You are on page 1of 95

TOPNOTCH MEDICAL BOARD PREP INTERNAL MEDICINE SUPEREXAM

For inquiries visit www.topnotchboardprep.com.ph or email us at topnotchmedicalboardprep@gmail.com


DEAR TOPNOTCH FRIENDS:

PLEASE FOLLOW THESE INSTRUCTIONS:

1. These questions are previous diagnostic, midterm, and finals exams of Topnotch, almost all of them made by Topnotch Board Exam Topnotchers.
2. Answer this Topnotch Superexam seriously 100-items at a time. Cover the “Explanations” Column. Do not immediately look at the answers from the
answer key. That’s not the correct way of answering sample exams. You need to treat these MCQs as exercises and not as handouts.
3. Time yourself. 1.5 hours per 100-item block.
4. After answering each 100-item block, refer to the Topnotch Answer Key for the correct answers. Please be careful of “frameshift mutations” when
checking your answers – check every 10 items. (the format of the answer key was designed for you to practice against “frameshift mutations”)
5. The Topnotch Superexams are EXERCISES for the actual med boards. They will not appear verbatim in your future exams. More than knowing what’s
the correct answer, it’s more important for you to:
a. Know why the other choices are wrong
b. Know why the other choices were included in the first place
c. Know the explanation to the correct answer
6. Sharpen your mind by answering the Topnotch Superexams. Most of these questions based on past feedback are more difficult than the actual questions
in the med boards. In these exams made by Board Exam Topnotchers, if you’re getting a score of 60/100 , that’s already a good score. More than 80/100
is outstanding.

Item QUESTION EXPLANATION AUTHOR TOPNOTCH
# EXAM
1 The most common cause of familial predisposition to Approximately 3% of the worlds population is KRISTEL TANHUI DIAGNOSTIC
thrombosis is: heterozygous for this mutation. (TOP 3 - AUG 2015 EXAM -
A. Hemophilia antibodies MED BOARDS; MARCH 2016
B. Protein C deficiency Source: Harrison’s Principles of Internal Medicine TOPNOTCH MD
C. Protein S deficiency 18th edition p525 FROM LA SALLE)
D. Factor V Leiden mutation
E. None of the above

2 A patient presents with neck stiffness and positive Meningeal involvement of tuberculous meningitis KRISTEL TANHUI DIAGNOSTIC
kernig’s and brudzinski’s sign. The spinal tap results is most pronounced at the base of the brain. (TOP 3 - AUG 2015 EXAM -
were suggestive of TB meningitis. Which of the MED BOARDS; MARCH 2016
following is the most likely CT scan finding? TB Meningitis – base of brain… TB meningitis – TOPNOTCH MD
A. Ring enhancing lesion base of brain… TB meningitis – base of brain FROM LA SALLE)
B. Hydrocephalus and abnormal enhancement of the
basal cisterns Source: Harrison’s Principles of Internal Medicine
C. Periventricular enhancement 18th edition p1348
D. Predisposition for the temporal lobes
E. Diffuse enhancement

3 Which of the following is best for monitoring Source: Harrison’s Principles of Internal Medicine KRISTEL TANHUI DIAGNOSTIC
therapeutic response in liver cancer? 18th edition p779 (TOP 3 - AUG 2015 EXAM -
A. AFP MED BOARDS; MARCH 2016
B. CEA TOPNOTCH MD
C. CA 19-9 FROM LA SALLE)
D. CA 125
E. Chromogranin a

4 Which of the following is a marker for lupus Antinuclear antibodies – Best screening test KRISTEL TANHUI DIAGNOSTIC
nephritis and is such most useful for assessing Anti-dsDNA – High titers are SLE specific and in (TOP 3 - AUG 2015 EXAM -
prognosis in SLE? some patients correlate with disease activity, MED BOARDS; MARCH 2016
A. Anti-histone nephritis, vasculitis TOPNOTCH MD
B. Anti-Sm Anti-Sm – Specific for SLE; no definite clinical FROM LA SALLE)
C. Anti-Ro correlations; most patients also have anti-RNP;
D. ANA more common in blacks and Asians than whites
E. Anti-dsDNA Anti-Ro – neonatal lupus with congenital heart
block
Anti-histone – drug induced lupus

Source: Harrison’s Principles of Internal Medicine
18th edition p2726
5 In patients with COPD with expiratory airflow In COPD, there is airway wall thickening of the KRISTEL TANHUI DIAGNOSTIC
limitation, the residual volume is: small airways. Also, there is alveolar wall (TOP 3 - AUG 2015 EXAM -
A. Increased destruction which leads to airway collapse and MED BOARDS; MARCH 2016
B. Decreased decrease in elastic recoil. Airway wall thickening TOPNOTCH MD
C. Same with airway collapse leads to airway obstruction. FROM LA SALLE)
D. Normalizes Airway obstruction with decrease elastic recoil
E. Initially increased then becomes decreased leads to airflow limitation causing air trapping and
hyperinflation. So, the patient experiences
difficulty in exhalation due to the obstruction.
In COPD, the lungs are hyperinflated thus the
diaphragm becomes depressed. This is because of
air trapping.
Notice that in COPD patients, there is a decrease in
the inspiratory capacity (IC) and an increase in the
residual volume (RV). The decrease in IC causes
dynamic hyperinflation.

Source: Harrison’s Principles of Internal Medicine
18th edition p2153

TOPNOTCH MEDICAL BOARD PREP INTERNAL MEDICINE SUPEREXAM Page 1 of 95


For inquiries visit www.topnotchboardprep.com.ph or email us at topnotchmedicalboardprep@gmail.com
TOPNOTCH MEDICAL BOARD PREP INTERNAL MEDICINE SUPEREXAM
For inquiries visit www.topnotchboardprep.com.ph or email us at topnotchmedicalboardprep@gmail.com
Item QUESTION EXPLANATION AUTHOR TOPNOTCH
# EXAM
6 A 30 year old obese patient presents with This is a case of obstructive sleep apnea. KRISTEL TANHUI DIAGNOSTIC
Hypertension. His roommate says he snores and (TOP 3 - AUG 2015 EXAM -
chokes during his sleep. During the day, the patient CPAP continuous positive airway pressure – MED BOARDS; MARCH 2016
would complain of headache and excessive daytime obstructive sleep apnea TOPNOTCH MD
sleepiness. The best ventilator management for this FROM LA SALLE)
patient is: BIPAP Bilevel positive airway pressure – COPD
A. CPAP
B. BIPAP AC mode – commonly used for initiation of
C. Assist control mechanical ventilation mechanical ventilation (unloads or rests the
D. SIMV (Synchronized intermittent mandatory respiratory muscles), absence of intact respiratory
ventilation) drive, completely unloads the patient’s respi
E. None. Patient does not need ventilatory support muscles requiring no work from the patient

SIMV – weaning, allows patients with intact
respiratory drive to exercise inspiratory muscles
between assisted breaths

Source: Harrison’s Principles of Internal Medicine
18th edition p2188, 2212
7 This describes acute decompensation after initiation Source: Harrison’s Principles of Internal Medicine KRISTEL TANHUI DIAGNOSTIC
of therapy for infection due to spirochetes such as in 18th edition p 1396 (TOP 3 - AUG 2015 EXAM -
leptospirosis: MED BOARDS; MARCH 2016
A. Drug resistance TOPNOTCH MD
B. Genetic variation FROM LA SALLE)
C. Jarische Herxheimer reaction
D. Acute drug reaction
E. All of the above

8 Migratory necrolytic erythema, glucose intolerance VIPoma/Verner Morrison syndrome– WDHA KRISTEL TANHUI DIAGNOSTIC
and thromboembolism are features of: Syndrome (watery diarrhea, Hypokalemia, (TOP 3 - AUG 2015 EXAM -
A. VIPoma Achlorydia) MED BOARDS; MARCH 2016
B. Somatostatinoma Somatostatinoma (3d’s) – Diabetes, diarrhea, TOPNOTCH MD
C. Gastrinoma Disease of the gallbladder (gallstone) FROM LA SALLE)
D. Glucagonoma Gastrinoma – multiple peptic ulcers
E. Insulinoma Insulinoma - hypoglycemia

Source: Harrison’s Principles of Internal Medicine
18th edition p3067
9 Occlusion of which of the following vessels results to Entire MCA occlusion at its origin (blocking both KRISTEL TANHUI DIAGNOSTIC
signs and symptoms of hemiplegia, homonymous its penetrating and cortical branches) and the (TOP 3 - AUG 2015 EXAM -
hemianopsia, hemianesthesia and gaze preference? distal collaterals are limited: contralateral MED BOARDS; MARCH 2016
A. Anterior cerebral artery hemiplegia, hemianesthesia, homonymous TOPNOTCH MD
B. Middle cerebral artery hemianopia, and a day or two of gaze preference FROM LA SALLE)
C. Posterior cerebral artery to the ipsilateral side. Dysarthria is common
D. Basilar artery because of facial weakness. When the dominant
E. Vertebral artery hemisphere is involved, global aphasia is present
also, and when the nondominant hemisphere is
affected, anosognosia, constructional apraxia, and
neglect are found

Source: Harrison’s Principles of Internal Medicine
18th edition p3284
10 Which of the following is not a part of the Bedside BISAP = BUN, Impaired mental status, SIRS, Age KRISTEL TANHUI DIAGNOSTIC
Index of Severity in Acute Pancreatitis? >60, Pleural effusion on radiography (TOP 3 - AUG 2015 EXAM -
A. BUN MED BOARDS; MARCH 2016
B. Mental Status The criteria for the severity of acute pancreatitis TOPNOTCH MD
C. Amylase was defined as organ failure of at least one organ FROM LA SALLE)
D. SIRS system and the presence of a local complication
E. Effusion in the pleural cavity such as necrosis, pseudocyst and abscess.

Source: Harrison’s Principles of Internal Medicine
18th edition p2639
11 Which of the following is the strongest predictor of Remember that tachypnea is the single most KRISTEL TANHUI DIAGNOSTIC
pneumonia? useful clinical sign for identifying pneumonia. (TOP 3 - AUG 2015 EXAM -
A. Tachycardia MED BOARDS; MARCH 2016
B. Tachypnea Source: Philippines CAP guidelines TOPNOTCH MD
C. Fever Palafox et al (2000). Diagnostic Value of FROM LA SALLE)
D. Chills Tachypnea in pneumonia defined radiologically.
E. Rhonchi

TOPNOTCH MEDICAL BOARD PREP INTERNAL MEDICINE SUPEREXAM Page 2 of 95


For inquiries visit www.topnotchboardprep.com.ph or email us at topnotchmedicalboardprep@gmail.com
TOPNOTCH MEDICAL BOARD PREP INTERNAL MEDICINE SUPEREXAM
For inquiries visit www.topnotchboardprep.com.ph or email us at topnotchmedicalboardprep@gmail.com
Item QUESTION EXPLANATION AUTHOR TOPNOTCH
# EXAM
12 A 57 year old alcoholic with chronic liver disease 2 complications of hepatic failure can only be KRISTEL TANHUI DIAGNOSTIC
presents with dyspnea that worsens on moving from treated with liver transplantation – (TOP 3 - AUG 2015 EXAM -
supine to upright position. This was supported by hepatopulmonary and hepatorenal syndrome. MED BOARDS; MARCH 2016
the finding of decreased O2 saturation on pulse TOPNOTCH MD
oximetry when moving from supine to upright Source: Harrison’s Principles of Internal Medicine FROM LA SALLE)
position. What is treatment of choice for this 18th edition p 2601
condition?
A. Liver transplantation
B. Continuous O2 support
C. Long acting Beta 2 agonist
D. Tiotropium
E. Portocaval shunt

13 A 31-year-old male with hemophilia A is admitted DDAVP causes the release of a number of factors KRISTEL TANHUI DIAGNOSTIC
with persistent gross hematuria. He denies recent and von Willebrand factor (TOP 3 - AUG 2015 EXAM -
trauma or any history of genitourinary pathology. from the liver and endothelial cells. This may be MED BOARDS; MARCH 2016
The examination is unremarkable. Hematocrit is useful for patients with mild hemophilia. FFP, TOPNOTCH MD
28%. All the following are treatments for hemophilia Cryoprecipitate and Recombinant Factor VIII can FROM LA SALLE)
A except: replace the lacking coagulation factor.
A. Desmopressin Plasmapheresis has no role in treatment of
B. Fresh frozen plasma Hemophilia A.
C. Cryoprecipitate
D. Recombinant Factor VIII Source: Harrison’s Principles of Internal Medicine
E. Plasmapheresis 18th edition p975

14 A patient who was diagnosed with acute MI develops Killip I – No signs of pulmonary edema KRISTEL TANHUI DIAGNOSTIC
dyspnea. On PE, vitals were HR 96, RR 22 and bp Killip II – Moderate heart failure as evidenced by (TOP 3 - AUG 2015 EXAM -
120/70 and an S3 gallop is appreciated. Chest xray rales at the lung bases, S3 gallop, tachypnea, signs MED BOARDS; MARCH 2016
was done which revealed pulmonary edema.What is of right sided heart failure including venous and TOPNOTCH MD
the Killip classification for the patient? hepatic congestion FROM LA SALLE)
A. Killip I Killip III – Pulmonary edema, severe heart failure
B. Killip II Killip IV – Shock
C. Killip III
D. Killip IV **There’s no killip V
E. Killip V
Source: Harrison’s Principles of Internal Medicine
18th edition p2031

15 A 45 year old male with HIV sought consult for We had cases that asked for patho like stuff in IM KRISTEL TANHUI DIAGNOSTIC
edema. PE revealed normal blood pressure and like this. (TOP 3 - AUG 2015 EXAM -
grade II bilateral edema. Auscultation was normal. A MED BOARDS; MARCH 2016
urinalysis was requested which revealed +4 This is a case of Focal segmental TOPNOTCH MD
proteinuria and negative RBC/WBC findings. Blood Glomerulosclerosis. FROM LA SALLE)
chemistry revealed serum albumin of 1.2g/dl and We are presented here with nephrotic syndrome.
elevated triglycerides. What is the most likely The most common causes of nephrotic syndrome
finding in light microscopy? in adults are either membranous glomerulopathy
A. Focal and segmental sclerosis and hyalinosis and FSGS. However, FSGS is associated with
B. GBM splitting HIV/AIDS, Heroin use, morbid obesity.
C. Subepithelial spike and dome
D. Diffuse capillary wall thickening Source: Robbins and Cotran Pathologic Basis of
E. Tram tracking Disease 8th ed p.918

16 A patient was recently diagnosed with carcinoid This is a case of pellagra, which is a deficiency of KRISTEL TANHUI DIAGNOSTIC
syndrome and seeks consult for diarrhea and Vitamin B3 or niacin. Pellagra presents with the (TOP 3 - AUG 2015 EXAM -
photosensitive dermatitis. On physical examination, classic D’s – diarrhea, dermatitis, dementia and MED BOARDS; MARCH 2016
an erythematous, pigmented skin rash in the death. TOPNOTCH MD
distribution of a broad collar (Casal necklace) was FROM LA SALLE)
noted. Patient is probably suffering from a deficiency NAD (niacin) is synthesized from the amino acid
of which of the following? tryptophan. In carcinoid syndrome, a serotonin
A. Vitamin A producing tumor diverts as much as 60% of the
B. Vitamin B1 body’s tryptophan metabolism away from the
C. Vitamin B3 synthesis NAD.
D. Vitamin B6 (in other words, serotonin and NAD are both
E. Vitamin E synthesized from tryptophan)

Source: Harper 27th ed p. 498
17 A 30-year-old male patient who recently underwent This is a case of tumor lysis syndrome which may KRISTEL TANHUI DIAGNOSTIC
chemotherapy for a leukemia, presented at the ER be precipitated by chemotherapy usually of (TOP 3 - AUG 2015 EXAM -
with signs and symptoms of acute renal failure and leukemias and lymphomas. The rapid death of MED BOARDS; MARCH 2016
seizures. Labs revealed hypocalcemia, massive amounts of cancer cells causes release of TOPNOTCH MD
hyperuricemia, hyperphosphatemia and intracellular ions most significantly Calcium, FROM LA SALLE)
hyperkalemia. Which of the following may be given phosphates, potassium as well as uric acid. This
to prevent this condition? causes derangement of electrolytes precipitating
A. Dexrazoxane seizures and arrhythmias. The elevated uric acid
B. Leucovorin levels can cause urate nephropathy and acute
C. Amifostine renal failure. Allopurinol may given
D. Allopurinol prophylactically.
E. Aspirin
Source: Harrison’s Principles of Internal Medicine
18th edition p2274

TOPNOTCH MEDICAL BOARD PREP INTERNAL MEDICINE SUPEREXAM Page 3 of 95


For inquiries visit www.topnotchboardprep.com.ph or email us at topnotchmedicalboardprep@gmail.com
TOPNOTCH MEDICAL BOARD PREP INTERNAL MEDICINE SUPEREXAM
For inquiries visit www.topnotchboardprep.com.ph or email us at topnotchmedicalboardprep@gmail.com
Item QUESTION EXPLANATION AUTHOR TOPNOTCH
# EXAM
18 A 65 year old diabetic male with a 15 year history of The drug of choice for DM gastroparesis is KRISTEL TANHUI DIAGNOSTIC
poorly controlled DM seeks consult for constantly Metoclopromide. (TOP 3 - AUG 2015 EXAM -
feeling fullness even after only a few bites. This is MED BOARDS; MARCH 2016
occassionaly accompanied by nausea or abdominal Source: Topnotch handout on Pharmacology TOPNOTCH MD
pain. He otherwise feels fine. An ultrasound was FROM LA SALLE)
done which reveals normal findings. He was finally
diagnosis with diabetic gastroparesis. What is the
drug of choice for this condition?
A. Metoclopromide
B. Ranitidine
C. Omeprazole
D. Lactulose
E. Senna concentrate

19 Which of the following is used in the diagnosis of an The diagnosis of anaphylactic reaction depends on KRISTEL TANHUI DIAGNOSTIC
anaphylactic reaction? a history revealing the onset of symptoms and (TOP 3 - AUG 2015 EXAM -
A. Epinephrine assays signs within minutes after the responsible MED BOARDS; MARCH 2016
B. Tryptase assay material is encountered. TOPNOTCH MD
C. Fibrinogen assay FROM LA SALLE)
D. Serotonin assay Source: Harrison’s Principles of Internal Medicine
E. Nitrous oxide (NO) assay 18th edition p2710

20 Which blood type is associated with gastric PUD – Blood type O KRISTEL TANHUI DIAGNOSTIC
adenocarcinoma? Gastric CA – Blood type A (TOP 3 - AUG 2015 EXAM -
A. Type A MED BOARDS; MARCH 2016
B. Type B Source: Harrison’s Principles of Internal Medicine TOPNOTCH MD
C. Type AB 18th edition p766 FROM LA SALLE)
D. Type O
E. Blood type is not associated with gastric adenoCA

21 Acne-form like rash can occur with treatment of SIMILAR TO PREVIOUS BOARD EXAM LESTER BRYAN CO MIDTERM 1
which anti-Koch's medication? CONCEPT/PRINCIPLE. There were more than a (TOP 10 - AUG EXAM -
A. Rifampicin fair share of Dermatology questions in our IM 2015 MED MARCH 2016
B. Pyrazinamide Exam, and some of them are nigh unanswerable BOARDS;
C. Isoniazid like in this case. TOPNOTCH MD
D. Streptomycin FROM UST)
E. Ethambutol

22 A 60-year-old man, a heavy smoker, presents for Emphysema is an example of COPD. Due to the LESTER BRYAN CO MIDTERM 1
advice to stop smoking. On physical examination, he destruction of alveolar walls, a lack of elastic recoil (TOP 10 - AUG EXAM -
is thin and has a ruddy complexion. He has a causes air to become trapped in alveoli, and, thus, 2015 MED MARCH 2016
productive cough and a barrel-shaped chest. He sits airflow obstruction occurs on expiration. In COPD, BOARDS;
leaning forward with his lips pursed to facilitate his FEV1 is decreased, whereas FVC is normal or TOPNOTCH MD
breathing. Which of the following is the most likely increased; therefore, patients with COPD have a FROM UST)
histologic finding in the lungs? decreased FEV1:FVC ratio.
A. Bronchial smooth muscle hypertrophy with
proliferation of eosinophils
B. Diffuse alveolar damage with leakage of protein-
rich fluid into alveolar spaces
C. Dilation of air spaces with destruction of alveolar
walls
D. Hyperplasia of bronchial mucus-secreting
submucosal glands
E. Permanent bronchial dilation caused by chronic
infection, with bronchi filled with mucus and
neutrophils
23 A 62-year-old man has had an elevated hematocrit The very elevated hemoglobin and hematocrit, LESTER BRYAN CO MIDTERM 1
for at least 3 years. His past medical history and which strongly correlate with true elevations in (TOP 10 - AUG EXAM -
review of systems are negative, except for mild, well- RBC mass rather than plasma contraction, are 2015 MED MARCH 2016
controlled hypertension. His latest complete blood consistent with polycythemia. Polycythemia then BOARDS;
count reveals the following: hemoglobin 18 mg/dL; is broken down into autonomous or primary TOPNOTCH MD
hematocrit 56%; and WBC count 17,500/mm3 with polycythemia vera, a stem cell disease, in which FROM UST)
platelets 800,000/mm3. On further investigation, the marrow is autonomously creating too many
which of the following findings is the most typical cells, versus reactive or secondary forms, in which
and expected? the marrow is responding to increased
A. Ringed sideroblasts on bone marrow examination erythropoietin from some alteration of normal
B. Arterial blood oxygen saturation less than 88% physiology. The elevations of the other cell lines
C. Presence of a Philadelphia chromosome on suggest polycythemia vera because this stem cell
cytogenetic testing disease involves all marrow cell lines. Ringed
D. Very low to absent erythropoietin titer sideroblasts and Pelger–Huet cells are seen in
E. Many Pelger–Huet cells on peripheral blood smear myelodysplasia, which is another condition
resulting from abnormal marrow clonal stem cells.
However, in this condition, cytopenias rather than
increases in counts are expected. On occasion,
CML,
another marrow stem cell clonal proliferative
disease, may manifest with elevated counts—
specifically the WBC count and, to a lesser extent,
platelets. In CML, a hemoglobin and hematocrit
elevated to this degree would be unusual and are
much more typical of polycythemia vera.

TOPNOTCH MEDICAL BOARD PREP INTERNAL MEDICINE SUPEREXAM Page 4 of 95


For inquiries visit www.topnotchboardprep.com.ph or email us at topnotchmedicalboardprep@gmail.com
TOPNOTCH MEDICAL BOARD PREP INTERNAL MEDICINE SUPEREXAM
For inquiries visit www.topnotchboardprep.com.ph or email us at topnotchmedicalboardprep@gmail.com
Item QUESTION EXPLANATION AUTHOR TOPNOTCH
# EXAM
24 A 42-year-old woman presents to clinic for a routine SIMILAR TO PREVIOUS BOARD EXAM LESTER BRYAN CO MIDTERM 1
visit. Her father and brother were diagnosed with CONCEPT/PRINCIPLE. Screening for colon cancer (TOP 10 - AUG EXAM -
colon cancer at ages 49 and 43, respectively. She in average-risk patients should begin at age 50 2015 MED MARCH 2016
denies abdominal pain, weight loss, melena, years; however, this patient has two fi rst-degree BOARDS;
hematochezia, or changes in stool caliber. The best relatives with colorectal cancer, making her high TOPNOTCH MD
option for colorectal cancer screening in this patient risk for developing the disease. Patients with a FROM UST)
is: family history of early colon cancer should begin
A. Colonoscopy beginning at age 50 years receiving screening at age 40 years or 10 years
B. CEA and CA 19-9 testing now before the age at which cancer was diagnosed in
C. Screening for Helicobacter pylori infection and the family member, whichever is first. CEA and CA
treatment, if positive 19-9 levels may be elevated in patients with
D. Barium enema with CEA and CA 19-9 testing colorectal neoplasms, but this is not specific and is
beginning at age 50 years not used for screening. H. pylori is a risk factor for
E. Colonoscopy now gastric cancer, but there is no evidence of
increased risk of colorectal cancers.
25 A 62-year-old woman with congestive heart failure With the exception of the LDH findings, all of the LESTER BRYAN CO MIDTERM 1
develops pneumonia and a large pleural effusion. pleural fluid fi ndings listed indicate the presence (TOP 10 - AUG EXAM -
Thoracentesis is performed in an effort to establish of an exudate. Exudates are caused by 2015 MED MARCH 2016
whether the pleural effusion is due to CHF or inflammation or disease of the pleural surface or BOARDS;
pneumonia. Which of the following fi ndings would by lymphatic obstruction (e.g., due to tuberculosis, TOPNOTCH MD
indicate that the pleural effusion is due to CHF? lung cancer, or pneumonia). Transudates are FROM UST)
A. A protein content of 6 g/dL with a serum protein caused by elevated systemic or pulmonary venous
level of 8 g/dL pressure or by decreased plasma oncotic pressure
B. A pH of 7.13 (e.g., due to CHF or nephrotic syndrome).
C. A glucose content of 20 mg/dL Therefore, in establishing the etiology of a pleural
D. A lactate dehydrogenase content of 100 mg/dL effusion, it is useful to determine whether the fl
(with a serum LDH level of 420 mg/dL) uid is a transudate or an exudate. This
E. A pleural fluid to serum protein ratio of 0.7 determination often can be made on the basis of a
chemical analysis of the pleural fluid. A pleural
fluid-to-serum protein ratio of more than 0.5, an
LDH content of more than two-thirds the upper
limit for serum, or a pleural fluid-to-serum LDH
ratio of greater than 0.6 usually indicates the
presence of an exudate. Pleural fluid pH values of
less than 7.2 and a pleural fluid glucose content of
less than 20 mg/dL also are associated with
inflammatory effusions (exudates).
26 A 30-year-old man presents with hypertension, SIMILAR TO PREVIOUS BOARD EXAM LESTER BRYAN CO MIDTERM 1
hematuria, palpable renal masses, and flank pain. He CONCEPT/PRINCIPLE. Adult polycystic kidney (TOP 10 - AUG EXAM -
states that a kidney disease runs in his family, and disease is the most common inherited disorder of 2015 MED MARCH 2016
his mother and maternal grandfather have it. the kidney and is characterized by autosomal BOARDS;
Laboratory abnormalities confirm renal failure. dominant inheritance. The disease is characterized TOPNOTCH MD
Which of the following is associated with the by partial replacement of the renal parenchyma FROM UST)
disorder described here? with cysts. An important association of adult
A. Autosomal recessive inheritance polycystic kidney disease is berry aneurysm of the
B. Berry aneurysm of the circle of Willis circle of Willis.
C. Clinical manifestation most commonly at birth
D. Multiple small medullary cysts in both kidneys
E. Presence of uric acid stones

27 Following a bar fight, a 22-year-old man is brought An epidural hematoma is an arterial hemorrhage LESTER BRYAN CO MIDTERM 1
unconscious to the emergency department. Several between the dura and the skull, most often (TOP 10 - AUG EXAM -
minutes earlier, he had been hit on the head with a resulting from skull fracture and laceration of the 2015 MED MARCH 2016
heavy iron club and had been briefly unconscious, middle meningeal artery. Epidural hematomas are BOARDS;
but had then apparently recovered. One or two characterized clinically by a short period of TOPNOTCH MD
minutes later, he had again lost consciousness. consciousness (lucid interval) followed by loss of FROM UST)
Which of the following is the most likely diagnosis? consciousness and signs of cerebral compression.
A. Epidural hematoma A subdural hematoma is venous hemorrhage
B. Subarachnoid hemorrhage underneath the dura, resulting from laceration of
C. Subdural hematoma the bridging veins. Subdural hematomas are
D. Transient ischemic attack characterized clinically by gradual signs of
E. Stroke cerebral compression occurring hours, days, or
weeks after injury. Subarachnoid hemorrhage is
commonly associated with rupture of a berry
aneurysm in the circle of
Willis. A transient ischemic attack is a brief
episode of impaired neurologic function caused by
a brief disturbance in cerebral circulation.

TOPNOTCH MEDICAL BOARD PREP INTERNAL MEDICINE SUPEREXAM Page 5 of 95


For inquiries visit www.topnotchboardprep.com.ph or email us at topnotchmedicalboardprep@gmail.com
TOPNOTCH MEDICAL BOARD PREP INTERNAL MEDICINE SUPEREXAM
For inquiries visit www.topnotchboardprep.com.ph or email us at topnotchmedicalboardprep@gmail.com
Item QUESTION EXPLANATION AUTHOR TOPNOTCH
# EXAM
28 A 64-year-old man enters the hospital because of SIMILAR TO PREVIOUS BOARD EXAM LESTER BRYAN CO MIDTERM 1
renal insuffi ciency. Until 6 months earlier, when he CONCEPT/PRINCIPLE. Questions in our IM exam (TOP 10 - AUG EXAM -
developed persistent back pain, he was in good tended to be long and focused on obscure 2015 MED MARCH 2016
health. At that time, he was found to be severely diseases, so manage your time wisely. Multiple BOARDS;
anemic, and his BUN and creatinine levels were myeloma is the most likely etiology. The TOPNOTCH MD
elevated (42 and 4.6 mg/dL, respectively). He now combination of hypercalcemia and acute kidney FROM UST)
undergoes further evaluation. He denies the use of injury raises the possibility of multiple myeloma
any medications, any past history of renal injury, and as the bone breakdown secondary to tumor
any diffi culty in voiding. He does complain of involvement releases large amounts of calcium to
persistent weakness and easy fatigability, and his the extracellular fluid and hypercalcemia ensues.
back pain has become more severe over the last 2 The renal failure in myeloma is primarily related
weeks. On physical examination, blood pressure is to hypercalcemia combined with proteinaceous
120/80 mm Hg, pulse is 70 bpm, respiratory rate is cast formation within the renal tubules, producing
15 breaths per minute, and temperature is 37.0°C. a form of intratubular obstruction as well as a
Major physical fi ndings include severe pallor, as tubular infl ammatory lesion. The major diagnostic
well as clear evidence of muscle wasting. Urinalysis clue is the finding of a urinary dipstick that is
reveals 1 protein on dipstick testing and 4 on mildly positive for protein in the urine but a
sulfosalicylic acid testing. Microscopic examination sulfosalicylic acid test that is strongly positive.
of the urine reveals an occasional broad cast and an Dipstick testing does not detect the negatively
occasional granular cast. Laboratory studies give the charged light-chain proteins, only the albumin.
following results: BUN=61 mg/dL, creatinine = 5.1 The sulfosalicylic acid test detects all forms of
mg/dL, serum sodium = 141 mEq/L, serum proteins. Renovascular lesions and thrombotic
potassium = 5.6 mEq/L, serum chloride = 101 renal disease could present with this picture,
mEq/L, serum bicarbonate = 14 mEq/L, serum although they should not be associated with
calcium = 11.7 mg/dL, and serum phosphorus = 6.0 hypercalcemia and severe back pain, and findings
mg/dL. Which of the following is the most likely on examination of the urine would not include
cause of this condition? proteins. SLE can be associated with severe
A. Renovascular disease anemia and joint manifestations, but
B. Thrombotic renal disease
hypercalcemia is not part of the picture.
C. Multiple myeloma
D. Systemic lupus erythematosus
E. Analgesic nephropathy
29 A 34-year-old woman presents in the emergency A pulmonary thromboembolism results in areas of LESTER BRYAN CO MIDTERM 1
department with tachypnea and shortness of breath the lung that are ventilated, but not perfused, (TOP 10 - AUG EXAM -
of acute onset. The history reveals that she has been yielding ratios of infinity and an increase in 2015 MED MARCH 2016
taking oral ontraceptives for 9 years. A lung scan alveolar dead space. When the ratio equals ∞, the BOARDS;
demonstrates a perfusion defect in the left lower PAO2 of the affected alveoli will be the same as TOPNOTCH MD
lobe. Which of the following occurs if the blood flow that in the humidified inspired air because FROM UST)
to alveolar units is totally obstructed by a pulmonary atmospheric air enters the alveoli via the process
thromboembolism? of ventilation, but no gas exchange takes place
a. The V/Q ratio of the alveolus equals zero. because the alveoli are not perfused. Areas of the
b. The PO2 of the alveolus will be equal to that in the lung that are perfused but not ventilated
inspired air. constitute areas of shunting (venous admixture),
c. The PO2 of the alveolus will be equal to the mixed characterized as a ratio equal to 0, and having
venous PO2. PAO2 values that equilibrate with the mixed
d. There will be an increase in shunting in the lung. venous blood.
e. There will be a decrease in alveolar dead space.
30 A sexually active 24-year-old woman known to be The most likely cause of the pneumonia is LESTER BRYAN CO MIDTERM 1
HIV-1 infected has had a fever for 2 days and has a Streptococcus pneumoniae. Although P. carinii is (TOP 10 - AUG EXAM -
productive cough. Chest radiographs show an the most common serious opportunistic infection 2015 MED MARCH 2016
infiltrate in the right lung. Two weeks earlier, her in patients with HIV-1 infection, the productive BOARDS;
helper T cell count was 510/mm3. Testing for cough, localized infi ltrate, and brief duration TOPNOTCH MD
nontreponemal antigen (rapid plasma reagin) is argue against P. carinii pneumonia. In addition, the FROM UST)
positive at two dilutions, and treponemal antigen CD4 count of more than 500 cells/mm3 suggests
testing is also positive. Which of the following is the that serious
most likely cause of the pneumonia? opportunistic infections such as PCP,
A. Streptococcus pneumoniae cytomegalovirus, or Mycobacterium avium-
B. Pneumocystis carinii intracellulare are unlikely for some time. Syphilis,
C. Cytomegalovirus (CMV) which may have been present, almost never
D. Mycobacterium avium-intracellulare involves the lungs.
E. Syphilis
31 A 75-year-old man complains of chest pain while The murmur and delayed carotid upstrokes are LESTER BRYAN CO MIDTERM 1
climbing stairs. On physical examination, there is a typical of the fixed LV outflow obstruction of aortic (TOP 10 - AUG EXAM -
II/VI systolic ejection murmur that radiates to the stenosis. Pulmonary stenosis also can cause chest 2015 MED MARCH 2016
neck. The carotid upstrokes are delayed and pain and a systolic ejection murmur but would not BOARDS;
diminished in volume. The likely diagnosis is: cause carotid delay. Hypertrophic cardiomyopathy TOPNOTCH MD
A. Hypertrophic cardiomyopathy causes a spike and dome of the carotid upstrokes, FROM UST)
B. Aortic stenosis that is, a sharp upstroke followed by fall and a
C. Mitral stenosis flatter secondary rise. The murmur of mitral
D. Pulmonary stenosis stenosis is diastolic.
E. Pulmonic regurgitation

TOPNOTCH MEDICAL BOARD PREP INTERNAL MEDICINE SUPEREXAM Page 6 of 95


For inquiries visit www.topnotchboardprep.com.ph or email us at topnotchmedicalboardprep@gmail.com
TOPNOTCH MEDICAL BOARD PREP INTERNAL MEDICINE SUPEREXAM
For inquiries visit www.topnotchboardprep.com.ph or email us at topnotchmedicalboardprep@gmail.com
Item QUESTION EXPLANATION AUTHOR TOPNOTCH
# EXAM
32 A 65-year-old woman with a significant smoking Small cell carcinoma of the lung is the most LESTER BRYAN CO MIDTERM 1
history presents with cough and shortness of breath. aggressive type of bronchogenic carcinoma. The (TOP 10 - AUG EXAM -
Computed tomography of the chest reveals a central location of this cancer is usually central. This is an 2015 MED MARCH 2016
mass near the left mainstem bronchus. Biopsy of the undifferentiated tumor with small round blue cells BOARDS;
mass is performed. Histologic examination reveals and is least likely to be cured by surgery because it TOPNOTCH MD
small round blue cells, and a diagnosis of small cell is usually already metastatic at diagnosis. FROM UST)
carcinoma is made. Which of the Associated paraneoplastic syndromes include
following is a frequent characteristic of this form of secretion of adrenocorticotropic hormone and
lung cancer? antidiuretic hormone.
A. Generally amenable to surgical cure at time of
diagnosis
B. More common in women, and a less clear relation
to smoking than other forms of lung cancer
C. Secretes a parathyroid-like hormone
D. Secretes either corticotrophin or antidiuretic
hormone
E. Usually in a peripheral rather than in a central
location
33 A 21-year-old woman presents to the emergency The patient most likely had diabetic ketoacidosis, LESTER BRYAN CO MIDTERM 1
department complaining of abdominal pain. She in a setting of newly-diagnosed type 1 diabetes (TOP 10 - AUG EXAM -
states that she has been having increased urination, characterized by a lack of insulin that led to 2015 MED MARCH 2016
increased thirst, and a 10-lb weight loss. On exam hyperglycemia and other metabolic derangements. BOARDS;
she has a blood pressure of 80/60 mm Hg, rapid DKA results in high anion gap metabolic acidosis. TOPNOTCH MD
deep breaths, and dry mucous membranes. Which The liver produces more ketone bodies than the FROM UST)
lab abnormality fit with the patient’s clinical body can metabolize. The anion gap reflects an
presentation? elevation of the acetoacetate and hydroxybutyrate
A. Low potassium in the plasma. Serum potassium levels may be
B. Low hematocrit elevated initially, but often patients have a low
C. Normal Anion gap acidosis body store and require repletion. As the acidosis
D. Low serum osmolality corrects, potassium will shift into cells in response
E. Low sodium to insulin. Patients may appear to have an elevated
hematocrit secondary to hemoconcentration.
Serum osmolality is often elevated and serum
sodium is diluted secondary to osmotic pull of the
glucose that shifts fluid into the intravascular
space.
34 A 55-year-old man presents with a chronic rash over SIMILAR TO PREVIOUS BOARD EXAM LESTER BRYAN CO MIDTERM 1
the buttocks and hips that has been unresponsive to CONCEPT/PRINCIPLE. A papulosquamous (TOP 10 - AUG EXAM -
topical steroids. It has recently started to itch. eruption in a bathing trunk distribution can be 2015 MED MARCH 2016
Examination shows 6- to 12-cm erythematous, seen with mycosis fungoides (cutaneous T cell BOARDS;
scaling plaques in a “bathing trunk” distribution. lymphoma), tinea corporis, or ossibly psoriasis. TOPNOTCH MD
Potassium hydroxide preparation is negative for The skin biopsy shows diagnostic changes with FROM UST)
evidence of a fungal infection. A skin biopsy atypical lymphocytes infi ltrating the epidermis
indicates an atypical lymphocytic infi ltrate with (epidermotropism) and forming clusters within
evidence of epidermotropism and Pautrier the epidermis (Pautrier microabscess). Impetigo
microabscess formation. What is the most likely typically has golden crusts as a predominant
diagnosis? feature. Atopic dermatitis predominantly affects
A. Impetigo flexor areas.
B. Psoriasis
C. Mycosis fungoides
D. Tinea corporis
E. Atopic dermatitis
35 A 57-year-old man is admitted to the hospital The clinical situation strongly suggests pulmonary LESTER BRYAN CO MIDTERM 1
because of acute shortness of breath shortly after a embolism. In greater than 80% of cases, (TOP 10 - AUG EXAM -
12-hour automobile ride. Findings on physical pulmonary emboli arise from thrombosis in the 2015 MED MARCH 2016
examination are normal except for tachypnea and deep venous circulation (DVT ) of the lower BOARDS;
tachycardia. He does not have edema or popliteal extremities, but a normal lower extremity Doppler TOPNOTCH MD
tenderness. An electrocardiogram reveals sinus does not exclude the diagnosis. DVTs often begin FROM UST)
tachycardia but is otherwise normal. Which of the in the calf, where they rarely if ever cause
following statements is correct? clinically significant pulmonary embolic disease.
A. A normal D-dimer level excludes pulmonary However, thromboses that begin below the knee
embolus. frequently “grow,” or propagate, above the knee;
B. If there is no contraindication to anticoagulation, clots that dislodge from above the knee cause
full-dose heparin or enoxaparin should be started clinically significant pulmonary emboli. Untreated
pending further testing. pulmonary embolism is associated with a 30%
C. Normal findings on examination of the lower mortality rate. Interestingly, only about 50% of
extremities make pulmonary embolism unlikely. patients with DVT of the lower extremities have
D. Early treatment of pulmonary embolism has little clinical findings of swelling, warmth, erythema,
effect on overall mortality. pain, or palpable “cord.” When a clot does dislodge
E. A normal lower extremity venous Doppler study from the deep venous system and travels into the
will rule out a pulmonary embolus. pulmonary vasculature, the most common clinical
findings are tachypnea and tachycardia; chest pain
is less likely and usually indicates pulmonary
infarction. T he ABG is usually abnormal, and a
high percentage of patients exhibit low P CO2 with
respiratory alkalosis, and a widening of the
alveolar-arterial oxygen gradient. The ECG usually
shows sinus tachycardia, but atrial fibrillation,
pseudoinfarction in the inferior leads, and acute
right heart strain are also seen. Initial treatment
for suspected pulmonary embolic disease includes
prompt hospitalization and institution of
intravenous heparin or therapeutic dose
subcutaneous low molecular-weight heparin. It is
particularly important to make an early diagnosis
of pulmonary embolus, as intervention can

TOPNOTCH MEDICAL BOARD PREP INTERNAL MEDICINE SUPEREXAM Page 7 of 95


For inquiries visit www.topnotchboardprep.com.ph or email us at topnotchmedicalboardprep@gmail.com
TOPNOTCH MEDICAL BOARD PREP INTERNAL MEDICINE SUPEREXAM
For inquiries visit www.topnotchboardprep.com.ph or email us at topnotchmedicalboardprep@gmail.com
Item QUESTION EXPLANATION AUTHOR TOPNOTCH
# EXAM
decrease the mortality rate from 30% down to 5%.
A normal D-dimer level helps exclude pulmonary
embolus in the low-risk setting. T his patient,
however, has a high pretest probability of PE;
further testing (CT pulmonary angiogram, V/Q
lung scan) must be done to exclude this important
diagnosis.

36 A 62-year-old woman presents to her primary care SIMILAR TO PREVIOUS BOARD EXAM LESTER BRYAN CO MIDTERM 1
physician with a 2-month history of stiffness and CONCEPT/PRINCIPLE. Polymyalgia rheumatica is (TOP 10 - AUG EXAM -
aching in her neck, shoulders, and hips. She is almost always seen in patients older than 50 years 2015 MED MARCH 2016
referred to a rheumatologist. After doing a battery of and can be seen in isolation or in patients with BOARDS;
tests, he diagnoses the woman with polymyalgia giant cell arteritis. CK levels are not increased with TOPNOTCH MD
rheumatica and not fibromyalgia. Which of the the disease, and while patients may have stiffness FROM UST)
following findings is characteristic of polymyalgia and pain, muscle weakness is not seen and
rheumatica? electromyography and muscle biopsy are normal.
A. Absence of response to prednisone The characteristic laboratory finding in
B. Antibodies to vascular smooth muscle polymyalgia rheumatica is an elevated ESR. There
C. Increased serum creatine kinase levels is generally a prompt therapeutic response to low-
D. Increased erythrocyte sedimentation rate dose prednisone.
E. Persistent muscle weakness
37 A 56-year-old patient with advanced alcoholic The clinical features of spontaneous bacterial LESTER BRYAN CO MIDTERM 1
cirrhosis and known ascites is found to have peritonitis, which develops in a setting of (TOP 10 - AUG EXAM -
abdominal pain, fever to 102 F, and a peripheral preexisting ascites, include abdominal pain, fever, 2015 MED MARCH 2016
white blood cell count of 17,000 with a shift to the leukocytosis, and paralytic ileus. The initial ascitic BOARDS;
left. Which of the following statements regarding the fluid total protein count is usually less than 1 TOPNOTCH MD
primary diagnosis is correct? mm3/dL. The absolute polymorphonuclear count FROM UST)
A. It is more likely when ascitic fluid total protein in the ascitic fluid is generally greater than 250
exceeds 1 mg/dL. cells/dL. Bacterial peritonitis associated with a
B. It develops in the setting of preexisting ascites. perforated viscus is secondary bacterial
C. The ascitic polymorphonuclear count is less than peritonitis.
100 cells/mm3.
D. It is often associated with aspergillosis.
E. It is associated with a perforated viscus.
38 A 50-year-old woman complains of a 2-month This patient presents with characteristic features LESTER BRYAN CO MIDTERM 1
history of her hands becoming painful and turning of scleroderma, a chronic illness in which (TOP 10 - AUG EXAM -
white or blue in the cold; progressive skin tightness unregulated immunologic processes (perhaps 2015 MED MARCH 2016
and thickening of fingers, hands, and forearms; triggered by unknown environmental antigens) BOARDS;
shortness of breath on exertion; and a sensation of cause small-vessel endothelial damage and TOPNOTCH MD
lower chest burning and food sticking on widespread dermal and internal organ fi brosis. FROM UST)
swallowing. Antibody testing shows the presence of The small-vessel endothelial damage leads to
ANA and elevated titers of antibody to anti- secondary vascular reactivity (Raynaud’s
topoisomerase I. Which of the following phenomenon) and, possibly, ischemic tissue
pathogenetic explanations best fits this patient’s damage. The increased collagen synthesis by
illness? A. Infiltration of tissue fibroblasts, which leads to widespread fi
mucopolysaccharides into underlying subepithelial brosis, is not unregulated; rather, it is caused by
tissues cytokine and growth factor secretion from
B. Unregulated fibroblastic collagen synthesis
lymphocytes, mast cells, and platelets. There is no
C. Raynaud’s phenomenon leading first to ischemia evidence that patients with scleroderma have
and later to tissue fibrosis tissue mucopolysaccharide infi ltration or that
D. Vascular endothelial damage and immunologically there are tumors responsible for paraneoplastic
mediated tissue fibrosis dermal fibrosis. Immunologic clues include
E. Carcinomatous paraneoplastic process elevated titers of antitbodies to anti-
topoisomerase I (Scl-70).

TOPNOTCH MEDICAL BOARD PREP INTERNAL MEDICINE SUPEREXAM Page 8 of 95


For inquiries visit www.topnotchboardprep.com.ph or email us at topnotchmedicalboardprep@gmail.com
TOPNOTCH MEDICAL BOARD PREP INTERNAL MEDICINE SUPEREXAM
For inquiries visit www.topnotchboardprep.com.ph or email us at topnotchmedicalboardprep@gmail.com
Item QUESTION EXPLANATION AUTHOR TOPNOTCH
# EXAM
39 A 78-year-old man enters the hospital because of The incidence of prostatism in elderly men is so LESTER BRYAN CO MIDTERM 1
abnormalities of urination. Today he is passing large great that it must be considered the primary cause (TOP 10 - AUG EXAM -
amounts of urine; however, some days he passes no of renal insuffi ciency until proven otherwise. This 2015 MED MARCH 2016
urine at all. He now has a blood pressure of 180/90 patient’s history is classic, in that he had 1 or 2 BOARDS;
mm Hg, and otherwise his physical examination is days on which he seemed to pass no urine, TOPNOTCH MD
normal. Laboratory studies show a BUN of 120 followed by days of high urine flow, a pattern that FROM UST)
mg/dL and a serum creatinine of 4.2 mg/dL. is caused by the gradual accumulation of large
Urinalysis reveals a specific gravity of 1.010; urine amounts of urine in the collecting system under
that is negative for protein, glucose, ketone bodies, pressure, which eventually may overcome some
and blood; and an occasional WBC per high-power degree of obstruction. The high pressure is
field on microscopic examination. Which of the transmitted back to the kidneys and results in
following is the most likely cause of the renal renal insuffi ciency. Acute glomerulonephritis and
insufficiency? acute interstitial nephritis are ruled out by the
A. Obstructive uropathy normal results of urinalysis. The possibility of
B. Acute glomerulonephritis acute tubular necrosis (ATN) should be
C. Acute interstitial nephritis
considered, but no information in the history
D. Acute tubular necrosis suggests recent surgery or nephrotoxic drug
E. Chronic renal failure of unspecifi ed nature intake that would have produced ATN. The best
way to screen for obstructive uropathy is renal
ultrasonography, which would demonstrate
dilated upper tract calyces.
40 A 56-year-old man enters the emergency Congestive heart failure is a syndrome, and its LESTER BRYAN CO MIDTERM 1
department complaining of dyspnea that began cause should be sought and treated directly (TOP 10 - AUG EXAM -
about 3 weeks ago and has progressed so that he whenever possible. It is usually helpful to establish 2015 MED MARCH 2016
now has diffi culty walking across a room. He has whether the root cause is systolic or diastolic BOARDS;
begun sleeping on three pillows. On physical dysfunction, a distinction made easily with TOPNOTCH MD
examination: temperature is 99 F, pulse is 102 bpm, echocardiography. Diuretics form the mainstay of FROM UST)
blood pressure is 130/90 mm Hg, and respiratory therapy, but adding both ACE inhibitors and beta-
rate is 24. There is jugular venous distention, and blockers prolongs life.
estimated central venous pressure is 10 cm H2O.
Other findings include bibasilar rales and an S3
gallop. Which of the following is true about the
treatment of the condition of the patient?
A. The cause of the condition should be treated
whenever possible.
B. Systolic versus diastolic dysfunction usually
cannot be established.
C. ACE inhibitors improve symptoms but do not
prolong life.
D. Diuretics are the drugs of last resort.
E. Beta Blockers are dangerous and should be
avoided.
41 A 75-year-old man complains of chest pain while The murmur and delayed carotid upstrokes are GEORGE MICHAEL MIDTERM 2
climbing stairs. On physical examination, there is a typical of the fixed LV outflow obstruction of aortic SOSUAN (TOP 5 - EXAM -
II/VI systolic ejection murmur that radiates to the stenosis. Echocardiography with Doppler AUG 2015 MED MARCH 2016
neck. The carotid upstrokes are delayed and interrogation of the valve will show the aortic BOARDS;
diminished in volume. The best test to confi rm the stenosis, quantify its severity, and assess left TOPNOTCH MD
diagnosis is: ventricular function. FROM UST)
A An ECG
B An exercise stress test
C An echocardiogram
D A radionuclide ventriculogram
E A chest x-ray
42 A person who has been vaccinated against Hepatitis Vaccinated individuals will only seroconvert to GEORGE MICHAEL MIDTERM 2
B will have a reactive serologic test to positive anti-HBs. SOSUAN (TOP 5 - EXAM -
A. HBsAg AUG 2015 MED MARCH 2016
B. Anti-HBs BOARDS;
C. HBeAg TOPNOTCH MD
D. Anti-HBe FROM UST)
E. Anti-HBc

43 Which of the anti-TB medications has the potential Isoniazid causes dose-related pyridoxine GEORGE MICHAEL MIDTERM 2
to cause pyridoxine deiciency deficiency. SOSUAN (TOP 5 - EXAM -
A. Isoniazid AUG 2015 MED MARCH 2016
B. Rifampin BOARDS;
C. Pyrazinamide TOPNOTCH MD
D. Ethambutol FROM UST)
E. Streptomycin

44 E. coli infection causing hemolytic uremic syndrome Shiga-like toxin, also known as verotoxin, is a toxin GEORGE MICHAEL MIDTERM 2
(HUS) produces this toxin generated by some strains of Escherichia coli SOSUAN (TOP 5 - EXAM -
A. Tetratoxin causing hemolytic uremic syndrome. AUG 2015 MED MARCH 2016
B. Saxotoxin BOARDS;
C. Verotoxin TOPNOTCH MD
D. Beta-toxin FROM UST)
E. Lecitoxin

TOPNOTCH MEDICAL BOARD PREP INTERNAL MEDICINE SUPEREXAM Page 9 of 95


For inquiries visit www.topnotchboardprep.com.ph or email us at topnotchmedicalboardprep@gmail.com
TOPNOTCH MEDICAL BOARD PREP INTERNAL MEDICINE SUPEREXAM
For inquiries visit www.topnotchboardprep.com.ph or email us at topnotchmedicalboardprep@gmail.com
Item QUESTION EXPLANATION AUTHOR TOPNOTCH
# EXAM
45 What agar can be use to differentiate the strain of E. Sorbitol MacConkey Agar is a variant of traditional GEORGE MICHAEL MIDTERM 2
coli producing HUS from other strains? MacConkey Agar used in the detection of E. coli SOSUAN (TOP 5 - EXAM -
A. OH157:H7 Agar O157:H7. This strain differs from most other AUG 2015 MED MARCH 2016
B. Sorbitol MacConkey Agar strans of E. coli in being unable to ferment BOARDS;
C. Xylose Lysine Agar sorbitol. TOPNOTCH MD
D. Lysine Iron Agar FROM UST)
E. Lactose E. coli Agar

46 A 65 y/o M has had an elevated hematocrit for at The very elevated hematocrit, which strongly GEORGE MICHAEL MIDTERM 2
least 3 years. CBC reveals the following: hematocrit correlate with true elevations in red blood cell SOSUAN (TOP 5 - EXAM -
56%; and WBC count 17,500/mm3 with platelets (RBC) mass rather than plasma contraction, are AUG 2015 MED MARCH 2016
800,000/mm3. On further investigation, which of consistent with primary polycythemia. No BOARDS;
the following findings is the most typical and secondary causes such as tissue hypoxia (as with TOPNOTCH MD
expected? certain cardiac and pulmonary diseases) result FROM UST)
A. Bone marrow hypoplasia from physiologic increases in erythropoietin can
B O2 saturation less than 85% be seen in this case.
C Presence of a Philadelphia chromosome on
cytogenetic testing
D Very low to absent erythropoietin titer
E Many Pelger–Huet cells on peripheral blood
smear
47 Drug that targets CD2- antigen on surface of mature Trastuzumab - HER2/neu extracellular doman; GEORGE MICHAEL MIDTERM 2
B ceslls used to treat Non-Hodgkin's lymphoma Bevacizumab - VEGF; Alemtuzumab - CD52 SOSUAN (TOP 5 - EXAM -
A. Bevacizumab antigen on the surface of lymphocytes, monocytes AUG 2015 MED MARCH 2016
B. Rituximab and macrophages; Cetuximab - EGFR BOARDS;
C. Trastuzumab TOPNOTCH MD
D. Alemtuzumab FROM UST)
E. Cetuximab

48 Most common type of head and neck cancer 95% of head and neck cancers are squamous cell GEORGE MICHAEL MIDTERM 2
A. Squamous cell carcinoma carcinoma. SOSUAN (TOP 5 - EXAM -
B. Adenocarcinoma AUG 2015 MED MARCH 2016
C. Adenosquamous carcinoma BOARDS;
D. Lymphoma TOPNOTCH MD
E. Lipoma FROM UST)

49 What would be the best way to screen an SLE patient anti-DsDNA is the best wasy to screen for possible GEORGE MICHAEL MIDTERM 2
for possible renal involvement? SLE nephritis. SOSUAN (TOP 5 - EXAM -
A. Kidney biopsy AUG 2015 MED MARCH 2016
B. Ultrasound BOARDS;
C. Urinalysis TOPNOTCH MD
D. ANA FROM UST)
E. Anti-dsDNA

50 Cerebral blood vessel vasoconstriction is caused by GEORGE MICHAEL MIDTERM 2


which of the following? SOSUAN (TOP 5 - EXAM -
A. Hypercarbia AUG 2015 MED MARCH 2016
B. Hyperoxygenation BOARDS;
C. Hypoxia TOPNOTCH MD
D. Hypocarbia FROM UST)
E. NOTA

51 Which of the following is not a sign of advanced GEORGE MICHAEL MIDTERM 2


COPD? SOSUAN (TOP 5 - EXAM -
A. Clubbing AUG 2015 MED MARCH 2016
B. Right heart failure BOARDS;
C. Systemic wasting TOPNOTCH MD
D. Weight loss FROM UST)
E. Paradoxical inward movement of ribcage on
inspiration
52 Risk factors for testicular cancer include, except Risk factors include hypospadias, a cryptorchid GEORGE MICHAEL MIDTERM 2
A. Hypospadias testicle, abnormal testicular development, SOSUAN (TOP 5 - EXAM -
B. Cryptorchid testicle Klinefelter’s syndrome, and a prior history of AUG 2015 MED MARCH 2016
C. Abnormal testicular development testicular cancer. Familial testicular germ cell BOARDS;
D. Klinefelter's syndrome tumors have an autosomal recessive inheritance, TOPNOTCH MD
E. NOTA with 4- to 10-fold increased risk in first-degree FROM UST)
relatives. There may be an association between
testicular germ cell tumors and organochlorine
compounds.
53 Most common presentation of Renal Cell Carcinoma Clinical features GEORGE MICHAEL MIDTERM 2
A. Hematuria a. Hematuria (40%–70% of patients) SOSUAN (TOP 5 - EXAM -
B. Abdominal mass with flank pain b. Abdominal mass with fl ank pain (20%–40% of AUG 2015 MED MARCH 2016
C. Weight loss patients) BOARDS;
D. Fever and malasise c. Weight loss (30% of patients) TOPNOTCH MD
E. Paraneoplastic syndromes d. Fever, malaise, night sweats, or anemia (15%– FROM UST)
30% of patients)
e. Paraneoplastic syndromes, including
hypercalcemia, polycythemia, hyponatremia, or
hypertension

TOPNOTCH MEDICAL BOARD PREP INTERNAL MEDICINE SUPEREXAM Page 10 of 95


For inquiries visit www.topnotchboardprep.com.ph or email us at topnotchmedicalboardprep@gmail.com
TOPNOTCH MEDICAL BOARD PREP INTERNAL MEDICINE SUPEREXAM
For inquiries visit www.topnotchboardprep.com.ph or email us at topnotchmedicalboardprep@gmail.com
Item QUESTION EXPLANATION AUTHOR TOPNOTCH
# EXAM
54 Most common etiologic agent of bacterial infection GEORGE MICHAEL MIDTERM 2
in burned patients. SOSUAN (TOP 5 - EXAM -
A. Steptococcus pneumoniae AUG 2015 MED MARCH 2016
B. Klebsiella pneumoniae BOARDS;
C. Mycoplasma pneumoniae TOPNOTCH MD
D. Pseudomonas aeruginosa FROM UST)
E. AOTA

55 Incubation period of Dengue virus GEORGE MICHAEL MIDTERM 2


A. 1-5 days SOSUAN (TOP 5 - EXAM -
B. 2-7 days AUG 2015 MED MARCH 2016
C. 5-8 days BOARDS;
D. 7-10 days TOPNOTCH MD
E. NOTA FROM UST)

56 DOC for pulmonary Cryptococcus in DOC for pulmonary Cryptococcus in GEORGE MICHAEL MIDTERM 2
immunocompetent individual immunocompetent individual – fluconazole (200- SOSUAN (TOP 5 - EXAM -
A. Fluconazole 400 mg/d for 3-6 months) AUG 2015 MED MARCH 2016
B. Ketoconazole BOARDS;
C. Itraconazole TOPNOTCH MD
D. Voriconazole FROM UST)
E. NOTA

57 Mechanism of autoimmunity exhibited by antibodies GEORGE MICHAEL MIDTERM 2


to M protein of Streptococcus crossreacting with SOSUAN (TOP 5 - EXAM -
myosin AUG 2015 MED MARCH 2016
A. Sequestered antigen BOARDS;
B. Immune complex deposition TOPNOTCH MD
C. Molecular mimicry FROM UST)
D. Both A and C
E. AOTA

58 Treatment period of osteomyelitis GEORGE MICHAEL MIDTERM 2


A. 1-2 weeks SOSUAN (TOP 5 - EXAM -
B. 2-4 weeks AUG 2015 MED MARCH 2016
C. 4-6 weeks BOARDS;
D. 6-8 weeks TOPNOTCH MD
E. NOTA FROM UST)

59 Part of spine most commonyly involved in GEORGE MICHAEL MIDTERM 2


rheumatoid arthritis SOSUAN (TOP 5 - EXAM -
A. Atlanto-axial AUG 2015 MED MARCH 2016
B. Thoracic BOARDS;
C. Lumbar TOPNOTCH MD
D. Sacral FROM UST)
E. AOTA

60 The normal anti-reflux mechanism of the esophagus Three normal antireflux mechanisms of the GEORGE MICHAEL MIDTERM 2
includes the following esophagus SOSUAN (TOP 5 - EXAM -
A. Lower esophageal sphincte 1. Lower esophageal sphincter AUG 2015 MED MARCH 2016
B. Crural diaphragm 2. Crural diaphragm BOARDS;
C. Anatomical location of gastroesophageal 3. Anatomical location of gastroesophageal TOPNOTCH MD
junction below the diaphragmatic hiatus junction below the diaphragmatic hiatus FROM UST)
D. Both A and C
E. AOTA

61 What is the definition of clinically important weight Clinically important weight loss is defined as the JAN CHRISTIAN MIDTERM 3
loss? loss of FELICIANO (TOP 2 EXAM -
A. Loss of 5 lbs over a pd of 6-12 mos 10 pounds (4.5 kg) or >5% of one’s body weight - AUG 2015 MED MARCH 2016
B. Loss of 10 lbs over a pd of 6-12 mos over a period of 6–12 BOARDS;
C. Loss of >5% of BW over a pd of 3 mos months. All are lifted verbatim from Harrison's TOPNOTCH MD
D. Loss of >10% of BW over a pd of 3 mos Princieple of 19th edition same as the board FROM UST)
E. Both B and C exams.

62 All of the ff statements regarding principles of As stated vebatim in Harrison's 19th ed, pg 468- JAN CHRISTIAN MIDTERM 3
cancer diagnosis is true EXCEPT? 470, The most widely used system of staging is the FELICIANO (TOP 2 EXAM -
A. The diagnosis of cancer relies most heavily on TNM (tumor, node, metastasis) system for most - AUG 2015 MED MARCH 2016
invasive tissue biopsy and should never be made cancers. Other anatomic staging systems are used BOARDS;
without obtaining tissues for some tumors, e.g., the Dukes classification for TOPNOTCH MD
B. No noninvasive diagnostic test is sufficient to colorectal cancers, the International Federation of FROM UST)
define a disease process as cancer Gynecologists and Obstetricians classification for
C. The first priority in patient management after the gynecologic cancers, and the Ann Arbor
diagnosis of cancer is established and shared with classification for Hodgkin’s disease.
the patient is to determine the extent of disease.
D. The curability of a tumor usually is inversely
proportional to the tumor burden
E. The most widely used system of staging for
cancers such as Lung Cancer, Breast Cancer,
Hodgkin's disease and Colorectal cancer is the TNM
(tumor, node, metastasis) system

TOPNOTCH MEDICAL BOARD PREP INTERNAL MEDICINE SUPEREXAM Page 11 of 95


For inquiries visit www.topnotchboardprep.com.ph or email us at topnotchmedicalboardprep@gmail.com
TOPNOTCH MEDICAL BOARD PREP INTERNAL MEDICINE SUPEREXAM
For inquiries visit www.topnotchboardprep.com.ph or email us at topnotchmedicalboardprep@gmail.com
Item QUESTION EXPLANATION AUTHOR TOPNOTCH
# EXAM
63 What is the most common cause of right sided heart The most common cause of right sided HF is a JAN CHRISTIAN MIDTERM 3
failure? concomitant left sided HF. FELICIANO (TOP 2 EXAM -
A. Left sided heart failure - AUG 2015 MED MARCH 2016
B. COPD BOARDS;
C. Pulmonary hypertension TOPNOTCH MD
D. Coronary artery disease FROM UST)
E. Portal hypertension

64 A 75 year old male went to you complaining of The Framingham criteria for the diagnosis of heart JAN CHRISTIAN MIDTERM 3
bipedal edema, paroxysmal nocturnal dyspnea and failure consists of the concurrent presence of FELICIANO (TOP 2 EXAM -
exertional dyspnea. Physical exam showed grade 2 either 2 major criteria or 1 major and 2 minor - AUG 2015 MED MARCH 2016
bipedal edema, tachycardia and hepatomegaly. criteria. Major criteria include the BOARDS;
Based on the Framingham criteria, which salient following:Paroxysmal nocturnal dyspne, Neck vein TOPNOTCH MD
feature is a major criterion for the diagnosis of heart distention, Rales. Cardiomegaly, Acute pulmonary FROM UST)
failure? edema, Hepatojugular reflux, S3 gallop, Central
A. Bipedal edema venous pressure greater than 16 cm water. Minor
B. Paroxysmal nocturnal dyspnea criteria are as follows: Nocturnal cough, Dyspnea
C. Tachycardia on ordinary exertion, A decrease in vital capacity
D. Hepatomegaly by one third the maximal value recorded, Pleural
E. Exertional dyspnea effusion, Hepatomegaly, Tachycardia (rate of 120
bpm), Bilateral ankle edema. Weight loss of 4.5 kg
in 5 days in response to treatment is both a major
and minor criteria
65 In evaluaitng a patient with thrombocytopenia, a key key step is to review the peripheral blood smear JAN CHRISTIAN MIDTERM 3
step is to review the peripheral blood smear and to and to first rule out “pseudothrombocytopenia,” FELICIANO (TOP 2 EXAM -
first rule out what entity? particularly in a patient without an apparent cause - AUG 2015 MED MARCH 2016
A. Schistocytes for the thrombocytopenia. BOARDS;
B. Hypocalcemia Pseudothrombocytopenia is an in vitro artifact TOPNOTCH MD
C. Pseudothrombocytopenia resulting FROM UST)
D. Coagulation defects from platelet agglutination via antibodies (usually
E. Von Willebrand's disease IgG, but also IgM and IgA) when the calcium
content is decreased by blood collection in
ethylenediamine tetraacetic (EDTA) (the
anticoagulant present in tubes [purple top] used
to collect blood for complete blood counts.
66 Which of the ff is an atypical organism that causes Atypical organism that cause pneumoniae include JAN CHRISTIAN MIDTERM 3
pneumonia? Mycoplasma pneumoniae, Chlamydia pneumoniae FELICIANO (TOP 2 EXAM -
A. Haemophilus influenzae and Legionella pneumophilia - AUG 2015 MED MARCH 2016
B. Anaerobes BOARDS;
C. Legionella pneumophilia TOPNOTCH MD
D. Klebsiella pneumoniae FROM UST)
E. Pseudomonas aeruginosa

67 Which of the ff statements regarding urinary tract The most common manifestation of UTI is acute JAN CHRISTIAN MIDTERM 3
infection is correct? cystitis. Asymptomatic bacteruria is pyuria that FELICIANO (TOP 2 EXAM -
A. The most common manifestation of UTI is occurs in the absence of symptoms and usually - AUG 2015 MED MARCH 2016
asymptmatic bacteriuria does not require treatment. During the neonatal BOARDS;
B. Asymptomatic bacteruria is pyuria that occurs in period, the incidence of UTI is slightly higher in TOPNOTCH MD
the absence of symptoms and usually requires males than in females bec males infants more FROM UST)
treatment. commonly have congenital urinary tract
C. Generally all women will acquire atleast one UTI anomalies. Only 50-80% of women will acquire
during their lifetime- uncomplicated cystitis in msot atleast one UTI during their lifetime
cases
D. During the neonatal period, incidence of UTI is
higher in males than females.
E. None of the above
68 All of the ff factors are criteria to classify a patient as As stated vebatim in Harrison's 19th ed, pg 803, JAN CHRISTIAN MIDTERM 3
having health care associated penumonia EXCEPT? the clinical conditions assoc with HCAP include FELICIANO (TOP 2 EXAM -
A. Hospitalization for 2 or more days Hospitalization for 2 or more days within the - AUG 2015 MED MARCH 2016
B. Hospitalization within the prior 4 months prior 3 months, Antibiotic therapy in the BOARDS;
C. Antibiotic therapy in the preceding 3 mos preceding 3 mos, Chronic dialysis, Home wound TOPNOTCH MD
D. Chronic dialysis care, Nursing home residence, Home infusion FROM UST)
E. Home wound care therapy and a family member with MDR infection

69 A newly diagnosed diabetic patient came to the The px is suffering from hypoglycemic episode. Of JAN CHRISTIAN MIDTERM 3
emergency room because of a a 1 week history of the all class of antidiabetic drugs mentioned, only FELICIANO (TOP 2 EXAM -
freuqnt light headedness, cold sweats and sulfonoylureas, being an insulin secretagouge, can - AUG 2015 MED MARCH 2016
palpitations occuring 2 hours after taking an cause hypoglycemia BOARDS;
unknown medication given to him by his neghbor TOPNOTCH MD
for diabetes. Which is the most likely medication FROM UST)
given to the patient?
A. Sulfonylurea
B. Biguanide
C. Alpha glucosidase inhibitor
D. Thiazolidinedione
E. PPD4 inhibitor

70 The only absolute contraindication to the use of The only absolute contraindication to the use of JAN CHRISTIAN MIDTERM 3
nitrates in patients with UA/NSTEMI? nitrates in patients with UA/NSTEMI is use of FELICIANO (TOP 2 EXAM -
A. Use of sildenafil for the prior 3-5 days sildenafil for the prior 24-48 hrs and hypotension. - AUG 2015 MED MARCH 2016
B. Bradycardia BOARDS;
C. Intractable headaches TOPNOTCH MD
D. Concomitant pulmonary edema FROM UST)
E. Hypotension

TOPNOTCH MEDICAL BOARD PREP INTERNAL MEDICINE SUPEREXAM Page 12 of 95


For inquiries visit www.topnotchboardprep.com.ph or email us at topnotchmedicalboardprep@gmail.com
TOPNOTCH MEDICAL BOARD PREP INTERNAL MEDICINE SUPEREXAM
For inquiries visit www.topnotchboardprep.com.ph or email us at topnotchmedicalboardprep@gmail.com
Item QUESTION EXPLANATION AUTHOR TOPNOTCH
# EXAM
71 Several stimuli trigger airway narrowing, wheezing, As stated vebatim in Harrison's 19th ed, pg 1671, JAN CHRISTIAN MIDTERM 3
and dyspnea in asthmatic patients. What is the most the most common allergen to trigger asthma is FELICIANO (TOP 2 EXAM -
common asthma trigger? Dermatophagoides commonly known as house - AUG 2015 MED MARCH 2016
A. Atopy dust mites. BOARDS;
B. Dermatophagoides species TOPNOTCH MD
C. Viral infections FROM UST)
D. Air pollution
E. Pollen

72 One of the following is NOT a diagnostic crtieria for Clinical manifestions or resp. distress is not part of JAN CHRISTIAN MIDTERM 3
acute respiratory distress syndrome. the criteria. All others mentioned are the 4 criteria FELICIANO (TOP 2 EXAM -
A. Acute in onset of ARDS. Take not the new Berlin crtieria classifies - AUG 2015 MED MARCH 2016
B. Clinical respiratory distress ARDS into mild (P/F ratio of less than 300) BOARDS;
C. PaO2/FiO2 ratio of less than 300 moderate (P/F ratio of less than 200) and severe TOPNOTCH MD
D. Bilatreral alveolar or interstitial infiltrates in (P/F ratio of less than 100) FROM UST)
CXR
E. Absence of left atrial hypertension

73 This mode of mechanical ventilation wherein the Intermitent mandatory ventilation, wherein most JAN CHRISTIAN MIDTERM 3
operator sets the number of mandatory breaths of frequently used is SIMV, operator sets the number FELICIANO (TOP 2 EXAM -
fixed volume to be delivered by the ventilator but of mandatory breaths of fixed volume to be - AUG 2015 MED MARCH 2016
between those breaths the patient can breath delivered by the ventilator but between those BOARDS;
spontaneously? breaths the patient can breath. In this mode TOPNOTCH MD
A. Assist control ventilation mandatory breaths are deivered in synchrony FROM UST)
B. Pressure support ventilation with the patient's inspiratory efforts at a
C. Intermitent mandatory ventilation frequency determined by the operator.
D. Continuous positive airway pressure
E. Pressure control ventilation
74 All of the ff clinical entities leads primarily to Sepsis although theretically can lead to decreased JAN CHRISTIAN MIDTERM 3
prerenal kidney injury EXCEPT? effective circulating volume is more commonly FELICIANO (TOP 2 EXAM -
A. Sepsis associated with intrinsic kindey injury. Spesis - AUG 2015 MED MARCH 2016
B. Hypovolemia leads to endothelial damage which results in BOARDS;
C. Cirrhosis microvascular thrombosis, activation of reactive TOPNOTCH MD
D. Use of NSAIDS oxygen species and leukocyte adhesions and FROM UST)
E. Use of cyclosporine migration all of which may injure renal tubular
cells.

75 A normocytic normochromic anemia is almost A normocytic normochromic anemia is observed JAN CHRISTIAN MIDTERM 3
universal in what stage of chronic kidney disease? as early as stage 3 CKD and is almost universal by FELICIANO (TOP 2 EXAM -
A. Stage 1 stage 4. - AUG 2015 MED MARCH 2016
B. Stage 2 BOARDS;
C. Stage 3 TOPNOTCH MD
D. Stage 4 FROM UST)
E. Stage 5

76 What is the strongest risk factor for cardiovascular As stated vebatim in Harrison's 19th ed, pg 1817, JAN CHRISTIAN MIDTERM 3
morbidity and mortality in a CKD patient? left ventricular hypertrophy and dilated FELICIANO (TOP 2 EXAM -
A. Heart failure cardiomyopathy are among the strongest risk - AUG 2015 MED MARCH 2016
B. Hypertension factors for cardiovascular mortality and morbidity BOARDS;
C. Ischemic heart disease in patients with CKD and are thought to be related TOPNOTCH MD
D. Left ventricular hypertophy rpimariliy but not xclusively to prolonged FROM UST)
E. coronary artery disease hypertension and ECFV overload.

77 All of the statements regarding peptic ulcer disease Helicobacter pylori (Odds ratio of 3.7) and NSAID JAN CHRISTIAN MIDTERM 3
is correct EXCEPT? abuse (OR=3.3) are the most common risk factors FELICIANO (TOP 2 EXAM -
A. Ulcers are defined as breaks in the mucosal for PUD not smoking (OR 0f 1.55). All other - AUG 2015 MED MARCH 2016
surface >5mm in size with depth to the submucosal statements are correct. BOARDS;
layer TOPNOTCH MD
B. Helicobacter pylori and smoking are the most FROM UST)
common risk factors for PUD
C. Coronary heart diease is a stronger risk factor for
PUD than diabetes
D. Gastric ulcers tend to occur later in life than
duodenal ulcers
E. Duodenal ulcers occurs most often in the first
portion of the duodenum with 90% located within
3cm of the pylorus.
78 When patients present with ascities for the first The most important criteria in determining the JAN CHRISTIAN MIDTERM 3
time, it is recommended that a diagnostic etiology of the asictic fluid is through the use of FELICIANO (TOP 2 EXAM -
paracentesis be perfromed to characterize the fluid. the SAAG ratio (Serum ascites to albumin - AUG 2015 MED MARCH 2016
What is the most useful constituent of ascitic fluid to gradient). BOARDS;
aid in determining etiology of fluid? TOPNOTCH MD
A. Cytology FROM UST)
B. Fluid LDH
C. Fluid albumin
D. Gram stain and culture
E. Cell count and differential count

TOPNOTCH MEDICAL BOARD PREP INTERNAL MEDICINE SUPEREXAM Page 13 of 95


For inquiries visit www.topnotchboardprep.com.ph or email us at topnotchmedicalboardprep@gmail.com
TOPNOTCH MEDICAL BOARD PREP INTERNAL MEDICINE SUPEREXAM
For inquiries visit www.topnotchboardprep.com.ph or email us at topnotchmedicalboardprep@gmail.com
Item QUESTION EXPLANATION AUTHOR TOPNOTCH
# EXAM
79 This autoantibody in SLE best correlates with High titers of AntoidsDNA are specfic for SLE and JAN CHRISTIAN MIDTERM 3
disease activity, nephritis and vasculits? in some patients correlate well with disease FELICIANO (TOP 2 EXAM -
A. ANA activity, nephritis and vasculitis - AUG 2015 MED MARCH 2016
B. Anti-Smith BOARDS;
C. Anti Ro and Anti La TOPNOTCH MD
D. Anti histone FROM UST)
E. Anti dsDNA

80 A patient with cushinoid features who tested A person who tested positive for a low dose DSM JAN CHRISTIAN MIDTERM 3
negative with a low dose dexametheasone but tested postive for a high dose DSM has FELICIANO (TOP 2 EXAM -
suppresion test but tested positive with a high DSM Cushing's disease/pituiatry adenoma. A person - AUG 2015 MED MARCH 2016
test most likey has what form of Cushing's with ectopic ACTH syndrome and adrenal sources BOARDS;
syndrome? of cortisol will test negative in a high dose DSM TOPNOTCH MD
A. Cushing's disease test. FROM UST)
B. Exogenous steroid use
C. Ectopic ACTH syndrome
D. Adrenal hyperplasia
E. MEN syndrome
81 Bruce Wayne’s parents were shot outside the when more than 30% of the blood volume is lost ANDREW TIU (TOP FINAL EXAM -
theater by a thief. His dad lost a lot of blood in the suddenly, patients are unable to compensate with 1 - AUG 2015 MED MARCH 2016
crime scene. His HR = 120, RR = 32, and orthostatic the usual mechanisms of vascular contraction and BOARDS;
hypotension. How much of the blood volume is lost changes in regional blood flow. The patient prefers TOPNOTCH MD
by Bruce Wayne’s father? to remain supine and will show postural FROM CIM)
a. 5 - 10% hypotension and tachycardia. If >40%, signs of
b. 10 - 15% hypovolemic shock appear. If 10 - 15%, signs of
c. 15 - 30% vascular instability appear.
d. 30 - 40% Harrison’s Internal Medicine 18th edition p.449
e. 40 - 50%
82 2. Bruce Wayne saw his dad to have pale palmar Harrison’s Internal Medicine 18th edition p.449 ANDREW TIU (TOP FINAL EXAM -
creases. If CBC was taken, what would be the most 1 - AUG 2015 MED MARCH 2016
likely hemoglobin level? BOARDS;
a. 11g/dl TOPNOTCH MD
b. 10g/dl FROM CIM)
c. 9g/dl
d. 8g/dl
e. 7g/dl
83 3. Which of the following is true of the malaria B - 10 - 30,000 merozoites ANDREW TIU (TOP FINAL EXAM -
transmission cycle? C - gametocyte - zygote - ookinete 1 - AUG 2015 MED MARCH 2016
a. sporozoites from salivary gland of mosquito D - they become trophozoites in the RBC BOARDS;
invade the hepatocytes and begin sexual Harrison’s Internal Medicine 18th edition p.1688 TOPNOTCH MD
reproduction FROM CIM)
b. merogony produces 1,000-2,000 merozoites
c. Gametocytes are taken by the mosquito and then
form into a ookinete then a zygote
d. after entry into the bloodstream, merozoites
rapidly invade erythrocytes and become sporozoites
e. none of the above
84 4. Which of the following does an S3 signify? Harrison’s Internal Medicine 18th edition p.1827 ANDREW TIU (TOP FINAL EXAM -
a. aortic stenosis 1 - AUG 2015 MED MARCH 2016
b. aortic regurgitation BOARDS;
c. mitral stenosis TOPNOTCH MD
d. heart failure FROM CIM)
e. pulmonic stenosis

85 5. MJP, a 25 year old female, diagnosed with Crohn’s patient showed signs of iron deficiency anemia ANDREW TIU (TOP FINAL EXAM -
Disease came in for complaints of pallor, easy and hypocalcemia in which both are absorbed in 1 - AUG 2015 MED MARCH 2016
fatigability, and carpopedal spasm. Upon physical the duodenum BOARDS;
examination, (+) Chvostek’s sign and Trousseau’s TOPNOTCH MD
sign. Which of the following organs is most likely FROM CIM)
involved?
a. duodenum
b. jejunum
c. ileum
d. cecum
e. rectum
86 6. Which of the following would present with an hepatocellular CA and hepatitis also presents with ANDREW TIU (TOP FINAL EXAM -
elevated AFP? elevated AFP. 1 - AUG 2015 MED MARCH 2016
a. prostate cancer Harrison’s Internal Medicine 18th edition p.652 BOARDS;
b. cirrhosis TOPNOTCH MD
c. ovarian germ cell tumor FROM CIM)
d. both A and B
e. both B and C

87 7. Which of the following forms of shock would Harrison’s Internal Medicine 18th edition p.2219 ANDREW TIU (TOP FINAL EXAM -
present with an increase in cardiac output and 1 - AUG 2015 MED MARCH 2016
venous oxygen saturation? BOARDS;
a. hypovolemic TOPNOTCH MD
b. cardiogenic FROM CIM)
c. septic
d. neurogenic
e. hypoadrenal

TOPNOTCH MEDICAL BOARD PREP INTERNAL MEDICINE SUPEREXAM Page 14 of 95


For inquiries visit www.topnotchboardprep.com.ph or email us at topnotchmedicalboardprep@gmail.com
TOPNOTCH MEDICAL BOARD PREP INTERNAL MEDICINE SUPEREXAM
For inquiries visit www.topnotchboardprep.com.ph or email us at topnotchmedicalboardprep@gmail.com
Item QUESTION EXPLANATION AUTHOR TOPNOTCH
# EXAM
88 8. Which of the following is not a clinical feature of This refers to an extra fold of skin beneath the ANDREW TIU (TOP FINAL EXAM -
atopic dermatitis? lower eyelid. 1 - AUG 2015 MED MARCH 2016
a. lesions resemble eczematous dermatitis Harrison’s Internal Medicine 18th edition p.395 BOARDS;
b. course usually lasts more than 6 weeks TOPNOTCH MD
c. Dennie- Morgan folds refer to hyperpigmentation FROM CIM)
on flexural sites
d. lichenification of skin
e. exacerbations and remissions
89 9. Which of the following factors would increase the other factors include enzyme induction, smoking, ANDREW TIU (TOP FINAL EXAM -
clearance of theophylline? high protein, low carbohydrate diet, and 1 - AUG 2015 MED MARCH 2016
a. liver disease childhood. BOARDS;
b. barbecued meat Harrison’s Internal Medicine 18th edition p.2111 TOPNOTCH MD
c. high carbohydrate diet FROM CIM)
d. old age
e. coadministration of zafirlukast

90 10. Patient presented to the emergency room with Harrison’s Internal Medicine 18th edition p.2178 ANDREW TIU (TOP FINAL EXAM -
dyspnea. On physical examination, dullness over left 1 - AUG 2015 MED MARCH 2016
lung field with decreased tactile fremitus was noted. BOARDS;
Thoracentesis was done over the site and contents TOPNOTCH MD
were found to be exudative. Glucose was 40mg/dl. FROM CIM)
Which of the following is likely a differential?
a. esophageal rupture
b. bacterial infection
c. rheumatoid pleuritis
d. both a and b
e. both b and c
91 11. Which of the following is the appropriate A - every year; C - every 5 years; E - every 3 years ANDREW TIU (TOP FINAL EXAM -
screening strategies for average risk patients? Harrison’s Internal Medicine 18th edition p.2423 1 - AUG 2015 MED MARCH 2016
a. FOBT every 3 years BOARDS;
b. colonoscopy every 10 years TOPNOTCH MD
c. flexible sigmoidoscopy every 10 years FROM CIM)
d. colonoscopy every 5 years
e. fecal DNA testing every 5 years

92 12. which of the following are absolute age >70 years old is a relative contraindication ANDREW TIU (TOP FINAL EXAM -
contraindications of liver transplantation except? Harrison’s Internal Medicine 18th edition p.2608 1 - AUG 2015 MED MARCH 2016
a. untreated sepsis BOARDS;
b. alcohol abuse TOPNOTCH MD
c. 72 years old FROM CIM)
d. congestive heart failure
e. breast cancer stage IV

93 13. Which of the following initial tests must be TST is done with TNF alpha inhibitors, abatacept, ANDREW TIU (TOP FINAL EXAM -
performed prior to starting etanercept in patients anakinra, and tocilizumab 1 - AUG 2015 MED MARCH 2016
with rheumatoid arthritis? Harrison’s Internal Medicine 18th edition p.2748 BOARDS;
a. CBC TOPNOTCH MD
b. AST/ALT FROM CIM)
c. eye examination
d. PPD skin test
e.creatinine
94 14. Which of the following clinical characteristics B - HLA - DR3 and DRw52; C - negative serologic ANDREW TIU (TOP FINAL EXAM -
would most likely point to Sjogren’s syndrome? test for HIV; D - lymphoid infiltrates of salivary 1 - AUG 2015 MED MARCH 2016
a. joint deformities such as ulnar deviation and glands by CD4; E - predominant in middle aged BOARDS;
boutoniere deformity women TOPNOTCH MD
b. HLA - DR5 Harrison’s Internal Medicine 18th edition p.2772 FROM CIM)
c. positive serologic test for HIV
d. granulomas in salivary glands
e. predominant in young males
95 15. Which of the following conditions are not hypomagnesemia; others include ANDREW TIU (TOP FINAL EXAM -
associated with calcium pyrophosphate dihydrate hypophosphatasia, postmeniscectomy, gitelman’s 1 - AUG 2015 MED MARCH 2016
disease? syndrome BOARDS;
a. gout Harrison’s Internal Medicine 18th edition p.2839 TOPNOTCH MD
b. hypermagnesemia FROM CIM)
c. aging
d. hemochromatosis
e. hyperparathyroidism
96 16. Which of the following is not a risk factor for this is a risk factor for follicular thyroid carcinoma ANDREW TIU (TOP FINAL EXAM -
papillary thyroid carcinoma in patients presenting Harrison’s Internal Medicine 18th edition p.2934 1 - AUG 2015 MED MARCH 2016
with a thyroid nodule? BOARDS;
a. male TOPNOTCH MD
b. 50 years old FROM CIM)
c. hoarse voice
d. iodine deficiency
e. suspected lymph node involvement
97 17. Which of the following medications would have a B&E - have no effects on renin, aldosterone, and ANDREW TIU (TOP FINAL EXAM -
decrease effect of renin, increase effect of ARR; C&D - have opposite effects from beta 1 - AUG 2015 MED MARCH 2016
aldosterone and increase net effect of aldosterone- blockers BOARDS;
renin-ration (ARR)? Harrison’s Internal Medicine 18th edition p.2951 TOPNOTCH MD
a. beta blockers FROM CIM)
b. alpha blockers
c. ACE inhibitors
d. ARBs
e. CCB

TOPNOTCH MEDICAL BOARD PREP INTERNAL MEDICINE SUPEREXAM Page 15 of 95


For inquiries visit www.topnotchboardprep.com.ph or email us at topnotchmedicalboardprep@gmail.com
TOPNOTCH MEDICAL BOARD PREP INTERNAL MEDICINE SUPEREXAM
For inquiries visit www.topnotchboardprep.com.ph or email us at topnotchmedicalboardprep@gmail.com
Item QUESTION EXPLANATION AUTHOR TOPNOTCH
# EXAM
98 18. Which of the following drugs are NOT associated others include steroids, cyclosporine, cytotoxic ANDREW TIU (TOP FINAL EXAM -
with an increased risk of generalized osteoporosis in drugs, anticonvulsants, aromatase inhibitors, 1 - AUG 2015 MED MARCH 2016
adults? GnRH agonists, and heparin BOARDS;
a. aluminum Harrison’s Internal Medicine 18th edition p.3124 TOPNOTCH MD
b. lithium FROM CIM)
c. alcohol
d. levothyroxine
e. none of the above
99 19. Patient presented with pain in the lips, gums, Harrison’s Internal Medicine 18th edition p.3361 ANDREW TIU (TOP FINAL EXAM -
cheek for a few seconds which recurs frequently at 1 - AUG 2015 MED MARCH 2016
day and night. Pain occurs usually when chewing BOARDS;
food. On neurologic examination, there is no sensory TOPNOTCH MD
loss. Which of the following is the drug of choice for FROM CIM)
this condition?
a. phenytoin
b. carbamazepine
c. lamotrigine
d. valproic acid
e. baclofen
100 20. Among the inflammatory myopathies, which of Harrison’s Internal Medicine 18th edition p.3510 ANDREW TIU (TOP FINAL EXAM -
the following is associated with visceral malignancy? 1 - AUG 2015 MED MARCH 2016
a. polymyositis BOARDS;
b. dermatomyositis TOPNOTCH MD
c. inclusion body myositis FROM CIM)
d. myofasciitis
e. polymyalgia rheumatica

101 35 yr old woman with chronic renal failure and Page 213 of IM Platinum. Elevated PTH occuring in ANGELA PAULINE DIAGNOSTIC
undergoing hemodialysis 2-3x a day develops CKD patients is secondary hyperparahyroidism. P. CALIMAG- EXAM - AUG
tumoral calcinosis. Her PTH is markedly elevated LOYOLA (TOP 8 - 2015
while her ionized Ca is normal. The most probable FEB 2015 MED
diagnosis is: BOARDS;
A. Primary hyperparathyroidism TOPNOTCH MD
B. Secondary hyperparathyroidism FROM UST)
C. Tertiary hyperparathyroidism
D. Parathyroid carcinoma
E. Pseudohyperparathyroidism
102 Clinical features present in both MEN 2A and Page 208 of IM Platinum. Men 2A- Medullary ANGELA PAULINE DIAGNOSTIC
MEN2B syndromes: thyroid carcinoma, pheochromocytoma, P. CALIMAG- EXAM - AUG
A. Pituitary adenoma parathyroid hyperplasia. Men 2B- Medullary LOYOLA (TOP 8 - 2015
B. Parathyroid hyperplasia thyroid carcinoma, Pheochromocytoma, mucosal FEB 2015 MED
C. Pheochromocytoma and gastrointestinal neuromas, marfanoid BOARDS;
D. Marfanoid habitus features. Men1- Parathyroid TOPNOTCH MD
E. Pancreatic islet cell hyperplasia hyperplasia,pancreatic islet cell FROM UST)
hyperplasia/adenoma/carcinoma and pituitary
hyperplasia/adenoma.
103 A 55 y/o male presents at the OPD with cough, mild Page 112 of IM Platinum. Pneumonia has four ANGELA PAULINE DIAGNOSTIC
dyspnea, and fever of 37.8OC. Symptoms have been stages, namely consolidation, red hepatization, P. CALIMAG- EXAM - AUG
present for the past 3 days, and when over-the- grey hepatization and resolution. LOYOLA (TOP 8 - 2015
counter cold medications were having no effect, he Consolidation/Edema-Occurs in the first 24 FEB 2015 MED
decided to seek medical attention. On PE, BP hours; Cellular exudates containing neutrophils, BOARDS;
120/70, PR 96 and regular, and RR 23. Lung lymphocytes and fibrin replaces the alveolar air. TOPNOTCH MD
examination revealed the presence of right lower Capillaries in the surrounding alveolar walls FROM UST)
lung crackles with decreased breath sounds in the become congested. The infections spreads to the
RLLF. Laboratory studies revealed WBC of 15.6 with hilum and pleura fairly rapidly Pleurisy occurs.
a left shift, sodium of 140, potassium of 4.5, BUN of Marked by coughing and deep breathing.
22 and creatinine of 1.0. Chest x-ray revealed a right Red Hepatization- Occurs in the 2-3 days after
lower lobe infiltrate. The patient is in which phase of consolidation; At this point the consistency of the
CAP if there is predominance of neurophils, lungs resembles that of the liver. The lungs
abundant fibrin deposition and cessation of become hypeaemic. Alveolar capillaries are
erythrocyte extravasation?
engorged with blood. Fibrinous exudates fill the
A. Edema alveoli. This stage is "characterized by the
B. Red hepatization presence of many erythrocytes, neutrophils,
C. Gray hepatization desquamated epithelial cells, and fibrin within the
D. Resolution alveoli" Grey
E. Consolidation Hepatization- Occurs in the 2-3 days after Red
Hepatization. This is an avascular stage. The lung
appears "gray-brown to yellow because of
fibrinopurulent exudates, disintegration of red
cells, and hemosiderin"
The pressure of the exudates in the alveoli causes
compression of the capillaries. "Leukocytes
migrate into the congested alveoli"
Resolution- This stage is characterized by the
"resorption and restoration of the pulmonary
architecture". A large number of macrophages
enter the alveolar spaces. Phagocytosis of the
bacteria-laden leucocytes occurs. "Consolidation
tissue re-aerates and the fluid infiltrate causes
sputum". "Fibrinous inflammation may extend to
and across the pleural space, causing a rub heard
by auscultation, and it may lead to resolution or to
organization and pleural adhesions"

TOPNOTCH MEDICAL BOARD PREP INTERNAL MEDICINE SUPEREXAM Page 16 of 95


For inquiries visit www.topnotchboardprep.com.ph or email us at topnotchmedicalboardprep@gmail.com
TOPNOTCH MEDICAL BOARD PREP INTERNAL MEDICINE SUPEREXAM
For inquiries visit www.topnotchboardprep.com.ph or email us at topnotchmedicalboardprep@gmail.com
Item QUESTION EXPLANATION AUTHOR TOPNOTCH
# EXAM
104 GOLD COPD staging uses four categories of severity Page 109 of IM Platinum. 1: Mild COPD- ANGELA PAULINE DIAGNOSTIC
for COPD, based on the value of FEV1. A patient FEV1/FVC < 70%; FEV1 > or equal to 80% P. CALIMAG- EXAM - AUG
classified under Stage III, has a predicted FEV1 of:
predicted; With or without chronic symptoms LOYOLA (TOP 8 - 2015
A. >90 % (cough, sputum production) 2: Moderate COPD- FEB 2015 MED
B. >80 % FEV1/FVC < 70%; FEV1 between 50 and 80% BOARDS;
C. 60 % predicted; With or without chronic symptoms TOPNOTCH MD
D. 30 % (cough, sputum production) 3: Severe COPD- FROM UST)
E. <20 % FEV1/FVC < 70%; FEV1 between 30 and 50%
predicted; With or without chronic symptoms
(cough, sputum production) 4: Very Severe
COPD- FEV1/FVC < 70%; FEV1 < or equal to 30%
predicted or FEV1 < 50% predicted plus chronic
respiratory failure
105 A 75 y/o male who underwent left nephrectomy for Page 121 of IM Platinum. This patient is probably ANGELA PAULINE DIAGNOSTIC
renal cell carcinoma 6 months ago is admitted to the having a massive PE.Dyspnea is the most common P. CALIMAG- EXAM - AUG
emergency department with acute severe dyspnea symptom and tachypnea is the most frequent sign. LOYOLA (TOP 8 - 2015
and cyanosis. On PE, BP 100/60; PR 120, RR 25, In massive PE like this case, on 2D echo there FEB 2015 MED
Oxygen saturation is 75% while breathing room air would be RV hypokinesis. BOARDS;
and fails to rise under supplemental oxygen. The TOPNOTCH MD
patient is intubated and hooked to mechanically FROM UST)
ventilator with 100% oxygen, which results in
further drop of the arterial saturation to 65%
despite correct positioning of the tube. Chest X-ray
shows clear lungs without infiltrates. Based on the
patient's diagnosis, which of the following is a
possible finding in 2D echo?
A. RV hypokinesis
B. RV hyperkinesis
C. LV hypokinesis
D. LV hyperkinesis
E. No possible 2D echo findings
106 These kidney disorders are characterized by Page 176 of IM Platinum. These are characteristics ANGELA PAULINE DIAGNOSTIC
proteinuria > 3.0 g/day, hypoalbuminemia, of Nephrotic syndrome. A-D are diseases which P. CALIMAG- EXAM - AUG
hyperlipidemia and edema, except: A. present with nephrotic syndrome. Cresenteric GN LOYOLA (TOP 8 - 2015
Minimal change disease is a Nephritic syndrome, presenting with Oliguria, FEB 2015 MED
B. Membranous GN Hematuria, Hypertension and Azotemia. BOARDS;
C. Focal-segment GN TOPNOTCH MD
D. DM nephropathy FROM UST)
E. Cresenteric GN

107 Criteria used in the diagnosis of DM can be either of Page 186 of IM Platinum. Criteria for diagnosis ANGELA PAULINE DIAGNOSTIC
the following, except:
may either be one of the following HbA1C >/= P. CALIMAG- EXAM - AUG
A. HbA1C >/= 7.0% 6.5% , FPG >/= 7.0 mmol/L, OGTT 2 hour plasma LOYOLA (TOP 8 - 2015
B. FPG >/= 7.0 mmol/L glucose >/= 11.1 mmol/L and Random plasma FEB 2015 MED
C. OGTT 2 hour plasma glucose >/= 11.1 mmol/L glucose >/= 11.1 mmol/L + classic symptoms of BOARDS;
D. Random plasma glucose >/= 11.1 mmol/L + hyperglycemia or hyperglycemic crisis TOPNOTCH MD
classic symptoms of hyperglycemia or FROM UST)
hyperglycemic crisis
E. None of the above
108 Which laboratory test is the most sensitive test for Page 221 of IM Platinum. The most sensitive and ANGELA PAULINE DIAGNOSTIC
pheochromocytoma?
less susceptible to false elevations from stress, P. CALIMAG- EXAM - AUG
A. Urinary VMA including venipuncture is the measurement of LOYOLA (TOP 8 - 2015
B. Urinary Metanephrines plasma metanephrine. FEB 2015 MED
C. Urinary Catecholamines BOARDS;
D. Plasma Metanephrines TOPNOTCH MD
E. Plasma VMA FROM UST)

109 The following parameters are components of the Page 233 of IM Platinum. Child Pugh is used to ANGELA PAULINE DIAGNOSTIC
Child Pugh score, except:
assess the prognosis of chronic liver disease, P. CALIMAG- EXAM - AUG
A. Serum bilirubin mainly cirrhosis. Although it was originally used to LOYOLA (TOP 8 - 2015
B. Serum albumin predict mortality during surgery, it is now used to FEB 2015 MED
C. Serum creatinine determine the prognosis, as well as the required BOARDS;
D. Prothrombin time strength of treatment and the necessity of liver TOPNOTCH MD
E. Ascites transplantation. Its components are Serum FROM UST)
bilirubin, serum albumin, prothrombin time,
Ascites and hepatic encephalopathy.
110 A 71 y/o male patient had complained of swallowing Page 261 of IM Platinum. This is a case of "steak ANGELA PAULINE DIAGNOSTIC
difficulty after ingesting meat two days previously. house syndrome" or schatzki ring. This is one of P. CALIMAG- EXAM - AUG
Endoscopy was done and a meat bolus was found 30 the most common cause of intermittent food LOYOLA (TOP 8 - 2015
cm from the incisors, the bolus was removed and a impaction with meat as the typical instigator. FEB 2015 MED
lower esophageal ring at the esophagogastric BOARDS;
junction is seen. Biopsy reported that the tissue has TOPNOTCH MD
normal esophageal squamous epithelium on one FROM UST)
side and gastric columnar epithelium on the distal
side of the membrane. What is the probable
diagnosis?
A. Esophageal webs
B. Schatzki ring
C. Zenker diverticulum
D. Nutcracker esophagus
E. Diffuse esophageal spasm

TOPNOTCH MEDICAL BOARD PREP INTERNAL MEDICINE SUPEREXAM Page 17 of 95


For inquiries visit www.topnotchboardprep.com.ph or email us at topnotchmedicalboardprep@gmail.com
TOPNOTCH MEDICAL BOARD PREP INTERNAL MEDICINE SUPEREXAM
For inquiries visit www.topnotchboardprep.com.ph or email us at topnotchmedicalboardprep@gmail.com
Item QUESTION EXPLANATION AUTHOR TOPNOTCH
# EXAM
111 Hemmorhoidal plexuses which traverse the anal Page 265 of IM Platinum. The three positions of ANGELA PAULINE DIAGNOSTIC
canal are commonly located in the following the main hemmorhoidal cushions are, Left lateral, P. CALIMAG- EXAM - AUG
positions, except: A. Left lateral right anterior and right posterior. LOYOLA (TOP 8 - 2015
B. Right lateral FEB 2015 MED
C. Right anterior BOARDS;
D. Right posterior TOPNOTCH MD
E. None of the above FROM UST)

112 A 35 y/o male withprogressing flank pain radiating Page 180 of IM Platinum. The most common stone ANGELA PAULINE DIAGNOSTIC
to R testicle and diagnosed to have urolithiasis. The to expect are calcium stones. P. CALIMAG- EXAM - AUG
most common stone to expect is:
LOYOLA (TOP 8 - 2015
A. Calcium oxalate stone FEB 2015 MED
B. Struvite stone BOARDS;
C. Cystine stone TOPNOTCH MD
D. Uric acid stone FROM UST)
E. Magnesium-ammonium-phosphate stone

113 A 24 y/o female complains of palpitations. On Page 71 of IM Platinum. This is a case of Mitral ANGELA PAULINE DIAGNOSTIC
physical examination she has a midsystolic click valve prolapse. Typical findings on P. CALIMAG- EXAM - AUG
followed by a mid to late crescendo systolic murmur echocardiography are as follows: LOYOLA (TOP 8 - 2015
at the apex. A 2-D echocardiogram was requested. Classic MVP: The parasternal long-axis view FEB 2015 MED
This will most likely reveal:
shows > 2 mm superior displacement of the mitral BOARDS;
A. Doming and restriction of motion of the mitral leaflets into the left atrium during systole, with a TOPNOTCH MD
valve leaflets. leaflet thickness of at least 5 mm FROM UST)
B. Marked superior displacement of mitral leaflets Nonclassic MVP: Displacement is > 2 mm, with a
w/ coaptation point at or superior to annular plane maximal leaflet thickness of < 5 mm
C. Fluttering mitral valve leaflet Other echocardiographic findings that should be
D. Mitral valve leaflets thickness <4mm considered as criteria are leaflet thickening,
E. Chordae shortening redundancy, annular dilatation, and chordal
elongation
114 The auscultatory findings in patient w/ ASD will be: Because the pressure in the left atria initially ANGELA PAULINE DIAGNOSTIC
A. Holosystolic murmur at the pulmonic area w/ exceeds that in the right, the blood flows in a left P. CALIMAG- EXAM - AUG
loud pulmonic component of S2(P2) to right shunt. This high volume of blood next LOYOLA (TOP 8 - 2015
B. Holosystolic murmur at the base w/ wide and passes into the right ventricle, and the ejection of FEB 2015 MED
fixed split S2 the excess blood through a normal pulmonary BOARDS;
C. Holosystolic murmur at the 4th ICS at the R valve produces the prominent mid-systolic flow TOPNOTCH MD
parasternal area w/ (+) Carvallo sign murmur as heard in this sample. This murmur is FROM UST)
D. Pansystolic murmur along lower left sternal best heard over the “pulmonic area” of the chest,
border and may radiate into the back. The most
E. Crescendo-decrescendo murmur on the second characteristic feature of an atrial septal defect is
intercostal space along the left sternal border the fixed split S2. A split S2 is caused
physiologically during inspiration because the
increase in venous return overloads the right
ventricle and delays the closure of the pulmonary
valve. With an atrial septal defect, the right
ventricle can be thought of as continuously
overloaded because of the left to right shunt,
producing a widely split S2. Because the atria are
linked via the defect, inspiration produces no net
pressure change between them, and has no effect
on the splitting of S2. Thus, S2 is split to the same
degree during inspiration as expiration, and is said
to be “fixed.”
115 A manifestation of severe aortic regurgitation Page 72 of IM Platinum. ANGELA PAULINE DIAGNOSTIC
characterized as jarring of the body and bobbing of P. CALIMAG- EXAM - AUG
the head with each systole: A. Quincke's LOYOLA (TOP 8 - 2015
pulse FEB 2015 MED
B. Duroziez sign BOARDS;
C. De Musset sign TOPNOTCH MD
D. Traube sign FROM UST)
E. Austin Flint sign

116 A vascular phenomena included as a minor criteria Page 65 of IM Platinum. Janeway lesions are non- ANGELA PAULINE DIAGNOSTIC
in the diagnosis of Infective Endocarditis:
tender, painless, small hemorrhagic P. CALIMAG- EXAM - AUG
A. Osler's nodes macular/nodular lesions on the palms or soles. LOYOLA (TOP 8 - 2015
B. Roth's spots Osler's nodes and Roth spots are immunologic and FEB 2015 MED
C. Subcutaneous nodules not vascular. Erythema marginatum and BOARDS;
D. Janeway lesions subcutaneous nodules are not included in the TOPNOTCH MD
E. Erythema marginatum Dukes criteria but in the Jones criteria for RF, FROM UST)

117 The most common arrythmia causing out of hospital SIMILAR TO PREVIOUS BOARD EXAM ANGELA PAULINE DIAGNOSTIC
deaths from Myocardial Infarction is:
CONCEPT/PRINCIPL Page 59 of IM Platinum. Most P. CALIMAG- EXAM - AUG
A. Atrial fibrillation out of hospital deaths are from ventricular LOYOLA (TOP 8 - 2015
B. Asystole fibrillation. FEB 2015 MED
C. Premature ventricular contractions BOARDS;
D. Junctional rhythms TOPNOTCH MD
E. Ventricular fibrillation FROM UST)

TOPNOTCH MEDICAL BOARD PREP INTERNAL MEDICINE SUPEREXAM Page 18 of 95


For inquiries visit www.topnotchboardprep.com.ph or email us at topnotchmedicalboardprep@gmail.com
TOPNOTCH MEDICAL BOARD PREP INTERNAL MEDICINE SUPEREXAM
For inquiries visit www.topnotchboardprep.com.ph or email us at topnotchmedicalboardprep@gmail.com
Item QUESTION EXPLANATION AUTHOR TOPNOTCH
# EXAM
118 A 23 y/o patient is undergoing surgery for a SIMILAR TO PREVIOUS BOARD EXAM ANGELA PAULINE DIAGNOSTIC
zygomatic fracture. The surgeon inadvertently CONCEPT/PRINCIPLE. The oculocardiac reflex, P. CALIMAG- EXAM - AUG
applied too much pressure on the patient's eyeball. also known as Aschner phenomenon, Aschner LOYOLA (TOP 8 - 2015
The anesthesiologist then noted that the patient's reflex, or Aschner-Dagnini reflex, is a decrease in FEB 2015 MED
ECG tracing converted from sinus rhythm to sinus pulse rate associated with traction applied to BOARDS;
bradycardia. The anesthesiologist knows that this extraocular muscles and/or compression of the TOPNOTCH MD
phenomenon is called:
eyeball. The reflex is mediated by nerve FROM UST)
A. Aschner reflex connections between the ophthalmic branch of the
B. Vasovagal reflex trigeminal cranial nerve via the ciliary ganglion,
C. Oculobradycardia phenomenon and the vagus nerve of the parasympathetic
D. Neurocardiogenic syncope nervous system.
E. Oculovagal reflex

119 The most useful indicator/index of LV function is Page 39 of IM Platinum. The formula for EF is EF= ANGELA PAULINE DIAGNOSTIC
the ejection fraction, to compute for EF the following SV/EDV. P. CALIMAG- EXAM - AUG
parameter is needed: A. Mean arterial LOYOLA (TOP 8 - 2015
pressure in mmHg FEB 2015 MED
B. Systemic vascular resistance in dynes BOARDS;
C. Cardiac output TOPNOTCH MD
D. End systolic volume FROM UST)
E. Stroke volume

120 Dopamine has varying hemodynamic effects based Page 27 of IM Platinum. The renal dose of ANGELA PAULINE DIAGNOSTIC
on the dose. The dose needed to activate the DA1 dopamine is 1-2 mcg/kg/min. Inotropic dose is 2- P. CALIMAG- EXAM - AUG
and DA2 receptors causing splanchnic and renal 4 mcg/kg/min, >5mcg/kg/min is a LOYOLA (TOP 8 - 2015
vasodilation is:
vasoconstrictor dose. FEB 2015 MED
A. 8 mcg/kg/min BOARDS;
B. 6 mcg/kg/min TOPNOTCH MD
C. 4 mcg/kg/min FROM UST)
D. 1 mcg/kg/min
E. 10 mcg/kg/min

121 Patient presents with a high-pitched, blowing, In patients with severe AR, the aortic valve closure LYNN DARYL MIDTERM 1
diastolic murmur, heard best in the third intercostal sound (A2) is usually absent. An S3 and systolic FELICIANO EXAM - AUG
space along the left sternal border. This is most ejection sound are frequently audible, and VILLAMATER, MD 2015
likely a case of: occasionally an S4 also may be heard. The murmur (TOP 5 - FEB 2015
A. Ventricular septal defect of chronic AR is typically a high-pitched, blowing, MED BOARDS;
B. Mitral stenosis decrescendo diastolic murmur, heard best in the TOPNOTCH MD
C. Aortic regurgitation 3rd intercostal space along the left sternal border. FROM EAC)
D. Atrial septal defect In patients in whom the AR is caused by primary
E. Pulmonic stenosis valvular disease, the diastolic murmur is usually
louder along the left than the right sternal border.
However, when the murmur is heard best along
the right sternal border, it suggests that the AR is
caused by aneurysmal dilatation of the aortic root.
Note: At least 3 questions about heart murmurs
were asked in IM Feb 2015 boards, mentioning
only about the auscultation finding.
122 Ventricular tachycardia lasting for more than 30 Sustained polymorphic VT, ventricular flutter, and LYNN DARYL MIDTERM 1
seconds and not terminated by therapy will lead to? VF all lead to immediate hemodynamic collapse. FELICIANO EXAM - AUG
A. Hemodynamic collapse Emergency asynchronous defibrillation is VILLAMATER, MD 2015
B. Fatal arrythmia therefore required, with at least 200-J monophasic (TOP 5 - FEB 2015
C. Asynchronous rhythm or 100-J biphasic shock. (Harrison) SIMILAR TO MED BOARDS;
D. Recurrent arrythmia PREVIOUS BOARD EXAM CONCEPT. TOPNOTCH MD
E. VT storm FROM EAC)

123 Treatment for ischemic and nephrotoxic acute Many different approaches to attenuate injury or LYNN DARYL MIDTERM 1
kidney injury include: hasten recovery have been tested in ischemic and FELICIANO EXAM - AUG
A. None nephrotoxic AKI. These include ANP, low-dose VILLAMATER, MD 2015
B. Hydration dopamine, etc. Whereas many of these are (TOP 5 - FEB 2015
C. Antibiotics beneficial in experimental models of ischemic or MED BOARDS;
D. Loop diuretics nephrotoxic ATN, they have either failed to confer TOPNOTCH MD
E. Immunosuppressives consistent benefit or proved ineffective in humans. FROM EAC)
(Harrison's) SIMILAR TO PREVIOUS BOARD EXAM
CONCEPT
124 The most useful test for distinguishing prerenal The most useful test for distinguishing prerenal LYNN DARYL MIDTERM 1
acute renal failure from intrinsic renal failure is: ARF from ischemic or nephrotoxic intrinsic renal FELICIANO EXAM - AUG
A. BUN Creatinine Ratio ARF is the fractional excretion of sodium (FENa). VILLAMATER, MD 2015
B. FENa Prerenal ARF typically have a FENa of <1.0% (TOP 5 - FEB 2015
C. Serum Sodium (frequently <0.1%). The FENa tends to be high in MED BOARDS;
D. Creatinine clearance ischemic ATN. (Harrison's) SIMILAR TO TOPNOTCH MD
E. Serum creatinine PREVIOUS BOARD EXAM CONCEPT. FROM EAC)

125 Which of the following drugs have been shown to The following treatment modalities have been LYNN DARYL MIDTERM 1
decrease mortality in patients with heart failure? shown to decrease mortality in patients with heart FELICIANO EXAM - AUG
A. Digoxin failure: ACE-I/ARBS (EF<40), Beta blockers, VILLAMATER, MD 2015
B. Loop diuretics Spironolactone, AICD. (TOP 5 - FEB 2015
C. Nitroglycerine MED BOARDS;
D. Beta-blockers TOPNOTCH MD
E. All of the above FROM EAC)

TOPNOTCH MEDICAL BOARD PREP INTERNAL MEDICINE SUPEREXAM Page 19 of 95


For inquiries visit www.topnotchboardprep.com.ph or email us at topnotchmedicalboardprep@gmail.com
TOPNOTCH MEDICAL BOARD PREP INTERNAL MEDICINE SUPEREXAM
For inquiries visit www.topnotchboardprep.com.ph or email us at topnotchmedicalboardprep@gmail.com
Item QUESTION EXPLANATION AUTHOR TOPNOTCH
# EXAM
126 A 28-year-old female presented with headache and This is a case of meningococcemia. The drug of LYNN DARYL MIDTERM 1
cough. After a few days, her temperature rises choice for this condition is IV penicillin. (Note: FELICIANO EXAM - AUG
abruptly, now with body ache and petechial rash on SIMILAR TO PREVIOUS BOARD EXAM CONCEPT) VILLAMATER, MD 2015
the trunk and legs. What is the drug of choice for her (TOP 5 - FEB 2015
condition? MED BOARDS;
A. Penicillin TOPNOTCH MD
B. Amikacin FROM EAC)
C. Ciprofloxacin
D. Azithromycin
E. Ceftriaxone
127 What is the role of iodides in the treatment of Iodides inhibit thyroid hormone synthesis or LYNN DARYL MIDTERM 1
hyperthyroidism? release with the induction of hypothyroidism. FELICIANO EXAM - AUG
A. It inhibits 5-deiodinase VILLAMATER, MD 2015
B. It inhibits peripheral conversion of T4 to T3 (TOP 5 - FEB 2015
C. It inhibits thyroid hormone synthesis MED BOARDS;
D. It blocks coupling of iodotyrosines. TOPNOTCH MD
E. Inhibit the thyroid peroxidase-catalyzed FROM EAC)
reactions
128 Which of the following prophylactic antibiotics is Lifelong PROPHYLAXIS post-splenectomy include LYNN DARYL MIDTERM 1
especially indicated in a splenectomized patient? Phenoxymethylpenicillin, Amoxicilliin or if allergic FELICIANO EXAM - AUG
A. Penicillin to penicillin, give Clarithromycin. For EMPIRIC VILLAMATER, MD 2015
B. Ceftriaxone treatment of hospitalized hyposplenic or asplenic (TOP 5 - FEB 2015
C. Azithromycin patients acute infection, Ceftriaxone 2g q24 hrs is MED BOARDS;
D. Chloramphenicol given. SIMILAR TO PREVIOUS BOARD EXAM TOPNOTCH MD
E. Linezolid CONCEPT. FROM EAC)

129 A patient presents with the following serologic HBsAG (+) & Anti-HBc IgG positive : Chronic Hepa LYNN DARYL MIDTERM 1
pattern: HBsAG reactive, Anti-HAV IgM reactive, B carrier; Anti-HAV IgM reactive: Acute hepatitis FELICIANO EXAM - AUG
anti-HBC IgG positive. The patient is most likely A infection VILLAMATER, MD 2015
suffering from? (TOP 5 - FEB 2015
A. Acute hepatits A and B MED BOARDS;
B. Acute hepatitis B TOPNOTCH MD
C. Acute Hepatitis A FROM EAC)
D. Acute hepatitis A superimposed on Chronic
Hepatitis B
E. Acute Hepatitis B
130 Which of the following drugs is the first line Grade A antibiotic/first line drug for LYNN DARYL MIDTERM 1
treatment for acute uncomplicated cystitis in non- uncomplicated cystitis: Ciprofloxacin, Ofloxacin, FELICIANO EXAM - AUG
pregnant women? Norfloxacin, Levofloxacin, Gatifloxacin, VILLAMATER, MD 2015
A. Cefuroxime 250 mg BID x 3 days Nitrofurantoin, Cotrimoxazole. Options A,C and E (TOP 5 - FEB 2015
B. Amoxicillin 500 mg TID x 7 days are Grade B antibiotics. SIMILAR TO PREVIOUS MED BOARDS;
C. Co-amoxiclav 625 mg BID x 7 days BOARD EXAM CONCEPT. TOPNOTCH MD
D. Ciprofloxacin 250 mg BID x 3 days FROM EAC)
E. Cefixime 400 mg OD x 3 days

131 Which of the following is/are included in the Treatment of MDRTB is based on drug LYNN DARYL MIDTERM 1
treatment for MDRTB? susceptibilities. The following are drugs given for FELICIANO EXAM - AUG
A. Kanamycin MRDTB: High dose INH, Pyrazinamide, VILLAMATER, MD 2015
B. Erythromycin Ethambutol; Flouroquinolones (Levofloxacin, (TOP 5 - FEB 2015
C. Chloramphenicol Ciprofloxacin), Aminoglycosides (Amikacin, MED BOARDS;
D. Pyrimethamine Kanamycin, Capreomycin, Streptomycin); TOPNOTCH MD
E. Sulfamethoxazole Cycloserine, Terizidone, Para-aminosalicylic acid, FROM EAC)
Bedaquiline. (SIMILAR TO PREVIOUS BOARD
EXAM CONCEPT)
132 In the treatment of COPD patient with chronic Only 3 interventions influence the natural history LYNN DARYL MIDTERM 1
respiratory failure, which of the following of COPD: smoking cessation, oxygen in chronic FELICIANO EXAM - AUG
pharmacologic therapy improves survival? hypoxemia, and lung volume reduction surgery for VILLAMATER, MD 2015
A. Bronchodilators emphysema. (TOP 5 - FEB 2015
B. Inhaled corticosteroids MED BOARDS;
C. Long-acting anticholinergic TOPNOTCH MD
D. Leukotriene inhibitors FROM EAC)
E. Long-term O2 therapy

133 A 48-year old male, hospitalized due to sepsis, This is a case of Acute Respiratory Distress LYNN DARYL MIDTERM 1
suddenly develops severe dyspnea at rest and Syndrome. It is a clinical syndrome of severe FELICIANO EXAM - AUG
agitation. Physical findings showed hypoxemia, and dyspnea of rapid onset, hypoxemia, and diffuse VILLAMATER, MD 2015
cold extremities. Chest radiograph done revealed pulmonary infiltrates leading to respiratory (TOP 5 - FEB 2015
diffuse interstitial infiltrates wtih ground glass failure. The only grade A recommendation for the MED BOARDS;
changes. Which of the following treatment is management of ARDS is low tidal volume TOPNOTCH MD
important in the management of his condition? mechanical ventilation. (6 ml/kg) Source: FROM EAC)
A. Glucocorticoids Harrisons. (Note: SIMILAR TO PREVIOUS BOARD
B. Nitroglycerine EXAM CONCEPT)
C. Morphine
D. Nitric oxide
E. Low-tidal volume ventilation

TOPNOTCH MEDICAL BOARD PREP INTERNAL MEDICINE SUPEREXAM Page 20 of 95


For inquiries visit www.topnotchboardprep.com.ph or email us at topnotchmedicalboardprep@gmail.com
TOPNOTCH MEDICAL BOARD PREP INTERNAL MEDICINE SUPEREXAM
For inquiries visit www.topnotchboardprep.com.ph or email us at topnotchmedicalboardprep@gmail.com
Item QUESTION EXPLANATION AUTHOR TOPNOTCH
# EXAM
134 A 32-year old female, presents to the clinic with This is a case of SLE. SLE may present with LYNN DARYL MIDTERM 1
symptoms of fatigue, joint pains, and facial rash. On anemia (hemolytic), thrombocytopenia, FELICIANO EXAM - AUG
examination she was noted to be thin with malar leukopenia, or lymphopenia in the absence of VILLAMATER, MD 2015
skin changes. CBC with platelet count was requested offending drugs. Among these, the most common (TOP 5 - FEB 2015
which will most likely reveal: hematologic manifestaion of SLE is anemia, MED BOARDS;
A. Thrombocytopenia usually normochromic normocytic, reflecting TOPNOTCH MD
B. Leukocytosis chronic illness. (Harrison's) The best answer is FROM EAC)
C. Anemia Anemia.
D. Neutropenia
E. Lymphocytosis

135 This finding in chronic myelogenous leukemia can Leukomoid reaction mimics CML, as both LYNN DARYL MIDTERM 1
differentiate it from leukemoid reaction: condition wil reveal an elevated WBC. However, in FELICIANO EXAM - AUG
A. Increased WBC count leukomoid reaction, neutrophil/leukocyte alkaline VILLAMATER, MD 2015
B. Anemia phosphatase and CRP, as a result of inclease (TOP 5 - FEB 2015
C. Hypercellular marrow with increased inflammatory response will be elevated. These MED BOARDS;
granulocyte precursor parameters were low in CML. (Note: SIMILAR TO TOPNOTCH MD
D. Decrease leukocyte alkaline phosphatase PREVIOUS BOARD EXAM CONCEPT) FROM EAC)
E. Increased CRP

136 The most common organism isolated from the While enteric gram-negative bacilli such as LYNN DARYL MIDTERM 1
ascitic fluid of patients with spontaneous bacterial Escherichia coli are most commonly encountered, FELICIANO EXAM - AUG
peritonitis is? gram-positive organisms such as streptococci, VILLAMATER, MD 2015
A. Streptococcus pneumoniae enterococci, or even pneumococci are sometimes (TOP 5 - FEB 2015
B. Staphylococcu aureus found. Source: Harrison's MED BOARDS;
C. Escherichia coli TOPNOTCH MD
D. Bacteroides fragilis FROM EAC)
E. Klebsiella pneumoniae

137 A 33-year old male was recently diagnosed to have MEN 2A- medullary thyroid carcinoma, LYNN DARYL MIDTERM 1
medullary thyroid carcinoma. On further work-up, pheochromocytoma, hyperparathyroidism; MEN FELICIANO EXAM - AUG
he was noted to have elevated levels of serum 2B - Medullary thyroid carcinoma + VILLAMATER, MD 2015
calcium and decreased serum phosphate. His Pheochromocytoma + neuromas; MEN 1 - (TOP 5 - FEB 2015
abdominal CT scan showed a mass on the right pancreatic tumors, parathyroid glands neoplasia, MED BOARDS;
adrenal gland Which of the following is the most and pituitary tumors. TOPNOTCH MD
likely diagnosis ? FROM EAC)
A. MEN Type 1
B. MEN Type 2A
C. MENType 2B
D. Li Fraumeni Syndrome
E. von Hippel Lindau Syndrome

138 A 42-year-old female presented with hematuria, This is a case of PSGN. This is an immune- LYNN DARYL MIDTERM 1
edema, hypertension and scanty urine. She had a mediated disease.Option B is incorrect, both skin FELICIANO EXAM - AUG
history of the throat pain 2 weeks ago. Which of the and throat infection antedate glomerular disease. VILLAMATER, MD 2015
following is true regarding her condition? In the first week of symptoms, most patient will (TOP 5 - FEB 2015
A. This is an immune-mediated disease which have depressed level of C3. The overall prognosis MED BOARDS;
involves activation of complement in association of PSGN is good in adults and children. Complete TOPNOTCH MD
with cell-mediated injury. resolution of the hematuria and proteinuria in FROM EAC)
B. Throat infections, but not skin infections children occurs within 3–6 weeks of the onset of
antedate glomerular disease. nephritis. (Harrison's)
C. In the first week of symptoms, most patient will
have elevated levels of C3.
D. The prognosis in adults is poor.
E. Complete resolution of hematuria in children
occurs within 3-6 months of the onset of nephritis.
139 Which of the following organism is implicated in bile Infection of these three species is established by LYNN DARYL MIDTERM 1
duct carcinoma? ingestion of raw or inadequately cooked FELICIANO EXAM - AUG
A. Clonorchis sinensis freshwater fish harboring metacercariae. These VILLAMATER, MD 2015
B. Schistosoma japonicum organisms excyst in the duodenum, releasing (TOP 5 - FEB 2015
C. Paragonimus westermani larvae that travel through the ampulla of Vater and MED BOARDS;
D. Capillaria philippinensis mature into adult worms in bile canaliculi. TOPNOTCH MD
E. Fasciola hepatica Cholangiocarcinoma is epidemiologically related FROM EAC)
to C. sinensis infection in China and to O. viverrini
infection in northeastern Thailand. This
association has resulted in classification of these
infectious agents as human carcinogens.
(Harrison's)
140 Which of the following anti-microbial drug is Tetracycline, Nafcillin, and Ertapenem have no LYNN DARYL MIDTERM 1
effective for the treatment of Pseudomonas coverage for Pseudomonas infection. SIMILAR TO FELICIANO EXAM - AUG
infection? PREVIOUS BOARD EXAM CONCEPT. VILLAMATER, MD 2015
A. Tetracycline (TOP 5 - FEB 2015
B. Nafcillin MED BOARDS;
C. Ertapenem TOPNOTCH MD
D. Ceftazidime FROM EAC)
E. All of the above

141 The suppressive dose of propranolol needed in This is the tremor controlling dose of propranolol EDWARD HARRY MIDTERM 2
controlling the tremors from hyperthyroidism is: due to hyperthyroidism - Reference: IM Platinum VALLAJERA, MD EXAM - AUG
A. 20-40 mg tab q6 page199 (TOP 8 - FEB 2015 2015
B. 20-40 mg tab q4 MED BOARDS;
C. 100mg tab q6 TOPNOTCH MD
D. 100mg tab q4 FROM PERPETUAL
E. None of the above BINAN)

TOPNOTCH MEDICAL BOARD PREP INTERNAL MEDICINE SUPEREXAM Page 21 of 95


For inquiries visit www.topnotchboardprep.com.ph or email us at topnotchmedicalboardprep@gmail.com
TOPNOTCH MEDICAL BOARD PREP INTERNAL MEDICINE SUPEREXAM
For inquiries visit www.topnotchboardprep.com.ph or email us at topnotchmedicalboardprep@gmail.com
Item QUESTION EXPLANATION AUTHOR TOPNOTCH
# EXAM
142 You are treating a case of sepsis in which the culture Empiric therapy is giving the antibiotic treatment EDWARD HARRY MIDTERM 2
and sensitivity results are still unavailable, the with broad spectrum of activity for a suspected VALLAJERA, MD EXAM - AUG
practice of giving antibiotics before C & S becomes organism/s without the benefit of knowing the (TOP 8 - FEB 2015 2015
available is defined as susceptibility patterns of such organism/s. Once MED BOARDS;
A. Giving an antibiotic which has a broad spectrum the results of the C &S become available, the TOPNOTCH MD
of activity without knowing the specific organism therapy can then be tailored depending on the FROM PERPETUAL
B. Withholding the treatment while awaiting C & S susceptibility results. BINAN)
results
C. Tailoring the antibiotic regimen guided by the
results from the culture and sensitivity
D. Both A and C
E. None of the above

143 What is the empiric therapy for post-splenectomy Penicillin is used as empiric treatment in post EDWARD HARRY MIDTERM 2
sepsis? splenectomy sepsis as the organism most likely VALLAJERA, MD EXAM - AUG
A. Ciprofloxacin involved is S. pneumoniae which is a Gram (+) (TOP 8 - FEB 2015 2015
B. Penicillin bacteria and is susceptible to the effects of MED BOARDS;
C. Erythromycin penicillin TOPNOTCH MD
D. Tetracycline FROM PERPETUAL
E. None of the above BINAN)

144 An example of the triple drug regimen used in H. This is the triple drug therapy used in H. pylori EDWARD HARRY MIDTERM 2
pylori infection consists of: infection while option B is the quadruple therapy VALLAJERA, MD EXAM - AUG
A. PPI + Clarithromycin + Amoxicillin or and is used when the triple therapy has failed. (TOP 8 - FEB 2015 2015
Metronidazole MED BOARDS;
B. PPI + Metronidazole + Tetracycline + Bismuth TOPNOTCH MD
C. PPI + Ciprofloxacin + Clindamycin FROM PERPETUAL
D. A and B BINAN)
E. None of the above

145 The thyroid suppressive dose of metoprolol is Reference: IM Platinum page 202 EDWARD HARRY MIDTERM 2
A. 50 mg VALLAJERA, MD EXAM - AUG
B. 200 mg (TOP 8 - FEB 2015 2015
C. 300 mg MED BOARDS;
D. 400 mg TOPNOTCH MD
E. None of the above FROM PERPETUAL
BINAN)

146 Which of the following is not associated with Decreased capillary permeability is not associated EDWARD HARRY MIDTERM 2
inflammation with inflammation VALLAJERA, MD EXAM - AUG
A. Redness (TOP 8 - FEB 2015 2015
B. Vasodilation MED BOARDS;
C. Decreased capillary permeability TOPNOTCH MD
D. Pain FROM PERPETUAL
E. Heat BINAN)

147 The IgM antibody exists as pentamer. This basically The IgM pentamer is made up of 5 units of IgM EDWARD HARRY MIDTERM 2
means that it consists of 5 antibodies joined with each unit capable of binding 2 antigens on VALLAJERA, MD EXAM - AUG
together. Thus the IgM pentamer could bind to how each side. (TOP 8 - FEB 2015 2015
many antigens? MED BOARDS;
A. 5 TOPNOTCH MD
B. 15 FROM PERPETUAL
C. 8 BINAN)
D. 10
E. 9

148 Histamine increases blood flow and vascular Increase number of phagocytes being attracted to EDWARD HARRY MIDTERM 2
permeability. This would account for all of the the site of injury is not a function of histamine VALLAJERA, MD EXAM - AUG
following changes that occur during inflammation (TOP 8 - FEB 2015 2015
except MED BOARDS;
A. Redness of the inflamed tissue TOPNOTCH MD
B. Heat of the inflamed tissue FROM PERPETUAL
C. Increased number of phagocytes being attracted BINAN)
to the tissue
D. Increased formation of interstitial fluid at the site
of injury
E. None of the above
149 A sample of blood from patient RR shows a high Elevated levels of IL-1 an endogenous pyrogen EDWARD HARRY MIDTERM 2
concentration of IL-1. This would indicate that RR: would indicate that RR has fever. VALLAJERA, MD EXAM - AUG
A. Is running a fever (TOP 8 - FEB 2015 2015
B. Has a sore throat MED BOARDS;
C. Is producing T lymphocytes TOPNOTCH MD
D. Has hypotension FROM PERPETUAL
E. Has swollen lymph nodes BINAN)

150 A patient came in to your clinic for the interpretation EDWARD HARRY MIDTERM 2
of his hepatitis profile you noted that Anti-Hbc IgM VALLAJERA, MD EXAM - AUG
is positive, what does it mean? (TOP 8 - FEB 2015 2015
A. The patient is actively replicating Hepa B virus MED BOARDS;
B. The patient is at the window preiod following 6 TOPNOTCH MD
months of acute infection FROM PERPETUAL
C. The patient has recovered from Hepa B BINAN)
D. The patient was immunized before
E. It is insignificant

TOPNOTCH MEDICAL BOARD PREP INTERNAL MEDICINE SUPEREXAM Page 22 of 95


For inquiries visit www.topnotchboardprep.com.ph or email us at topnotchmedicalboardprep@gmail.com
TOPNOTCH MEDICAL BOARD PREP INTERNAL MEDICINE SUPEREXAM
For inquiries visit www.topnotchboardprep.com.ph or email us at topnotchmedicalboardprep@gmail.com
Item QUESTION EXPLANATION AUTHOR TOPNOTCH
# EXAM
151 JA, was suspected of having a hematologic Imatinib, a tyr kinase inhibitor is used for the EDWARD HARRY MIDTERM 2
malignancy, analysis showed that he has a t(9:22) treatment of CML. Abciximab is a GPIIb/IIIa VALLAJERA, MD EXAM - AUG
translocation forming a fusion product called bcr- inhibitor, infliximab is a TNF - alpha antibody used (TOP 8 - FEB 2015 2015
abl, what is the treatment for this case? in autoimmune diseases, omalizumab is an IgG MED BOARDS;
A. Abciximab antibody for IgE used in the treatment of TOPNOTCH MD
B. Infliximab moderate - severe asthma. FROM PERPETUAL
C. Omalizumab BINAN)
D. Imatinib
E. None of the above

152 Which of the following would deal with an B cells and the antibodies deal with extracellular EDWARD HARRY MIDTERM 2
extracellular pathogen in the lymph? pathogens from the lymph fluid draining a VALLAJERA, MD EXAM - AUG
A. B cells and natural killer cells particular region of the body. (TOP 8 - FEB 2015 2015
B. B cells and antibodies MED BOARDS;
C. Killer T cells only TOPNOTCH MD
D. Macrophages only FROM PERPETUAL
E. Helper T cells only BINAN)

153 If a virus invaded a liver cell, that liver cell would Both options would tell what a virally infected cell EDWARD HARRY MIDTERM 2
A. Display fragments of viral proteins on its MHC I would do. VALLAJERA, MD EXAM - AUG
protein (TOP 8 - FEB 2015 2015
B. Display fragments of viral proteins on its MHC II MED BOARDS;
protein TOPNOTCH MD
C. Begin making viral proteins and nucleic acids (e.g., FROM PERPETUAL
DNA, RNA). BINAN)
D. Both A and C
E. All of the above
154 Which of the following cells destroys body cells that NK cells are endowed with the ability to kill a EDWARD HARRY MIDTERM 2
have been infected by a virus variety of infected and tumor cells, without prior VALLAJERA, MD EXAM - AUG
A. Monocytes exposure to or activation by these microbes or (TOP 8 - FEB 2015 2015
B. B lymphocytes tumors. This ability makes NK cells an early line of MED BOARDS;
C. Natural killer cells defense against viral infections and, perhaps, some TOPNOTCH MD
D. T lymphocytes tumors. E is cytotoxic T cell or CD8 mediated FROM PERPETUAL
E. None of the above destruction of virally infected cells through the BINAN)
recognition of virally infected class 1 MHC
molecule of that cell (Robbins, 9th ed.)
155 Which of the following does not occur during a fever Plasma zinc levels are unaffected by fever. The rest EDWARD HARRY MIDTERM 2
A. WBC function is increased are actions of the body occuring during fever. VALLAJERA, MD EXAM - AUG
B. Bacterial metabolism is impaired. (TOP 8 - FEB 2015 2015
C. Plasma zinc levels rise MED BOARDS;
D. Plasma iron levels fall TOPNOTCH MD
E. There is release of cytokines FROM PERPETUAL
BINAN)

156 Which of the following is true of neutrophils? Most of the dead cells in pus are neutrophils EDWARD HARRY MIDTERM 2
A. They account for most of the dead cells in pus together with killed pathogens. VALLAJERA, MD EXAM - AUG
B. They're usually the last immune cells to enter (TOP 8 - FEB 2015 2015
infected tissues MED BOARDS;
C. They're considered part of the adaptive immune TOPNOTCH MD
system FROM PERPETUAL
D. Both A and C BINAN)
E. All of the above
157 What is the most common presenting symptom of The diverticulum may serve as a point for EDWARD HARRY MIDTERM 2
Meckel's diverticulum among adults intussussception. In children, it usually presents VALLAJERA, MD EXAM - AUG
A. Passage of currant jelly stools with bleeding painlessly in the stools (TOP 8 - FEB 2015 2015
B. Intestinal obstruction MED BOARDS;
C. Passage of blood painlessly in the stool TOPNOTCH MD
D. Abdominal pain FROM PERPETUAL
E. None of the above BINAN)

158 AO, a 40 year old man came in due to anemia, PE was This is a case of Vit. B12 deficiency, the EDWARD HARRY MIDTERM 2
unremarkable except there was weakness of legs, differentiating features with folate deficiency is VALLAJERA, MD EXAM - AUG
arms, trunk, tingling and numbness that the patient the presence of neurologic manifestations and (TOP 8 - FEB 2015 2015
said progressively worsens, CBC shows anemia and folate improves only the anemia while the MED BOARDS;
peripheral blood smear shows the presence of large neurologic symptoms would not improve. TOPNOTCH MD
RBCs. What is the treatment for this case? FROM PERPETUAL
A. Iron supplement BINAN)
B. Folate supplementation
C. Vit. B12 supplementation
D. Vit. C supplementation
E. None of the above

159 It is a group of disorders characterized by insulin EDWARD HARRY MIDTERM 2


resistance, impaired insulin secretion and excessive VALLAJERA, MD EXAM - AUG
hepatic glucose production (TOP 8 - FEB 2015 2015
A. Type 1 DM MED BOARDS;
B. Type 2 DM TOPNOTCH MD
C. Central Diabetes Insipidus FROM PERPETUAL
D. Nephrogenic Diabetes Insipidus BINAN)
E. None of the above

TOPNOTCH MEDICAL BOARD PREP INTERNAL MEDICINE SUPEREXAM Page 23 of 95


For inquiries visit www.topnotchboardprep.com.ph or email us at topnotchmedicalboardprep@gmail.com
TOPNOTCH MEDICAL BOARD PREP INTERNAL MEDICINE SUPEREXAM
For inquiries visit www.topnotchboardprep.com.ph or email us at topnotchmedicalboardprep@gmail.com
Item QUESTION EXPLANATION AUTHOR TOPNOTCH
# EXAM
160 MD, 35 year old male came in to your clinic due to Both MEN 2A and 2B have marfanoid habitus, EDWARD HARRY MIDTERM 2
palpitations, you noted marfanoid features and pheochrmocytoma and medullary thyroid CA, VALLAJERA, MD EXAM - AUG
elicited a strong family history of the same condition MEN 2A has parathyroid adenoma while MEN 2B (TOP 8 - FEB 2015 2015
among his ascendants in which there is also has mucosal neuromas. MED BOARDS;
medullary thyroid carcinoma and TOPNOTCH MD
pheochromocytoma, what is the most likely FROM PERPETUAL
diagnosis BINAN)
A. MEN I
B. MEN 2A
C. MEN 2B
D. Marfan syndrome
E. None of the above
161 This is characterized as head nodding which is found Corrigans pulse is bounding pulses. Durosier's HAROLD JAY S. MIDTERM 3
in patients with Aortic regurgitation. sign if femoral retrograde bruits. Traube's sign is BAYTEC, MD (TOP EXAM - AUG
A. Corrigan's pulse pistol shot femorals. Hills sign is pistol shot 10 - FEB 2015 MED 2015
B. Durosier's sign femorals BOARDS;
C. Traube's sign TOPNOTCH MD
D. De Mussets sign FROM FEU)
E. Hill's sign

162 Graham-Steel murmur is a murmur usually caused this murmur is usually caused by pulmonary HAROLD JAY S. MIDTERM 3
by pulmonary hypertension. This murmur is usually hypertension caused by the pulmonary valve BAYTEC, MD (TOP EXAM - AUG
heard in what valve abnormality? regurgitation 10 - FEB 2015 MED 2015
A. Pulmonary regurgitation BOARDS;
B. Pulmonary stenosis TOPNOTCH MD
C. Tricuspid Regurgitation FROM FEU)
D. Tricuspid stenosis
E. Aortic Regurgitation

163 A patient came in at the ER with altered mental E3V4M5 HAROLD JAY S. MIDTERM 3
status. According to his companion the patient was BAYTEC, MD (TOP EXAM - AUG
just watching TV and suddenly fell on the ground. On 10 - FEB 2015 MED 2015
assessment, you noticed that the patient opens his BOARDS;
eyes through loud voice, confused and localizes pain. TOPNOTCH MD
What is the GCS score of the patient? FROM FEU)
A. 10
B. 11
C. 12
D. 13
E. 14

164 A patient came in at the ER complaining of body U waves are seen in hypokalemia HAROLD JAY S. MIDTERM 3
malaise associated with polyuria. ECG shows BAYTEC, MD (TOP EXAM - AUG
prominent U waves. Which of the following is most 10 - FEB 2015 MED 2015
likely the cause of the disease. BOARDS;
A. hyperkalemia TOPNOTCH MD
B. hypokalemia FROM FEU)
C. hypocalcemia
D. hypercalcemia
E. hyponatremia

165 The contiguous ECG leads for the septal wall are: A. inferior wall… B. high lateral wall…. D..anterior HAROLD JAY S. MIDTERM 3
A. II, III, aVF wall. V5 V6 is lateral wall BAYTEC, MD (TOP EXAM - AUG
B. I, aVL 10 - FEB 2015 MED 2015
C. V1, V2 BOARDS;
D. V3, V4 TOPNOTCH MD
E. V4, V5 FROM FEU)

166 A patient in the ward is diagnosed with Multiple Pituitary neoplasia is a component of MEN 1 HAROLD JAY S. MIDTERM 3
Endocrine Neoplasia 2A. You are aware that this BAYTEC, MD (TOP EXAM - AUG
disease has the following components EXCEPT: 10 - FEB 2015 MED 2015
A. Medullary thyroid carcinoma BOARDS;
B. pheochromocytoma TOPNOTCH MD
C. hyperparathyroidism FROM FEU)
D. Pituitary neoplasia
E. none of the above

167 Which among the following is the most common HAROLD JAY S. MIDTERM 3
toxin associated with Dilated Cardiac Myopathy? BAYTEC, MD (TOP EXAM - AUG
A. Cigarette smoke 10 - FEB 2015 MED 2015
B. Alcohol BOARDS;
C. Shabu TOPNOTCH MD
D. Paracetamol FROM FEU)
E. Nifedipine

TOPNOTCH MEDICAL BOARD PREP INTERNAL MEDICINE SUPEREXAM Page 24 of 95


For inquiries visit www.topnotchboardprep.com.ph or email us at topnotchmedicalboardprep@gmail.com
TOPNOTCH MEDICAL BOARD PREP INTERNAL MEDICINE SUPEREXAM
For inquiries visit www.topnotchboardprep.com.ph or email us at topnotchmedicalboardprep@gmail.com
Item QUESTION EXPLANATION AUTHOR TOPNOTCH
# EXAM
168 The following are components of the classic triad for B, C, D are the classic components of Becks triad. HAROLD JAY S. MIDTERM 3
Cardiac Tamponade EXCEPT: Pulsus parodoxus can also be seen in Cardiac BAYTEC, MD (TOP EXAM - AUG
A. Pulsus parodoxus tamponade but it is not a part of the triad. 10 - FEB 2015 MED 2015
B. Decrease blood pressure BOARDS;
C. Neck vein engorgement TOPNOTCH MD
D. Muffled heart sound FROM FEU)
E. B and C

169 Janeway lesions, Osler's nodes, and Roth's spots are HAROLD JAY S. MIDTERM 3
usually seen in what disease? BAYTEC, MD (TOP EXAM - AUG
A. Rheumatic Fever 10 - FEB 2015 MED 2015
B. Kawasaki disease BOARDS;
C. Infective Endocarditis TOPNOTCH MD
D. SLE FROM FEU)
E. Dermatomyosis

170 Prolonged QT interval is usually caused by what HAROLD JAY S. MIDTERM 3


abnormality? BAYTEC, MD (TOP EXAM - AUG
A. hypokalemia 10 - FEB 2015 MED 2015
B. hyperkalemia BOARDS;
C. hypocalcemia TOPNOTCH MD
D. hypercalcemia FROM FEU)
E. Hypernatremia

171 A patient is currently on his 4th month of anti TB Isoniazid can cause peripheral neuropathy HAROLD JAY S. MIDTERM 3
regimen suddenly developed tingling sensations of BAYTEC, MD (TOP EXAM - AUG
his hands and feet. Which of the following drugs 10 - FEB 2015 MED 2015
might have caused the symptoms? BOARDS;
A. Rifampicin TOPNOTCH MD
B. Isoniazid FROM FEU)
C. Ethambutol
D. Pyrazynamide
E. Streptomycin

172 In relation to the above question, what should be only pyridoxine or B6 is given HAROLD JAY S. MIDTERM 3
given to improve the symptom? BAYTEC, MD (TOP EXAM - AUG
A. Vitamin B1 10 - FEB 2015 MED 2015
B. Vitamin B2 BOARDS;
C. Vitamin B6 TOPNOTCH MD
D. Vitamin B12 FROM FEU)
E. B Complex

173 which of the following drugs is LEAST likely to cause other choices are insulin secretagogue which can HAROLD JAY S. MIDTERM 3
hypoglycemia in a diabetic patient? cause hypoglycemia BAYTEC, MD (TOP EXAM - AUG
A. metformin 10 - FEB 2015 MED 2015
B. gliclazide BOARDS;
C. glibenclamide TOPNOTCH MD
D. repaglinide FROM FEU)
E. Nateglinide

174 This is a qualitative marker of HBV replication and HAROLD JAY S. MIDTERM 3
relative infectivity. Its disappearance may be a BAYTEC, MD (TOP EXAM - AUG
harbinger of clinical improvement and resolution of 10 - FEB 2015 MED 2015
infection BOARDS;
A. HBsAg TOPNOTCH MD
B. Anti HBs FROM FEU)
C. Anti HBc
D. HBV DNA
E. HBeAg

175 Which among the following is the most common HAROLD JAY S. MIDTERM 3
cause of acute pancreatitis BAYTEC, MD (TOP EXAM - AUG
A. Trauma 10 - FEB 2015 MED 2015
B. Drugs BOARDS;
C. gallstone TOPNOTCH MD
D. alcohol FROM FEU)
E. Smoking

176 what is the single most common risk factor for HAROLD JAY S. MIDTERM 3
hepatitis C? BAYTEC, MD (TOP EXAM - AUG
A. Injection drug use 10 - FEB 2015 MED 2015
B. hemodialysis BOARDS;
C. Promiscuous activity TOPNOTCH MD
D. Maternal-fetal transmission FROM FEU)
E. Unprotected sex

TOPNOTCH MEDICAL BOARD PREP INTERNAL MEDICINE SUPEREXAM Page 25 of 95


For inquiries visit www.topnotchboardprep.com.ph or email us at topnotchmedicalboardprep@gmail.com
TOPNOTCH MEDICAL BOARD PREP INTERNAL MEDICINE SUPEREXAM
For inquiries visit www.topnotchboardprep.com.ph or email us at topnotchmedicalboardprep@gmail.com
Item QUESTION EXPLANATION AUTHOR TOPNOTCH
# EXAM
177 This is the single BEST acute measure of hepatic HAROLD JAY S. MIDTERM 3
synthetic function and helpful in both the diagnosis BAYTEC, MD (TOP EXAM - AUG
and assessing prognosis of acute parenchymal liver 10 - FEB 2015 MED 2015
disease BOARDS;
A. Total albumin TOPNOTCH MD
B. LDH FROM FEU)
C. AST
D. ALT
E. Coagulation factors

178 All of the following are criteria for Systemic . Harrison’s 18th edition chapter 271 page 2223 HAROLD JAY S. MIDTERM 3
Inflammatory Response Syndrome or SIRS except table 271-1 and IM platinum page 297. according BAYTEC, MD (TOP EXAM - AUG
A. Fever of >38C to these sources tachypnea should be >24. I was 10 - FEB 2015 MED 2015
B. Hypothermia of <36C actually shocked as i crossed reference it with BOARDS;
C. Tachypnea of >24 other sources like Schwartz 9th ed and medscape TOPNOTCH MD
D. Tachycardia of >90 in which they considered tachypnea of >20. It is FROM FEU)
E. Leukocytosis>10,000 odd that harrisons and medscape have the same
source which is the american college of chest
physicians but they have different values
179 This is the most active drug against M leprae and HAROLD JAY S. MIDTERM 3
inhibits Folate synthesis. BAYTEC, MD (TOP EXAM - AUG
A. Dapsone 10 - FEB 2015 MED 2015
B. Rifampicin BOARDS;
C. Clofazimine TOPNOTCH MD
D. Isoniazid FROM FEU)
E. Sulfamethoxazole

180 Which among the following findings is consistent for CAP MR-- RR of 30 or above, PR of 125 or above, HAROLD JAY S. MIDTERM 3
Moderate Risk Community Acquired Pneumonia in a Temp of equal or more than 40 or less than or BAYTEC, MD (TOP EXAM - AUG
Diabetic patient? equal to 36, altered mental state of acute onset. 10 - FEB 2015 MED 2015
A. RR of 28 Suspected aspiration, decompensated BOARDS;
B. PR of 120 comorbidities. RBS of 300 means that there is TOPNOTCH MD
C. Temperature of 39C decompensation of the DM which poses more risk FROM FEU)
D. BP of 100/60 to infection
E. RBS of 300

181 What is the drug of choice for post-splenectomy SIMILAR TO PREVIOUS BOARD EXAM JEAN PAOLO M. FINAL EXAM -
sepsis? CONCEPT/PRINCIPLE.. Ceftriaxone is the drug of DELFINO, MD (TOP AUG 2015
A. Penicillin 20-30M units/day IV continuous choice for post-splenectomy sepsis 10 - FEB 2015 MED
infusion BOARDS;
B. Ceftriaxone 1-2g/day IV/IM OD TOPNOTCH MD
C. Cefuroxime 1.5g IV/IM q8 FROM FATIMA)
D. Vancomycin 500mg IV q6
E. Clindamycin 2g/day IV/IM q6-q12
182 In pleural effusion, the following are factors the following are factors indicating the likely need JEAN PAOLO M. FINAL EXAM -
indicating the likely need for a procedure more for a procedure more invasive than a DELFINO, MD (TOP AUG 2015
invasive than a thoracentesis except? thoracentesis: Loculated pleural fluid 10 - FEB 2015 MED
A. Loculated pleural fluid Pleural fluid pH <7.20 BOARDS;
B. Pleural fluid pH <7.20 Pleural fluid glucose <3.3 mmol/L (<60 mg/dL) TOPNOTCH MD
C. Pleural fluid glucose >3.3 mmol/L (<60 mg/dL) Positive Gram stain or culture of the pleural fluid FROM FATIMA)
D. Positive Gram stain or culture of the pleural fluid Presence of gross pus in the pleural space. There is
E. Presence of gross pus in the pleural space no typo in choice B.
the following are factors indicating the likely need
for a procedure more invasive than a
thoracentesis: Loculated pleural fluid
Pleural fluid pH <7.20
Pleural fluid glucose .3 mmol/L (<60 mg/dL)
Positive Gram stain or culture of the pleural fluid
Presence of gross pus in the pleural space

183 62 year old female presented with chronic cough Chest CT is more specific for bronchiectasis and is JEAN PAOLO M. FINAL EXAM -
and sputum production. Chest x-ray revealed "tram the imaging modality of choice for confirming the DELFINO, MD (TOP AUG 2015
tracks" appearance. What diagnostic test will you diagnosis. CT findings include airway dilation, lack 10 - FEB 2015 MED
request to confirm the diagnosis? of bronchial tapering, bronchial wall thickening in BOARDS;
A. Bronchoscopy dilated airways, inspissated secretions (e.g., the TOPNOTCH MD
B. Sputum culture "tree-in-bud" pattern), or cysts emanating from FROM FATIMA)
C. Pulmonary angiography the bronchial wall (especially pronounced in cystic
D. Chest CT scan bronchiectasis
E. No further tests necessary

184 The liver enzyme test of a 41 year old female patient Alkaline phosphatase, 5'nucleotidase, gamma JEAN PAOLO M. FINAL EXAM -
with jaundice revealed the following results: AST= glutamyl transpeptidase are enzymes that reflect DELFINO, MD (TOP AUG 2015
40 IU/L, ALT= 55 IU/L, alkaline phosphatase and cholestasis. If these enzymes are found to be 10 - FEB 2015 MED
gamma-glutamyl transpeptidase are elevated. What elevated, the 1st thing to do is ultrasound to look BOARDS;
should you request next? for dilated intra/extrahepatic biliary tree and to TOPNOTCH MD
A. serum albumin identify gallstones FROM FATIMA)
B. ultrasound
C. liver biopsy
D. prothrombin time
E. serum bilirubin

TOPNOTCH MEDICAL BOARD PREP INTERNAL MEDICINE SUPEREXAM Page 26 of 95


For inquiries visit www.topnotchboardprep.com.ph or email us at topnotchmedicalboardprep@gmail.com
TOPNOTCH MEDICAL BOARD PREP INTERNAL MEDICINE SUPEREXAM
For inquiries visit www.topnotchboardprep.com.ph or email us at topnotchmedicalboardprep@gmail.com
Item QUESTION EXPLANATION AUTHOR TOPNOTCH
# EXAM
185 What is the most common manifestation of Neisseria A nonblanching rash (petechial or purpuric) JEAN PAOLO M. FINAL EXAM -
meningitides infection? develops in >80% of cases of meningococcal DELFINO, MD (TOP AUG 2015
A. rash disease. 10 - FEB 2015 MED
B. headache BOARDS;
C. fever TOPNOTCH MD
D. bleeding FROM FATIMA)
E. hypotension

186 The following are expected laboratory findings in Tumor lysis syndrome (TLS) is characterized by JEAN PAOLO M. FINAL EXAM -
tumor lysis syndrome except? hyperuricemia, hyperkalemia, DELFINO, MD (TOP AUG 2015
A. Hyperuricemia hyperphosphatemia, and hypocalcemia and is 10 - FEB 2015 MED
B. Hyperphosphatemia caused by the destruction of a large number of BOARDS;
C. Hypocalcemia rapidly proliferating neoplastic cells. It is most TOPNOTCH MD
D. Hyperkalemia often associated with treatment of leukemia. FROM FATIMA)
E. None of the above

187 27 year old male patient presented with headache Hypertension + hypokalemia = Conns disease JEAN PAOLO M. FINAL EXAM -
and weakness. BP is elevated, other PE findings are DELFINO, MD (TOP AUG 2015
normal. CBC within normal limits, Na= 149, K= 2.5. 10 - FEB 2015 MED
What is your primary consideration in this case? BOARDS;
A. Conn's Syndrome TOPNOTCH MD
B. Essential Hypertension FROM FATIMA)
C. Cushing Syndrome
D. Cushing's Disease
E. Pheochromocytoma

188 60 year old female patient, with no known mnemonics for myxedema coma: 5H- JEAN PAOLO M. FINAL EXAM -
comorbities suddenly became lethargic and hypoglycemia, hyponatremia, hypothermia, DELFINO, MD (TOP AUG 2015
progressed to coma. According to her relatives, they hypoventilation, hypotension 10 - FEB 2015 MED
noticed that the patient had long-standing fatigue, BOARDS;
cold intolerance and weight gain. Impression was TOPNOTCH MD
myxedema coma. What other findings would you FROM FATIMA)
expect to see in this patient?
A. BP 140/90
B. Temp= 38 deg C
C. RR= 28
D. Na= 132
E. hyperglycemia

189 Chikungunya virus belongs to what family? Chikungunya belongs to togaviridae family. JEAN PAOLO M. FINAL EXAM -
A. Reoviridae DELFINO, MD (TOP AUG 2015
B. Flaviviridae 10 - FEB 2015 MED
C. Togaviridae BOARDS;
D. Herpesviridae TOPNOTCH MD
E. Filoviridae FROM FATIMA)

190 What is the most common location of JEAN PAOLO M. FINAL EXAM -
extrapulmonary TB? DELFINO, MD (TOP AUG 2015
A. Lymph node 10 - FEB 2015 MED
B. Bones BOARDS;
C. Intestines TOPNOTCH MD
D. Ovary FROM FATIMA)
E. Brain

191 Patient presented with pale palpebral conjunctiva TIBC is increased in iron deficiency anemia JEAN PAOLO M. FINAL EXAM -
and skin pallor. Hgb Hct were decreased. PBS DELFINO, MD (TOP AUG 2015
showed decreased MCV, MCH. If you are suspecting 10 - FEB 2015 MED
iron deficiency anemia, which of the following is an BOARDS;
unlikely finding? TOPNOTCH MD
A. Decreased serum iron FROM FATIMA)
B. Decreased TIBC
C. Decreased ferritin
D. B and C
E. All of the above
192 A patient comes to you with the following laboratory p.245 IM platinum 1st edition JEAN PAOLO M. FINAL EXAM -
finding- Anti-HAV (-), Anti-HCV (-), HBsAg (-), Anti- DELFINO, MD (TOP AUG 2015
HBs (+), Anti-HBc (+), HBeAg (-), Anti-Hbe (+). What 10 - FEB 2015 MED
is your interpretation? BOARDS;
A. Patient has hepatitis A TOPNOTCH MD
B. Recovery from hepatitis B FROM FATIMA)
C. Vaccination with hepatitis B
D. Chronic hepatitis B with high infectivity
E. Acute hepatitis B with high infectivity

193 What is the most common cause of systolic CAD is the the most common cause of systolic JEAN PAOLO M. FINAL EXAM -
dysfunction that leads to left-sided heart failure? dysfunction that leads to left-sided heart failure. DELFINO, MD (TOP AUG 2015
A. Valvular heart disease VHD and dilated cardiomyopathy are other causes 10 - FEB 2015 MED
B. Dilated cardiomyopathy of systolic heart failure BOARDS;
C. Coronary artery disease TOPNOTCH MD
D. Thyrotoxicosis FROM FATIMA)
E. Aging

TOPNOTCH MEDICAL BOARD PREP INTERNAL MEDICINE SUPEREXAM Page 27 of 95


For inquiries visit www.topnotchboardprep.com.ph or email us at topnotchmedicalboardprep@gmail.com
TOPNOTCH MEDICAL BOARD PREP INTERNAL MEDICINE SUPEREXAM
For inquiries visit www.topnotchboardprep.com.ph or email us at topnotchmedicalboardprep@gmail.com
Item QUESTION EXPLANATION AUTHOR TOPNOTCH
# EXAM
194 Hypovolemic hyponatremia is seen in which of the JEAN PAOLO M. FINAL EXAM -
following condition? DELFINO, MD (TOP AUG 2015
A. Addison's Disease 10 - FEB 2015 MED
B. Conn's Syndrome BOARDS;
C. SIADH TOPNOTCH MD
D. Diabetes insipidus FROM FATIMA)
E. Diuretic therapy

195 A patient has easy fatigability, easy bruising, Non-suppression in low dose dexamethasone test JEAN PAOLO M. FINAL EXAM -
hypertension, central obesity and moon facies. You confirms cushing syndrome. High dose DELFINO, MD (TOP AUG 2015
suspect Cushing Syndrome. After giving low dose dexamethasone test is used to determine the cause 10 - FEB 2015 MED
dexamethasone, cortisol levels were not suppressed. of elevated cortisol. If cortisol is suppressed in BOARDS;
After high dose dexamethasone test, cortisol levels High dose dexamethasone test, then a pituitary TOPNOTCH MD
decreased. What should you request next? cause is considered, so you should request for a FROM FATIMA)
A. Cranial MRI cranial MRI
B. Abdominal CT scan
C. Inferior petrosal sinus sampling
D. Abdominal ultrasound
E. Chest X-ray

196 What is the most important virulence factor of E.coli strains of E. coli that cause invasive symptomatic JEAN PAOLO M. FINAL EXAM -
in causing urinary tract infection? infection of the urinary tract in otherwise normal DELFINO, MD (TOP AUG 2015
A. capsule hosts often possess and express genetic virulence 10 - FEB 2015 MED
B. endotoxin factors, including surface adhesins that mediate BOARDS;
C. P-fimbriae binding to specific receptors on the surface of TOPNOTCH MD
D. enterotoxin uroepithelial cells. The best-studied adhesins are FROM FATIMA)
E. H-antigen the P fimbriae, hairlike protein structures that
interact with a specific receptor on renal epithelial
cells.
197 A 25 year old male complains of arthritis and eye Diagnosis is Reiter's syndrome. It usually develops JEAN PAOLO M. FINAL EXAM -
irritation. There was previous episodes of burning after a non-gonococcal urethritis infection or GI DELFINO, MD (TOP AUG 2015
sensation upon urination. PE revealed swelling of tract infection with the following organisms, 10 - FEB 2015 MED
the right knee and a lesion in the glans penis. Which Salmonella, Shigella, Campylobacter, Yersinia BOARDS;
of the following is correct? enterocolitica. TOPNOTCH MD
A. Culture from the glans penis lesion would yield FROM FATIMA)
Neisseria gonorrhea
B. Rheumatoid factor is likely to be positive
C. Patient had a GI tract infection several weeks
ago
D. ANA is positive
E. None of the above
198 40 year old female develops severe pain and S.aureus is the most common cause of septic JEAN PAOLO M. FINAL EXAM -
swelling in the right elbow. She is not sexually active. arthritis in adults. If the patient is sexually active, DELFINO, MD (TOP AUG 2015
You suspect septic arthritis. What is the most likely then N.gonorrhea is the most likely cause. 10 - FEB 2015 MED
organism to cause septic arthritis in this case? BOARDS;
A. Neisseria gonorrhea TOPNOTCH MD
B. Escherichia coli FROM FATIMA)
C. Streptococcus pneumoniae
D. Streptococcus pyogenes
E. Staphylococcus aureus

199 30 year old male patient developed bipedal edema Diagnosis is MGN. Secondary membranous JEAN PAOLO M. FINAL EXAM -
and hypertension. Work-up revealed glomerulonephritis causes are the following: DELFINO, MD (TOP AUG 2015
hypoalbuminemia, proteinuria, hematuria and Infection: Hepatitis B and C, syphilis, malaria, 10 - FEB 2015 MED
hypercholesterolemia. Liver enzymes are normal. schistosomiasis, leprosy, filariasis BOARDS;
Renal biopsy revealed uniform thickening of the Cancer: Breast, colon, lung, stomach, kidney, TOPNOTCH MD
basement membrane along the peripheral capillary esophagus, neuroblastoma FROM FATIMA)
loops seen by light microscopy. Subepithelial Drugs: gold, mercury, penicillamine,
deposits are seen in electron microscopy. This nonsteroidal anti-inflammatory agents,
condition is associated with which of the following? probenecid
A. Schistosomiasis
B. Gastric cancer
C. Hepatitis B
D. All of the above
E. B and C

200 Patient with pallor is evaluated for anemia. CBC Anemia with elevated reticulocyte index is caused JEAN PAOLO M. FINAL EXAM -
revealed decreased hemoglobin and hematocrit. by any hemolytic process or by hemorrhage. DELFINO, MD (TOP AUG 2015
MCV and MCH are both normal. Reticulocyte index is 10 - FEB 2015 MED
greater than 2.5. Which of the following is not BOARDS;
considered in the differential diagnosis? TOPNOTCH MD
A. Paroxysmal Nocturnal Hemoglobinuria FROM FATIMA)
B. Hereditary spherocytosis
C. Autoimmune hemolytic anemia
D. Blood loss
E. None of the above

TOPNOTCH MEDICAL BOARD PREP INTERNAL MEDICINE SUPEREXAM Page 28 of 95


For inquiries visit www.topnotchboardprep.com.ph or email us at topnotchmedicalboardprep@gmail.com
TOPNOTCH MEDICAL BOARD PREP INTERNAL MEDICINE SUPEREXAM
For inquiries visit www.topnotchboardprep.com.ph or email us at topnotchmedicalboardprep@gmail.com
Item QUESTION EXPLANATION AUTHOR TOPNOTCH
# EXAM
201 The diagnosis of metabolic syndrome rests on Instead of LDL, FBS is the fifth criterion in the GRACE ARVIOLA, DIAGNOSTIC
measurement of the following parameters EXCEPT: diagnosis of metabolic syndrome. MD (TOP 3 - AUG EXAM - FEB
A. Triglycerides 2014 MED 2015
B. Waist circumference BOARDS;
C. Blood pressure TOPNOTCH MD)
D. HDL
E. LDL

202 When and how often is lipid screening among adults Current ATP III guidelines recommend screening GRACE ARVIOLA, DIAGNOSTIC
recommended? in all adults >20 years of age. The screen should MD (TOP 3 - AUG EXAM - FEB
A. Start at 20 years old, then annually include a fasting lipid profile repeated every 5 2014 MED 2015
B. Start at 20 years old, then every 5 years years. BOARDS;
C. Start at 30 years old, then annually TOPNOTCH MD)
D. Start at 30 years old, then every 5 years
E. Start at 40 years old, then annually

203 Auscultation of a patient with a known valvular This is Carvallo sign of tricuspid regurgitation. GRACE ARVIOLA, DIAGNOSTIC
heart defect revealed a holosystolic murmur noted MD (TOP 3 - AUG EXAM - FEB
at the left parasternal border. The murmur 2014 MED 2015
characteristically increased in intensity during BOARDS;
inspiration. The valve affected is: TOPNOTCH MD)
A. Aortic valve
B. Pulmonary valve
C. Mitral valve
D. Tricuspid valve
E. Both aortic and mitral valve

204 Standard treatment of a patient with suspected The standard recommendation for patients with GRACE ARVIOLA, DIAGNOSTIC
multidrug-resistant healthcare-associated risk factors for MDR infection is for three MD (TOP 3 - AUG EXAM - FEB
pneumonia consist of: antibiotics: two directed at P> aeruginosa and one 2014 MED 2015
A. 1 Anti-pseudomonal + 1 MRSA-active antibiotics at MRSA. BOARDS;
B. 1 Anti-pseudomonal + 2 MRSA-active antibiotics TOPNOTCH MD)
C. 2 Anti-pseudomonal + 1 MRSA-active antibiotics
D. 2 Anti-pseudomonal + 2 MRSA-active antibiotics
E. None of the above

205 Adenosine deaminase is used in the diagnosis of: Determination of the pleural concentration of GRACE ARVIOLA, DIAGNOSTIC
A. TB pleural effusion adenosine deaminase (ADA) is a useful screening MD (TOP 3 - AUG EXAM - FEB
B. Lung adenocarcinoma test: tuberculosis is virtually excluded if the value 2014 MED 2015
C. COPD is very low. BOARDS;
D. Asthma TOPNOTCH MD)
E. Silicosis

206 A 56 year-old patient suffering from sudden onset of This is McConnell's sign of pulmonary embolism. GRACE ARVIOLA, DIAGNOSTIC
dyspnea underwent 2D-echocardiography. Imaging MD (TOP 3 - AUG EXAM - FEB
showed a hypokinetic right ventricular free wall 2014 MED 2015
with a normal apex. This indirect sign is highly BOARDS;
suggestive of: TOPNOTCH MD)
A. COPD exacerbation
B. Bronchial asthma
C. Pulmonary embolism
D. Congestive heart failure
E. Malignant pleural effusion
207 In the computation of a patient's corrected plasma Plasma Na concentration falls by 1.6 to 2.4 mM for GRACE ARVIOLA, DIAGNOSTIC
sodium concentration, what other laboratory value every 100 mg/dL increase in glucose due to MD (TOP 3 - AUG EXAM - FEB
must be considered? glucose-induced water efflux from cells. This true 2014 MED 2015
A. Albumin hyponatremia resolves after correction of BOARDS;
B. Calcium ion concentration hyperglycemia. TOPNOTCH MD)
C. Random blood sugar
D. Potassium
E. Blood pH

208 The anemia seen in chronic kidney disease is A normocytic, normochromic anemia is observed GRACE ARVIOLA, DIAGNOSTIC
usually: as early as CKD stage 3 and is almost universal by MD (TOP 3 - AUG EXAM - FEB
A. Microcytic, hypochromic stage 4. 2014 MED 2015
B. Normocytic, normochromic BOARDS;
C. Macrocytic, hyperchromic TOPNOTCH MD)
D. Normocytic, hypochromic
E. Microcytic, hyperchromic

209 Which of the following conditions does NOT Studies have shown that low-calcium diets GRACE ARVIOLA, DIAGNOSTIC
predispose to urolithiasis? increase the risk of incident stone formation, MD (TOP 3 - AUG EXAM - FEB
A. Bacterial infection perhaps by reducing the amount of calcium in the 2014 MED 2015
B. Dehydration intestine to bind oxalate, thereby incerasing urine BOARDS;
C. Metabolic syndrome oxalate levels. TOPNOTCH MD)
D. Diet high in calcium
E. Gout

TOPNOTCH MEDICAL BOARD PREP INTERNAL MEDICINE SUPEREXAM Page 29 of 95


For inquiries visit www.topnotchboardprep.com.ph or email us at topnotchmedicalboardprep@gmail.com
TOPNOTCH MEDICAL BOARD PREP INTERNAL MEDICINE SUPEREXAM
For inquiries visit www.topnotchboardprep.com.ph or email us at topnotchmedicalboardprep@gmail.com
Item QUESTION EXPLANATION AUTHOR TOPNOTCH
# EXAM
210 In the management of diabetic ketoacidosis, what If the initial serum potassium is <3.3 mEq/L, do GRACE ARVIOLA, DIAGNOSTIC
condition can potentially withold the administration not administer insulin until the potassium is MD (TOP 3 - AUG EXAM - FEB
of insulin? corrected. Administering insulin in this setting will 2014 MED 2015
A. Hypernatremia aggravate the hypokalemia as insulin causes BOARDS;
B. Hypokalemia intracellular shift of potassium. TOPNOTCH MD)
C. Metabolic acidosis
D. Impaired mental status
E. Severe hyperglycemia

211 After instituting treatment, when do you expect the The TSH response is gradual and should be GRACE ARVIOLA, DIAGNOSTIC
TSH response to thyroid hormone replacement measured about two months after instituting MD (TOP 3 - AUG EXAM - FEB
therapy in cases of hypothyroidism to occur? treatment or after any subsequent change in 2014 MED 2015
A. 2 weeks levothyroxine dosage. BOARDS;
B. 4 weeks TOPNOTCH MD)
C. 2 months
D. 6 months
E. 1 year

212 Which laboratory finding is NOT consistent with If the defect is due to pituitary, gestational, or GRACE ARVIOLA, DIAGNOSTIC
diabetes insipidus? nephrogenic DI, the polyuria results in a small (1– MD (TOP 3 - AUG EXAM - FEB
A. Urine output >50 mL/kg/day 2%) decrease in body water and a commensurate 2014 MED 2015
B. Urine osmolarity <300 mOsm/L increase in plasma osmolarity and sodium BOARDS;
C. Blood pressure of 90/60 concentration that stimulate thirst and a TOPNOTCH MD)
D. Plasma osmolarity of 290 mOsm/L compensatory increase in water intake. As a
E. Plasma sodium ion concentration of 160 mEq/L result, hypernatremia and other overt physical or
laboratory signs of dehydration do not develop
unless the patient also has a defect in thirst or fails
to drink for some other reason.
213 A duodenal ulcer is considered refractory if it is not A refractory gastric ulcer is one that fails to heal GRACE ARVIOLA, DIAGNOSTIC
healed after how many weeks of therapy? after 12 weeks of therapy. MD (TOP 3 - AUG EXAM - FEB
A. 4 weeks 2014 MED 2015
B. 6 weeks BOARDS;
C. 8 weeks TOPNOTCH MD)
D. 10 weeks
E. 12 weeks

214 The Rome II criteria is used to diagnose: Recurrent abdominal pain or discomfort at least 3 GRACE ARVIOLA, DIAGNOSTIC
A. Crohn's disease days per month in the last three months MD (TOP 3 - AUG EXAM - FEB
B. Exudative pleural effusion associated with two or more of the following: 1] 2014 MED 2015
C. Infective endocarditis improvement with defecation; 2] onset associated BOARDS;
D. Acute kidney injury with a change in frequency of stool; and 3] onset TOPNOTCH MD)
E. Irritable bowel syndrome associated with a change in form (appearance) of
stool

215 This is the antibiotic of choice in cases of Treatment is with a second-generation GRACE ARVIOLA, DIAGNOSTIC
spontaneous bacterial peritonitis. cephalosporin, with cefotaxime being the most MD (TOP 3 - AUG EXAM - FEB
A. Metronidazole commonly used. 2014 MED 2015
B. Clindamycin BOARDS;
C. Amikacin TOPNOTCH MD)
D. Cefotaxime
E. TMP-SMZ

216 What is the standard test for CSF examination in The VDRL test remains the standard for examining GRACE ARVIOLA, DIAGNOSTIC
suspected cases of neurosyphilis? CSF. It is highly specific, and when reactive, is MD (TOP 3 - AUG EXAM - FEB
A. RPR considered diagnostic of neurosyphilis. However, 2014 MED 2015
B. VDRL this test is insensitive and may be nonreactive BOARDS;
C. MHA-TP even in cases of symptomatic neurosyphilis. TOPNOTCH MD)
D. FTA-ABS
E. Biopsy

217 In an asymptomatic HIV-positive patient, at what At present, a reasonable course of action is to GRACE ARVIOLA, DIAGNOSTIC
CD4 count is antiretroviral therapy warranted? initiate antiretroviral therapy in anyone with the MD (TOP 3 - AUG EXAM - FEB
A. <500 acute HIV syndrome; all pregnant women; patients 2014 MED 2015
B. <450 with an AIDS-defining illness; patients with HIV- BOARDS;
C. <400 associated nephropathy; patients with hepatitis B TOPNOTCH MD)
D. <350 infection when treatment for hepatitis B is
E. <300 indicated; and patients with asymptomatic disease
with CD4+ T-cell counts of <500 u/L. NOTE: in
Harrison's 17th edition, the cut-off value was <350
u/L.
218 Which form of arthritis carries the highest incidence Patients with RA have the highest incidence of GRACE ARVIOLA, DIAGNOSTIC
of infective arthritis? infective arthritis (most often secondary to S. MD (TOP 3 - AUG EXAM - FEB
A. Ankylosing spondylitis aureus) because of chronically inflamed joints, 2014 MED 2015
B. Lupus arthritis glucocorticoid therapy, and frequent breakdown BOARDS;
C. Rheumatoid arthritis of rheumatoid nodules, vasculitic ulcers, and skin TOPNOTCH MD)
D. Gouty arthritis overlying deformed joints.
E. Osteoarthritis

TOPNOTCH MEDICAL BOARD PREP INTERNAL MEDICINE SUPEREXAM Page 30 of 95


For inquiries visit www.topnotchboardprep.com.ph or email us at topnotchmedicalboardprep@gmail.com
TOPNOTCH MEDICAL BOARD PREP INTERNAL MEDICINE SUPEREXAM
For inquiries visit www.topnotchboardprep.com.ph or email us at topnotchmedicalboardprep@gmail.com
Item QUESTION EXPLANATION AUTHOR TOPNOTCH
# EXAM
219 Falsely normal vitamin B12 levels are seen in: Bacterial overgrowth, liver disease, and GRACE ARVIOLA, DIAGNOSTIC
A. Pregnancy myeloproliferative disorders cause a falsely MD (TOP 3 - AUG EXAM - FEB
B. Folate deficiency normal vitamin B12 level. Folate deficiency, 2014 MED 2015
C. OCP use pregnancy, and OCP use cause falsely low vitamin BOARDS;
D. Bacterial overgrowth B12 levels. TOPNOTCH MD)
E. Phenytoin use

220 Pure red cell aplasia is associated with what tumor? A small minority of pure red cell aplasia cases GRACE ARVIOLA, DIAGNOSTIC
A. Small cell carcinoma of the lung occur with a thymoma. Resection of the thymoma MD (TOP 3 - AUG EXAM - FEB
B. Squamous cell carcinoma of the lung leads to hematologic improvement in about half of 2014 MED 2015
C. Thymoma the patients, possibly because the tumor is a BOARDS;
D. Colon adenocarcinoma source of marrow suppressive cells. TOPNOTCH MD)
E. Renal cell carcinoma

221 which of the following is a class I recommendation Class I indication for aspirin or warfarin therapy in LEAN ANGELO MIDTERM
for the use of aspirin and oral anticoagulants in patients with MVP: ASA for cerebral TIA; warfarin SILVERIO, MD EXAM 1 - FEB
mitral valve prolapse? therapy for patients aged >65y/o in atrial (TOP 4 - AUG 2014 2015
A. Warfarin therapy after stroke fibrillation with hypertension, mitral regurgitation MED BOARDS;
B. Aspirin therapy for patients in sinus rhythm or hx of heart failure. ASA for patients <65 y/o TOPNOTCH MD),
with echocardiographic evidence of high risk mitral with atrial fibrillation, warfarin therapy after MD
valve prolapse stroke. SIMILAR TO PREVIOUS BOARD EXAM
C. overlap warfarin therapy along with aspirin for CONCEPT/PRINCIPLE
patient with transient ischemic attack
D. aspirin therapy after stroke in patients with
contraindication to anticoagulants
E. all of the above

222 which of the following is a false correlation between finding of a large fused cv wave indicates tricuspid LEAN ANGELO MIDTERM
JVP waves and its indicated condition? regurgitation. Tricuspid stenosis produces large a SILVERIO, MD EXAM 1 - FEB
A. Large fused cv wave - tricuspid stenosis wave in JVP (TOP 4 - AUG 2014 2015
B. Rapid x+y descent - constrictive pericarditis MED BOARDS;
C. Kussmaul sign - pericardial tamponade, RV TOPNOTCH MD),
failure MD
D. Cannon a wave- complete heart block
E. large a wave - primary pulmonary hypertension

223 which of the following is true about chronic stable LEAN ANGELO MIDTERM
angina ? SILVERIO, MD EXAM 1 - FEB
A. The most important predictor of prognosis is left (TOP 4 - AUG 2014 2015
ventricular function MED BOARDS;
B. Myocardial ischemia is caused by increased TOPNOTCH MD),
myocardial oxygen demand MD
C. Resting blood flow does not cause ischemia
unless stenosis is >95%
D. during ischemia, diastolic dysfunction precedes
ECG changes and regional wall motion abnormalities
E. all of the above
224 which of the following is an absolute absolute contraindication for fibrinolysis in LEAN ANGELO MIDTERM
contraindication for fibrinolysis in STEMI? STEMI: any prior ICH, known structural SILVERIO, MD EXAM 1 - FEB
A. Thalamic hemorrhage 10 years ago cerebrovascular lesion, known malignant (TOP 4 - AUG 2014 2015
B. Current use of anticoagulants intracranial neoplasm, ischemic stroke within 3 MED BOARDS;
C. Active peptic ulcer months, suspected aortic dissection, active TOPNOTCH MD),
D. Traumatic or prolonged (>10mins) CPR or bleeding or bleeding diatheses ( excluding MD
major surgery (<3 weeks) menses), significant closed head or facial trauma
E. all of the above within 3months.

225 the following statement is true about acromegaly measuring the serum concentration of IGF-1 is the LEAN ANGELO MIDTERM
except? best screening test for acromegaly. It is always SILVERIO, MD EXAM 1 - FEB
A. Increased risk of premalignant colon polyps and increased in patient with active acromegaly. (TOP 4 - AUG 2014 2015
colon cancer Physiologic increased is seen in the following MED BOARDS;
B. A random serum level of GH is not helpful in condition: pregnancy, adolescence, sleep apnea. TOPNOTCH MD),
establishing the diagnosis MD
C. Surgical excision is the treatment of choice
D. serum concentration of IGF1 is increased in only
50% of patient with active acromegaly
E. none of the above
226 which of the following statement is true about psychogenic polydipsia does not present with LEAN ANGELO MIDTERM
diabetes insipidus except? nocturia. This is the most distinguishing symptom SILVERIO, MD EXAM 1 - FEB
A. The absence of response to water deprivation ( that separates it from a true structural or (TOP 4 - AUG 2014 2015
urine osmolality <300mosm/kg) is diagnostic of physiologic abnormality. MED BOARDS;
diabetes insipidus TOPNOTCH MD),
B. Cortisol is necessary hormone for kidney in MD
terms of excreting excess water load
C. the presence of nocturia cannot rule out
psychogenic polydipsia
D. central diabetes insipidus can be distinguished
from nephrogenic type by the response to
exogenous desmopressin
E. none of the above

TOPNOTCH MEDICAL BOARD PREP INTERNAL MEDICINE SUPEREXAM Page 31 of 95


For inquiries visit www.topnotchboardprep.com.ph or email us at topnotchmedicalboardprep@gmail.com
TOPNOTCH MEDICAL BOARD PREP INTERNAL MEDICINE SUPEREXAM
For inquiries visit www.topnotchboardprep.com.ph or email us at topnotchmedicalboardprep@gmail.com
Item QUESTION EXPLANATION AUTHOR TOPNOTCH
# EXAM
227 in severe hypercalcemia wherein the primary cause in cases of sever hypercalcemia with unknown LEAN ANGELO MIDTERM
cannot be detected, what is the most important primary, calcium concentration should be SILVERIO, MD EXAM 1 - FEB
therapeutic intervention ? decreased, aggressive rehydration with volume (TOP 4 - AUG 2014 2015
A. dialysis espansion promotes calciuresis or saline diuresis MED BOARDS;
B. Calcitonin IV and is the most important therapeutic TOPNOTCH MD),
C. Pamidronate plus loop diuretics intervention. Loop diuretics helps promote MD
D. Saline diuresis calcium excretion only after rehydration is done.
E. All of the above is important pamidronate decreases calcium concentration by
inhibiting bone resorption and has a marked and
prolonged effect on calcium concentration,
however its effect is delayed. dialysis is reserved
for patients with renal failure while calcitonin is
used rarely because of its modest effect andthe
rapid onset of tachyphylaxis.
228 The following condition are associated with type A type A gastritis involves the fundus and/or body of LEAN ANGELO MIDTERM
gastritis except ? the stomach. It is associated with autoimmune, SILVERIO, MD EXAM 1 - FEB
A. Gastric carcinoids atrophic gastritis and pernicious anemia. The (TOP 4 - AUG 2014 2015
B. Pernicious anemia serum levels of gastrin since achlorydia develops MED BOARDS;
C. H pylori infection in this patient. Gastrin is potent growth factor for TOPNOTCH MD),
D. Atrophic gastritis the development of gastric carcinoid tumors and MD
E. Hypergastrinemia gastric polyps. on the otherhand, type B gastritis
involves primarily the antrum and it is associated
with H pylori infection.
229 True about inflammatory bowel disease except? Ulcerative colitis is a continuous inflammatort LEAN ANGELO MIDTERM
A. ulcerative colitis is a continuous process process that etends from the anal verge to the SILVERIO, MD EXAM 1 - FEB
B. Ulcerative colitis is associated with intestinal more proximal colon. Crohns is a segmental type (TOP 4 - AUG 2014 2015
fistula, strictures, and perianal disease of inflammation. UC does not form fistules and MED BOARDS;
C. elevated levels of alkaline phosphatase in a perianal disease is uncommon. Stricture of the TOPNOTCH MD),
patient with UC suggest the presence of primary intestine are common with Crohns but rate in UC. MD
schlerosing colangitis presence of it mandates ruling out carcinoma.
D. the presence of strictures in patient with active
UC suggest colonic adenocarcinoma
E. none of the above

230 which of the following is a poor prognostic indicator poor prognostic markers for alcoholic hepatitis: LEAN ANGELO MIDTERM
for alcoholic hepatitis? Discriminant Function of >32, encephalopathy, SILVERIO, MD EXAM 1 - FEB
A. Discriminant function of 28 ascites, renal failure, prolonged PT, bilirubin (TOP 4 - AUG 2014 2015
B. Prolonged PT >20mg/dl. MED BOARDS;
C. Total bilirubin of 18mg/dl TOPNOTCH MD),
D. AST > 400U/L MD
E. Severe jaundice

231 which of the folowing condition is associated with A SAAG value of >1.1g/dl is almost always LEAN ANGELO MIDTERM
SAAG >1.1g/dl? indicate portal hypertension. SILVERIO, MD EXAM 1 - FEB
A. Nephrotic syndrome (TOP 4 - AUG 2014 2015
B. Colonic malignancy MED BOARDS;
C. Genitourinary TB TOPNOTCH MD),
D. Right sided heart failure MD
E. All of the above

232 what is the primary treatment for thrombotic TTP is a type of microangiopathies wherein there LEAN ANGELO MIDTERM
thrombocytopenic purpura? is a deficiency in the vWF cleaving protease SILVERIO, MD EXAM 1 - FEB
A. Aspirin plus dipyridamole (ADAMTS13). This result to large vWF multimers (TOP 4 - AUG 2014 2015
B. Platelet transfusion in the plasma promoting aggregant effect. The only MED BOARDS;
C. plasmapharesis treatment for this case is plasma exchange ( TOPNOTCH MD),
D. Intravenous IVIG plasmapheresis with infusion of FFP or MD
E. Splenectomy cryosupernatant infusion. platelet transfusion is
generally contraindicated unless invasive
procedures is required. other ancillary treatment
however with unknown benefit include:
dipyridamole, ASA, prednisone. IVIG is reserved
for refractory cases.
233 A 67 y/o male was worked up for persistent MGUS is the most common dysproproteinemia, M LEAN ANGELO MIDTERM
proteinemia. The laboratory results showed the ff: M protein is < 3g/dl and the plasma cell is < 10% ( SILVERIO, MD EXAM 1 - FEB
protein 2.4 g/dl, BMA 8% plasma cells, serum MM >10% BMA plasma cell). Patient is (TOP 4 - AUG 2014 2015
creatinine 0.9mg/dl, Hgb 145, bone radiograph asymptomatic. Serum creatinine, hemoglobin and MED BOARDS;
shows osteoporotic changes of the vertebra. what is bone radiographs are normal. TOPNOTCH MD),
your primary diagnosis? MD
A. Multiple myeloma
B. waldenstrom macroglobulinemia
C. MGUS
D. plasmacytoma
E. none of the above
234 A 47 y/o male asymptomatic presents in your clinic this is a classic case of CML. Characterized by LEAN ANGELO MIDTERM
with an incidental finding of WBC count 110x109 /L, increased WBC count, granulocytes in all stages of SILVERIO, MD EXAM 1 - FEB
basophilia, eosinophilia and obliterated traube maturation. Presence of philadelphia chromosome (TOP 4 - AUG 2014 2015
space. Cytogenetic studies showed t(9,22). Which of is the hallmark for this condition. This is caused by MED BOARDS;
the following is true about his condition except? translocation of bcr -abl gene. Leukocyte alkaline TOPNOTCH MD),
A. the standard therapy for this condition is phosphatase ( marker of functional activity of MD
Imatinib granulocytes) is low compared to reactive
B. leukocyte alkaline phosphatase score is low or leukocytosis. there is an increaesed Vitamin B12
zero level due to increased level of transcobalamin I.
C. increased serum Vitamin B12 level Imatinib is the treatment of choice.
D. philadelphia chromosome is the hallmark of this
condition
E. all of the above
TOPNOTCH MEDICAL BOARD PREP INTERNAL MEDICINE SUPEREXAM Page 32 of 95
For inquiries visit www.topnotchboardprep.com.ph or email us at topnotchmedicalboardprep@gmail.com
TOPNOTCH MEDICAL BOARD PREP INTERNAL MEDICINE SUPEREXAM
For inquiries visit www.topnotchboardprep.com.ph or email us at topnotchmedicalboardprep@gmail.com
Item QUESTION EXPLANATION AUTHOR TOPNOTCH
# EXAM
235 68 y/o male with DM nephropathy with creatinine of proteinuria with negative urine dipstick LEAN ANGELO MIDTERM
2.4 then suddenly came in creatinine of 9 mg/dl accompanied by low back pain and suddenc SILVERIO, MD EXAM 1 - FEB
without any other hx of blood loss or diarrhea. he increase in creatinine is a classic manifestation of (TOP 4 - AUG 2014 2015
also have anemia and Low back pain in the past 6 myeloma kidney. MED BOARDS;
mo, 24 hr urine protein showed proteinuria TOPNOTCH MD),
iof10g/day, the urine dipstick is negative ? MD
A. Multiple Myeloma
B. FSGS
C. RPGN
D. Uremia
E. any of the above

236 which laboratory finding will distinguish acute Acute tubular necrosis: Urine Osm - <350 LEAN ANGELO MIDTERM
tubular necrosis from pre renal azotemia? mOsm/L; BUN/Crea ratio - <15; urine Na >30 SILVERIO, MD EXAM 1 - FEB
A. Urine spgr 1.031 mg/dl; FeNa >3% (TOP 4 - AUG 2014 2015
B. BUN/Creatinine 30:1 MED BOARDS;
C. Urine osmolality of 300 mOsm/L TOPNOTCH MD),
D. FeNa% 0.8% MD
E. Urine Na 20mEq/L

237 which of the following will decrease in urinary loss of antithrombin is main causative LEAN ANGELO MIDTERM
glomerulonephritis presenting with nephrotic factor why nephrotic syndrome is prone to SILVERIO, MD EXAM 1 - FEB
syndrome? thrombotic states. Loss of albumin leads to (TOP 4 - AUG 2014 2015
A. Urinary excretion of antithrombin III compensatory increase in the synthesis of MED BOARDS;
B. Atrial natriuretic peptide lipoprotein leading to increase in VLDL fraction. TOPNOTCH MD),
C. vasopressin ADH is increase as well as aldosterone to MD
D. VLDL compensate for sodium and water loss. ANP is
E. None of the above decreased at this state.

238 which of the following is not a manifestation of fanconi syndrome is a renal disease affecting the LEAN ANGELO MIDTERM
Fanconi Syndrome? proximal renal tubule. Glucose, amino acid, SILVERIO, MD EXAM 1 - FEB
A. anemia phosphate,uric acid,bicarbonate are completely (TOP 4 - AUG 2014 2015
B. acidosis passes in urine instead of being reabsorbed. MED BOARDS;
C. aminoaciduria TOPNOTCH MD),
D. glycosuria MD
E. Hyperuricosuria

239 which of the following is true about acquired anemia is the most common hematologic LEAN ANGELO MIDTERM
Immunodeficiency syndrome except? abnormality in HIV patients and in the absence of SILVERIO, MD EXAM 1 - FEB
A. Diagnosis is attained if anyone with HIV a specific treatable cause is independently (TOP 4 - AUG 2014 2015
infection develop diseases associated with T cell associated with poor prognosis. MED BOARDS;
defect and CD4 count of <200 /ul. TOPNOTCH MD),
B. Trimethoprim/sulfamethoxazole should be MD
started for primary prophylaxis if CD4 count reaches
<200/ul
C. leukopenia is the most common hematologic
abnormality in HIV
D. invasive cervical cancer and generalized wasting
>10% is an AIDS defining illness.
E. immune reconstitution syndrome is frequently
seen in the setting of mycobacterial infection
240 this type of respiratory failure results in alveolar type 1, acute hypoxemic respiratory failure -d/t LEAN ANGELO MIDTERM
hypoventilation secondary to impaired respiratory alveolar flooding and subsequent intrapulmonary SILVERIO, MD EXAM 1 - FEB
drive? shunt physiology. Type 3-d/t lung atelectasis. (TOP 4 - AUG 2014 2015
A. Type 1 Type 4 - hypoperfusion of respiratory muscles in MED BOARDS;
B. Type 2 patient in shock. SIMILAR TO PREVIOUS BOARD TOPNOTCH MD),
C. Type 3 EXAM CONCEPT/PRINCIPLE MD
D. Type 4
E. None of the above

241 A 50 year old male 10 pack year smoker and remissions and exacerbations, (+) family history, KEVIN BRYAN LO, MIDTERM 2
hypertensive consulted due to dyspnea. History PE and history point more to asthma MD (TOP 7 - AUG EXAM - FEB
revealed multiple episodic attacks of dyspnea and 2014 MED 2015
coughing over the past few years. He said that he has BOARDS;
had siblings with similar coughing episodes as well. TOPNOTCH MD)
He has verbalized that the dyspnea and coughing
would frequently accompany colds and an episode of
flu would last a few weeks and would eventually
resolve only to return again after a few months.
Initial PE revealed symmetrical chest expansion,
occasional wheezes on both lung bases, no other
remarkable findings, most likely condition is
A. COPD - emphysema
B. bronchial asthma
C. congestive heart failure
D. anginal equivalent
E. post nasal drip syndrome

TOPNOTCH MEDICAL BOARD PREP INTERNAL MEDICINE SUPEREXAM Page 33 of 95


For inquiries visit www.topnotchboardprep.com.ph or email us at topnotchmedicalboardprep@gmail.com
TOPNOTCH MEDICAL BOARD PREP INTERNAL MEDICINE SUPEREXAM
For inquiries visit www.topnotchboardprep.com.ph or email us at topnotchmedicalboardprep@gmail.com
Item QUESTION EXPLANATION AUTHOR TOPNOTCH
# EXAM
242 30 year old female patient diagnosed case of SIMILAR TO PREVIOUS BOARD EXAM KEVIN BRYAN LO, MIDTERM 2
rheumatic heart disease with mitral regurgitation CONCEPT/PRINCIPLE, no culture isolated MD (TOP 7 - AUG EXAM - FEB
presented with new onset fever over the past 5 days, organism, give broad spectrum IV antibiotic that 2014 MED 2015
dyspnea, easy fatigability, 2D echo revealed covers for the most common organisms BOARDS;
vegetations on the mitral valve, however blood TOPNOTCH MD)
cultures returned negative, which of the following
antibiotics would most likely be used for this
patient?
A. vancomycin
B. ampicillin
C. ceftriaxone
D. gentamicin
E. penicillin G
243 Dilated cardiomyopathy is most commonly brought SIMILAR TO PREVIOUS BOARD EXAM KEVIN BRYAN LO, MIDTERM 2
about by which of the following? CONCEPT/PRINCIPLE, hypertensive heart disease MD (TOP 7 - AUG EXAM - FEB
A. infectious etiology still most common cause 2014 MED 2015
B. genetic factors BOARDS;
C. hypertensive heart disease TOPNOTCH MD)
D. autoimmune destruction
E. immunologic dysfunction

244 Which of the following hepatitis virus causes the SIMILAR TO PREVIOUS BOARD EXAM KEVIN BRYAN LO, MIDTERM 2
most cases of chronic liver disease in humans? CONCEPT/PRINCIPLE, only about 1% of cases of MD (TOP 7 - AUG EXAM - FEB
A. hepatitis A Hep B go on to become chronic and cause CLD 2014 MED 2015
B. hepatitis B BOARDS;
C. hepatitis C TOPNOTCH MD)
D. hepatitis D
E. hepatitis E

245 Which of the following drugs are classified to have a SIMILAR TO PREVIOUS BOARD EXAM KEVIN BRYAN LO, MIDTERM 2
narrow therapeutic index, is both nephrotoxic and CONCEPT/PRINCIPLE, only vancomycin reflects MD (TOP 7 - AUG EXAM - FEB
ototoxic at the same time? all characteristics, (aminoglycosides also have the 2014 MED 2015
A. vancomycin same characteristics) BOARDS;
B. imipinem cilastatin TOPNOTCH MD)
C. furosemide
D. cyclophosphamide
E. cisplatin

246 Nitroglycerin is usually given to relieve chest pain SIMILAR TO PREVIOUS BOARD EXAM KEVIN BRYAN LO, MIDTERM 2
during an acute anginal attack, which of the CONCEPT/PRINCIPLE, venodilator, decrease MD (TOP 7 - AUG EXAM - FEB
following is its predominant mechanism of action? preload 2014 MED 2015
A. acts as a venodilator BOARDS;
B. negative inotropic TOPNOTCH MD)
C. negative chronotropic
D. decreases peripheral vascular resistance
E. prevents platelet aggregation

247 A 32 year old female patient was noted to have SIMILAR TO PREVIOUS BOARD EXAM KEVIN BRYAN LO, MIDTERM 2
rheumatic heart disease. Patient comes in for follow CONCEPT/PRINCIPLE, AR = wide pulse pressure MD (TOP 7 - AUG EXAM - FEB
up check up since she was referred back by her 2014 MED 2015
cardiologist to her primary care physician, after BOARDS;
routine vital signs checking,BP130/60 heart rate of TOPNOTCH MD)
96 RR of 20, the patient told her primary care
physician that she was told by her cardiologist that
she had a valvular problem developed as
complication of her condition, the primary care
physician most likely thinks this is?
A. mitral stenosis
B. mitral regurgitation
C. aortic stenosis
D. aortic regurgitation
E. none of the above
248 32 year old female patient treated for SLE went into SIMILAR TO PREVIOUS BOARD EXAM KEVIN BRYAN LO, MIDTERM 2
remission a few weeks ago and discontinued her CONCEPT/PRINCIPLE, skin changes and MD (TOP 7 - AUG EXAM - FEB
medications when she got well, 5 days prior, she hypotensive episodes = primary adrenal 2014 MED 2015
started having cough and colds and body malaise, insufficiency BOARDS;
she reported recent episodes of light headedness, TOPNOTCH MD)
muscle weakness, weight loss. BP was 120/80 at
sitting but on standing dropped to 80/60, there was
some noted hyperpigmentation of the skin on the
dorsum of the arms, which of the following
conditions is most likely?
A. SLE flare
B. sepsis secondary to pneumonia
C. dehydration
D. medication side effects
E. adrenal insufficiency

TOPNOTCH MEDICAL BOARD PREP INTERNAL MEDICINE SUPEREXAM Page 34 of 95


For inquiries visit www.topnotchboardprep.com.ph or email us at topnotchmedicalboardprep@gmail.com
TOPNOTCH MEDICAL BOARD PREP INTERNAL MEDICINE SUPEREXAM
For inquiries visit www.topnotchboardprep.com.ph or email us at topnotchmedicalboardprep@gmail.com
Item QUESTION EXPLANATION AUTHOR TOPNOTCH
# EXAM
249 60 year old male patient came in due to easy SIMILAR TO PREVIOUS BOARD EXAM KEVIN BRYAN LO, MIDTERM 2
fatigability, no other symptoms noted on ROS. On CONCEPT/PRINCIPLE, unexplaine anemia in MD (TOP 7 - AUG EXAM - FEB
examination, patient was noted to be pale, no oral elderly rule out occult GI malignancy 2014 MED 2015
lesions, no palpable cervical lympadenopathies, BOARDS;
chest, abdominal PE findings were unremarkable, TOPNOTCH MD)
DRE revealed smooth rectal vault, (-) blood on
tactating finger, CBC revealed hgb 9.5g/dL which of
the following is the most likely pathology?
A. chronic renal failure
B. lymphoma
C. occult GI malignancy
D. Pulmonary TB
E. HIV

250 40 year old female patient come in with neck mass of SIMILAR TO PREVIOUS BOARD EXAM KEVIN BRYAN LO, MIDTERM 2
3 years duration, noted gradual increase in size, she CONCEPT/PRINCIPLE, multinodular goiter, MD (TOP 7 - AUG EXAM - FEB
also complained of occasional difficulty swallowing subtotal thyroidectomy is the best treatment of 2014 MED 2015
and complained of the cosmetic disturbance due to choice for this condition BOARDS;
the mass in her neck. The patient was worked up TOPNOTCH MD)
and evaluated to be euthyroid, ultrasound revealed
multinodular goiter, which of the following
treatments is most applicable?
A. subtotal thyroidectomy
B. radioactive iodine I131
C. thyroid hormone levothyroxine supplementation
D. PTU antithyroid medication
E. SSKI (saturated solution of potassium iodide)

251 A 55 year old male patient admitted for severe SIMILAR TO PREVIOUS BOARD EXAM KEVIN BRYAN LO, MIDTERM 2
pneumonia went into septic shock, which of the CONCEPT/PRINCIPLE, about 2 similar question in MD (TOP 7 - AUG EXAM - FEB
following is a possible effect on the patient's kidney? this form of a physio question, personally I did not 2014 MED 2015
A. increase GFR decrease sodium and water study for IM as a board subject, study your BOARDS;
retention pharma, patho, micro, physio and you will be ok TOPNOTCH MD)
B. decrease GFR increase sodium and water plus your internship clerkship experiences as well,
retention one of the easiest subjects on the boards IM
C. no change in GFR decrease in sodium and water platinum would do ok as well which I read during
retention internship
D. no change in GFR increase in sodium and water
retention
E. none of the aove
252 A 56 year old male marathon runner collapsed on SIMILAR TO PREVIOUS BOARD EXAM KEVIN BRYAN LO, MIDTERM 2
track, when he was brought in for medical CONCEPT/PRINCIPLE, pre renal due to shock MD (TOP 7 - AUG EXAM - FEB
assistance, he was found to be very dehydrated and dehydration 2014 MED 2015
hypotensive, assessment was heat exhaustion and BOARDS;
severe dehydration, which type of renal failure will TOPNOTCH MD)
this patient most likely develop if the condition is
allowed to progress?
A. pre renal
B. intra renal
C. post renal
D. all of the above
E. none of the above

253 A 40 year old female patient came into the clinic SIMILAR TO PREVIOUS BOARD EXAM KEVIN BRYAN LO, MIDTERM 2
complaining of epigastric pain 2-3 hours after eating, CONCEPT/PRINCIPLE, for GERD need to be high MD (TOP 7 - AUG EXAM - FEB
this was accompanied by burning sensation going dose proton pump inhibitors 2014 MED 2015
upward into the chest, which of the following BOARDS;
treatments is most suitable for this patient? TOPNOTCH MD)
A. AlMgOH take 1 tablet as needed
B. omeprazole 20mg/tablet 1 tablet OD
C. omeprazole 40mg/tablet 1 tablet OD
D. prednisone 10mg/tab 1 tab OD
E. ISDN 5mg/tab 1 tab sublingual

254 50 year old female patient known diabetic and thiazolidinediones can cause excess water KEVIN BRYAN LO, MIDTERM 2
hypertensive for 10 years recently complained of retention which may exacerbate patient's MD (TOP 7 - AUG EXAM - FEB
episodes of exertional dyspnea, bipedal edema, beginning heart failure thus is contraindicated 2014 MED 2015
nocturnal cough and 2 pillow orthopnea, which of BOARDS;
the following oral hypoglycemic medications are TOPNOTCH MD)
contraindicated for this patient?
A. metformin
B. glimepiride
C. sitagliptin
D. acarbose
E. pioglitazone

TOPNOTCH MEDICAL BOARD PREP INTERNAL MEDICINE SUPEREXAM Page 35 of 95


For inquiries visit www.topnotchboardprep.com.ph or email us at topnotchmedicalboardprep@gmail.com
TOPNOTCH MEDICAL BOARD PREP INTERNAL MEDICINE SUPEREXAM
For inquiries visit www.topnotchboardprep.com.ph or email us at topnotchmedicalboardprep@gmail.com
Item QUESTION EXPLANATION AUTHOR TOPNOTCH
# EXAM
255 24 year old military recruit residing with his platoon SIMILAR TO PREVIOUS BOARD EXAM KEVIN BRYAN LO, MIDTERM 2
developed high fever, nuchal rigidity, altered CONCEPT/PRINCIPLE, drug of choice is still PenG MD (TOP 7 - AUG EXAM - FEB
consciousness and rapid onset of purpuric rashes for meningo 2014 MED 2015
over the trunk and lower extremities, the drug of BOARDS;
choice for this condition is? TOPNOTCH MD)
A. penicillin G
B. ceftriaxone
C. ampicillin sulbactam
D. azithromycin
E. clindamycin

256 A 65 year old male 20 pack year smoking history A more on asthma, C pneumonia, D trachea should KEVIN BRYAN LO, MIDTERM 2
presents with a history of increasing dyspnea, be on the same side with atelectasis, E more on MD (TOP 7 - AUG EXAM - FEB
weight loss, occasional intermittent coughing. Chest pneumonia 2014 MED 2015
xray was requested by his primary physician which BOARDS;
revealed a hilar mass on the right hemithorax TOPNOTCH MD)
causing collapse of the right upper lung lobe, which
of the following was the most likely PE findings in
this patient?
A. bilateral crackles over lung bases occasional
wheezing bilateral lung bases
B. decreased breath sounds over right upper lung
field, occasional wheezing right hemithorax
C. dullness on right lung base, increased tactile and
vocal fremiti
D. tracheal shifted to the left, decreased breath
sounds over the right hemithorax
E. (+) bronchophony and egophony right lung base,
occasional crackles right lung
257 In relation to the question above, which of the SIMILAR TO PREVIOUS BOARD EXAM KEVIN BRYAN LO, MIDTERM 2
following diagnostic procedures is most appropriate CONCEPT/PRINCIPLE, accessible hilar lung mass MD (TOP 7 - AUG EXAM - FEB
for patient's condition? can be accessed through bronchoscopy 2014 MED 2015
A. CT guided aspiration biopsy BOARDS;
B. transbronchial biopsy TOPNOTCH MD)
C. resection biopsy via thoracotomy incision
D. VATS guided lung biopsy
E. none of the above

258 Which of the following conditions below could SIMILAR TO PREVIOUS BOARD EXAM KEVIN BRYAN LO, MIDTERM 2
potentially produce hypovolemic hyponatremia? CONCEPT/PRINCIPLE, A and B, D are MD (TOP 7 - AUG EXAM - FEB
A. SIADH hypervolemic 2014 MED 2015
B. water intoxication BOARDS;
C. diuretic therapy TOPNOTCH MD)
D. Conn's syndrome
E. none of the above

259 Which of the following is the test of choice to urea breath test to document cure KEVIN BRYAN LO, MIDTERM 2
document H. Pylori eradication following treatment? MD (TOP 7 - AUG EXAM - FEB
A. rapid urease test 2014 MED 2015
B. urea breath test BOARDS;
C. gastric mucosal biopsy TOPNOTCH MD)
D. H.pylori serological test
E. all of the above

260 50 year old male patient smoker and alcoholic with SIMILAR TO PREVIOUS BOARD EXAM KEVIN BRYAN LO, MIDTERM 2
history of chronic epigastric pain passed black tarry CONCEPT/PRINCIPLE, 14 hours for melena MD (TOP 7 - AUG EXAM - FEB
stools, the physician knows that the stool was 2014 MED 2015
probably in the GI tract for at least how long? BOARDS;
A. 14 hours TOPNOTCH MD)
B. 16 hours
C. 20 hours
D. 24 hours
E. 72 hours

261 A 54 year old male who has been hospitalized for This is a case of ARDS which results from RAYMUND MIDTERM 3
severe acute pancreatitis complained of onset of endothelial damage and leakage of protein-rich MARTIN LI, MD EXAM - FEB
dyspnea and tachypnea. Chest X-ray reveals diffuse material into the alveolar spaces. There are many (TOP 1 - AUG 2014 2015
bilateral infiltrates and stat ABG revealed severe causes, one of which is severe pancreatitis. MED BOARDS;
hypoxemia. Pulmonary capillary wedge pressure TOPNOTCH MD)
was measured and was unremarkable. What is the
most likely pathophysiologic mechanism of this
condition?
A. Bronchial mucous plugging
B. Alveolar wall destruction
C. Left ventricular infarction
D. Endothelial damage
E. Interstitial fibrosis

TOPNOTCH MEDICAL BOARD PREP INTERNAL MEDICINE SUPEREXAM Page 36 of 95


For inquiries visit www.topnotchboardprep.com.ph or email us at topnotchmedicalboardprep@gmail.com
TOPNOTCH MEDICAL BOARD PREP INTERNAL MEDICINE SUPEREXAM
For inquiries visit www.topnotchboardprep.com.ph or email us at topnotchmedicalboardprep@gmail.com
Item QUESTION EXPLANATION AUTHOR TOPNOTCH
# EXAM
262 A patient with known coronary artery disease is SIMILAR TO PREVIOUS BOARD EXAM RAYMUND MIDTERM 3
prescribed with a maintenance dose of aspirin. CONCEPT/PRINCIPLE. Prostaglandin synthase I or MARTIN LI, MD EXAM - FEB
Which of the following is the mode of action of this COX1 - irreversibly acetylated by aspirin to (TOP 1 - AUG 2014 2015
drug in preventing future ischemic events? decrease synthesis of thromboxane. Prostaglandin MED BOARDS;
A. Inhibition of cyclooxygenase 2-mediated synthesis synthase II or COX2 is selectively expressed in TOPNOTCH MD)
of prostaglandins inflammatory cells and mediate inflammation.
B. Irreversible acetylation of prostacyclin Aspirin inhibits both but its action on
C. Inhibition of prostaglandin synthase 1 prostaglandin synthase I or COX1 is what helps
D. Direct blockade of thromboxane action on prevent platelet aggregation.
platelets
E. Reversible acetylation of cyclooxygenase
263 Given the following clicical picture: (+) HbsAg, (- Review hepatitis laboratory profile RAYMUND MIDTERM 3
)anti-Hbs, (+) anti-Hbc IgG, (-) HbeAg, (+) anti-Hbe, MARTIN LI, MD EXAM - FEB
(+) anti-HbA IgG, (+) anti-HbC, what is the (TOP 1 - AUG 2014 2015
diagnosis? MED BOARDS;
A. Past Hepatitis A infection, Chronic Hepatitis B TOPNOTCH MD)
carrier, Hepatitis C infection
B. Acute Hepatitis A infection, Chronic Hepatitis B
carrier, Hepatitis C infection
C. Past Hepatitis A infection, Chronic Active Hepatitis
B, Hepatitis C infection
D. Acute Hepatitis A infection, Chronic Hepatitis B
carrier, Resolved Hepatitis C infection
E. Past Hepatitis A infection, Past Hepatitis B
vaccination, Hepatitis C infection

264 A 63 year old male patient comes to your clinic Urate appears as needle like crystals with negative RAYMUND MIDTERM 3
complaining of redness and painful swelling of the birefringerence under polarized light MARTIN LI, MD EXAM - FEB
right knee. Synovial fluid analysis revealed an (TOP 1 - AUG 2014 2015
inflammatory picture and analysis under polarized MED BOARDS;
microscopy shows negative birefringerence. What is TOPNOTCH MD)
the most likely etiology?
A. Autoimmune pathology
B. Crystal arthropathy
C. Dissemininated gonococcal infection
D. Wear and tear
E. None of the above
265 A chronic alcoholic was hospitalized for lethargy and This is a case of hepatic encephalopathy. Lactulose RAYMUND MIDTERM 3
confusion. Physical examination findings include is converted by colonic bacteria into organic acids, MARTIN LI, MD EXAM - FEB
icteric sclerae, positive fluid wave and shifting which traps ammonia and prevents diffusion back (TOP 1 - AUG 2014 2015
dullness, bilateral pedal edema, and flapping into the circulation MED BOARDS;
tremors of the hand. Which of the following will help TOPNOTCH MD)
improve that patient's current reason for admission?
A. Alkalinization of urine
B. Colonic acidification
C. Large-volume paracentesis
D. Adequate intravenous hydration
E. TIPS
266 A patient presents with anemia and enlarged spleen Hereditary spherocytosis is an extravascular type RAYMUND MIDTERM 3
upon palpation. Peripheral smear reveals RBC's with pf hemolytic anemia which may cause gallstone MARTIN LI, MD EXAM - FEB
lack of central pallor. Family history is also formation due to accumulation of unconjugated (TOP 1 - AUG 2014 2015
significant for the condition. If a splenectomy is bilirubin. Splenectomy will prevent this. MED BOARDS;
performed on this patient, which of the following TOPNOTCH MD)
may be prevented?
A. Portal vein hypertension
B. Neoplastic transformation
C. Gallstone formation
D. Thrombocytosis
E. Invasive bacterial infection
267 In a patient who recently had myocardial infarction, Fibrinous pericarditis may 1-3 days after MI as a RAYMUND MIDTERM 3
when do you expect the possibility of an response to the necrotic tissue. But the MARTIN LI, MD EXAM - FEB
autoimmune-mediated type of pericarditis after MI? autoimmune type of pericarditis called Dressler's (TOP 1 - AUG 2014 2015
A. 12-24 hours syndrome occurs weeks to months after MI MED BOARDS;
B. 1-3 days TOPNOTCH MD)
C. 4-7 days
D. 7-14 days
E. >2 weeks

268 An elderly hospitalized patient who has been Symptomatic pulmonary embolism will cause RAYMUND MIDTERM 3
confined to bed complained of acute onset dypnea, tachypnea, hypoxemia, and respiratory alkalosis MARTIN LI, MD EXAM - FEB
tachypnea and shortness of breath. Further (TOP 1 - AUG 2014 2015
examination revealed left calf pain and tenderness. MED BOARDS;
What will be the expected ABG findings? TOPNOTCH MD)
A. pH=7.31, pCO2=50, pO2=98
B. pH=7.29, pCO2=29, pO2=76
C. pH=7.39, pCO2=39, pO2=88
D. pH=7.52, pCO2=49, pO2=54
E. pH=7.49, pCO2=29, pO2=72

TOPNOTCH MEDICAL BOARD PREP INTERNAL MEDICINE SUPEREXAM Page 37 of 95


For inquiries visit www.topnotchboardprep.com.ph or email us at topnotchmedicalboardprep@gmail.com
TOPNOTCH MEDICAL BOARD PREP INTERNAL MEDICINE SUPEREXAM
For inquiries visit www.topnotchboardprep.com.ph or email us at topnotchmedicalboardprep@gmail.com
Item QUESTION EXPLANATION AUTHOR TOPNOTCH
# EXAM
269 A 5 year old patient is brought to the ER by her DKA will cause shift of potassium from intacellular RAYMUND MIDTERM 3
mother due to complaints of polyuria, nausea and to extracellular causing hyperkalemia. The MARTIN LI, MD EXAM - FEB
vomiting, and abdominal pain. Respiration is noted increase in osmotic pressure will cause brisk (TOP 1 - AUG 2014 2015
to be deep and rapid and urine ketones is positive. If diuresis and loss of Na causing hyponatremia MED BOARDS;
serum chemistry levels are measured, which of the TOPNOTCH MD)
following will be expected?
A. Increased K, Deacresed Na, Increased Glucose
B. Increased K, Increased Na, Increased Glucose
C. Decreased K, Decreased Na, Increased Glucose
D. Increased K, Increased Na, Normal Glucose
E. Decreased K, Decreased Na, Decreased Glucose
270 What is the drug of choice in a patient with Hydrocortisone IV is the drug of choice in acute RAYMUND MIDTERM 3
Addison's disease presenting with acute adrenal adrenal crisis MARTIN LI, MD EXAM - FEB
crisis? (TOP 1 - AUG 2014 2015
A. Prednisone MED BOARDS;
B. Prednisolone TOPNOTCH MD)
C. Hydrocortisone
D. Betamethasone
E. Dexamethasone

271 A 23 year old female patient complaining of Antibiotic therapy helps eradicate H. pylori and RAYMUND MIDTERM 3
epigastric pain underwent endoscopy which primarily decreases recurrence risk MARTIN LI, MD EXAM - FEB
revealed a duodenal ulcer. She was started on (TOP 1 - AUG 2014 2015
Proton pump inhibitor therapy. The addition of MED BOARDS;
antibiotics to the treament regimen will help: TOPNOTCH MD)
A. Decrease recurrence risk
B. Relieve symptoms
C. Facilitate ulcer healing
D. Prevent disease transmission
E. Prevent malignant transformation

272 What is the most common cause of acute kidney RAYMUND MIDTERM 3
injury? MARTIN LI, MD EXAM - FEB
A. Intrinsic AKI (TOP 1 - AUG 2014 2015
B. Postrenal AKI MED BOARDS;
C. Acute Tubular Necrosis TOPNOTCH MD)
D. Prerenal AKI
E. Obstructive uropathy

273 A 23 year old female patient who was diagnosed Agranulocytosis is a rare idiosyncratic reaction of RAYMUND MIDTERM 3
with Grave's disease is being maintained on a thionamides (ex. Methimazole) MARTIN LI, MD EXAM - FEB
thionamide medication. In order to rapidly detect a (TOP 1 - AUG 2014 2015
rare idiosyncratic adverse reaction that is associated MED BOARDS;
with this drug, which laboratory tests should be TOPNOTCH MD)
monitored?
A. Liver function test
B. Alkaline phosphatase
C. CBC
D. Serum potassium
E. Serum creatinine
274 A 72 year old male patient who is a native of Baguio Age is the primary predisposing factor for RAYMUND MIDTERM 3
City is complaining of knee pain that is worse in the osteoarthritis MARTIN LI, MD EXAM - FEB
afternoon and during exertion. He lives a sedentary (TOP 1 - AUG 2014 2015
lifestyle and is overweight. Family history reveals MED BOARDS;
that this runs in the family. There are no other TOPNOTCH MD)
associated symptoms. Which of the following is the
most important risk factor for causing this disease in
this patient?
A. Cold climate
B. Gender
C. Obesity
D. Family history
E. Age

275 A 25 year old patient who is being treated for a Reiter's syndrome/Reactive arthritis - RAYMUND MIDTERM 3
urinary tract infection develops conjunctival redness seronegative, associated with HLA B27, triad of MARTIN LI, MD EXAM - FEB
and difficulty in ambulation due to a swollen knee. urethritis, conjunctivitis, and arthritis (TOP 1 - AUG 2014 2015
Which of the following is true about this condition? MED BOARDS;
A. Reactive arthritis TOPNOTCH MD)
B. Associated with HLA-B27
C. Seronegative type of arthritis
D. A and B
E. All of the above

TOPNOTCH MEDICAL BOARD PREP INTERNAL MEDICINE SUPEREXAM Page 38 of 95


For inquiries visit www.topnotchboardprep.com.ph or email us at topnotchmedicalboardprep@gmail.com
TOPNOTCH MEDICAL BOARD PREP INTERNAL MEDICINE SUPEREXAM
For inquiries visit www.topnotchboardprep.com.ph or email us at topnotchmedicalboardprep@gmail.com
Item QUESTION EXPLANATION AUTHOR TOPNOTCH
# EXAM
276 A elderly female patient who recently suffered from Broca's area - expressive aphasia RAYMUND MIDTERM 3
left-sided hemiparesis is able to comprehend speech MARTIN LI, MD EXAM - FEB
but is unable to repeat words and has difficulty (TOP 1 - AUG 2014 2015
using words and sentences. Which of the following MED BOARDS;
areas of the cortex is most likely affected? TOPNOTCH MD)
A. Inferior frontal gyrus
B. Superior frontal gyrus
C. Superior temporal gyrus
D. Posterior parietal gyrus
E. None of the above

277 A 72 year old patient presents at the ER with transudative (ex. CHF, nephrotic): PF LDH<200, PF RAYMUND MIDTERM 3
complaints of dypnea and chest discomfort. P.E. protein <3, PF-serum protein ratio <0.5, PR-serum MARTIN LI, MD EXAM - FEB
findings include decreased dullness upon percussion LDH ratio of <0.6, PF-serum protein gradient of (TOP 1 - AUG 2014 2015
and decreased breath sounds and vocal fremitus. >31g/L MED BOARDS;
She has a history of chronic hypertension. Chest x- TOPNOTCH MD)
ray reveals cardiomegaly and blunting of the
costophrenic angle. Which of the following findings
is most consistent with the patient's most likely
diagnosis upon thoracentesis?
A. Pleural fluid LDH of >200 U/L
B. Pleural fluid protein of >3 g
C. Pleural fluid and serum protein gradient of
>31g/L
D. Pleural -serum LDH ratio >0.6
E. All of the above
278 In the recovery phase of acute renal failure, one Recovery phase is the diuretic phase placing RAYMUND MIDTERM 3
should be most concerned of which of the following patient at risk for hypokalemia MARTIN LI, MD EXAM - FEB
possible complications? (TOP 1 - AUG 2014 2015
A. Volume overload MED BOARDS;
B. Hypokalemia TOPNOTCH MD)
C. Hypernatremia
D. Hyperuricemia
E. Hyperkalemia

279 A 54 year old anxious and diaphoretic male patient Inferior infarct - blockage of right coronary artery RAYMUND MIDTERM 3
presents at the ER complaining of severe substernal which supplies SA and AV node MARTIN LI, MD EXAM - FEB
chest pain that is unrelieved by rest. Physical (TOP 1 - AUG 2014 2015
examination reveals bradycardia and ECG shows ST MED BOARDS;
segment elevation of II, III, aVF. BP is 130/90, HR is TOPNOTCH MD)
52, and RR is 23. Which of the following is the most
consistent with the patient's scenario?
A. Cardiogenic shock secondary to massive
infarction
B. Blockage of the vessel supplying the SA and AV
node
C. Compensatory parasympathetic response
D. Pericardial tamponade
E. Coronary atherosclerosis affecting the left
anterior descending artery
280 A patient who was diagnosed with bacterial Infectious mononucleosis (EBV) - Almost all RAYMUND MIDTERM 3
pharyngitis is treated with amoxicillin and develops patients with mononucleosis that are treated with MARTIN LI, MD EXAM - FEB
a pruritic maculopapular rash. Patient claims that he amoxicillin/ampicillin develops a generalized (TOP 1 - AUG 2014 2015
was treated with amoxicillin several years ago pruritic maculopapular rash MED BOARDS;
without any reaction. What is your primary etiologic TOPNOTCH MD)
consideration?
A. RNA virus
B. Catalase positive cocci
C. DNA virus
D. Fungal infection
E. Drug-induced hypersensitivity reaction
281 A 58 year old male was brough to the ER due to a Dx: TB meningitis. TB of the central nervous ERIC ROYD FINAL EXAM -
one week history of progressive headache and fever. system is seen most often in young children but TALAVERA, MD FEB 2015
He has been diagnosed with Pulmonary also develops in adults, especially those infected (TOP 1 - AUG 2014
Tuberculosis but has been unable to start his with HIV. The disease often presents subtly as MED BOARDS;
medications due to financial difficulties. On headache and slight mental changes after a TOPNOTCH MD)
examination, he was drowsy but arousable with note prodrome of weeks of low-grade fever, malaise,
of nuchal rigidity and a positive Brudzinski sign. anorexia, and irritability. Since meningeal
Lumbar puncture was done which showed an involvement is pronounced at the base of the
elevated WBC count of lymphocytic predominance, brain, paresis of cranial nerves (ocular nerves in
an elevated protein level and a decreased glucose particular) is a frequent finding, and the
level. Which of the following statements is true involvement of cerebral arteries may produce
regarding the most probable condition of the focal ischemia. Lumbar puncture is the
patient? cornerstone of diagnosis. Adjunctive
A. Polymerase chain reaction (PCR) is the gold dexamethasone significantly enhanced the
standard for diagnosis chances of survival among persons >14 years of
B. The disease often presents abruptly with a acute age but did not reduce the frequency of neurologic
onset of severe headache and vomiting sequelae.
C. Adjunctive treatment with dexamethasone
promotes significantly reduces the frequency of
neurologic sequelae
D. It is more common in elderly than in young
children
E. Involvement is more pronounced at the base of
the brain, with paresis of cranial nerves being a

TOPNOTCH MEDICAL BOARD PREP INTERNAL MEDICINE SUPEREXAM Page 39 of 95


For inquiries visit www.topnotchboardprep.com.ph or email us at topnotchmedicalboardprep@gmail.com
TOPNOTCH MEDICAL BOARD PREP INTERNAL MEDICINE SUPEREXAM
For inquiries visit www.topnotchboardprep.com.ph or email us at topnotchmedicalboardprep@gmail.com
Item QUESTION EXPLANATION AUTHOR TOPNOTCH
# EXAM
frequent finding

282 Which of the following joints is spared in OA affects certain joints, yet spares others. Usually ERIC ROYD FINAL EXAM -
Osteoarthritis? spared are the wrist, elbow, and ankle. Some TALAVERA, MD FEB 2015
A. Knee joints, like the ankles, may be spared because their (TOP 1 - AUG 2014
B. Ankle articular cartilage may be uniquely resistant to MED BOARDS;
C. Cervical Vertebrae loading stresses TOPNOTCH MD)
D. Proximal interphalangeal joint
E. Hip

283 A 40 year old female presented with progressive Dx: Nephrotic syndrome Nephrotic syndrome ERIC ROYD FINAL EXAM -
weight gain and marked swelling of her lower classically presents with heavy proteinuria, TALAVERA, MD FEB 2015
extremities. She has no known co-morbids. PE is minimal hematuria, hypoalbuminemia, (TOP 1 - AUG 2014
normal except for the presence of a grade 2 bipedal hypercholesterolemia, edema, and hypertension. If MED BOARDS;
edema. CBC, BUN, Electrolytes and Creatinine were left undiagnosed or untreated, some of these TOPNOTCH MD)
all normal. Her urinalysis showed a 4+ albumin, oval syndromes will progressively damage enough
fat bodies and free fat droplets. Her 24 hour urine glomeruli to cause a fall in GFR, producing renal
albumin excretion was elevated at 8.6 grams. Which failure.
of the following findings would also be expected in
this patient?
A. Elevated bilirubin levels
B. Elevated serum cholesterol
C. Elevated serum albumin
D. Elevated blood uric acid
E. None of the above
284 Which of the following clinical features argues Alteration in bowel habits is the most consistent ERIC ROYD FINAL EXAM -
against the diagnosis of Irritable Bowel Syndrome clinical feature in IBS. Abdominal Pain is often TALAVERA, MD FEB 2015
(IBS)? exacerbated by eating or emotional stress and (TOP 1 - AUG 2014
A. Bloody diarrhea improved by passage of flatus or stools. Patients MED BOARDS;
B. Alteration in bowel habits with IBS frequently complain of abdominal TOPNOTCH MD)
C. Exacerbation of abdominal pain by stress distention and increased belching or flatulence.
D. Resolution of abdomina pain on passage of stool Bleeding is not a feature of IBS unless
E. Increased belching or flatulence hemorrhoids are present.

285 A 40 year old female was referred for hematologic Dx: CML. Initial management of patients with ERIC ROYD FINAL EXAM -
evaluation because of findings of splenomegaly and chemotherapy is currently reserved for rapid TALAVERA, MD FEB 2015
a markedly elevated WBC count of 180,000/uL with lowering of WBCs, reduction of symptoms, and (TOP 1 - AUG 2014
the following differential count: Blasts 2%, reversal of symptomatic splenomegaly. MED BOARDS;
Promyelocyte 15%, Metamyelocyte 20%, Myelocyte Hydroxyurea, a ribonucleotide reductase inhibitor, TOPNOTCH MD)
10%, Bands 8%, Segmenters 30%, Lymphocytes induces rapid disease control.
10% and Basophils 5%. A primary diagnosis of CML
was made by the hematologist. Which of the
following treatment options for this case induces
rapid disease control by lowering the WBC count?
A. Imatinib
B. Cytarabine
C. Doxorubicin
D. Hydroxyurea
E. Stem cell transplantation
286 Which of the following statements concerning Because patients are volume-constricted, liberal ERIC ROYD FINAL EXAM -
Pheochromocytoma is false? salt intake and hydration are necessary to avoid TALAVERA, MD FEB 2015
A. Hypertension is the dominant sign orthostasis. Among the presenting symptoms, (TOP 1 - AUG 2014
B. Catecholamine crises can lead to heart failure, episodes of palpitations, headaches, and profuse MED BOARDS;
pulmonary edema, arrhythmias, and intracranial sweating are typical and constitute a classic triad. TOPNOTCH MD)
hemorrhage The dominant sign is hypertension. Classically,
C. Elevated plasma and urinary levels of patients have episodic hypertension, but sustained
catecholamines and metanephrines are the hypertension is also common. Catecholamine
cornerstone for the diagnosis crises can lead to heart failure, pulmonary edema,
D. Complete tumor removal is the ultimate arrhythmias, and intracranial hemorrhage.
therapeutic goal
E. Restriction of salt intake is a necessary pre-
operative preparation

TOPNOTCH MEDICAL BOARD PREP INTERNAL MEDICINE SUPEREXAM Page 40 of 95


For inquiries visit www.topnotchboardprep.com.ph or email us at topnotchmedicalboardprep@gmail.com
TOPNOTCH MEDICAL BOARD PREP INTERNAL MEDICINE SUPEREXAM
For inquiries visit www.topnotchboardprep.com.ph or email us at topnotchmedicalboardprep@gmail.com
Item QUESTION EXPLANATION AUTHOR TOPNOTCH
# EXAM
287 A 60 year old male sought consult due to muscle Dx: Secondary Hyperaldosteronism (secondary to ERIC ROYD FINAL EXAM -
weakness. His BP was elevated at 180/110 with an Renal Artery Stenosis). Decreased RBF will TALAVERA, MD FEB 2015
abdominal bruit noted. Blood tests revealed stimulate renin release which will stimulate (TOP 1 - AUG 2014
hypokalemia. An arterial duplex scan was done aldosterone release. Aldosterone promotes MED BOARDS;
which showed a decreased blood flow through the sodium absorption (hence an elevated BP) and K TOPNOTCH MD)
right renal artery. What are the expected plasma excretion (hence hypokalemia which presented as
renin activity (PRA) and aldosterone level for this muscle weakness)
case?
A. Low PRA and low aldosterone
B. High PRA and low aldosterone
C. High PRA and high aldosterone
D. Low PRA and high aldosterone
E. Normal PRA and low aldosterone
288 The murmur of mitral regurgitation most commonly systolic murmur of at least grade III/VI intensity ERIC ROYD FINAL EXAM -
radiates to what location? is the most characteristic auscultatory finding in TALAVERA, MD FEB 2015
A. Axilla chronic severe MR. It is usually holosystolic (see (TOP 1 - AUG 2014
B. Right parasternal area Fig. 227-5A), but as previously noted it is MED BOARDS;
C. Neck decrescendo and ceases in mid- to late systole in TOPNOTCH MD)
D. Epigastric area patients with acute severe MR. The systolic
E. Left parasternal area murmur of chronic MR is usually most prominent
at the apex and radiates to the axilla.

289 A 29 year old female presented with weigh loss, The presence of severe ophthalmopathy requires ERIC ROYD FINAL EXAM -
palpitation, tremors and heat intolerance. She is caution, and some authorities advocate the use of TALAVERA, MD FEB 2015
tachycardic at 115/bpm with the presence of prednisone, 40 mg/d, at the time of radioiodine (TOP 1 - AUG 2014
moderate to severe proptosis. The thyroid gland is treatment, tapered over 2–3 months to prevent MED BOARDS;
diffusely enlarged and non tender. Reflexes are exacerbation of ophthalmopathy. TOPNOTCH MD)
hyperactive. The rest of the examination is normal.
Which of the following treatment options may
worsen her proptosis?
A. PTU
B. Methimazole
C. Propranolol
D. RAI
E. Prednisone
290 A "locked-in" state of preseved consciousness with The picture of complete basilar occlusion, ERIC ROYD FINAL EXAM -
quadriplegia and cranial nerve signs suggests however, is easy to recognize as a constellation of TALAVERA, MD FEB 2015
complete infarction of what area in the central bilateral long tract signs (sensory and motor) with (TOP 1 - AUG 2014
nervous system? signs of cranial nerve and cerebellar dysfunction. MED BOARDS;
A. Thalamus and Basal Ganglia A "locked-in" state of preserved consciousness TOPNOTCH MD)
B. Cerebellum and Medulla with quadriplegia and cranial nerve signs suggests
C. Hypothalamus and Pituitary complete pontine and lower midbrain infarction.
D. Pons and Lower Midbrain The therapeutic goal is to identify impending
E. Occipital lobe of the cerebral hemisphere basilar occlusion before devastating infarction
occurs.
291 A 56 year old female developed dysphagia to both Intermittent dysphagia that occurs only with solid ERIC ROYD FINAL EXAM -
solids and liquids from the onset. Which of the food implies structural dysphagia, whereas TALAVERA, MD FEB 2015
following disorders would be most compatible with constant dysphagia with both liquids and solids (TOP 1 - AUG 2014
this case? strongly suggests a motor abnormality such as in MED BOARDS;
A. Schatzki ring achalasia TOPNOTCH MD)
B. Peptic Stricture
C. Achalasia
D. Esophageal carcinoma
E. Eosinophilic esophagitis

292 What is the only pharmacologic therapy Supplemental O2 is the only pharmacologic ERIC ROYD FINAL EXAM -
demonstrated to unequivocally decrease mortality therapy demonstrated to unequivocally decrease TALAVERA, MD FEB 2015
rates in patients with COPD ? mortality rates in patients with COPD. For patients (TOP 1 - AUG 2014
A. LABA with resting hypoxemia the use of O2 has been MED BOARDS;
B. Corticosteroids demonstrated to have a significant impact on TOPNOTCH MD)
C. Supplemental Oxygen mortality rate. Patients meeting these criteria
D. Theophylline should be on continual oxygen supplementation,
E. Ipratropium as the mortality benefit is proportional to the
number of hours/day oxygen is used
293 A 25 year old male, known case of Hemophilia A The formation of alloantibodies to FVIII or FIX is ERIC ROYD FINAL EXAM -
with a history of multiple Factor VIII concentrate currently the major complication of hemophilia TALAVERA, MD FEB 2015
transfusion, was admitted due to bilateral knee treatment. The prevalence of inhibitors to FVIII is (TOP 1 - AUG 2014
swelling. Despite transfusion with adequate levels of estimated to be between 5 and 10% of all cases MED BOARDS;
Factor VIII concentrate, levels of the said factor and 20% of severe hemophilia A patients. TOPNOTCH MD)
remain to be decreased. Which of the following Inhibitors usually appear early in life, at a median
statements regarding the patient's current of 2 years of age, and after 10 cumulative days of
predicament is true? exposure. However, intensive replacement
A. It is an uncommon complication of hemophilia therapy such as for major surgery, intracranial
treatment bleeding, or trauma increases the risk of inhibitor
B. It is a transient condition which will formation for patients of all ages. The clinical
spontaneously resolve diagnosis of an inhibitor is suspected when
C. The laboratory test required to confirm its patients do not respond to factor replacement at
presence is an aPTT mixed with a normal plasma therapeutic doses. .
D. Intensive replacement therapy such as for major
surgery, intracranial bleeding, or trauma are risk
factors for the development of this phenomenon
E. Both C and D are correct

TOPNOTCH MEDICAL BOARD PREP INTERNAL MEDICINE SUPEREXAM Page 41 of 95


For inquiries visit www.topnotchboardprep.com.ph or email us at topnotchmedicalboardprep@gmail.com
TOPNOTCH MEDICAL BOARD PREP INTERNAL MEDICINE SUPEREXAM
For inquiries visit www.topnotchboardprep.com.ph or email us at topnotchmedicalboardprep@gmail.com
Item QUESTION EXPLANATION AUTHOR TOPNOTCH
# EXAM
294 Which of the following is a clinical manifestation of In warm, moist, intertriginous areas (commonly ERIC ROYD FINAL EXAM -
secondary syphilis? the perianal region, vulva, and scrotum), papules TALAVERA, MD FEB 2015
A. Condylomata lata can enlarge to produce broad, moist, pink or gray- (TOP 1 - AUG 2014
B. Gumma white, highly infectious lesions [condylomata lata MED BOARDS;
C. Tabes dorsalis (see Fig. e7-20)] in 10% of patients with TOPNOTCH MD)
D. Argyll Robertson Pupil secondary syphilis.
E. Chancre

295 A 56 year old male, with no known co-morbids If SAAG <1.1: think of infection, malignancy and ERIC ROYD FINAL EXAM -
presented with gradual abdominal enlargement. On nephrotic syndrome. SAAG >1.1 almost always TALAVERA, MD FEB 2015
PE, the abdomen was tense with a positive fluid secondary to portal HTN (TOP 1 - AUG 2014
wave and shifting dullness. The rest of the MED BOARDS;
examination was essentially unremarkable. A TOPNOTCH MD)
diagnostic paracentesis was done which showed a
serum ascites albumin gradient (SAAG) of > 1.1
g/dL. Which of the following clincial conditions are
compatible with such finding?
A. TB peritonitis
B. Pancreatic ascites
C. Peritoneal carcinomatosis
D. Portal Hypertension
E. None of the above
296 What is the target blood pressure in chronic kidney CKD patients with diabetes or proteinuria >1 g per ERIC ROYD FINAL EXAM -
disease (CKD) patients with diabetes or proteinuria 24 h, blood pressure should be reduced to 125/75, TALAVERA, MD FEB 2015
of > 1g/24hr? if achievable without prohibitive adverse effects. (TOP 1 - AUG 2014
A. 130/85 mmHg MED BOARDS;
B. 125/75 mmHg TOPNOTCH MD)
C. 120/80 mmHg
D. 130/80 mmHg
E. 140/90 mmHg

297 A 66 year old female, known hypertensive and The National Institute of Neurological Disorders ERIC ROYD FINAL EXAM -
diabetic, was brough to the ER due to a sudden onset and Stroke (NINDS) recombinant tPA (rtPA) TALAVERA, MD FEB 2015
of left sided weakness. Vital signs are as follows: BP Stroke Study showed a clear benefit for IV rtPA in (TOP 1 - AUG 2014
is 120/70, PR 97, RR 20, T 36.5. Neurologic selected patients with acute stroke. The NINDS MED BOARDS;
examination showed preferential gaze to the right, study used IV rtPA (0.9 mg/kg to a 90-mg max; TOPNOTCH MD)
shallow left nasolabial fold, MMT of 2/5 on both left 10% as a bolus, then the remainder over 60
upper and lower extremites and a positive Babinski minutes) versus placebo in patients with ischemic
on the left. A plain cranial CT scan was done which stroke within 3 hours of onset.
ruled out the presence of intracerebral hemorrhage.
To be considered a candidate for thrombolytic
therapy, the onset of symptoms to time of drug
administration should be less than or equal to how
many hours ?
A. 8 hours
B. 2 hours
C. 3 hours
D. 10 hours
E. 14 hours
298 What is the most common manifestation of diffuse The most common manifestation of diffuse CNS ERIC ROYD FINAL EXAM -
CNS lupus? lupus is cognitive dysfunction, including TALAVERA, MD FEB 2015
A. Headache difficulties with memory and reasoning. (TOP 1 - AUG 2014
B. Cognitive dysfunction MED BOARDS;
C. Psychosis TOPNOTCH MD)
D. Seizures
E. Myelopathy

299 A 23 year old male sought consult due to polyuria. If water deprivation test resulted in concentration ERIC ROYD FINAL EXAM -
PE was essentially normal. Blood tests showed a of urine: think of primary polydipsia. If TALAVERA, MD FEB 2015
normal FBS and HBA1C. Plasma osmolality and administration of vasopressin resulted in (TOP 1 - AUG 2014
serum sodium were both elevated. Urinalysis was concentration of urine: think of central DI MED BOARDS;
done which showed a decreased specific gravity. TOPNOTCH MD)
Water deprivation test was done which did not
concentrate the urine. Vasopressin was then
administered however it also failed to concentrate
the urine. What is the most likely diagnosis for this
case?
A. SIADH
B. Central diabetes insipidus
C. Psychogenic polydipsia
D. Nephrogenic diabetes insipidus
E. None of the above
300 Which of the following tests can be done to ASPGN will present with a low C3 level while it is ERIC ROYD FINAL EXAM -
distinguish IgA nephropathy with acute post normal in IgA nephropathy TALAVERA, MD FEB 2015
streptococcal glomerulonephritis (APSGN)? (TOP 1 - AUG 2014
A. KUB ultrasound MED BOARDS;
B. Serum Creatinine TOPNOTCH MD)
C. Lipid profile
D. CBC
E. Serum C3 level

TOPNOTCH MEDICAL BOARD PREP INTERNAL MEDICINE SUPEREXAM Page 42 of 95


For inquiries visit www.topnotchboardprep.com.ph or email us at topnotchmedicalboardprep@gmail.com
TOPNOTCH MEDICAL BOARD PREP INTERNAL MEDICINE SUPEREXAM
For inquiries visit www.topnotchboardprep.com.ph or email us at topnotchmedicalboardprep@gmail.com
Item QUESTION EXPLANATION AUTHOR TOPNOTCH
# EXAM
301 the presence of rapid x+y descent in jugular venous the following are JVP abnormalities and its LEAN ANGELO BACK-UP
waveform is correlated in what cardiac condition? correlated cardiac conditions: increase JVP- SILVERIO, MD MIDTERM
A. Atrioventricular dissociation Congestive heart failure; large a wave- tricuspid (TOP 4 - AUG 2014 EXAM - FEB
B. Tricuspid stenosis stenosis, RVH, pulmonary hypertension, cannon a MED BOARDS; 2015
C. Tricuspid regurgitation wave - atrioventricular dissociation, rapid x+y TOPNOTCH MD),
D. Constrictive pericarditis descent - constrictive pericarditis, kussmaul sign- MD
E. Pericardial tamponade pericardial tamponade, RVH, and constrictive
pericarditis; large fused cv wave - tricuspid
regurgitation. Mayo clinic IM concise textbook 1st
ed p 37
302 which of the following is not an absolute absolute contraindication for fibrinolysis in LEAN ANGELO BACK-UP
contraindication for fibrinolysis in STEMI? STEMI: any prior ICH, known strucutral SILVERIO, MD MIDTERM
A. Presence of malignant intracranial neoplasm cerebrovascular lesion, known malignant (TOP 4 - AUG 2014 EXAM - FEB
B. Active peptic ulcer disease intracranial neoplasm, ischemic stroke within 3 MED BOARDS; 2015
C. Suspected aortic dissection months EXCEPT acute ischemic stroke in evolution TOPNOTCH MD),
D. Concomittant ischemic stroke in evolution ( < (<3hrs), suspected aortic dissection, active MD
1hr) bleeding or bleeding diathesis ( excluding
E. bleeding diathesis menses), and significant closed head or facial
trauma.Mayo clinic IM concise textbook 1st ed p
106
303 A 42 y/o female complaining of dry nonproductive thymoma is associated with the following LEAN ANGELO BACK-UP
cough of 3 weeks duration. This is accompanied by condition: myasthenia gravis, pure red cell aplasia, SILVERIO, MD MIDTERM
weight loss and fatigue. She is a chronic smoker with hypogammaglobulinemia, polymyositis,SLE, (TOP 4 - AUG 2014 EXAM - FEB
20 pack years and an alcoholic beverage drinker. CT sjogren, ulcerative colitis , pernicious anemia, MED BOARDS; 2015
scan was requested showed absence of any scleroderma, and panhypopituitarism. TOPNOTCH MD),
suspicious lung lesion but an incidental finding of Erythrocytosis secondary to increase production MD
thymoma was seen. the latter may be associated of erythropoeitin is a paraneoplastic syndrome
with the following except? associated with hepatocellular carcinoma and
A. pure red cell aplasia renal cell neoplasm.
B. erythrocytosis
C. hypogammaglobulinemia
D. polymyositis
E. none of the above

304 what is a recommended therapy for patients renal cell carcinoma is known to be resistant to LEAN ANGELO BACK-UP
diagnosed with metastatic renal cell carcinoma? traditional chemotherapeutic agents. Cytokine SILVERIO, MD MIDTERM
A. Gemcitabine containing chemotherapy therapy with interleukin 2 or interferon gamma (TOP 4 - AUG 2014 EXAM - FEB
B. Radical nephrectomy with adjuvant produces regression only in about 10-20% of MED BOARDS; 2015
chemotherapy patient with metastatic disease. recently, sunitinib TOPNOTCH MD),
C. Interferon gamma based therapy or sorafenib were proved to be superior to other MD
D. Sorafenib biologic based therapy and now the first line
E. palliative managment. therapy for metastatic RCC. Harrison 18th ed.
chapter 94
305 An asymptomatic 64 y/o female went for consult patients with high risk adenomatous polyp ( LEAN ANGELO BACK-UP
secondary to incidental finding of one 1.8cm sessille villous, and/or >2.5cm) should have a follow up SILVERIO, MD MIDTERM
villous adenoma on her transverse colon during colonoscopy or radiographic study in 3 years. CT (TOP 4 - AUG 2014 EXAM - FEB
routine colonoscopy. What is the next step in scan is only done for staging once colon cancer is MED BOARDS; 2015
managing this case? diagnosed and not just by presence of polyps TOPNOTCH MD),
A. repeat colonoscopy after 10 years alone. Routine colonoscopy after 10 years is MD
B. abdominal CT scan with contrast recommended if the initial screening is negative.
C. transverse colectomy surgery is recommended if the diagnosis is colonic
D. colonoscopy after 3 years malignancy.
E. Reassurance
306 A 49 y/o (+) htn, (+) T2DM male was admitted at the aztreonam, beta lactams and cephalosporin are LEAN ANGELO BACK-UP
ICU secondary to MVA. After 5 days of generally ineffective in Legionella infection since SILVERIO, MD MIDTERM
hospitalization. Patient experienced nonproductive the latter is an intracellular bacteria. (TOP 4 - AUG 2014 EXAM - FEB
cough and fever. Supine CXR revealed new bibasilar MED BOARDS; 2015
pulmonary infiltrates. urine antigen is positive for TOPNOTCH MD),
legionella infection. which of the following MD
medication is not effective in this present case?
A. Levofloxacin
B. Aztreonam
C. Cotrimoxazole
D. Doxycycline
E. Clindamycin

307 Which of the following is an indication for adjunct Initial antibiotic management of PCP is LEAN ANGELO BACK-UP
glucocorticoids in a patient diagnosed with characterized by worsening of condition due to SILVERIO, MD MIDTERM
Pneumocystics Jiroveci infection? lysis of organism and immune mounted response (TOP 4 - AUG 2014 EXAM - FEB
A. Aa gradient of 42mmHg to the intracellular contents. adjunct MED BOARDS; 2015
B. Acute Respiratory distress syndrome glucocorticoid administration is recommended in TOPNOTCH MD),
C. pneumothorax moderate or severe cases of PCP infection as MD
D. Lack of improvement after 3 days of treatment determined by room air PaO2 < 70mmHg and Aa
E. all of the above gradient of >35mmHg.

308 which of the following manuevers would be In hypertrophic cardiomyopathy, there is an LEAN ANGELO BACK-UP
expected to increase the intensity of murmur in assymetric hypertrophy of the interventricular SILVERIO, MD MIDTERM
hypertrophic obstructive cardiomyopathy? septum, creating a dynamic outflow tract (TOP 4 - AUG 2014 EXAM - FEB
A. Squatting obstruction. Maneuvers that will decrease the left MED BOARDS; 2015
B. Valsalva ventricular volume will increase the intensity of TOPNOTCH MD),
C. handgrip the murmur. In the choices, only valsalva MD
D. A and C maneuver will decrease the left ventricular
E. B and C preload.

TOPNOTCH MEDICAL BOARD PREP INTERNAL MEDICINE SUPEREXAM Page 43 of 95


For inquiries visit www.topnotchboardprep.com.ph or email us at topnotchmedicalboardprep@gmail.com
TOPNOTCH MEDICAL BOARD PREP INTERNAL MEDICINE SUPEREXAM
For inquiries visit www.topnotchboardprep.com.ph or email us at topnotchmedicalboardprep@gmail.com
Item QUESTION EXPLANATION AUTHOR TOPNOTCH
# EXAM
309 Which of the following is not a potential this patient may develop short bowel syndrome LEAN ANGELO BACK-UP
complication of patient who underwent massive following resection of larger portion of the small SILVERIO, MD MIDTERM
resection of small bowel due to superior mesenteric and large intestine. Multiple factors contribute to (TOP 4 - AUG 2014 EXAM - FEB
artery occlusion? diarrhea and steatorrhea including gastric acid MED BOARDS; 2015
A. Calcium oxalate renal stones hypersecretion, increased bile acids in the small TOPNOTCH MD),
B. Cholesterol gallstones bowel due to decreased absorption in the MD
C. Progressive coronary atherosclerosis remaining bowel. and lactose intolerance due to
D. gastric ulcers gastric acid hypersecretion. the propensity to
E. none of the above develop oxalate renal stones is because of the
binding of free fatty acids in the colon to the
calcium. thus the calcium is not free to bind to
oxalate and the latter is easily absorbed in the
large intestine. The increase bile acid pool size will
yield to supersaturation of gallbladder contents
leading to cholesterol gallstones. loss of inhibitory
transmitters coming from the small intestine will
make the gastric acid secretion unchecked. there is
no relation between atherosclerosis and
shortbowel syndrome.
310 what is the earliest radiographic finding in ( SIMILAR TO PREVIOUS BOARD EXAM LEAN ANGELO BACK-UP
rheumatoid arthritis ? CONCEPT/PRINCIPLE). Harrisons 18 th chapter SILVERIO, MD MIDTERM
A. Periarticular soft tissue swelling 321 (TOP 4 - AUG 2014 EXAM - FEB
B. joint space loss MED BOARDS; 2015
C. Periarticular osteopenia TOPNOTCH MD),
D. Capsular thickening MD
E. Subchondral erosions

311 the following statement is true about acromegaly measuring the serum concentration of IGF-1 is the LEAN ANGELO BACK-UP
except? best screening test for acromegaly. It is always SILVERIO, MD MIDTERM
A. Increased risk of premalignant colon polyps and increased in patient with active acromegaly. (TOP 4 - AUG 2014 EXAM - FEB
colon cancer Physiologic increased is seen in the following MED BOARDS; 2015
B. A random serum level of GH is not helpful in condition: pregnancy, adolescence, sleep apnea. TOPNOTCH MD),
establishing the diagnosis MD
C. Surgical excision is the treatment of choice
D. serum concentration of IGF1 is increased in only
50% of patient with active acromegaly
E. none of the above

312 The following condition are associated with type A type A gastritis involves the fundus and/or body of LEAN ANGELO BACK-UP
gastritis except ? the stomach. It is associated with autoimmune, SILVERIO, MD MIDTERM
A. Gastric carcinoids atrophic gastritis and pernicious anemia. The (TOP 4 - AUG 2014 EXAM - FEB
B. Pernicious anemia serum levels of gastrin since achlorydia develops MED BOARDS; 2015
C. H pylori infection in this patient. Gastrin is potent growth factor for TOPNOTCH MD),
D. Atrophic gastritis the development of gastric carcinoid tumors and MD
E. Hypergastrinemia gastric polyps. on the otherhand, type B gastritis
involves primarily the antrum and it is associated
with H pylori infection.
313 which of the following is not part of the treatment ADA treatment goals for DM: Hba1c <7.0%, LEAN ANGELO BACK-UP
goals in diabetes mellitus according ADA? Preprandial glucose of 70-130mg/dl, postprandial SILVERIO, MD MIDTERM
A. HBA1c <6.5% glucose <180mg/dl. BP <130/80mmHg, LDL (TOP 4 - AUG 2014 EXAM - FEB
B. TAG <130mg/dl <100mg/dl, HDL >40mg/dl (men), >50mg/dl MED BOARDS; 2015
C. BP <120/80mmHg (women), TAG <150mg/dl. Harrison 18th ed p TOPNOTCH MD),
D. HDL >60mg/dl. E. All of the above 2990 MD

314 Which of the following is not part of Bedside Index BISAP scoring is predictive of hospital mortality LEAN ANGELO BACK-UP
of Severity in Acute Pancreatitis? secondary to acute pancreatitis. The presence of 3 SILVERIO, MD MIDTERM
A. Confusion or more is donates a higher risk. BUN> 22mg%, (TOP 4 - AUG 2014 EXAM - FEB
B. Age >60years old impaired mental status, SIRS 2/4, Age >60 y/o, MED BOARDS; 2015
C. Prsence of pleural effusion Pleural effusion. Harrison 18th d page 2640 TOPNOTCH MD),
D. BMI > 30 MD
E. BUN > 22mg%

315 which of the following drugs have a direct toxic drug induced liver injury can be categorized into LEAN ANGELO BACK-UP
effect on hepatocytes ? two types: direct toxic effect and idiosyncratic SILVERIO, MD MIDTERM
A. halothane effect. The former is predictable, dose related (TOP 4 - AUG 2014 EXAM - FEB
B. simvastatin pattern of injury and short time to effect duration. MED BOARDS; 2015
C. Acetaminophen The most common example is acetaminophen ( TOPNOTCH MD),
D. isoniazid NAPQI) others are Carbon tetrachloride, MD
E. All of the above trichloroethylene,tetracycline, and amanitin.
examples of drugs that causes idiosyncratic liver
injury are the following: halothane, isothane,
isoniazid, statins and chlorpromazine.
316 which in the following condition is not associated digital clubbing is most commonly associated with LEAN ANGELO BACK-UP
with digital clubbing? advanced lung diseases such as bronchiectasis, SILVERIO, MD MIDTERM
A. hyperthryroidism cystic fibrosis, sarcoidosis, idiopathic pulmonary (TOP 4 - AUG 2014 EXAM - FEB
B. Hepatocellular carcinoma fibrosis, tuberculosis, empyema , and lung abscess. MED BOARDS; 2015
C. Idiopathic pulmonary fibrosis extrapulmonary causes include cyanotic TOPNOTCH MD),
D. Chronic obstructive pulmonary disease congenital heart disease, subacute bacterial MD
E. None of the above endocarditis, crohns disease, ulcerative colitis, and
untreated hyperthyroidism. COPD does not cause
clubbing.

TOPNOTCH MEDICAL BOARD PREP INTERNAL MEDICINE SUPEREXAM Page 44 of 95


For inquiries visit www.topnotchboardprep.com.ph or email us at topnotchmedicalboardprep@gmail.com
TOPNOTCH MEDICAL BOARD PREP INTERNAL MEDICINE SUPEREXAM
For inquiries visit www.topnotchboardprep.com.ph or email us at topnotchmedicalboardprep@gmail.com
Item QUESTION EXPLANATION AUTHOR TOPNOTCH
# EXAM
317 A 47 y/o male asymptomatic presents in your clinic this is a classic case of CML. Characterized by LEAN ANGELO BACK-UP
with an incidental finding of WBC count 110x109 /L, increased WBC count, granulocytes in all stages of SILVERIO, MD MIDTERM
basophilia, eosinophilia and obliterated traube maturation. Presence of philadelphia chromosome (TOP 4 - AUG 2014 EXAM - FEB
space. Cytogenetic studies showed t(9,22). Which of is the hallmark for this condition. This is caused by MED BOARDS; 2015
the following is true about his condition except? translocation of bcr -abl gene. Leukocyte alkaline TOPNOTCH MD),
A. the standard therapy for this condition is phosphatase ( marker of functional activity of MD
Imatinib granulocytes) is low compared to reactive
B. leukocyte alkaline phosphatase score is low or leukocytosis. there is an increaesed Vitamin B12
zero level due to increased level of transcobalamin I.
C. increased serum Vitamin B12 level Imatinib is the treatment of choice
D. philadelphia chromosome is the hallmark of this
condition
E. all of the above
318 the most common genetic cause of End stage Renal Harrisons 18th ed p 2359 LEAN ANGELO BACK-UP
disease in children and adolesence? SILVERIO, MD MIDTERM
A. ARPKD (TOP 4 - AUG 2014 EXAM - FEB
B. ADPKD MED BOARDS; 2015
C. Nephronopthisis TOPNOTCH MD),
D. Medullary cystic kidney disease MD
E. Medullary sponge kidney

319 which of the following is not an recommended recommended empiric antibiotics for HCAP LEAN ANGELO BACK-UP
empiric antibiotic treatment for patients diagnosed without risk factors for MDR Pathogen ( pls see SILVERIO, MD MIDTERM
with Healthcare associated Pneumonia and without table 257-1 Harrison 18th ed page 2130) are the (TOP 4 - AUG 2014 EXAM - FEB
risk factors for MDR pathogens? following: ceftriaxone, moxifloxacin,levofloxacin, MED BOARDS; 2015
A. ceftriaxone ciprofloxacin, ampicillin sulbactam, and TOPNOTCH MD),
B. meropenem ertapenem. Harrison 18th ed p 2139 ( SIMILAR TO MD
C. moxifloxacin PREVIOUS BOARD EXAM CONCEPT/PRINCIPLE)
D. ampicillin/sulbactam
E. none of the above

320 minimum definition of clinically important Harrison 18th ed pag 641. LEAN ANGELO BACK-UP
involuntary weight loss is? SILVERIO, MD MIDTERM
A. >5% loss of body weight in 6-12 months (TOP 4 - AUG 2014 EXAM - FEB
B. >10% loss of body weight in 5 months MED BOARDS; 2015
C. >5% loss of body weight in 1 month TOPNOTCH MD),
D. >10% loss of body weight in 6-12 months MD
E. none of the above

321 What is the major symptom of acute pancreatitis? Pain is characteristically steady and boring in DEBBIE ROSE BACK-UP
A. abdominal pain character, located in the epigastrium and TANENGSY, MD MIDTERM
B. dyspnea periumbilical region and often radiates to the (TOP 5 - AUG 2014 EXAM - FEB
C. hypotension back, chest, flank, and lower abdomen. C & D are MED BOARDS; 2015
D. abdominal tenderness not symptoms, but are signs. Harrison's Principles TOPNOTCH MD)
of Internal Medicine 18th edition, p. 2636
322 Supportive care measure/s that form the hallmark in DEBBIE ROSE BACK-UP
the treatment in acute pancreatitis, except: TANENGSY, MD MIDTERM
A. bowel rest (TOP 5 - AUG 2014 EXAM - FEB
B. hydration with crystalloid MED BOARDS; 2015
C. analgesia TOPNOTCH MD)
D. pancreatic enzyme supplementation
E. none of the above

323 The most common valvular abnormality in Harrison's Principles of Internal Medicine 18th DEBBIE ROSE BACK-UP
rheumatoid arthritis? edition, p. 2739 TANENGSY, MD MIDTERM
A. tricupid regurgitation (TOP 5 - AUG 2014 EXAM - FEB
B. mitral stenosis MED BOARDS; 2015
C. mitral regurgitation TOPNOTCH MD)
D. aortic regurgitation
324 In the treatment of nongonococcal bacterial arthritis, Weight bearing should be avoided until signs of DEBBIE ROSE BACK-UP
the following may be done, except: inflammation have subsided, but frequent passive TANENGSY, MD MIDTERM
A. needle aspiration motion of joint is indicated to maintain full (TOP 5 - AUG 2014 EXAM - FEB
B. weight bearing mobility. Arthrotomy best manages septic arthritis MED BOARDS; 2015
C. intravenous bactericidal agents of the hip in young children. Harrison's Principles TOPNOTCH MD)
D. arthrotomy of Internal Medicine 18th edition, p. 2845

325 True regarding diabetic ketoacidosis, except: Relative insulin deficiency and inadequate fluid DEBBIE ROSE BACK-UP
A. fluid deficit is often 3-5 L intake are underlying causes of HHS. HHS free TANENGSY, MD MIDTERM
B. Insulin deficiency and glucagon excess are water deficit: 9 - 10 L. Bicarbonate replacement (TOP 5 - AUG 2014 EXAM - FEB
necessary for its development often impairs heart function, decreases tissue MED BOARDS; 2015
C. Inadequate fluid intake is the underlying cause oxygenation, and promotes hypokalemia. TOPNOTCH MD)
D. initial intravenous fluid replacement: 0.9% NSS Harrison's Principles of Internal Medicine 18th
E. Bicarbonate replacement is not necessary. edition, p. 2977-2979

326 This is the single best measure of hepatic synthetic Serum albumin is not a good indicator of acute DEBBIE ROSE BACK-UP
function: hepatic dysfunction because of its slow turnover TANENGSY, MD MIDTERM
A. serum globulins (T1/2: 18-20 days). Alkaline phosphatase (the (TOP 5 - AUG 2014 EXAM - FEB
B. serum albumin liver isoenzyme) is increased in cholestasis. MED BOARDS; 2015
C. clotting factors Harrison's Principles of Internal Medicine 18th TOPNOTCH MD)
D. alkaline phosphatase edition, p. 2529-2530
E. liver span

TOPNOTCH MEDICAL BOARD PREP INTERNAL MEDICINE SUPEREXAM Page 45 of 95


For inquiries visit www.topnotchboardprep.com.ph or email us at topnotchmedicalboardprep@gmail.com
TOPNOTCH MEDICAL BOARD PREP INTERNAL MEDICINE SUPEREXAM
For inquiries visit www.topnotchboardprep.com.ph or email us at topnotchmedicalboardprep@gmail.com
Item QUESTION EXPLANATION AUTHOR TOPNOTCH
# EXAM
327 True of Crohn disease but not true of ulcerative ANCA is more frequently associated with DEBBIE ROSE BACK-UP
colitis: ulcerative colitis. Crohn - ASCA (+) Harrison's TANENGSY, MD MIDTERM
A. gross blood and mucus in stool Principles of Internal Medicine 18th edition, p. (TOP 5 - AUG 2014 EXAM - FEB
B. systemic symptoms 2486 table 295-5 MED BOARDS; 2015
C. pain TOPNOTCH MD)
D. recurrence after surgery
E. ANCA (+)

328 True in the treatment of uric acid lithiasis, except: Raising the urine pH above 6.5 won't provide DEBBIE ROSE BACK-UP
A. raise urine pH above 6.5 additional benefit but increases the risk of calcium TANENGSY, MD MIDTERM
B. lower excessive uric acid secretion phosphate stone formation. Harrison's Principles (TOP 5 - AUG 2014 EXAM - FEB
C. give supplemental alkali of Internal Medicine 18th edition, p. 2386 MED BOARDS; 2015
D. low purine diet TOPNOTCH MD)
E. allopurinol

329 The most common hematologic abnormality in Harrison's Principles of Internal Medicine 18th DEBBIE ROSE BACK-UP
rheumatoid arthritis is: edition, p. 2740 TANENGSY, MD MIDTERM
A. thrombocytopenia (TOP 5 - AUG 2014 EXAM - FEB
B. neutropenia MED BOARDS; 2015
C. thrombocytosis TOPNOTCH MD)
D. microcytic, hypochromic anemia
E. normocytic, normochromic anemia

330 Usually the earliest manifestation of The earliest manifestations of ophthalmopathy in DEBBIE ROSE BACK-UP
ophthalmopathy in Grave's Disease: Grave's Disease are usually a sensation of TANENGSY, MD MIDTERM
A. periorbital edema grittiness, eye discomfort, and excess tearing. (TOP 5 - AUG 2014 EXAM - FEB
B. chemosis Harrison's Principles of Internal Medicine 18th MED BOARDS; 2015
C. proptosis edition, p. 2924 TOPNOTCH MD)
D. excess tearing
E. diplopia

331 In the diagnosis of pheochromocytoma, the DEBBIE ROSE BACK-UP


measurement of this is the most sensitive and is less TANENGSY, MD MIDTERM
susceptible to false (+) increase from stress: (TOP 5 - AUG 2014 EXAM - FEB
A. 24hour urine VMA MED BOARDS; 2015
B. 24hour urine metanephrines TOPNOTCH MD)
C. plasma metanephrine
D. 24hour urine catecholamine
E. plasma catecholamine

332 One of the following is not a contraindication to the Harrison's Principles of Internal Medicine 18th DEBBIE ROSE BACK-UP
administration of IV rtPA for acute ischemic stroke: edition, p. 3273 table 370-1 TANENGSY, MD MIDTERM
A. coma (TOP 5 - AUG 2014 EXAM - FEB
B. GI bleed in the preceeding 21 days MED BOARDS; 2015
C. prior stroke in 3 months TOPNOTCH MD)
D. onset to adminstration in </= 3 hours
E. recent MI

333 The most common cause of cerebral embolism Harrison's Principles of Internal Medicine 18th DEBBIE ROSE BACK-UP
overall: edition, p. 3274 TANENGSY, MD MIDTERM
A. MI (TOP 5 - AUG 2014 EXAM - FEB
B. prosthetic valve MED BOARDS; 2015
C. rheumatic heart disease TOPNOTCH MD)
D. nonrheumatic atrial fibrillation
E. ischemic cardiomyopathy

334 The initial treatment of cirrhotic ascites is: Harrison's Principles of Internal Medicine 18th DEBBIE ROSE BACK-UP
A. furosemide edition, p. 332 TANENGSY, MD MIDTERM
B. restriction of salt intake (TOP 5 - AUG 2014 EXAM - FEB
C. spironolactone and furosemide MED BOARDS; 2015
D. transjugular intrahepatic peritoneal shunt TOPNOTCH MD)
E. restriction of water intake

335 The presence of scleral icterus indicates a serum Harrison's Principles of Internal Medicine 18th DEBBIE ROSE BACK-UP
bilirubin of at least: edition, p. 324 TANENGSY, MD MIDTERM
A. 1 mg/dL (TOP 5 - AUG 2014 EXAM - FEB
B. 1.5 mg/dL MED BOARDS; 2015
C. 2 mg/dL TOPNOTCH MD)
D. 2.5 mg/dL
E. 3 mg/dL

336 The hemoglobin level in patient with palmar crease Harrison's Principles of Internal Medicine 18th DEBBIE ROSE BACK-UP
pallor is at most: edition, p. 449 TANENGSY, MD MIDTERM
A. <11 g/dL (TOP 5 - AUG 2014 EXAM - FEB
B. <10 g/dL MED BOARDS; 2015
C. <9 g/dL TOPNOTCH MD)
D. <8 g/dL
337 The clinical hallmark of acne vulgaris: Harrison's Principles of Internal Medicine 18th DEBBIE ROSE BACK-UP
A. comedones edition, p.404 TANENGSY, MD MIDTERM
B. scarring (TOP 5 - AUG 2014 EXAM - FEB
C. pustules MED BOARDS; 2015
D. telangiectasias TOPNOTCH MD)
E. papules

TOPNOTCH MEDICAL BOARD PREP INTERNAL MEDICINE SUPEREXAM Page 46 of 95


For inquiries visit www.topnotchboardprep.com.ph or email us at topnotchmedicalboardprep@gmail.com
TOPNOTCH MEDICAL BOARD PREP INTERNAL MEDICINE SUPEREXAM
For inquiries visit www.topnotchboardprep.com.ph or email us at topnotchmedicalboardprep@gmail.com
Item QUESTION EXPLANATION AUTHOR TOPNOTCH
# EXAM
338 Which of the following strictly defines a fever? A & B pertain to hyperpyrexia. A PM temperature DEBBIE ROSE BACK-UP
A. > 41.5C of 37.7C also defines a fever. Harrison's Principles TANENGSY, MD MIDTERM
B. > 106.7F of Internal Medicine 18th edition, p. 143 (TOP 5 - AUG 2014 EXAM - FEB
C. AM temperature of >37.2C MED BOARDS; 2015
D. PM temperature of >37.5C TOPNOTCH MD)
E. none of the above

339 Causes of hypovolemic hyponatremia include the Nephrotic syndrome is a cause of hypervolemic DEBBIE ROSE BACK-UP
following, except: hyponatremia. TANENGSY, MD MIDTERM
A. primary hypoaldosteronism (TOP 5 - AUG 2014 EXAM - FEB
B. diarrhea MED BOARDS; 2015
C. burns TOPNOTCH MD)
D. nephrotic syndrome
E. none of the above

340 A true statement regarding potassium level The use of intravenous administration should be DEBBIE ROSE BACK-UP
derangements: limited to patients unable to utilize the enteral TANENGSY, MD MIDTERM
A. Oral replacement with KCl is the mainstay of route or in the setting of severe complications. A (TOP 5 - AUG 2014 EXAM - FEB
therapy for hypokalemia. decrease in renal K excretion is the most common MED BOARDS; 2015
B. Reduced tissue uptake is the most common cause underlying cause of hyperkalemia. Intravenous TOPNOTCH MD)
of hyperkalemia. bicarbonate should be reserved for patients with
C. Intravenous bicarbonate is routinely used in the hyperkalemia concomitant metabolic acidosis.
treatment of hyperkalemia.
D. none of the above
341 What is the significance of (+) PPD? JESSICA MAE BACK-UP
A. Exposure to TB SANCHEZ, MD MIDTERM
B. Infection with TB (TOP 4 - AUG 2014 EXAM - FEB
C. Active TB MED BOARDS; 2015
D. Highly infectious TB TOPNOTCH MD)

342 Why is monotherapy with isoniazid highly JESSICA MAE BACK-UP


discouraged in treatment of active TB? SANCHEZ, MD MIDTERM
A. Increases the risk for drug resistance (TOP 4 - AUG 2014 EXAM - FEB
B. Has not been proven effective as MED BOARDS; 2015
chemoprophylaxis TOPNOTCH MD)
C. Due to hepatotoxicity
D. Increased occurrence of peripheral neuropathy if
not combined with other medications
343 Which autoantibody is highly specific for SLE? JESSICA MAE BACK-UP
A. Anti-histone SANCHEZ, MD MIDTERM
B. Anti-RNP (TOP 4 - AUG 2014 EXAM - FEB
C. Anti-Ro/SSA MED BOARDS; 2015
D. Anti-Sm TOPNOTCH MD)

344 A newly diagnosed diabetic patient was brought to Sulfonylureas, especially those of the 1st JESSICA MAE BACK-UP
the ER because of loss of consciousness. His CBG is generation, are notorious for causing SANCHEZ, MD MIDTERM
35 mg/dL. He was recently started on a medication hypoglycaemia. (TOP 4 - AUG 2014 EXAM - FEB
for diabetes by their family physician. Which is the MED BOARDS; 2015
most likely medication given to him? TOPNOTCH MD)
A. Thiazolidinedione
B. Sulfonylurea
C. Alpha glucosidase inhibitor
D. Metformin
345 A 20 year old female consults due to tea-colored C3 levels are decreased in acute PSGN JESSICA MAE BACK-UP
urine and bipedal edema. She had recent fever and SANCHEZ, MD MIDTERM
sore throat. Her BP is 150/90 and serum creatinine (TOP 4 - AUG 2014 EXAM - FEB
is 1 mg/dL. C3 level is 130 mg/dL (NV = 83-177 MED BOARDS; 2015
mg/dL). What feature would suggest IgA TOPNOTCH MD)
nephropathy over acute post-streptococcal
glomerulonephritis?
A. C3 levels
B. Bipedal
C. If the fever and cough occurred 16 days ago
D. Her blood pressure
346 A 45 year old lawyer came to the ER because of This is a case of hypertensive emergency for which JESSICA MAE BACK-UP
headache and right sided weakness. BP was noted to IV antihypertensives are indicated. SANCHEZ, MD MIDTERM
be 220/120, HR 110/bpm. Which antihypertensive (TOP 4 - AUG 2014 EXAM - FEB
agent is recommended? MED BOARDS; 2015
A. Clonidine sublingual TOPNOTCH MD)
B. Esmolol IV
C. Nicardipine IV
D. Nifedipine sublingual
347 Which of the following is an absolute JESSICA MAE BACK-UP
contraindication to fibrinolytic therapy? SANCHEZ, MD MIDTERM
A. Suspected aortic dissection (TOP 4 - AUG 2014 EXAM - FEB
B. Hypertension with SBP > 160 mmHg and/or DBP MED BOARDS; 2015
>90 mmHg TOPNOTCH MD)
C. Recent invasive surgical procedure (<2 weeks)
D. Prolonged CPR

TOPNOTCH MEDICAL BOARD PREP INTERNAL MEDICINE SUPEREXAM Page 47 of 95


For inquiries visit www.topnotchboardprep.com.ph or email us at topnotchmedicalboardprep@gmail.com
TOPNOTCH MEDICAL BOARD PREP INTERNAL MEDICINE SUPEREXAM
For inquiries visit www.topnotchboardprep.com.ph or email us at topnotchmedicalboardprep@gmail.com
Item QUESTION EXPLANATION AUTHOR TOPNOTCH
# EXAM
348 A 20 year old female student complains of JESSICA MAE BACK-UP
palpitations. On PE, she has a mid systolic click SANCHEZ, MD MIDTERM
followed by a mid to late crescendo murmur at the (TOP 4 - AUG 2014 EXAM - FEB
apex. The diagnosis is most probably: MED BOARDS; 2015
A. Tricuspid regurgitation TOPNOTCH MD)
B. Mitral valve prolapse
C. Mitral stenosis
D. Atrial septal defect
349 A 20 year old female student complains of In MVP: JESSICA MAE BACK-UP
palpitations. On PE, she has a mid systolic click Squatting - increases venous return = delayed SANCHEZ, MD MIDTERM
followed by a mid to late crescendo murmur at the systolic click = shorter murmur (TOP 4 - AUG 2014 EXAM - FEB
apex. If she was instructed to squat, this will result Standing or Valsalva maneuver - decreases venous MED BOARDS; 2015
to: return = earlier systolic click = prolonged murmur TOPNOTCH MD)
A. Increase in venous return and delay in the
occurrence of the systolic click
B. Decrease in venous return and delay in the
occurrence of the systolic click
C. Increase in venous return and early occurrence of
the systolic click
D. Decrease in venous return and early occurrence of
the systolic click
350 What is the cardinal symptom of heart failure? JESSICA MAE BACK-UP
A. Fatigue SANCHEZ, MD MIDTERM
B. Orthopnea (TOP 4 - AUG 2014 EXAM - FEB
C. Paroxysmal noctural dyspnea MED BOARDS; 2015
D. Edema TOPNOTCH MD)

351 What is the most useful index of LV function? JESSICA MAE BACK-UP
A. Stroke volume SANCHEZ, MD MIDTERM
B. End diastolic volume (TOP 4 - AUG 2014 EXAM - FEB
C. Ejection fraction MED BOARDS; 2015
D. End systolic volume TOPNOTCH MD)

352 A 55 year old male, both diabetic and hypertensive, Diabetic nephropathy present with normal size to JESSICA MAE BACK-UP
has chronic renal failure due to diabetic enlarged kidneys on ultrasound. A patient who has SANCHEZ, MD MIDTERM
nephropathy. Which of the following findings will developed diabetic nephropathy most likely also (TOP 4 - AUG 2014 EXAM - FEB
most likely also be present in the patient? has diabetic retinopathy as well. MED BOARDS; 2015
A. Absence of proteinuria TOPNOTCH MD)
B. Contracted small kidneys on ultrasound
C. Diabetic retinopathy
D. Red cell casts in the urine sediment
353 A 55 year old man is found to have a serum calcium The patient’s hypercalcemia was most likely JESSICA MAE BACK-UP
of 13 mg/dL after complaining of fatigue and thirst caused by elevated levels of PTHrp, a SANCHEZ, MD MIDTERM
for the past month. A CXR revealed a 4 cm mass in paraneoplastic syndrome associated with (TOP 4 - AUG 2014 EXAM - FEB
the right lower lobe of the lung. Which of the squamous cell carcinoma. MED BOARDS; 2015
following is the most likely cause of his TOPNOTCH MD)
hypercalcemia?
A. ACTH
B. ADH
C. PTH
D. PTH related protein
354 A 57 year old man is evaluated through endoscopy Majority of ulcers are caused by Helicobacter JESSICA MAE BACK-UP
after an episode of hematemesis. Findings showed a pylori, which is associated with poor SANCHEZ, MD MIDTERM
3 cm duodenal ulcer. Which of the following socioeconomic status. (TOP 4 - AUG 2014 EXAM - FEB
statements is correct regarding this finding? MED BOARDS; 2015
A. The lesion should be biopsied because duodenal Reference: Harrison’s Principles of Internal TOPNOTCH MD)
ulcers have increased risk of carcinoma Medicine, Self-Assessment and Board Review,
B. Poor socioeconomic status is a risk factor for the 18th ed.
development of this condition
C.The patient is not at risk for any associated
cancers
D. Antral gastritis is rarely found with this condition
355 Which of the following is the most common JESSICA MAE BACK-UP
symptom or sign of liver disease? SANCHEZ, MD MIDTERM
A. Fatigue (TOP 4 - AUG 2014 EXAM - FEB
B. Itching MED BOARDS; 2015
C. Jaundice TOPNOTCH MD)
D. Nausea
356 In women, what is the average amount of reported For women, drinking more than 1 alcoholic drink JESSICA MAE BACK-UP
daily alcohol intake that is associated with the daily (so at least 2 drinks), is associated with SANCHEZ, MD MIDTERM
development of chronic liver disease? development of chronic liver disease. For men, (TOP 4 - AUG 2014 EXAM - FEB
A. 1 drink more than 2 drinks (or at least 3 drinks) is the cut MED BOARDS; 2015
B. 2 drinks off. TOPNOTCH MD)
C. 3 drinks
D. 6 drinks
357 A 24 year old is newly diagnosed with SLE. Which of The system most commonly involved is the JESSICA MAE BACK-UP
the following organ system complications is she musculoskeletal system, with 95% of patients SANCHEZ, MD MIDTERM
most likely to have over the course of her lifetime? having involvement, usually as arthralgias or (TOP 4 - AUG 2014 EXAM - FEB
A. Cardiopulmonary myalgias. MED BOARDS; 2015
B. Renal TOPNOTCH MD)
C. Hematologic Reference: Harrison’s Principles of Internal
D. Musculoskeletal Medicine, Self-Assessment and Board Review,
18th ed.

TOPNOTCH MEDICAL BOARD PREP INTERNAL MEDICINE SUPEREXAM Page 48 of 95


For inquiries visit www.topnotchboardprep.com.ph or email us at topnotchmedicalboardprep@gmail.com
TOPNOTCH MEDICAL BOARD PREP INTERNAL MEDICINE SUPEREXAM
For inquiries visit www.topnotchboardprep.com.ph or email us at topnotchmedicalboardprep@gmail.com
Item QUESTION EXPLANATION AUTHOR TOPNOTCH
# EXAM
358 A 32 year old woman with known SLE was noted to Diagnosis is Libman-Sacks endocarditis, known for JESSICA MAE BACK-UP
have a new cardiac murmur on routine follow-up having sterile vegetations because it’s not thought SANCHEZ, MD MIDTERM
with her rheumatologist. She is asymptomatic, to be due to microbial infection. (TOP 4 - AUG 2014 EXAM - FEB
without any fever, weight loss, nor history of MED BOARDS; 2015
previous cardiac disease. On 2D echo, a vegetation TOPNOTCH MD)
on the mitral valve is demonstrated. Which of the
following statements is true?
A. Blood cultures are unlikely to be positive
B. Steroid therapy has been proven to improve this
condition
C. Pericarditis is frequently present concomitantly
D. The lesion has a low risk of embolisation
359 A 45 year old man reports to his physician that his GH is secreted in a highly pulsatile fashion, JESSICA MAE BACK-UP
wife noted coarsening of his facial features over therefore obtaining a random sample is not SANCHEZ, MD MIDTERM
several years. In addition, he also reports low libido reliable. IGF-1, the downstream mediator of GH, is (TOP 4 - AUG 2014 EXAM - FEB
and energy. PE shows frontal bossing and enlarged used as a marker instead. MED BOARDS; 2015
hands. An MRI confirms that he has a pituitary mass. TOPNOTCH MD)
Which of the following screening tests should be
ordered to diagnose the cause of the mass?
A. 24 hour urinary free cortisol
B. ACTH assay
C. Growth hormone level
D. Serum IGF-1 level
360 Which of the following proteins is the primary JESSICA MAE BACK-UP
source of bound T4 in the plasma? SANCHEZ, MD MIDTERM
A. Albumin (TOP 4 - AUG 2014 EXAM - FEB
B. Gamma globulin MED BOARDS; 2015
C. Thyroid peroxidase TOPNOTCH MD)
D. Thyroxine-binding globulin
361 The classic lesion of a secondary The major disorders of bone disease can be MAIRRE JAMES BACK-UP
hyperparathyroidism in patients with CKD which is classified into those associated with high bone GADDI, MD (TOP 4 MIDTERM
associated with high bone turnover turnover with increased PTH levels (including - AUG 2013 MED EXAM - FEB
A. Calciphylaxis osteitis fibrosa cystica, the classic lesion of BOARDS; 2015
B. Osteitis fibrosa cystica secondary hyperparathyroidism) and low bone TOPNOTCH MD)
C. Adynamic bone disease turnover with low or normal PTH levels
D. Osteomalacia (adynamic bone disease and osteomalacia). HPIM
E. None of the above 18th pg 2313

362 Which of the following is/are TRUE regarding A: proximal leg DVT; C: hypoxemia (decreased MAIRRE JAMES BACK-UP
pulmonry embolism? arterial PO2) and an increased alveolar-arterial O2 GADDI, MD (TOP 4 MIDTERM
A. About half of patietns with pelvic vein thrombosis tension gradient, which represents the inefficiency - AUG 2013 MED EXAM - FEB
or distal leg deep venous thrombosis develop of O2 transfer across the lungs; D: right sided BOARDS; 2015
pulmonary embolism heart failure HPIM 18th pg 2171 TOPNOTCH MD)
B. Isolated calf vein thrombi pose a much lower risk
of pulmonary embolism but are the most common
source of paradoxical embolism
C. The msot common gas exchange abnormalities are
hypercarbia and an increased alveolar oxygen
gradient
D. Progressive left sided heart failure is the usual
cause of death in PE
E. All are TRUE
363 The extrapulmonary site LEAST commonly involved In order of frequency, the extrapulmonary sites MAIRRE JAMES BACK-UP
in TB is/are the most commonly involved in TB are the lymph GADDI, MD (TOP 4 MIDTERM
A. Lymph nodes nodes, pleura, genitourinary tract, bones and - AUG 2013 MED EXAM - FEB
B. Peritoneum joints, meninges, peritoneum, and pericardium. BOARDS; 2015
C. Bones and joints HPIM 18th pg 1346 TOPNOTCH MD)
D. Meninges
E. Pericardium

364 In a patient with paroxysmal nocturnal The definitive diagnosis of PNH must be based on MAIRRE JAMES BACK-UP
hemoglobinuria, their RBCs are deficient in which of the demonstration that a substantial proportion of GADDI, MD (TOP 4 MIDTERM
the following surface proteins? the patient's red cells have an increased - AUG 2013 MED EXAM - FEB
A. CD4 and CD8 susceptibility to complement (C), due to the BOARDS; 2015
B. CD55 and CD59 deficiency on their surface of proteins TOPNOTCH MD)
C. CD51 and CD55 (particularly CD59 and CD55) that normally
D. CD19 and CD21 protect the red cells from activated C. HPIM 18th
E. All of the above pg 884

365 Most common complication of PUD? Perforation - 2nd most common; outlet MAIRRE JAMES BACK-UP
A. Gatric outlet obstruction obstruction - least common HPIM 18th pg 2445 GADDI, MD (TOP 4 MIDTERM
B. Perforation - AUG 2013 MED EXAM - FEB
C. Bleeding BOARDS; 2015
D. B and C TOPNOTCH MD)
E. All of the above

366 The following are used in the management of Treatment of infectious bronchiectasis is directed MAIRRE JAMES BACK-UP
bronchiectasis EXCEPT? at the control of active infection and GADDI, MD (TOP 4 MIDTERM
A. Antibiotics improvements in secretion clearance and - AUG 2013 MED EXAM - FEB
B. Mucolytic agents bronchial hygiene so as to decrease the microbial BOARDS; 2015
C. Bronchodilators load within the airways and minimize the risk of TOPNOTCH MD)
D. Surgery repeated infection. For refractory cases surgery
E. No exception can be considered, with resection of a focal area of
suppuration. In advanced cases, lung
transplantation can be considered. HPIM 18th pg

TOPNOTCH MEDICAL BOARD PREP INTERNAL MEDICINE SUPEREXAM Page 49 of 95


For inquiries visit www.topnotchboardprep.com.ph or email us at topnotchmedicalboardprep@gmail.com
TOPNOTCH MEDICAL BOARD PREP INTERNAL MEDICINE SUPEREXAM
For inquiries visit www.topnotchboardprep.com.ph or email us at topnotchmedicalboardprep@gmail.com
Item QUESTION EXPLANATION AUTHOR TOPNOTCH
# EXAM
2143-2144

367 A 55-year old male smoker came to you for chronic Advanced disease may be accompanied by MAIRRE JAMES BACK-UP
cough with associated sputum production and systemic wasting, with significant weight loss, GADDI, MD (TOP 4 MIDTERM
exertional dyspnea. He experienced these symptoms bitemporal wasting, and diffuse loss of - AUG 2013 MED EXAM - FEB
for years but appeared to be worsening. On PE, you subcutaneous adipose tissue. Some patients with BOARDS; 2015
noted a prolonged expiratory phase and wheezing. advanced disease have paradoxical inward TOPNOTCH MD)
Physical findings of advanced disease include the movement of the rib cage with inspiration
following EXCEPT? (Hoover's sign), the result of alteration of the
A. Systemic wasting vector of diaphragmatic contraction on the rib
B. Bitemporal wasting cage as a result of chronic hyperinflation. HPIM
C. Hoover’s sign 18th pg 2156
D. Clubbing of digits
E. No exception

368 The most discriminating symptom of duodenal Pain that awakes the patient from sleep (between MAIRRE JAMES BACK-UP
ulcers is? midnight and 3 a.m.) is the most discriminating GADDI, MD (TOP 4 MIDTERM
A. Epigastric pain awakening patient from sleep symptom, with two-thirds of DU patients - AUG 2013 MED EXAM - FEB
B. Burning gnawing epigastric pain describing this complaint HPIM 18th pg 2445 BOARDS; 2015
C. Epigastric pain 90 minutes to 3 hours after a meal TOPNOTCH MD)
D. Epigastric pain relieved by antacids or food
E. Nausea and vomiting

369 What is the most common form of diabetic The most common form of diabetic neuropathy is MAIRRE JAMES BACK-UP
neuropathy? distal symmetric polyneuropathy. It most GADDI, MD (TOP 4 MIDTERM
A. Proximal symmetric polyneuropathy frequently presents with distal sensory loss, but - AUG 2013 MED EXAM - FEB
B. Proximal asymmetric polyneuropathy up to 50% of patients do not have symptoms of BOARDS; 2015
C. Distal symmetric polyneuropathy neuropathy. HPIM 18th pg 2984 TOPNOTCH MD)
D. Distal asymmetric polyneuropathy
E. None of the above

370 Amiodarone can have which of the following effect/s Amiodarone has the following effects on thyroid MAIRRE JAMES BACK-UP
on thyroid function? function: (1) acute, transient suppression of GADDI, MD (TOP 4 MIDTERM
A. Chronic suppression of thyroid function thyroid function; (2) hypothyroidism in patients - AUG 2013 MED EXAM - FEB
B. Suppression of autoimmune Grave’s disease susceptible to the inhibitory effects of a high BOARDS; 2015
C. Thyrotoxicosis caused by a Jod-Basedow effect iodine load; and (3) thyrotoxicosis that may be TOPNOTCH MD)
D. Hypothyroidism in patients susceptible to the caused by either a Jod-Basedow effect from the
stimulatory effects of a high iodine load iodine load, in the setting of MNG or incipient
E. None of the above Graves’ disease, or a thyroiditis-like condition.
HPIM 18th pg 2930
371 A 45/F presented to you with a two week history of Acute thyroiditis is rare and due to suppurative MAIRRE JAMES BACK-UP
fever, jaw pain and sore throat. Further history infection of the thyroid. In children and young GADDI, MD (TOP 4 MIDTERM
revealed that, she had a flu-like illness a few weeks adults, the most common cause is the presence of - AUG 2013 MED EXAM - FEB
prior to the onset of sore throat. On PE, patient is a piriform sinus, a remnant of the fourth branchial BOARDS; 2015
febrile, with an apparent neck mass. Upon palpation, pouch. Subacute thyroiditis is also termed de TOPNOTCH MD)
there was tenderness on the anterior neck area. Quervain’s thyroiditis, granulomatous thyroiditis,
Which of the following is/are TRUE? or viral thyroiditis. Many viruses have been
A. The most common cause of acute thyroiditis is implicated, including mumps, coxsackie, influenza,
due to the presence of a piriform sinus, a remnant of adenoviruses, and echoviruses, but attempts to
the third branchial pouch. identify the virus in an individual patient are often
B. A long standing goiter and degeneration in a unsuccessful and do not influence management.
thyroid malignancy are risk factors in the young HPIM 18th ed pg 2927-2928
C. Patient’s condition is also termed as de Quervain’s
thyroiditis, granulomatous thyroiditis or viral
thyroiditis
D. A specific viral etiology is usualy identifiable
E. None of the above
372 Hyperuricemia is a common adverse effect of which Hyperuricemia is a common adverse effect of MAIRRE JAMES BACK-UP
anti-TB medication? pyrazinamide therapy that usually can be GADDI, MD (TOP 4 MIDTERM
A. Isoniazid managed conservatively. Clinical gout is rare. - AUG 2013 MED EXAM - FEB
B. Rifampicin HPIM 18th ed pg 1376 BOARDS; 2015
C. Pyrazinamide TOPNOTCH MD)
D. Ethambutol
E. Streptomycin

373 The pathogenesis of idiopathic TTP is related to a Idiopathic TTP is related to a deficiency of, or MAIRRE JAMES BACK-UP
deficiency of, or antibodies to: antibodies to the metalloprotease ADAMTS13, that GADDI, MD (TOP 4 MIDTERM
A. ADAMTS13 cleaves VWF. VWF is normally secreted as ultra- - AUG 2013 MED EXAM - FEB
B. von Willebrand factor large multimers, which are then cleaved by BOARDS; 2015
C. GpIb-IX-V ADAMTS13. The persistence of ultra-large VWF TOPNOTCH MD)
D. Gp IIb-IIIa molecules is thought to contribute to pathogenic
E. Platelets platelet adhesion and aggregation HPIM 18th pg
969

TOPNOTCH MEDICAL BOARD PREP INTERNAL MEDICINE SUPEREXAM Page 50 of 95


For inquiries visit www.topnotchboardprep.com.ph or email us at topnotchmedicalboardprep@gmail.com
TOPNOTCH MEDICAL BOARD PREP INTERNAL MEDICINE SUPEREXAM
For inquiries visit www.topnotchboardprep.com.ph or email us at topnotchmedicalboardprep@gmail.com
Item QUESTION EXPLANATION AUTHOR TOPNOTCH
# EXAM
374 What is the signature tachycardia of patients with Multifocal AT (MAT) is the signature tachycardia MAIRRE JAMES BACK-UP
significant pulmonary disease? of patients with significant pulmonary disease. GADDI, MD (TOP 4 MIDTERM
A. Atrial flutter The atrial rhythm is characterized by at least three - AUG 2013 MED EXAM - FEB
B. Multifocal atrial tachycardia distinct P-wave morphologies and often at least BOARDS; 2015
C. Ventricular premature complexes three different PR intervals, and the associated TOPNOTCH MD)
D. Atrial premature complexes atrial and ventricular rates are typically between
E. Atrioventricular Junctional Tachycardias 100 and 150 beats per minute. HPIM 18th pg 1886

375 Metabolic syndrome can be diagnosed by fulfilling 3 Central obesity: Waist circumference >102 cm MAIRRE JAMES BACK-UP
or more of the criteria defined by the NCEP:ATPIII in (M), >88 cm (F) GADDI, MD (TOP 4 MIDTERM
2001. Which of the following parameters can Hypertriglyceridemia: Triglycerides ≥150 mg/dL - AUG 2013 MED EXAM - FEB
following can satisfy the criteria? OR specific medication BOARDS; 2015
A. TG level of 120 on fenofibrate Low HDL cholesterol: <40 mg/dL and <50 mg/dL, TOPNOTCH MD)
B. HDL < 40 mg/dL for males respectively, OR specific medication
C. BP reading of 120/90 Hypertension: Blood pressure ≥130 mm systolic
D. B and C OR ≥85 mm diastolic OR specific medication
E. All of the above Fasting plasma glucose ≥100 mg/dL or specific
medication OR previously diagnosed Type 2
diabetes
376 An obese 34/M presents with crushing chest pain Cardiac syndrome X is used to describe patients MAIRRE JAMES BACK-UP
after having a fight with his partner. He was with angina-like chest pain and ischemic- GADDI, MD (TOP 4 MIDTERM
immediately brought to the ER and an ECG was done appearing ST-segment depression during stress - AUG 2013 MED EXAM - FEB
which showed ST-segment depression on V1-V6. despite normal coronary arteriograms. Some data BOARDS; 2015
After 30 minutes, the pain eventually subsided. A indicate that many such patients have limited TOPNOTCH MD)
coronary angiogram was subsequently done which changes in coronary flow in response to pacing
showed no abnormalities. What is your diagnosis? stress or coronary vasodilators. Prinzmetal angina
A. Prinzmetal variant angina would show ST-segment elevation during an
B. Cardiac syndrome X episode HPIM 18th pg 102 & 2020
C. Unstable angina
D. NSTEMI
E. None of the above

377 Absolute contraindications to fibrinolytic therapy in BP >180mmHg/ > 110mmHg at any time during MAIRRE JAMES BACK-UP
STEMI include which of the following? acute presentation; history of hemorrhagic CVD at GADDI, MD (TOP 4 MIDTERM
A. Elevated BP of >220 mmHg systolic and/or >110 any time of nonhemorrhagic CVD within the past - AUG 2013 MED EXAM - FEB
mmHg diastolic at any time during the acute year; active internal bleeding excluding menses; BOARDS; 2015
presentation suspicion of aortic dissection HPIM 18th 2028 TOPNOTCH MD)
B. History of a nonhemorrhagic stroke within the
last two years
C. Active internal bleeding including menses
D. A and B
E. None of the above
378 Hemoptysis from tuberculosis results from: Hemoptysis develops in 20–30% of cases, and MAIRRE JAMES BACK-UP
A. erosion of a blood vessel in the wall of a cavity massive hemoptysis may ensue as a consequence GADDI, MD (TOP 4 MIDTERM
B. rupture of a dilated vessel in a cavity of the erosion of a blood vessel in the wall of a - AUG 2013 MED EXAM - FEB
C. aspergilloma formation cavity. Hemoptysis, however, may also result from BOARDS; 2015
D. A and B rupture of a dilated vessel in a cavity TOPNOTCH MD)
E. All of the above (Rasmussen’s aneurysm) or from aspergilloma
formation in an old cavity. HPIM 18th 1345

379 Tthe following can precipitate acute Factors That May Precipitate Acute MAIRRE JAMES BACK-UP
decompensation in patients with CHF EXCEPT? Decompensation in Patients With GADDI, MD (TOP 4 MIDTERM
A. Liver failure Chronic Heart Failure include dietary indiscretion, - AUG 2013 MED EXAM - FEB
B. NSAID use myocardial ischemia/infarction, arrhythmias BOARDS; 2015
C. Infection (tachycardia or bradycardia), discontinuation of TOPNOTCH MD)
D. Pregnancy HF therapy, infection, anemia, initiation of
E. No exception medications that worsen HF: {Calcium antagonists
(verapamil, diltiazem), Beta blockers,
Nonsteroidal anti-inflammatory drugs,
antiarrhythmic agents [all class I agents, sotalol
(class III)] Anti-TNF antibodies}, alcohol
consumption, pregnancy, worsening hypertension
and acute valvular insufficiency HPIM 18th pg
1907
380 Which of the following is/are TRUE regarding Associated with HLA B27; B - The outer annular MAIRRE JAMES BACK-UP
ankylosing spondylitis (AS)? fibers are eroded and eventually replaced by bone, GADDI, MD (TOP 4 MIDTERM
A. Associated with HLA-B25 forming the beginning of a syndesmophyte, which - AUG 2013 MED EXAM - FEB
B. Outer annular fibers are eroded and eventually then grows by continued endochon- dral BOARDS; 2015
replaced by bone ultimately bridging the adjacent ossification, ultimately bridging the adjacent TOPNOTCH MD)
vertebral bodies vertebral bodies. Ascending progression of this
C. NSAIDs are the first line of pharmacologic therapy process leads to the “bamboo spine.” HPIM 18th
for AS. pg 2774 and 2777
D. B and C
E. All of the above
381 What is the recommended duration of treatment for SIMILAR TO PREVIOUS BOARD EXAM SCOTT RILEY ONG, BACK-UP
extrapulmonary tuberculosis? CONCEPT/PRINCIPLE. Extrapulmonary MD (TOP 5 - AUG MIDTERM
A. 6-9 months tuberculosis is generally treated with anti-Koch's 2014 MED EXAM - FEB
B. 9-12 months regimen for 6-9 months, except for CNS BOARDS; 2015
C. 12-15 months tuberculosis, which is treated for 9-12 months. TOPNOTCH MD)
D. 15-18 months
E. 2 years

TOPNOTCH MEDICAL BOARD PREP INTERNAL MEDICINE SUPEREXAM Page 51 of 95


For inquiries visit www.topnotchboardprep.com.ph or email us at topnotchmedicalboardprep@gmail.com
TOPNOTCH MEDICAL BOARD PREP INTERNAL MEDICINE SUPEREXAM
For inquiries visit www.topnotchboardprep.com.ph or email us at topnotchmedicalboardprep@gmail.com
Item QUESTION EXPLANATION AUTHOR TOPNOTCH
# EXAM
382 Which of the following ECG findings is consistent SCOTT RILEY ONG, BACK-UP
with hyperkalemia? MD (TOP 5 - AUG MIDTERM
A. Tall, peaked T wave 2014 MED EXAM - FEB
B. Widened QRS complex BOARDS; 2015
C. Prolonged PR interval TOPNOTCH MD)
D. Appearance of sine waves
E. All of the above

383 Which of the following acid-base abnormalities SCOTT RILEY ONG, BACK-UP
would you expect to see in a patient sufferring from MD (TOP 5 - AUG MIDTERM
aspirin toxicity? 2014 MED EXAM - FEB
A. High anion gap acidosis BOARDS; 2015
B. Metabolic alkalosis TOPNOTCH MD)
C. Respiratory acidosis
D. Respiratory alkalosis
E. Both A and D

384 Which of the following drugs constitute the SCOTT RILEY ONG, BACK-UP
cornerstone of treatment for congestive heart MD (TOP 5 - AUG MIDTERM
failure? 2014 MED EXAM - FEB
A. ACEI and digoxin BOARDS; 2015
B. Furosemide and ACEI TOPNOTCH MD)
C. ACEI and beta blocker
D. aspirin and digoxin
E. beta blocker and thiazide

385 Which of the following CNS diseases are SIMILAR TO PREVIOUS BOARD EXAM SCOTT RILEY ONG, BACK-UP
characterized by a pathologic triad of demyelination, CONCEPT/PRINCIPLE. MD (TOP 5 - AUG MIDTERM
inflammation and gliosis? 2014 MED EXAM - FEB
A. Ischemic stroke BOARDS; 2015
B. Parkinson's disease TOPNOTCH MD)
C. Multiple sclerosis
D. Guillain-Barre syndrome
E. Progressive multifocal leukoencephalopathy

386 A 56-year old obese male was admitted for ST- CK-MB returns to normal in 2-3 days following a SCOTT RILEY ONG, BACK-UP
elevation myocardial infarction. On his 7th hospital myocardial infarction. New-onset elevation may MD (TOP 5 - AUG MIDTERM
day, he was referred for sudden decrease in indicate reinfarction. 2014 MED EXAM - FEB
sensorium, hypotension and irregular pulses. You BOARDS; 2015
suspected a reinfarction. Which of the following TOPNOTCH MD)
cardiac enzyme would confirm your diagnosis?
A. CK-MB
B. Troponin I
C. Lactate dehydrogenase
D. AST
E. Myoglobin
387 Among patients with prosthetic valves, which of the IV drug user and right-sided acute IE: S. aureus. SCOTT RILEY ONG, BACK-UP
following organisms is the most common cause of Native valves and left-sided subacute IE: viridans MD (TOP 5 - AUG MIDTERM
infective endocarditis streptococci. Prosthetic valves: S. epidermidis. GI 2014 MED EXAM - FEB
A. Staphylococcus aureus malignancy: enterococci. BOARDS; 2015
B. viridans streptococci TOPNOTCH MD)
C. enterococci
D. Staphylococcus epidermidis
E. beta-hemolytic streptococci

388 Which of the following is the most commonly SIMILAR TO PREVIOUS BOARD EXAM SCOTT RILEY ONG, BACK-UP
involved organ in sarcoidosis? CONCEPT/PRINCIPLE. MD (TOP 5 - AUG MIDTERM
A. Skin 2014 MED EXAM - FEB
B. Heart BOARDS; 2015
C. Lung TOPNOTCH MD)
D. Kidney
E. Liver

389 Which of the following imaging findings would be SIMILAR TO PREVIOUS BOARD EXAM SCOTT RILEY ONG, BACK-UP
most consistent with sarcoidosis? CONCEPT/PRINCIPLE. MD (TOP 5 - AUG MIDTERM
A. Enlarged mass of confluent lymph nodes in the 2014 MED EXAM - FEB
anterior mediastinum BOARDS; 2015
B. Bilateral symmetric hilar and mediastinal TOPNOTCH MD)
lymphadenopathy with discrete node enlargement
C. Bilateral asymmetric hilar lymph node
enlargement, usually more pronounced on the right
D. A conglomerate mass of lymph nodes in the
bilateral hila
E. Any of the above can suggest sarcoidosis
390 Graham-Steel murmur is characteristically heard in SCOTT RILEY ONG, BACK-UP
which of the following conditions? MD (TOP 5 - AUG MIDTERM
A. Pulmonic regurgitation 2014 MED EXAM - FEB
B. Tricuspid regurgitation BOARDS; 2015
C. Aortic regurgitation TOPNOTCH MD)
D. Mitral stenosis
E. Ventricular septal defect

TOPNOTCH MEDICAL BOARD PREP INTERNAL MEDICINE SUPEREXAM Page 52 of 95


For inquiries visit www.topnotchboardprep.com.ph or email us at topnotchmedicalboardprep@gmail.com
TOPNOTCH MEDICAL BOARD PREP INTERNAL MEDICINE SUPEREXAM
For inquiries visit www.topnotchboardprep.com.ph or email us at topnotchmedicalboardprep@gmail.com
Item QUESTION EXPLANATION AUTHOR TOPNOTCH
# EXAM
391 Among elderly patients, which of the following is the However, colorectal cancer is the most worrisome SCOTT RILEY ONG, BACK-UP
most common cause of lower GI bleeding? cause of lower GI bleeding and must first be ruled MD (TOP 5 - AUG MIDTERM
A. Diverticulosis out in any elderly patient with such complaint. 2014 MED EXAM - FEB
B. Colorectal cancer BOARDS; 2015
C. Anal fissure TOPNOTCH MD)
D. Ulcerative colitis
E. Amoebic colitis

392 A 26-year old patient presented with 1-month Category III: new smear (-) without extensive SCOTT RILEY ONG, BACK-UP
history of productive cough with associated weight parenchymal disease MD (TOP 5 - AUG MIDTERM
loss and night sweats. Chest radiograph showed 2014 MED EXAM - FEB
minimal reticulonodular infiltrates in the right lung BOARDS; 2015
apex. Sputum examinations were all negative TOPNOTCH MD)
despite adequate sputum collection. You began the
patient on a 7-day regimen of amoxicillin therapy
However, symptoms persisted despite good
compliance. You then decided to start him on anti-
Koch's treatment. Under WHO guidelines, the patient
is classified as:
A. Category I
B. Category II
C. Category III
D. You should have ordered TB culture first.
E. You should have started the patient on another 7-
day trial with azithromycin.

393 Which of the following clinical features would Criteria for moderate-risk CAP: RR > 30, HR >125, SCOTT RILEY ONG, BACK-UP
warrant a classifcation of moderate-risk community- BP < 90/60, T <36 or >40, altered mental state, MD (TOP 5 - AUG MIDTERM
acquired pneumonia? decompensated comorbidity, suspected 2014 MED EXAM - FEB
A. RR 24 breaths/min aspiration, multilobar infiltrate, pleurall effusion, BOARDS; 2015
B. HR 100 beats/min abscess formation TOPNOTCH MD)
C. Temperature 38.0oC
D. Bilateral lung infiltrates on chest radiograph
E. All of the above

394 Which of the following laboratory parameters would The other parameters suggest an intrinsic renal SCOTT RILEY ONG, BACK-UP
suggest pre-renal etiology for azotemia? etiology. MD (TOP 5 - AUG MIDTERM
A. BUN:creatinine ratio > 20 2014 MED EXAM - FEB
B. FeNa > 2% BOARDS; 2015
C. Urine osmolarity < 350 mOsm/kg TOPNOTCH MD)
D. Urine:plasma creatinine ratio < 20
E. All of the above

395 Which of the following laboratory findings are SCOTT RILEY ONG, BACK-UP
consistent with tumor lysis syndrome? MD (TOP 5 - AUG MIDTERM
A. Hyperkalemia 2014 MED EXAM - FEB
B. Hyperphosphatemia BOARDS; 2015
C. Hyperuricemia TOPNOTCH MD)
D. Hypocalcemia
E. All of the above

396 At what stage of chronic kidney disease will you Normocytic, normochomic anemia can be seen as SCOTT RILEY ONG, BACK-UP
expect to fiirst see normocytic, normochromic early as stage III CKD and is universal by stage IV. MD (TOP 5 - AUG MIDTERM
anemia? 2014 MED EXAM - FEB
A. Stage I BOARDS; 2015
B. Stage II TOPNOTCH MD)
C. Stage III
D. Stage IV
E. Stage V

397 Which of the following bacteria is known as the most Empiric drug of choice for SBP: cefotaxime SCOTT RILEY ONG, BACK-UP
common etiologic agent in spontaneous bacterial MD (TOP 5 - AUG MIDTERM
peritonitis? 2014 MED EXAM - FEB
A. Bacteroides fragilis BOARDS; 2015
B. Enterococcus faecalis TOPNOTCH MD)
C. Escherichia coli
D. viridans streptococci
E. Enterobacter spp.

398 The presence of bifid pulse is associated with which Cardiac tamponade: pulsus paradoxus, pulsus SCOTT RILEY ONG, BACK-UP
of the following conditions? alternans. Aortic stenosis: pulsus parvus et tardus. MD (TOP 5 - AUG MIDTERM
A. Cardiac tamponade Aortic regurgitation: wide pulse pressure. 2014 MED EXAM - FEB
B. Aortic regurgitation BOARDS; 2015
C. Aortic stenosis TOPNOTCH MD)
D. Pericarditis
E. Hypertrophic obstructive cardiomyopathy

TOPNOTCH MEDICAL BOARD PREP INTERNAL MEDICINE SUPEREXAM Page 53 of 95


For inquiries visit www.topnotchboardprep.com.ph or email us at topnotchmedicalboardprep@gmail.com
TOPNOTCH MEDICAL BOARD PREP INTERNAL MEDICINE SUPEREXAM
For inquiries visit www.topnotchboardprep.com.ph or email us at topnotchmedicalboardprep@gmail.com
Item QUESTION EXPLANATION AUTHOR TOPNOTCH
# EXAM
399 In a patient who has recently recovered from an RF without carditis: 5 years after attack or until 21 SCOTT RILEY ONG, BACK-UP
episode of rheumatic fever without any sequelae, years old. RF with mild carditis: 10 years after MD (TOP 5 - AUG MIDTERM
prophylactic treatment with benzathin penicillin 1.2 attack or until 21 years old. RF with moderate to 2014 MED EXAM - FEB
units IM every 2-4 weeks is recommended until severe carditis: 10 years after attack or until 40 BOARDS; 2015
when? years old, or even lifetime TOPNOTCH MD)
A. For 3 years after attack or until 21 years old
B. For 5 years after attack or until 21 years old
C. For 10 years after attack or until 21 years old
D. For 10 years after attack or until 40 years old
E. For 15 years after attack or until 40 years old
400 Which of the following is a posthepatic etiology for Options A-C are intrahepatic causes. Option E is a SCOTT RILEY ONG, BACK-UP
portal hypertension? prehepatic cause. MD (TOP 5 - AUG MIDTERM
A. Congenital hepatic fibrosis 2014 MED EXAM - FEB
B. Alcoholic liver cirrhosis BOARDS; 2015
C. Schistosomiasis TOPNOTCH MD)
D. Budd-Chiari syndrome
E. Portal vein thrombosis

401 A patient came to you with recurrent anal fistula, SIMILAR TO PREVIOUS BOARD EXAM JOSE CARLO DIAGNOSTIC
you noted on her radiologic studies that there are CONCEPT/PRINCIPLE MASANGKAY III, EXAM - AUG
noticeable "skip lesions". Which of the following MD (TOP 8 - FEB 2014
medications may be given to the patient? 2014 MED
A. Sulfadiazine BOARDS;
B. Sulfasalazine TOPNOTCH MD)
C. Sulfamethoxazole
D. Salicylic Acid
E. Mycophenolate mofetil

402 A 40 year old asymptomatic patient has just Patient is already asymptomatic, CXR findings in a JOSE CARLO DIAGNOSTIC
completed 1 week treatment for CAP-moderate risk, CAP treated patient takes 4-12 weeks to clear. MASANGKAY III, EXAM - AUG
Upon follow-up you requested a Chest Xray still with MD (TOP 8 - FEB 2014
noted lobar infiltrates. What would be your next step 2014 MED
of management? BOARDS;
A. Extend antimicrobial therapy TOPNOTCH MD)
B. Change antimicrobial therapy to a broader
spectrum
C. Consider PTB
D. Re-admit, do sputum culture and sensitivity and
do re-treatment
E. None of the above
403 You encountered an ECG tracing with ST elevations Diffuse ST elevation is seen in Pericarditis. JOSE CARLO DIAGNOSTIC
in all leads. What will be your primary MASANGKAY III, EXAM - AUG
consideration? MD (TOP 8 - FEB 2014
A. Inferior wall MI 2014 MED
B. High lateral wall MI BOARDS;
C. Anterior Wall MI TOPNOTCH MD)
D. Pancardiac MI
E. None of the above

404 A chest Xray was done in a patient with severe Hampton's Hump is a peripheral wedge shaped JOSE CARLO DIAGNOSTIC
dyspnea, the radiologic report stated a noticed density on CXR, usually above the diaphragm seen MASANGKAY III, EXAM - AUG
Hampton's Hump above the diaphragm. What would in Pulmonary embolism MD (TOP 8 - FEB 2014
be your next step of management? 2014 MED
A. Antibiotic therapy BOARDS;
B. Further investigation for a primary malignancy TOPNOTCH MD)
C. Refer to surgery for immediate removal
D. Fibrinolytic therapy
E. Give a bronchodilator with Corticosteroids

405 Metabolic Syndrome is defined using the following TG >150 mg/dL is more proper. JOSE CARLO DIAGNOSTIC
criteria, EXCEPT: MASANGKAY III, EXAM - AUG
A. Waist line more than 40 in in Males MD (TOP 8 - FEB 2014
B. TG> 200 mg/dL 2014 MED
C. HDL less than 40mg/dl in Males BOARDS;
D. BP >/= 130/ >/= 85 TOPNOTCH MD)
E. FBS >100 mg/dL

406 A patient with a history of RHD came in with a 2d JOSE CARLO DIAGNOSTIC
Echocardiography result revealing an Aortic MASANGKAY III, EXAM - AUG
Regurgitation, upon your physical examination you MD (TOP 8 - FEB 2014
noticed that the patient's Upper extremity BP was 2014 MED
140/100 while his Lower extremity BP was 100/70. BOARDS;
This sign is termed as a: TOPNOTCH MD)
A. Hill's Sign
B. Demusset's Sign
C. Quincke's Sign
D. Corrigan's Sign
E. Duroziez's Sign
407 Transfusion-associated Viral Hepatitis is due to: Although both Hepatitis B and C can be JOSE CARLO DIAGNOSTIC
A. Hepatitis A transmitted thru blood transfusion, Hepatitis C is MASANGKAY III, EXAM - AUG
B. Hepatitis B more associated with blood transfusions. MD (TOP 8 - FEB 2014
C. Hepatitis C (SIMILAR TO PREVIOUS BOARD EXAM 2014 MED
D. Hepatitis D CONCEPT/PRINCIPLE) BOARDS;
E. Hepatitis E TOPNOTCH MD)

TOPNOTCH MEDICAL BOARD PREP INTERNAL MEDICINE SUPEREXAM Page 54 of 95


For inquiries visit www.topnotchboardprep.com.ph or email us at topnotchmedicalboardprep@gmail.com
TOPNOTCH MEDICAL BOARD PREP INTERNAL MEDICINE SUPEREXAM
For inquiries visit www.topnotchboardprep.com.ph or email us at topnotchmedicalboardprep@gmail.com
Item QUESTION EXPLANATION AUTHOR TOPNOTCH
# EXAM
408 An elderly female patient is suffering from The description is compatible with CPPD crystals JOSE CARLO DIAGNOSTIC
occasional arthralgias on her right knee, the of Pseudogout. MASANGKAY III, EXAM - AUG
Rheumatologist performed arthrocentesis with MD (TOP 8 - FEB 2014
microscopic findings of rhomboid and rod-like 2014 MED
crystals with weak positive birefringence, which of BOARDS;
the following would be the Rheumatologist's TOPNOTCH MD)
primary consideration?
A. Gout
B. Pseudogout
C. Rheumatoid Arthritis
D. Acute Bacterial Arthritis
E. Osteoarthritis

409 Which of the following is NOT a risk factor for JOSE CARLO DIAGNOSTIC
OSTEOPOROSIS? MASANGKAY III, EXAM - AUG
A. Family History MD (TOP 8 - FEB 2014
B. s/p TAHBSO 2014 MED
C. Injury BOARDS;
D. Obesity TOPNOTCH MD)
E. Age

410 A patient undergoing therapy for Crohn's Disease, a TNF-alpha inhibitor prevents granulation JOSE CARLO DIAGNOSTIC
had a reactivation of his Latent TB, what Drug may formation hence reactivation of TB occurs. MASANGKAY III, EXAM - AUG
have been responsible? MD (TOP 8 - FEB 2014
A. Rituximab 2014 MED
B. Sulfasalazine BOARDS;
C. 5-ASA TOPNOTCH MD)
D. Infliximab
E. Alosetron

411 A patient with manifestations of Cushing's JOSE CARLO DIAGNOSTIC


Syndrome, to determine the source of his disease a MASANGKAY III, EXAM - AUG
Dexamethasone Suppression Test was done: Cortisol MD (TOP 8 - FEB 2014
Levels were not suppressed by Low dose 2014 MED
Dexamethasone meanwhile High Dose BOARDS;
Dexamethasone suppressed Cortisol levels. What is TOPNOTCH MD)
the probable primary disease?
A. Pituitary CS
B. Adrenal CS
C. Ectopic CS
D. Exogenous Steroid Use
E. Addison's Disease

412 This heart sound, coincides with early diastole or JOSE CARLO DIAGNOSTIC
rapid ventricular filling, commonly seen in a dilated MASANGKAY III, EXAM - AUG
cardiomyopathy: MD (TOP 8 - FEB 2014
A. S1 2014 MED
B. S2 BOARDS;
C. S3 TOPNOTCH MD)
D. S4
E. C and D

413 Among which of the following thyroid function tests SIMILAR TO PREVIOUS BOARD EXAM JOSE CARLO DIAGNOSTIC
is most accurate in diagnosing Hyperthyroidism? CONCEPT/PRINCIPLE. The free fraction of MASANGKAY III, EXAM - AUG
A. TSH Thyroid hormines is the "true" Thyroid level. MD (TOP 8 - FEB 2014
B. TRH 2014 MED
C. Total T3 BOARDS;
D. Free T4 TOPNOTCH MD)
E. Calcitonin

414 To prevent Nitroglycerin tolerance which of the a 6-8-hour NTG free period every 24 hours will JOSE CARLO DIAGNOSTIC
following should be done? prevent NTG tolerance. MASANGKAY III, EXAM - AUG
A. Increase dose of NTG MD (TOP 8 - FEB 2014
B. Stop NTG for 6-8 hours a day 2014 MED
C. Decrease Dose of NTG BOARDS;
D. Replace NTG with Sodium Nitroprusside TOPNOTCH MD)
E. Do Nitroglycerin desensitization

415 Which of the following is not seen in Diabetes SIMILAR TO PREVIOUS BOARD EXAM JOSE CARLO DIAGNOSTIC
insipidus? CONCEPT/PRINCIPLE. A: urine osmolality should MASANGKAY III, EXAM - AUG
A. Increased Urine osmolality after water not increase after water deprivation test; B and C MD (TOP 8 - FEB 2014
deprivation test are both seen in DI; D and E are seen in a 2014 MED
B. Random Urine Specific gravity of 1.005 nephrogenic type of Diabetes insipidus. BOARDS;
C. Serum osmolality of 390 mOsm/L TOPNOTCH MD)
D. No response to desmopressin
E. Specific Gravity of 1.030 after HCTZ
administration
416 The following are characteristics of a Type I Diabetes Based on USMLE step 1. JOSE CARLO DIAGNOSTIC
mellitus, EXCEPT: MASANGKAY III, EXAM - AUG
A. Islet Amyloid Deposit on histology MD (TOP 8 - FEB 2014
B. Associated with HLA system 2014 MED
C. Severe Glucose intolerance BOARDS;
D. Decreased C-peptide levels TOPNOTCH MD)
E. Weak genetic predisposition

TOPNOTCH MEDICAL BOARD PREP INTERNAL MEDICINE SUPEREXAM Page 55 of 95


For inquiries visit www.topnotchboardprep.com.ph or email us at topnotchmedicalboardprep@gmail.com
TOPNOTCH MEDICAL BOARD PREP INTERNAL MEDICINE SUPEREXAM
For inquiries visit www.topnotchboardprep.com.ph or email us at topnotchmedicalboardprep@gmail.com
Item QUESTION EXPLANATION AUTHOR TOPNOTCH
# EXAM
417 SIADH presents with the following, EXCEPT: Serum osmolarity should be less than the urine. JOSE CARLO DIAGNOSTIC
A. Excessive water retention MASANGKAY III, EXAM - AUG
B. Maybe seen in Lung Cancer MD (TOP 8 - FEB 2014
C. Hyponatremia 2014 MED
D. Serum osmolarity more than the urine BOARDS;
osmolarity TOPNOTCH MD)
E. Treated with Demeclocycline

418 A patient presenting with a Parathyroid adenoma, PPP(parathyroid, Pituitary and Pancreatic tumors) JOSE CARLO DIAGNOSTIC
Prolactinoma, and Insulinoma, with several stomach are seen in MEN 1 or Wermer's Syndrome. MASANGKAY III, EXAM - AUG
ulcers and a history of Urolithiasis is presenting with MD (TOP 8 - FEB 2014
what syndrome? 2014 MED
A. MEN 1 BOARDS;
B. MEN 2A TOPNOTCH MD)
C. MEN 2B
D. Sipple's Syndrome
E. Simmond's Syndrome

419 A woman on maintenance for Hyperthyroidism JOSE CARLO DIAGNOSTIC


unknowingly became pregnant. After giving birth, MASANGKAY III, EXAM - AUG
the Pediatrician told her that her child has Aplasia MD (TOP 8 - FEB 2014
cutis congenita. What drug was responsible? 2014 MED
A. PTU BOARDS;
B. Methimazole TOPNOTCH MD)
C. Propranolol
D. I 131
E. Lugol's Iodine

420 What stage of Pneumonia corresponds with a the resolution phase is described with a JOSE CARLO DIAGNOSTIC
predominance of macrophages? predominance of macrophages and cessation of MASANGKAY III, EXAM - AUG
A. Edema inflammatory response. MD (TOP 8 - FEB 2014
B. Red hepatization 2014 MED
C. Gray hepatization BOARDS;
D. Resolution TOPNOTCH MD)
E. None of the above

421 A chest radiograph showing "water bottle" sign, *IM is much like an "anything under the sun" type WEBSTER MIDTERM 1
suggesting a multichambered heart enlargement is of exam. It will test your ability to integrate the ALINDOG, MD EXAM - AUG
most consistent with: basic subjects (biochem, physio, ana, patho, (TOP 3 - FEB 2014 2014
A. Cardiac tamponade pharma) into clinical perspective. But again, just MED BOARDS;
B. Myocardial infarction similar with the other subjects, it has many TOPNOTCH MD)
C. Endocarditis SIMILAR TO PREVIOUS BOARD EXAM
D. Congestive heart failure CONCEPT/PRINCIPLEs (not really of IM, but of
physio, pharma, etc) and very few case analysis.

422 The anti-TB drug which is found to be more active Must know some notable side effects of anti-TB WEBSTER MIDTERM 1
against slowly replicating bacilli and is known to drugs: Hepatotoxicity (H>R>Z); GI upset, orange ALINDOG, MD EXAM - AUG
cause hyperuricemia in some TB patients is: urine, flu-like, anemia, thrombocytopenia (TOP 3 - FEB 2014 2014
A. Isoniazid (Rifampicin); Ototoxicity, vestibular impairment, MED BOARDS;
B. Rifampicin local irritation due to injection (Streptomycin); TOPNOTCH MD)
C. Pyrazinamide Peripheral neuropathy, psychosis, convulsion
D. Ethambutol (Isoniazid); Optic neuritis (Ethambutol)

423 The recognized primary mediator of bronchial SRSA are leukotrienes C4, D4, E4. Histamine, WEBSTER MIDTERM 1
asthma: although associated with hypersensitivity states, is ALINDOG, MD EXAM - AUG
A. Histamine not a mediator of asthma, and hence anti- (TOP 3 - FEB 2014 2014
B. Acetylcholine histamines are not used as reliever or controller MED BOARDS;
C. Dust mites for asthma. Dust mites are not mediators, but TOPNOTCH MD)
D. Slow reacting substances of anaphylaxis triggers.

424 Of the following clinical findings, which will The CREST syndrome includes subcutaneous WEBSTER MIDTERM 1
differentiate diffuse systemic scleroderma from the calcinosis, Reynaud's phenomenon (choice A), ALINDOG, MD EXAM - AUG
CREST variant? esophageal dysmotility, sclerodactyly and (TOP 3 - FEB 2014 2014
A. Hands turning painful and white or blue in the telangiectasia (also present in the systemic type MED BOARDS;
cold but less common). Another defining finding in TOPNOTCH MD)
B. Distal skin thickening systemic sclerosis is that its pulmonary
C. Renal involvement involvement is most likely to be severe than in
D. Esophageal dysmotility CREST.
E. All of the above
425 This urinalysis finding is pathognomonic of an end- Broad waxy casts suggest presence of significant WEBSTER MIDTERM 1
stage renal disease; a very poor prognostic sign: tubular degeneration and implies a poor ALINDOG, MD EXAM - AUG
A. Hyaline casts prognosis. Hyaline casts are composed almost (TOP 3 - FEB 2014 2014
B. "Coffin lid" crystals entirely of Tamm-Horsfall protein and a 0-2/lpf MED BOARDS;
C. Broad waxy casts count is considered normal; increased amounts TOPNOTCH MD)
D. Oil droplets are found during exercise, fever, dehydration, and
heart failure. Coffin lid crystals are typical of triple
phosphate. Oil droplets on the other hand, may
only be artefactual during urinalysis.

TOPNOTCH MEDICAL BOARD PREP INTERNAL MEDICINE SUPEREXAM Page 56 of 95


For inquiries visit www.topnotchboardprep.com.ph or email us at topnotchmedicalboardprep@gmail.com
TOPNOTCH MEDICAL BOARD PREP INTERNAL MEDICINE SUPEREXAM
For inquiries visit www.topnotchboardprep.com.ph or email us at topnotchmedicalboardprep@gmail.com
Item QUESTION EXPLANATION AUTHOR TOPNOTCH
# EXAM
426 The Chinese restaurant syndrome, a condition WEBSTER MIDTERM 1
characterized by burning and tightness in the upper ALINDOG, MD EXAM - AUG
body accompanied by diaphoresis, nausea, flushing (TOP 3 - FEB 2014 2014
and cramps is attributed to: MED BOARDS;
A. Monosodium glutamate intoxication TOPNOTCH MD)
B. Puffer fish poisoning
C. Fried rice (Yangchow) contaminated with
enteric bacteria
D. Rancid cooking oil poisoning

427 A 26 year-old male came in for consult due to painful WEBSTER MIDTERM 1
urethral discharge. He admits sexual promiscuity ALINDOG, MD EXAM - AUG
and practice of unsafe sex. Which of the following (TOP 3 - FEB 2014 2014
infectious agents most likely accounts for his MED BOARDS;
condition? TOPNOTCH MD)
A. Treponema
B. Chlamydia
C. Trichomonas
D. Granuloma inguinale

428 Which of the following statements about insulin Glucose stimulation of insulin secretion starts with WEBSTER MIDTERM 1
synthesis and secretion is not true? entry of glucose molecules into pancreatic beta ALINDOG, MD EXAM - AUG
A. Blood glucose level of >70 mg/dl stimulates cells by means of a facilitative glucose transporter (TOP 3 - FEB 2014 2014
insulin synthesis. (GLUT 2). Options A and B are true. (HPIM, DM MED BOARDS;
B. Glucose phosphorylation by glucokinase is the Chapter 18th edition) TOPNOTCH MD)
rate-limiting step in glucose-regulated insulin
secretion.
C. Glucose is transported to the pancreatic beta
cells by an active glucose transport.
D. All of the above
E. None of the above
429 A 32 year-old man was found to have chronic fatigue Hepatitis B is associated with polyarteritis (PAN). WEBSTER MIDTERM 1
and AST elevations 4x the upper limit. He had a ALINDOG, MD EXAM - AUG
series of blood transfusions 4 years ago in a (TOP 3 - FEB 2014 2014
secondary hospital in a remote island due to MED BOARDS;
vehicular trauma. Vascular biopsy was done and TOPNOTCH MD)
showed polyarteritis. Which of the following is the
most likely cause of his condition?
A. Hepatitis A
B. Hepatitis B
C. Hepatitis C
D. Hepatitis D

430 A 63 year-old previous MI male patient was seen at This is a case of acute mesenteric ischemia which WEBSTER MIDTERM 1
the emergency room after experiencing severe is more common in patients with history or ALINDOG, MD EXAM - AUG
abdominal pain that is out of proportion from evidence of arterial vascular compromise (TOP 3 - FEB 2014 2014
physical findings. On exam he has soft abdomen with including MI and peripheral arterial diseases. The MED BOARDS;
hypoactive bowels. An abdominal radiograph superior mesenteric artery is usually affected. TOPNOTCH MD)
reveals 'thumbprinting" or bowel loops. Which of "Thumbrpinting" or bowel looping is a common
the following is consistent with the most likely finding in Xrays. Patients will often have
diagnosis? abdominal PE findings that are relatively benign
A. Metabolic alkalosis is also present. compared with the patient's symptoms. Metabolic
B. CBC will show leukopenia but with relative acidosis will usually be present as well.
increase in bands.
C. There is involvement of the superior mesenteric
artery.
D. Barium enema can be used to reverse the
condition.
E. None of the above.
431 Which of the following drugs can cause acute WEBSTER MIDTERM 1
pericarditis? ALINDOG, MD EXAM - AUG
A. Isoniazid (TOP 3 - FEB 2014 2014
B. Hydralazine MED BOARDS;
C. Procainamide TOPNOTCH MD)
D. All of the above

432 The best way to confirm the diagnosis of aortic Echo study with Doppler can detect the valvular WEBSTER MIDTERM 1
stenosis is by performing: problem, quantify its severity and assess left ALINDOG, MD EXAM - AUG
A. An electrocardiogram ventricular function. Although a ventriculogram (TOP 3 - FEB 2014 2014
B. An echocardiogram can tell about ventricular function, it cannot assess MED BOARDS;
C. An exercise stress test disease severity. ECG is non-specific whereas a TOPNOTCH MD)
D. A radionuclide ventriculogram stress test is contraindicated in symptomatic
patients.

TOPNOTCH MEDICAL BOARD PREP INTERNAL MEDICINE SUPEREXAM Page 57 of 95


For inquiries visit www.topnotchboardprep.com.ph or email us at topnotchmedicalboardprep@gmail.com
TOPNOTCH MEDICAL BOARD PREP INTERNAL MEDICINE SUPEREXAM
For inquiries visit www.topnotchboardprep.com.ph or email us at topnotchmedicalboardprep@gmail.com
Item QUESTION EXPLANATION AUTHOR TOPNOTCH
# EXAM
433 A 60 year-old hypertensive male patient came in for Any elderly male patient, who has anemia WEBSTER MIDTERM 1
regular follow-up. He has no new complaints except especially if its microcytic, and does not have a ALINDOG, MD EXAM - AUG
for easy fatigability and weakness. On examination, demonstrable source of anemia warrants (TOP 3 - FEB 2014 2014
there is no hepatomegaly or jaundice but there is investigation for a colonic mass or tumor (for MED BOARDS;
pallor. CBC was ordered showing a hemoglobin of 9 occult blood loss). And in this case, the best test TOPNOTCH MD)
g/dl with MCV less than 80 fl. Liver function tests will be a colonoscopy.
and iron studies were all normal. The next
procedure that should be performed is:
A. Colonoscopy
B. Bone marrow aspirate
C. Bone scan
D. Liver biopsy

434 A liver function panel shows: NORMAL AST, ALT, Cirrhosis is the most consistent. There is WEBSTER MIDTERM 1
and LDH; LOW total protein and albumin; HIGH considerable destruction of hepatocytes in ALINDOG, MD EXAM - AUG
bilirubin and ammonia. The most likely hepatic cirrhosis which can lead to "normal" hepatic (TOP 3 - FEB 2014 2014
condition based on these lab results pattern is: enzymes (AST, ALT) in the blood, in contrast to MED BOARDS;
A. Hepatitis elevated levels as seen when there is on going TOPNOTCH MD)
B. Biliary obstruction destruction of liver cells (hepatitis). The
C. Passive hepatic congestion destruction of hepatocytes also leads to
D. Cirrhosis diminished synthesizing functions of the liver (low
E. None of the above proteins/albumin, low coagulation factors);
similarly, its ability to detoxify and excrete wastes
is also compromised (high bilirubin and
ammonia).
435 Evidence of pulmonary embolism may also be seen WEBSTER MIDTERM 1
through a chest radiograph. A sign where there is a ALINDOG, MD EXAM - AUG
peripheral wedge-shaped density above the (TOP 3 - FEB 2014 2014
diaphragm is known as: MED BOARDS;
A. Westermark's TOPNOTCH MD)
B. Palla's sign
C. Humpton's
D. McConnell's
E.

436 Which of the following pleural fluid analysis findings WEBSTER MIDTERM 1
is consistent with a transudative effusion? ALINDOG, MD EXAM - AUG
A. PF pro-BNP of >1500 pg/ml (TOP 3 - FEB 2014 2014
B. Loculated PF MED BOARDS;
C. PF pH <7.2 TOPNOTCH MD)
D. Positive culture

437 Chvostek's sign is seen in: WEBSTER MIDTERM 1


A. Hypocalcemia ALINDOG, MD EXAM - AUG
B. Hypercalcemia (TOP 3 - FEB 2014 2014
C. Hypermagnesemia MED BOARDS;
D. Hyperkalemia TOPNOTCH MD)

438 In which conditions is a thyroid bruit may be Grave's disease renders the thyroid gland WEBSTER MIDTERM 1
present? hypervascular, which is responsible for the ALINDOG, MD EXAM - AUG
A. Ectopic thyroid gland turbulent blood flow producing the bruit. It is best (TOP 3 - FEB 2014 2014
B. Thyroglossal duct cyst heard on the lateral lobes of the thyroid. MED BOARDS;
C. Infectious thyroiditis TOPNOTCH MD)
D. Grave's disease

439 The stigmata of liver cirrhosis include all of the Choosing the best answer, presence of esophageal WEBSTER MIDTERM 1
following except: varices is a pathognomonic finding in portal ALINDOG, MD EXAM - AUG
A. Testicular atrophy hypertension rather than cirrhosis. The two are (TOP 3 - FEB 2014 2014
B. Spider angiomata different and separate entities. MED BOARDS;
C. Gynecomastia TOPNOTCH MD)
D. Esophageal varices
E. None of the above

440 Idiopathic pancreatic beta cell destruction which Type 1a = autoimmune beta cell destruction; 1b = WEBSTER MIDTERM 1
leads to absolute insulin deficiency is classified as: non-autoimmune/idiopathic destruction. MODY ALINDOG, MD EXAM - AUG
A. DM type 1a means maturity-onset diabetes of the young and is (TOP 3 - FEB 2014 2014
B. DM type 1b characterized by impaired insulin secretion MED BOARDS;
C. DM type 2 secondary to defective factors or enzymes TOPNOTCH MD)
D. MODY type 2 involved in normal beta cell functions. MODY 2, in
particular, is characterized by problem in
glucokinase.

441 Uremic fetor is caused by: Uremic fetor is a urine-like odor on the breath; JULIET KRISTINE MIDTERM 2
A. Breakdown of urea to ammonia in saliva derives from the breakdown of urea to ammonia EVANGELISTA, MD EXAM - AUG
B. Bacterial conversion of ammonia in the in saliva and is often associated with an (TOP 9 - FEB 2014 2014
intestines unpleasant metallic taste (dysgeusia). MED BOARDS;
C. Accumulation of ammonia in the blood due to TOPNOTCH MD)
decreased clearance by the kidneys
D. A and C
E. All of the above

TOPNOTCH MEDICAL BOARD PREP INTERNAL MEDICINE SUPEREXAM Page 58 of 95


For inquiries visit www.topnotchboardprep.com.ph or email us at topnotchmedicalboardprep@gmail.com
TOPNOTCH MEDICAL BOARD PREP INTERNAL MEDICINE SUPEREXAM
For inquiries visit www.topnotchboardprep.com.ph or email us at topnotchmedicalboardprep@gmail.com
Item QUESTION EXPLANATION AUTHOR TOPNOTCH
# EXAM
442 A 42 year-old male patient, nonhypertensive, Mitral stenosis findings are loud S1, accentuated JULIET KRISTINE MIDTERM 2
nondiabetic visits in the out-patient department. On P2, apical diastolic rumble and murmur and EVANGELISTA, MD EXAM - AUG
auscultation, an accentuation of first heart sound, opening snap. These are secondary to Rheumatic (TOP 9 - FEB 2014 2014
opening snap and mid-diastolic rumbling murmur at Heart Disease. MED BOARDS;
the apex were heard. The above findings pertain that TOPNOTCH MD)
the patient is suffering from:
A. Mitral regurgitation
B. Mitral stenosis
C. Pulmonary stenosis
D. Aortic regurgitation
E. Aortic stenosis
443 The main substance/s causing vasodilation in Histamine is the main substance causing JULIET KRISTINE MIDTERM 2
urticaria is/are: vasodilation in urticaria also in Type I EVANGELISTA, MD EXAM - AUG
A. Serotonin hypersensitivity. Wheal is a classic demonstration (TOP 9 - FEB 2014 2014
B. Bradykinin of histamine effect. MED BOARDS;
C. Histamine TOPNOTCH MD)
D. Prostaglandins
E. All of the above

444 A 35 year-old female, hypertensive and diabetic Hydralazine causes drug-induced lupus thus JULIET KRISTINE MIDTERM 2
came into your clinis because of erythematous rash should be withdrawn. EVANGELISTA, MD EXAM - AUG
across the bridge of her nose. Her BP was 180/100. (TOP 9 - FEB 2014 2014
Among her maintenance medications, this medicine MED BOARDS;
should be withdrawn: TOPNOTCH MD)
A. Propranolol
B. Hydralazine
C. Nifedipine
D. Losartan
E. Captopril

445 A 55 year-old male, nonhypertensive, nodiabetic was Blood alcohol level: 50-100=sedation, high, slower JULIET KRISTINE MIDTERM 2
brought to the Emergency Room confused and reaction times, 100-200=impaired motor function, EVANGELISTA, MD EXAM - AUG
disoriented with fluctuating consciousness and slurred speech, ataxia, 200-300=emesis, stupor, (TOP 9 - FEB 2014 2014
perception. If the diagnosis is alcohol delirium, what 300-400=coma, >500=respiratory depression, MED BOARDS;
is the blood alcohol level of this patient? death TOPNOTCH MD)
A. 50-100
B. 100-200
C. 200-300
D. 300-400
E. >500

446 Bone marrow biopsy was done to a 20 year-old Pancytopenia is a syndrome of chronic primary JULIET KRISTINE MIDTERM 2
female with recurrent infections. Pancytopenia with hematopoietic failure. Bone marrow biopsy EVANGELISTA, MD EXAM - AUG
an empty marrow is seen. Diagnosis for the above reveals an empty marrow. (TOP 9 - FEB 2014 2014
findings: MED BOARDS;
A. Acute leukemia TOPNOTCH MD)
B. Myelodysplastic syndrome
C. Aplastic anemia
D. Pure red cell aplasia
E. Paroxysmal nocturnal hemoglobinuria

447 Examination of the synovial fluid of the knee of a In osteoarthritis, the findings of the synovial fluid JULIET KRISTINE MIDTERM 2
patient with osteoarthritis will reveal the following are are clear viscous fluid, negative culture, low EVANGELISTA, MD EXAM - AUG
except: WBC. CPPD crystals are found in pseudogout. (TOP 9 - FEB 2014 2014
A. Clear viscous fluid MED BOARDS;
B. Negative culture TOPNOTCH MD)
C. Low levels of WBC count
D. (+) CPPD crystals
E. None of the above

448 Superior vena cava syndrome is characterized by the Superior vena cava syndrome is a severe JULIET KRISTINE MIDTERM 2
following: reduction invenous retirn from the head, neck and EVANGELISTA, MD EXAM - AUG
A. Anhidrosis, miosis, ptosis of affected side upper extremities due to obstruction of SVC blood (TOP 9 - FEB 2014 2014
B. Edema and rubor of face, neck and upper chest flow. Most common etiologies are lung cancer, MED BOARDS;
C. Prenic nerve paralysis and elevation of lymphoma, and metastatic tumors. TOPNOTCH MD)
hemidiaphragm
D. Shoulder pain radiating to ulnar distribution of
the arm
E. All of the above
449 In patients with HIV infection, the generally For prognostication, CD4 count value tells the JULIET KRISTINE MIDTERM 2
accepted indicator of the immunologic competence immune status of patients infected with HIV. EVANGELISTA, MD EXAM - AUG
is: (TOP 9 - FEB 2014 2014
A. Immunoglobulin level MED BOARDS;
B. Plasma viremia level TOPNOTCH MD)
C. CD4 T lymphocyte count
D. A and C
E. B and C

TOPNOTCH MEDICAL BOARD PREP INTERNAL MEDICINE SUPEREXAM Page 59 of 95


For inquiries visit www.topnotchboardprep.com.ph or email us at topnotchmedicalboardprep@gmail.com
TOPNOTCH MEDICAL BOARD PREP INTERNAL MEDICINE SUPEREXAM
For inquiries visit www.topnotchboardprep.com.ph or email us at topnotchmedicalboardprep@gmail.com
Item QUESTION EXPLANATION AUTHOR TOPNOTCH
# EXAM
450 Which of the following drugs can dissolve the Streptokinase is a thrombolytic, a tissue JULIET KRISTINE MIDTERM 2
thrombus in acute coronary syndrome? plasminogen activator analog, which converts EVANGELISTA, MD EXAM - AUG
A. Low molecular weight heparin plasminogen to plasmin, which degrades the fibrin (TOP 9 - FEB 2014 2014
B. Unfractionated heparin and fibrinogen, causing thrombolysis. MED BOARDS;
C. Aspirin TOPNOTCH MD)
D. Streptokinase
E. Abciximab

451 In large retrosternal goiter, when the arms are Pemberton sign is venous distention over the neck JULIET KRISTINE MIDTERM 2
raised above the head, this cause the thyroid mass to and difficulty breathing especially when the arms EVANGELISTA, MD EXAM - AUG
impinge on the blood vessels causing flushing of the are raised in large retrosternal goiters. (TOP 9 - FEB 2014 2014
face and syncope. This is also known as: MED BOARDS;
A. Basedow sign TOPNOTCH MD)
B. Reidel's sign
C. Pemberton sign
D. McConnell's sign
E. Cullen's sign

452 a 48 year-old man with a history of alcohol abuse Esophageal reflux secondary to hiatal hernia will JULIET KRISTINE MIDTERM 2
presents to the emergency room vomiting bright red not present with bright red blood vomitus and is EVANGELISTA, MD EXAM - AUG
blood. All the following should be included in the not related to alcohol abuse. (TOP 9 - FEB 2014 2014
differential diagnosis except: MED BOARDS;
A. Ruptured esophageal varices TOPNOTCH MD)
B. Esophageal reflux secondary to hiatal hernia
C. Boerhaave's syndrome
D. Mallory weiss syndrome
E. None of the above

453 The differentiating agent that do not produce DIC All trans-retinoic acid allows DNA transcription JULIET KRISTINE MIDTERM 2
and promotes maturation of promyelocytes in AML and differentiation of immature leukemic EVANGELISTA, MD EXAM - AUG
M3 is: promyelocytes into mature granulocytes. It is (TOP 9 - FEB 2014 2014
A. Vitamin A useful in patients with Acute Myelogenous MED BOARDS;
B. Tretinoin Leukemia M3. TOPNOTCH MD)
C. All cis-retinoic acid
D. All trans-retinoic acid
E. Retinol

454 A transient, pruritic linear wheal with a flare at a site Dermographism is a transient, pruritic linear JULIET KRISTINE MIDTERM 2
in which the skin is briskly stroked with a firm wheal with a flare at a site in which the skin is EVANGELISTA, MD EXAM - AUG
object is known as: briskly stroked with a firm object. (TOP 9 - FEB 2014 2014
A. Dermatosis MED BOARDS;
B. Dermographics TOPNOTCH MD)
C. Dermatography
D. Dermographism
E. Dermoid

455 In a patient presenting with upper respiratory tract Coxsackievirus B is the most common cause of JULIET KRISTINE MIDTERM 2
infection, the sudden onset of chest pain with myocarditis and pericarditis manifested as fever, EVANGELISTA, MD EXAM - AUG
pericardial friction rub and heart failure is usually chest pain and signs of congestive heart failure. (TOP 9 - FEB 2014 2014
due to: MED BOARDS;
A. Hepatitis B virus TOPNOTCH MD)
B. Herpesviruses
C. Coxsackievirus
D. Measles virus
E. Adenovirus

456 Patients who are undergoing dialysis for the first The dialysis disequilibrium syndrome is a rare but JULIET KRISTINE MIDTERM 2
time will experience this condition due to rapid serious complication of hemodialysis. Despite the EVANGELISTA, MD EXAM - AUG
removal of toxic metabolites: fact that maintenance hemodialysis has been a (TOP 9 - FEB 2014 2014
A. Dumping syndrome routine procedure for over years, this syndrome MED BOARDS;
B. First use syndrome remains poorly understood. The signs and TOPNOTCH MD)
C. Anaphylactic shock symptoms vary widely from restlessness and
D. Recirculation headache to coma and death.
E. Disequilibrium syndrome

457 A 58 year-old male was diagnosed with Membranous GN is the most common adult cause JULIET KRISTINE MIDTERM 2
membranous GN. Kidney biopsy of this patient will of nephrotic syndrome. It is characterized by EVANGELISTA, MD EXAM - AUG
show: diffuse capillary and BM thickening, spike and (TOP 9 - FEB 2014 2014
A. Effacement of foot processes dome appearance, subepithelial IgG and C3 MED BOARDS;
B. Subendothelial deposits deposits. TOPNOTCH MD)
C. Subepithelial deposits
D. IgG linear antibodies
E. Kimmelsteil-Wilson bodies

458 Pulmonary infections is common among patients Pseudomonas aeruginosa is the most common JULIET KRISTINE MIDTERM 2
with cystic fibrosis. Therapy should be directed cause of nosocomial pneumonia in patients with EVANGELISTA, MD EXAM - AUG
against which of the following organism: cystic fibrosis patients. Renal colic typically begins (TOP 9 - FEB 2014 2014
A. Pseudomonas aeruginosa in the abdomen and often radiates to the MED BOARDS;
B. Streptococcus penumoniae hypochondrium or the groin. It is typically colicky TOPNOTCH MD)
C. Chlamydia pneumoniae due to the presence of a stone in the ureter or at
D. Klebsiella pneumoniae the pelviureteric junction.
E. All of the above

TOPNOTCH MEDICAL BOARD PREP INTERNAL MEDICINE SUPEREXAM Page 60 of 95


For inquiries visit www.topnotchboardprep.com.ph or email us at topnotchmedicalboardprep@gmail.com
TOPNOTCH MEDICAL BOARD PREP INTERNAL MEDICINE SUPEREXAM
For inquiries visit www.topnotchboardprep.com.ph or email us at topnotchmedicalboardprep@gmail.com
Item QUESTION EXPLANATION AUTHOR TOPNOTCH
# EXAM
459 A 25 year-old man came to Emergency Room due to Renal colic typically begins in the abdomen and JULIET KRISTINE MIDTERM 2
severe intermittent flank pain radiating to groin. often radiates to the hypochondrium or the groin. EVANGELISTA, MD EXAM - AUG
Pain was described as "renal colic" usually indicates: It is typically colicky due to the presence of a stone (TOP 9 - FEB 2014 2014
A. Kidney stone in the ureter or at the pelviureteric junction. MED BOARDS;
B. Ureteral stone Perforation is the second most common TOPNOTCH MD)
C. Bladder stone complication of peptic ulcer disease. Initial
D. Pyelonephritis procedure of choice which is safe and easy to
E. Cystitis document pneumoperitoneum is Plain xray of the
abdomen. Barium swallow is contraindicated in
the presence of perforation.
460 Ruptured peptic ulcer with pneumoperitoneum is Perforation is the second most common JULIET KRISTINE MIDTERM 2
most easily and safely diagnosed by: complication of peptic ulcer disease. Initial EVANGELISTA, MD EXAM - AUG
A. Barium swallow procedure of choice which is safe and easy to (TOP 9 - FEB 2014 2014
B. Ultrasound document pneumoperitoneum is Plain xray of the MED BOARDS;
C. Plain xray of the abdomen abdomen. Barium swallow is contraindicated in TOPNOTCH MD)
D. CT scan the presence of perforation.
E. A and C

461 In tumor lysis syndrome, the following are the LUISA SARANILLO, BACK-UP
electrolyte abnormalities except: MD (TOP 6 - FEB MIDTERM
A. hypekalemia 2014 MED EXAM AUG
B. hyperphosphatemia BOARDS; 2014 - FOR
C. hyperuricemia TOPNOTCH MD) INCLUSION IN
D. hypocalcemia THE SAMPLEX
E. Hypercalcemia

462 A 50 year old female presented with acute cough, Pneumatocele formation is consistent with S. LUISA SARANILLO, BACK-UP
fever and tachypnea. On chest x-ray, there was aureus. MD (TOP 6 - FEB MIDTERM
bilateral infiltrates and pneumatoceles. What is the 2014 MED EXAM AUG
most likely etiologic agent? BOARDS; 2014 - FOR
A. Staphylococcus aureus TOPNOTCH MD) INCLUSION IN
B. Streptococcus pneumoniae THE SAMPLEX
C. Haemophilus influenzae
D. Mycoplasma pneumoniae
E. Klebsiella pneumoniae

463 One of the following is not consistent with the waist circumference of >102cm is for males, while LUISA SARANILLO, BACK-UP
criteria for metabolic syndrome. for females it is >88cm MD (TOP 6 - FEB MIDTERM
A. Triglyceride level of 200 2014 MED EXAM AUG
B. Patient on metformin BOARDS; 2014 - FOR
C. Blood pressure of 135/90 TOPNOTCH MD) INCLUSION IN
D. Female waist circumference of >102 cm THE SAMPLEX
E. HDL level of <40 mg/dl in males

464 The following echocardiographic findings are LUISA SARANILLO, BACK-UP


considered positive for infective endocarditis: MD (TOP 6 - FEB MIDTERM
A. Intracardiac mass 2014 MED EXAM AUG
B. abscess BOARDS; 2014 - FOR
C. New dehiscence of prosthetic valve TOPNOTCH MD) INCLUSION IN
D. New valvular regurgitation THE SAMPLEX
E. All of the choices

465 What is the causative agent of malignant tertian P. falciparum - malignant tertian malaria: P. vivax LUISA SARANILLO, BACK-UP
malaria? and ovale - benign tertian malaria; P. malariae - MD (TOP 6 - FEB MIDTERM
A. Plasmodium falciparum quartan malaria. 2014 MED EXAM AUG
B. Plasmodium vivax BOARDS; 2014 - FOR
C. Plasmodium ovale TOPNOTCH MD) INCLUSION IN
D. Plasmodium malariae THE SAMPLEX
E. B and C

466 A 30 year old male construction worker was This is a mild case of leptospirosis. The first line LUISA SARANILLO, BACK-UP
admitted due to fever of 3 days, associated with drug is Doxycycline 100 mg BID PO, while MD (TOP 6 - FEB MIDTERM
abdominal pain, diarrhea, myalgia, and headache. On amoxicillin is the alternative drug. For severe 2014 MED EXAM AUG
Physical examination, there was conjuctival cases, Penicillin G 1.5MU q6-8h IV is the first line BOARDS; 2014 - FOR
suffusion and calf tenderness. History revealed that drug. The rest of the choices are alternative drugs TOPNOTCH MD) INCLUSION IN
he waded in flood water. Vital signs were normal, for severe cases. THE SAMPLEX
good urine output, no jaundice, and no signs of
meningeal irritation. What is the first line drug?
A. Amoxicillin 500mg q6hrs PO
B. Ampicillin 1g q6h IV
C. Penicillin G 1.5MU q6h IV
D. Doxycycline 100mg BID PO
E. Azithromycin 500mg OD PO

467 Components of multiple endocrine neoplasia 2B components of MEN 2B are: medullary thyroid LUISA SARANILLO, BACK-UP
except: carcinoma, pheochromocytoma, mucosal and GI MD (TOP 6 - FEB MIDTERM
A. Medullary thyroid carcinoma neuroma, and marfanoid features. Parathyroid 2014 MED EXAM AUG
B. pheochromocytoma adenoma is a component of MEN 2A BOARDS; 2014 - FOR
C. Parathyroid adenoma TOPNOTCH MD) INCLUSION IN
D. Mucosal and Gastrointestinal neuroma THE SAMPLEX
E. Marfanoid features

TOPNOTCH MEDICAL BOARD PREP INTERNAL MEDICINE SUPEREXAM Page 61 of 95


For inquiries visit www.topnotchboardprep.com.ph or email us at topnotchmedicalboardprep@gmail.com
TOPNOTCH MEDICAL BOARD PREP INTERNAL MEDICINE SUPEREXAM
For inquiries visit www.topnotchboardprep.com.ph or email us at topnotchmedicalboardprep@gmail.com
Item QUESTION EXPLANATION AUTHOR TOPNOTCH
# EXAM
468 Coronary artery disease is the most common cause CAD is the most common cause of systolic LUISA SARANILLO, BACK-UP
of: dysfunction, while concentric LVH is the most MD (TOP 6 - FEB MIDTERM
A. Systolic dyfunction common cause of diastolic dysfunction. 2014 MED EXAM AUG
B. Diastolic dysfunction BOARDS; 2014 - FOR
C. cardiomyopathy TOPNOTCH MD) INCLUSION IN
D. Left ventricular hypertrophy THE SAMPLEX
E. All of the choices

469 Patient came in complaining of dysuria, hematuria, LUISA SARANILLO, BACK-UP


and oliguria. How will you define oliguria? MD (TOP 6 - FEB MIDTERM
A. No urine output for 6 hours 2014 MED EXAM AUG
B. No urine output for 12 hours BOARDS; 2014 - FOR
C. Urine output of <400ml/24 hours TOPNOTCH MD) INCLUSION IN
D. Urine output of < 600ml/24 hours THE SAMPLEX
E. None of the choices

470 In spirometry, patients with Chronic Obstructive In COPD: FEV1 and FEV1/FVC ratio are reduced, LUISA SARANILLO, BACK-UP
Pulmonary Disease have: while residual volume, total lung capacity, and MD (TOP 6 - FEB MIDTERM
A. Increased FEV1 functional residual capacity are increased. 2014 MED EXAM AUG
B. Increased Residual volume BOARDS; 2014 - FOR
C. Decreased total lung capacity TOPNOTCH MD) INCLUSION IN
D. Decreased functional residual capacity THE SAMPLEX
E. Increased FEV1/FVC ratio

471 In the natural history of ARDS, in what pahase will There are 3 phases in ARDS: exudative, LUISA SARANILLO, BACK-UP
most patients recover or wean from mechanical proliferative, and fibrotic phase. Most patients will MD (TOP 6 - FEB MIDTERM
support? recover during the proliferative phase. 2014 MED EXAM AUG
A. Exudative phase BOARDS; 2014 - FOR
B. Proliferative phase TOPNOTCH MD) INCLUSION IN
C. Fibrotic phase THE SAMPLEX
D. Resolution phase
E. None of the choices

472 A 36 year old female had vomiting and profuse RIFLE criteria for AKI: risk - urine output LUISA SARANILLO, BACK-UP
diarrhea for 3 days at home without any medications <0.5ml/kg/h for 6 hrs; injury - urine output of MD (TOP 6 - FEB MIDTERM
taken. She was brought to the emergency <0.5ml/kg/h for 12 hrs; failure - urine output of 2014 MED EXAM AUG
department severely dehydrated without urine <0.3ml/kg/h for 24 hours or anuria for 12 hours; BOARDS; 2014 - FOR
output for 15 hours. Her serum creatinine was Loss - complete loss of kidney function for TOPNOTCH MD) INCLUSION IN
5mg/dl. Using the RIFLE criteria for acute kidney >4weeks; ESRD - complete loss of kidney function THE SAMPLEX
injury, the patient is in what stage? for >3 months.
A. risk
B. injury
C. failure
D. loss
E. end stage renal disease

473 A 40 year old female came in for consultation due to this is a case of anemia due to bleeding. LUISA SARANILLO, BACK-UP
menorrhagia for 6 months. She complained of Transfusion is done when Hemoglobin is <7g/dl, MD (TOP 6 - FEB MIDTERM
dizziness upon standing and changing positions. She or hemoglobin of <10g/dl in symptomatic anemic 2014 MED EXAM AUG
was pale, with heart rate of 120bpm, respiratory patient - presence of tachycardia, dyspnea, BOARDS; 2014 - FOR
rate of 24cpm. Hemoglobin level was 8g/dl. What is postural hypotension, chest pain, and syncope. TOPNOTCH MD) INCLUSION IN
the best management? THE SAMPLEX
A. surgery
B. ferrous sulfate tablet TID
C. IV iron
D. transfuse with packed RBC
E. transfuse with Packed RBC together with
platelet concentrate
474 Patient presents with polyuria, polydipsia, and The criteria for the diagnosis of DM are the LUISA SARANILLO, BACK-UP
weight loss. If you are suspecting diabetes mellitus, following: HBA1C ≥6.5%; FBS ≥ 126mg/dl MD (TOP 6 - FEB MIDTERM
which of the following will confirm your diagnosis. (7mmol/L); 2hr plasma glucose during OGTT ≥ 2014 MED EXAM AUG
A. HBA1C of 6% 200mg/dl (11.1mmol/L); RBS ≥ 200mg/dl BOARDS; 2014 - FOR
B. FBS of 6.9mmol/L (11.1mmol/L) + classic symptoms of TOPNOTCH MD) INCLUSION IN
C. 2-hour plasma glucose during an OGTT is hyperglycemia. THE SAMPLEX
220mg/dl
D. random blood sugar of 11mmol/L
E. any of the choices

475 A 28 year old male presented with acute abdominal This is a case of nephrolithiasis, and the standard LUISA SARANILLO, BACK-UP
pain. He can't find position of comfort, and keeps on diagnostic procedure is helical CT scan without MD (TOP 6 - FEB MIDTERM
moving in the bed rolling up himself trying to ease contrast 2014 MED EXAM AUG
the pain. He also had hematuria and dysuria. What is BOARDS; 2014 - FOR
the standard diagnostic procedure to confirm your TOPNOTCH MD) INCLUSION IN
diagnosis? THE SAMPLEX
A. helical CT scan with contrast
B. helical CT scan without contrast
C. urinalysis
D. ultrasound
E. TURP

TOPNOTCH MEDICAL BOARD PREP INTERNAL MEDICINE SUPEREXAM Page 62 of 95


For inquiries visit www.topnotchboardprep.com.ph or email us at topnotchmedicalboardprep@gmail.com
TOPNOTCH MEDICAL BOARD PREP INTERNAL MEDICINE SUPEREXAM
For inquiries visit www.topnotchboardprep.com.ph or email us at topnotchmedicalboardprep@gmail.com
Item QUESTION EXPLANATION AUTHOR TOPNOTCH
# EXAM
476 A 54 year old female patient presents with an RAI therapy provides the quickest resolution of LUISA SARANILLO, BACK-UP
anterior neck mass. She had sleepless nights and hyperthyroidism MD (TOP 6 - FEB MIDTERM
profuse sweating. She had palpitations and tremors. 2014 MED EXAM AUG
She had weight loss of about 5kg despite eating BOARDS; 2014 - FOR
more than her regular meal. She asked you, what TOPNOTCH MD) INCLUSION IN
management will give the quickest resolution of her THE SAMPLEX
symptoms?
A. propylthiouracil
B. methimazole
C. surgery
D. radioactive iodine therapy
E. propranolol

477 A 28 year old male alcoholic came in due to vomiting this is a case of pancreatitis, in which there are LUISA SARANILLO, BACK-UP
and abdominal pain around the periumbilical region two distinctive signs: cullen's sign - blue MD (TOP 6 - FEB MIDTERM
boring in character radiating to the back, which discoloration around the umbilicus; turner's sign - 2014 MED EXAM AUG
intensifies when lying supine. On examination you green brown discoloration of the flanks. BOARDS; 2014 - FOR
noticed a blue discoloration around the umbilicus TOPNOTCH MD) INCLUSION IN
which is called: THE SAMPLEX
A. turner's sign
B. cullen's sign
C. Dunphy's sign
D. courvoisier's sign
E. murphy's sign
478 What is the best screening test for systemic lupus For SLE, ANA is the best screening test. Anti- LUISA SARANILLO, BACK-UP
erythematosus? DsDNA is highly specific and correlates with the MD (TOP 6 - FEB MIDTERM
A. Anti-DsDNA disease activity, and Anti-Sm is specific for SLE. 2014 MED EXAM AUG
B. Anti-Sm BOARDS; 2014 - FOR
C. ANA TOPNOTCH MD) INCLUSION IN
D. C3 THE SAMPLEX
E. Any of the choices

479 The following are the indications for dialysis except: LUISA SARANILLO, BACK-UP
A. Intractable volume overload MD (TOP 6 - FEB MIDTERM
B. Intractable hyperkalemia 2014 MED EXAM AUG
C. Intractable acidosis BOARDS; 2014 - FOR
D. Uremic encephalopathy TOPNOTCH MD) INCLUSION IN
E. None of the choices THE SAMPLEX

480 A 45 year old farmer smoker, and alcoholic had a this is a new smear positive PTB. The treatment is LUISA SARANILLO, BACK-UP
one month productive cough, associated with divided into 2 phases: the intensive phase MD (TOP 6 - FEB MIDTERM
intermittent fever, chest pain, anorexia and fatigue. wherein, isoniazid, rifampicin, ethambutol, and 2014 MED EXAM AUG
If this is a new case with a smear-positive, what pyrazinamide are given for 4 months; and the BOARDS; 2014 - FOR
drugs are given during the maintenance phase of maintenace phase wherein isoniazid and TOPNOTCH MD) INCLUSION IN
treatment? rifampicin are given for 2 months. THE SAMPLEX
A. isoniazid and rifampicin
B. pyrazinamide and isoniazid
C. ethambutol and isoniazid
D. rifampicin, ethambutol, and isoniazid
E. isoniazid, rifampicin, pyrazinamide, and
ethambutol
481 DJ had just been administered an injection of pollen Subcutaneous epinephrine would be the first ANGELIS ANDREA FINAL EXAM -
extract as prescribed by her allergologist. She medication used. Additional treatment can include COCOS, MD (TOP 1 AUG 2014
started to complain about nausea and you note that plasma expanders, diphenhydramine and - FEB 2014 MED
her face is flushed and her voice sounds muffled and cimetidine, as indicated. Corticosteroids should be BOARDS;
strained. Which of the following is the first priority started early but the effect is delayed. TOPNOTCH MD)
in managing this episode of anaphylaxis?
A. intravenous injection of 1:10,000 epinephrine
B. subcutaneous injection of 1:1,000 epinephrine
C. intravenous corticosteroid
D. intramuscular injection of diphenhydramine
482 A 40-year-old female receiving a drug from In contrast, propranolol causes an increase in the ANGELIS ANDREA FINAL EXAM -
supraventricular tachycardia shows prolongation of PR interval. COCOS, MD (TOP 1 AUG 2014
the QT interval on ECG. An antiarrhytmic drug which - FEB 2014 MED
causes QT prolongation include: BOARDS;
A. lidocaine TOPNOTCH MD)
B. propranolol
C. flecainide
D. quinidine

483 A 20-year-old man presents with recurrent upper CVID is due to a defect in B-cell mutation and has ANGELIS ANDREA FINAL EXAM -
respiratory tract infections. What immunodeficiency various causes. It can be acquired during the 20s COCOS, MD (TOP 1 AUG 2014
does he have if through further studies he was to 30s. - FEB 2014 MED
shown to have normal number of B cells but very BOARDS;
low to absent plasma cells? TOPNOTCH MD)
A. Bruton's agammaglobulinemia
B. Severe combined immunodeficiency
C. Common variable immunodeficiency
D. Wiskott Aldrich Syndrome

TOPNOTCH MEDICAL BOARD PREP INTERNAL MEDICINE SUPEREXAM Page 63 of 95


For inquiries visit www.topnotchboardprep.com.ph or email us at topnotchmedicalboardprep@gmail.com
TOPNOTCH MEDICAL BOARD PREP INTERNAL MEDICINE SUPEREXAM
For inquiries visit www.topnotchboardprep.com.ph or email us at topnotchmedicalboardprep@gmail.com
Item QUESTION EXPLANATION AUTHOR TOPNOTCH
# EXAM
484 Benzene causes: SIMILAR TO PREVIOUS BOARD EXAM ANGELIS ANDREA FINAL EXAM -
A. leukemia CONCEPT/PRINCIPLE. Benzene is a risk factor for COCOS, MD (TOP 1 AUG 2014
B. lymphoma BOTH aplastic anemia and leukemia (some - FEB 2014 MED
C. aplastic anemia reference include lymphoma also). Some BOARDS;
D. polycythemia vera questions in the boards actually have 2 best TOPNOTCH MD)
answers so you just answer one and pray that
your choice is what the board of medicine has in
mind. :)
485 A boy presents with hemarthrosis and gross Hemophilia A is the most common type of ANGELIS ANDREA FINAL EXAM -
hematuria. He has an uncle with the same condition. hemophilia. It is an X-linked disorder. Most COCOS, MD (TOP 1 AUG 2014
On further examination, his partial thromboplastin patients present with major bleeding like - FEB 2014 MED
time (PTT) is increased. Considering the most likely hemarthrosis, and internal bleeding. Factor VIII is BOARDS;
diagnosis, which of the following should be the treatment of choice in managing these TOPNOTCH MD)
administered to the patient? patients. Other alternatives would be
A. Factor VIII concentrate cryoprecipitate then fresh frozen plasma. SIMILAR
B. cryoprecipitate TO PREVIOUS BOARD EXAM
C. cryosupernatant CONCEPT/PRINCIPLE.
D. fresh frozen plasma
486 Multiple endocrine neoplasia (MEN) 2A and 2B have Parathyroid adenoma is a feature of MEN2A only. ANGELIS ANDREA FINAL EXAM -
these features in common, EXCEPT: Know the features of the MEN syndromes. COCOS, MD (TOP 1 AUG 2014
A. Medullary thyroid carcinoma SIMILAR TO PREVIOUS BOARD EXAM - FEB 2014 MED
B. Pheochromocytoma CONCEPT/PRINCIPLE. BOARDS;
C. Parathyroid adenoma TOPNOTCH MD)
D. There is no exception

487 What is the average GFR per day under normal The average GFR per day is 180 (some references, ANGELIS ANDREA FINAL EXAM -
conditions in the healthy adult? 185) liters per day. COCOS, MD (TOP 1 AUG 2014
A. 80 liters - FEB 2014 MED
B. 200 liters BOARDS;
C. 180 liters TOPNOTCH MD)
D. 120 liters

488 A patient was diagnosed to have idiopathic Idiopathic pulmonary fibrosis is a type of ANGELIS ANDREA FINAL EXAM -
pulmonary fibrosis. What is the hallmark restrictive lung disease, the hallmark of which is COCOS, MD (TOP 1 AUG 2014
spirometric finding for this disease? decreased total lung capacity. This is verbatim - FEB 2014 MED
A. Decreased vital capacity from Harrisons. SIMILAR TO PREVIOUS BOARD BOARDS;
B. Decreased functonal residual capacity EXAM CONCEPT/PRINCIPLE. TOPNOTCH MD)
C. Decreased inspiratory reserve volume
D. Decreased total lung capacity

489 Dihydropiridines reduce the blood pressure of a Dihydropiridines like amlodipine and nifedipine ANGELIS ANDREA FINAL EXAM -
hypertensive patient by: act on the blood vessels. Nondihydropiridines like COCOS, MD (TOP 1 AUG 2014
A. Decreasing peripheral vascular resistance verapamil act on the cardiac muscles. - FEB 2014 MED
B. Increasing cardiac output BOARDS;
C. Decreasing calcium influx into cardiac muscles TOPNOTCH MD)
D. Decreasing renal blood flow
490 A lethargic 23-year-old female was brought to the Naloxone is the antidote for opioid overdose, NAC ANGELIS ANDREA FINAL EXAM -
ER who intentionally took an unknown number of for paracetamol overdose, and 100% oxygen for COCOS, MD (TOP 1 AUG 2014
sedatives. The appropriate antidote would be: carbon monoxide poisoning. - FEB 2014 MED
A. naloxone BOARDS;
B. flumazenil TOPNOTCH MD)
C. N-acetylcysteine
D. 100% oxygen

491 Ataxia, confusion and ophthalmophlegia are clinical Vitamin B1 or Thiamine deficiency causes ANGELIS ANDREA FINAL EXAM -
features of a syndrome that result from severe Wernicke-Korsakoff's syndrome. COCOS, MD (TOP 1 AUG 2014
deprivation of: - FEB 2014 MED
A. Ethyl alcohol BOARDS;
B. Methanol TOPNOTCH MD)
C. Thiamine
D. Riboflavin

492 A negative HBSAg, positive anti-HBC and positive A negative HBSAg, negative anti-HBC and positive ANGELIS ANDREA FINAL EXAM -
anti-HbS is interpreted as: anti-HbS, on the other hand, is interpreted as COCOS, MD (TOP 1 AUG 2014
A. Immune due to natural infection immune due to Hep B immunization. - FEB 2014 MED
B. Immune due to hepatitis B vaccination BOARDS;
C. Acutely infected TOPNOTCH MD)
D. Chronically infected

493 A patient from Palawan complains of paroxysms of Splenomegaly is a characteristic finding in patients ANGELIS ANDREA FINAL EXAM -
chills, fever, and profuse sweating. What additional with malaria. COCOS, MD (TOP 1 AUG 2014
physical finding is consistent with the most likely - FEB 2014 MED
diagnosis? BOARDS;
A. petechiae TOPNOTCH MD)
B. jaundice
C. hepatomegaly
D. Splenomegaly

494 The most common site of TB in pregnancy: In general, the most common site in all age groups ANGELIS ANDREA FINAL EXAM -
A. Pulmonary for tuberculosis infection is still pulmonary. COCOS, MD (TOP 1 AUG 2014
B. GIT - FEB 2014 MED
C. GUT BOARDS;
D. Placenta TOPNOTCH MD)

TOPNOTCH MEDICAL BOARD PREP INTERNAL MEDICINE SUPEREXAM Page 64 of 95


For inquiries visit www.topnotchboardprep.com.ph or email us at topnotchmedicalboardprep@gmail.com
TOPNOTCH MEDICAL BOARD PREP INTERNAL MEDICINE SUPEREXAM
For inquiries visit www.topnotchboardprep.com.ph or email us at topnotchmedicalboardprep@gmail.com
Item QUESTION EXPLANATION AUTHOR TOPNOTCH
# EXAM
495 The hallmark of dengue hemorrhagic fever is: The other choices can also be seen in dengue ANGELIS ANDREA FINAL EXAM -
A. thrombocytopenia hemorrhagic fever/severe dengue. COCOS, MD (TOP 1 AUG 2014
B. hemoconcentration - FEB 2014 MED
C. plasma leakage BOARDS;
D. Bleeding TOPNOTCH MD)

496 According to the CDC, which among the following is Bronchi, trachea, lungs or esophageal candidiasis ANGELIS ANDREA FINAL EXAM -
NOT an AIDS-defining condition? are considered as aids-defining illnesses, but not COCOS, MD (TOP 1 AUG 2014
A. oral candidiasis oral candidiasis. - FEB 2014 MED
B. cytomegalovirus retinitis BOARDS;
C. pneumocystic jirovecci pneumonia TOPNOTCH MD)
D. progressive multifocal leukoencephalopathy

497 A patient presents with recurrent febrile episodes A similar question was asked recurrently during ANGELIS ANDREA FINAL EXAM -
and migratory joint pains. On physical examination, our exam. Acute rheumatic fever can be diagnosed COCOS, MD (TOP 1 AUG 2014
there is a systolic murmur over the 4th ICS left mid- if the patient fulfills 2 major or 1 major and 2 - FEB 2014 MED
clavicular area. To confirm your diagnosis, you minor manifestations plus evidence of a preceding BOARDS;
should order which of the following diagnostics? GAS infection through ASO or anti-Dnase B titers. TOPNOTCH MD)
A. ASO titers
B. throat swab culture
C. anti-Dnase A titers
D. ESR

498 This condition presents with crypt abscesses and This should be differentiated from Crohn's ANGELIS ANDREA FINAL EXAM -
friable mucosal pseudopolyps. There is also lead- disease, the features of which are: cobblestone COCOS, MD (TOP 1 AUG 2014
pipe appearance on imaging. What disease is this? mucosa, transmural inflammation, skip lesions, - FEB 2014 MED
A. Crohn's disease strictures and fistulas. BOARDS;
B. Ulcerative colitis TOPNOTCH MD)
C. Irritable bowel syndrome
D. Peutz-Jeghers syndrome

499 All of the following can cause hemorrhagic cystitis, Kanamycin, an aminoglycoside, is nephrotoxic but ANGELIS ANDREA FINAL EXAM -
EXCEPT: does not affect the bladder. Other causes of COCOS, MD (TOP 1 AUG 2014
A. cyclophosphamide hemorrhagic cystitis include E. coli, papovavirus, - FEB 2014 MED
B. adenovirus influenza A, methicillin, piperacillin etc. BOARDS;
C. radiation TOPNOTCH MD)
D. kanamycin
500 A female patient presents with charcot's classic triad Natalizumab is a humanized monoclonal antibody ANGELIS ANDREA FINAL EXAM -
of scanning speech, intention tremor and nystagmus. that belongs to a class known as alpha-4 integrin COCOS, MD (TOP 1 AUG 2014
Which of the following may be of benefit to this inhibitors. It binds to the cell surface receptors - FEB 2014 MED
patient? known as alpha-4-beta-1 and alpha-4-beta-7. It BOARDS;
A. natalizumab has been shown to reduce the rates of relapse and TOPNOTCH MD)
B. IVIG disease progression in multiple sclerosis.
C. valproic acid
D. Memantine

501 Which of the following statements regarding pulsus B. Pulsus alternans; C. Homan's sign JAN CHARMAINE BACK-UP
paradoxus is true? PALOMAR, MD MIDTERM
A. It is the fall in systolic blood pressure by (TOP 9 - FEB 2014 EXAM AUG
>10mmHg with inspiration. MED BOARDS; 2014
B. It is the beat to beat variability of pulse TOPNOTCH MD)
amplitude seen in severe LV systolic heart failure.
C. It refers to posterior calf pain on active
dorsiflexion of the foot againsts resistance
suggestive of DVT.
D. all of the above
E. none of the above
502 Imelda, a 76 year old vendor went to visit your clinic Normal <120/<80mmHg; Pre-hypertension=120- JAN CHARMAINE BACK-UP
complaining of dizziness. Vital signs revealed 139/80-89mmHg; Stage 1= 140-159/90-99mmHg; PALOMAR, MD MIDTERM
BP=150/110mmHg, HR=78bpm; RR=20rpm; and Stage 2=>160/>100mmHg; Isolated Systolic (TOP 9 - FEB 2014 EXAM AUG
Temperature=36.8C. Classify hypertension of hypertension=>140/<90mmHg MED BOARDS; 2014
patient: TOPNOTCH MD)
A. Normal
B. Pre-hypertension
C. Stage 1 hypertension
D. Stage 2 hypertension
E. Isolated Systolic hypertension
503 Metabolic syndrome refers to metabolic Waist circumference >102cm in males; JAN CHARMAINE BACK-UP
abnormalities that confer an increased risk of TAG>150mg/dL or use of specific med; low HDL; PALOMAR, MD MIDTERM
cardiovascular disease and diabetes mellitus. Which BP >130/85mmHg; FBS>100mg/dL or previously (TOP 9 - FEB 2014 EXAM AUG
of the following does not meet the criteria for diagnosed T2DM MED BOARDS; 2014
metabolic syndrome? TOPNOTCH MD)
A. Waist circumference of >88cm in a female
B. Triglyceride >100mg/dL
C. HDL <40mg/dL and <50mg/dL in a male and
female, respectively
D. BP of 140/90mmHg
E. Fasting blood glucose of 150 mg/dL

TOPNOTCH MEDICAL BOARD PREP INTERNAL MEDICINE SUPEREXAM Page 65 of 95


For inquiries visit www.topnotchboardprep.com.ph or email us at topnotchmedicalboardprep@gmail.com
TOPNOTCH MEDICAL BOARD PREP INTERNAL MEDICINE SUPEREXAM
For inquiries visit www.topnotchboardprep.com.ph or email us at topnotchmedicalboardprep@gmail.com
Item QUESTION EXPLANATION AUTHOR TOPNOTCH
# EXAM
504 A 59 year old man was rushed to the emergency JAN CHARMAINE BACK-UP
room because of severe chest pain described as PALOMAR, MD MIDTERM
heaviness . Pain radiates to left shoulders and arm. (TOP 9 - FEB 2014 EXAM AUG
What ancillary procedure should you request? MED BOARDS; 2014
A. Stress testing TOPNOTCH MD)
B. 2D Echo
C. 12- lead ECG
D. CBC with platelet
E. CT scan of the chest

505 This drug acts as an HMG-CoA reductase inhibitor SIMILAR TO PREVIOUS BOARD EXAM JAN CHARMAINE BACK-UP
which results to plaque stabilization, lowering of CONCEPT/PRINCIPLE PALOMAR, MD MIDTERM
LDL cholesterol and TAG and raising HDL (TOP 9 - FEB 2014 EXAM AUG
cholesterol: MED BOARDS; 2014
A. Fenofibrate TOPNOTCH MD)
B. Clopidogrel
C. Metoprolol
D. Nifedipine
E. Atorvastatin

506 In which of the following patients is Coronary artery IM platinum 55 JAN CHARMAINE BACK-UP
bypass grafting indicated? PALOMAR, MD MIDTERM
A. Elena, 53 years old, with 3-vessel coronary artery (TOP 9 - FEB 2014 EXAM AUG
disease MED BOARDS; 2014
B. Mario, 47 years old with 70% occlusion of the left TOPNOTCH MD)
anterior descending and left circumflex artery
C. Inang, 61 years old with stenosis of the left main
coronary artery
D. All of the above
E. None of the above
507 The following patients cannot be given streptokinase Absolute contraindications to thrombolysis: any JAN CHARMAINE BACK-UP
and tissue plasminogen activators, except: prior intracranial hemorrhage, non-hemorrhagic PALOMAR, MD MIDTERM
A. Riza, 49 years old, complaining of severe chest stroke or other CV event withn the past year, (TOP 9 - FEB 2014 EXAM AUG
heaviness, with ST elevation in leads II, III, AVF, day marked hypertension (SBP>180 or DBP>110) at MED BOARDS; 2014
2 of menses any time during the acute presentation, suspected TOPNOTCH MD)
B. Minda, 67 years old, suspected aortic dissection aortic dissection, and active bleeding or bleeding
C. Roel, 52 year old with blood pressure of diathesis (excluding menses)
190/120mmHg at the ER
D. Rommel, 74 years old with history of
hemorrhagic stroke in the basal ganglia 20 years ago
E. Sanse, 67 years old, with history of ischemic
stroke in the thalamus 8 months ago

508 The following clinical findings can be appreciated in A. De Musset sign - jarring of the body and JAN CHARMAINE BACK-UP
a patient with aortic regurgitation, except: bobbing of the head with each systole in severe AR PALOMAR, MD MIDTERM
A. De Musset sign B. Gallavardin effect - in AS, low-pitched (TOP 9 - FEB 2014 EXAM AUG
B. Gallavardin effect midsystolic ejection murmur at 2nd R ICS that may MED BOARDS; 2014
C. Corrigan's pulse be transmitted to the apex resembling murmur of TOPNOTCH MD)
D. Duroziez sign MR
E. Austin Flint murmur C. Corrigan's pulse - water hammer pulse
D. Duroziez sign - To and fro murmur when
femoral artery is compressed
E. Austin Flint murmur- soft low-pitched
rumbling mid to late diastolic murmur
509 Jemima is a 23 year old sales clerk who developed A. Resonant to percussion, normal fremitus, no JAN CHARMAINE BACK-UP
fever days prior to consult with associated cough, adventitious sounds noted - Normal PALOMAR, MD MIDTERM
colds and difficulty of breathing. The impression was B. Hyperresonant to percussion, decreased (TOP 9 - FEB 2014 EXAM AUG
Moderate Risk Pneumonia, what is the expected fremitus, with occassional wheezing-COPD MED BOARDS; 2014
chest exam findings? C. Dull to percussion, Increased fremitus, with TOPNOTCH MD)
A. Resonant to percussion, normal fremitus, no bibasal crackles -Pneumonia
adventitious sounds noted D. Dull to percussion, decreased fremitus, with
B. Hyperresonant to percussion, decreased pleural friction rub-Pleural effusion
fremitus, with occassional wheezing E. Resonant to percussion, normal fremitus, with
C. Dull to percussion, Increased fremitus, with wheezes-Asthma
bibasal crackles
D. Dull to percussion, decreased fremitus, with
pleural friction rub
E. Resonant to percussion, normal fremitus, with
wheezes
510 This refers to a previously treated pulmonary (IM platinum 99) Failure- a patient, who while on JAN CHARMAINE BACK-UP
tuberculosis patient who has been declared cured treatment, is sputum smear positive at 5 months PALOMAR, MD MIDTERM
and is now diagnosed with culture positive or later during the course of treatment; Return (TOP 9 - FEB 2014 EXAM AUG
tuberculosis: after default- a patient who returns to treatment MED BOARDS; 2014
A. New with positive bacteriology following interruption TOPNOTCH MD)
B. Relapse of treatment for 2 months or more; Transfer in -
C. Failure patient who was transferred from another facility
D. Return after default with proper referral slip to continue treatment -IM
E. Transfer in 99

TOPNOTCH MEDICAL BOARD PREP INTERNAL MEDICINE SUPEREXAM Page 66 of 95


For inquiries visit www.topnotchboardprep.com.ph or email us at topnotchmedicalboardprep@gmail.com
TOPNOTCH MEDICAL BOARD PREP INTERNAL MEDICINE SUPEREXAM
For inquiries visit www.topnotchboardprep.com.ph or email us at topnotchmedicalboardprep@gmail.com
Item QUESTION EXPLANATION AUTHOR TOPNOTCH
# EXAM
511 The presence of this antigen determines Rh JAN CHARMAINE BACK-UP
positivity: PALOMAR, MD MIDTERM
A. D antigen (TOP 9 - FEB 2014 EXAM AUG
B. d antigen MED BOARDS; 2014
C. C antigen TOPNOTCH MD)
D. c antigen
E. H antigen

512 What is the single best test to request in a patient TSH is supressed. IM platinum 198 JAN CHARMAINE BACK-UP
suspected of hyperthyroidism? PALOMAR, MD MIDTERM
A. T3, T4 (TOP 9 - FEB 2014 EXAM AUG
B. TSH MED BOARDS; 2014
C. FT3 TOPNOTCH MD)
D. Thyroglobulin
E. TSI

513 Arterial blood gas result of a patient at the ER reads JAN CHARMAINE BACK-UP
pH of 7.10, pCO2=68mmHg and plasma PALOMAR, MD MIDTERM
bicarbonate=32mmol/L. Interpret: (TOP 9 - FEB 2014 EXAM AUG
A. Respiratory acidosis with renal compensation MED BOARDS; 2014
B. Metabolic acidosis with respiratory TOPNOTCH MD)
compensaton
C. Respiratory alkalosis with renal compensation
D. Metabolic alkalosis with respiratory
compensaton
E. None of the above
514 This management yields the quickest resolution of RAI damages gland thru cytotoxic effect. Absolute JAN CHARMAINE BACK-UP
the hyperthyroid state, however, it leads to contraindications: pregnancy and breast feeding PALOMAR, MD MIDTERM
hypothyroidism requiring lifelong thyroid (TOP 9 - FEB 2014 EXAM AUG
replacement therapy: MED BOARDS; 2014
A. Propylthiouracil TOPNOTCH MD)
B. Methimazole
C. Radioactive Iodine therapy
D. Propranolol
E. None of the above

515 The following are chronic macrovascular *Acute complications of DM- DKA, HHS; *Chronic JAN CHARMAINE BACK-UP
complications of diabetes mellitus, except: microvascular complications-Retinopathy, PALOMAR, MD MIDTERM
A. Diabetic ketoacidosis Neuropathy, Nephropathy; *Chronic (TOP 9 - FEB 2014 EXAM AUG
B. Retinopathy macrovascular complications- CAD, PAD, MED BOARDS; 2014
C. Neuropathy Cerebrovascular disease TOPNOTCH MD)
D. Nephropathy
E. All of the above

516 Minda, 36 year old G2P1 25 weeks AOG, come to HbA1C- reflects the glycemic status over the prior JAN CHARMAINE BACK-UP
your clinic for follow up check up. You diagnosed her 3 months; Fructosamine assay- reflects the PALOMAR, MD MIDTERM
to have gestational diabetes last month for which glycemic status over the prior 2 weeks (TOP 9 - FEB 2014 EXAM AUG
you prescribed Metformin 500mg tab once a day. MED BOARDS; 2014
You want to know the glycemic status over the prior TOPNOTCH MD)
2 weeks. What test should you request?
A. FBS
B. 2 hour OGTT
C. Random blood sugar
D. HbA1C
E. Fructosamine assay
517 These antacids can cause diarrhea and constipation JAN CHARMAINE BACK-UP
respectively: PALOMAR, MD MIDTERM
A. Magnesium hydroxide and aluminum hydroxide (TOP 9 - FEB 2014 EXAM AUG
B. Aluminum hydroxide and magnesium hydroxide MED BOARDS; 2014
C. Cimetidine and ranitidine TOPNOTCH MD)
D. Ranitidine and cimetidine
E. None of the above

518 What is the gold standard in the diagnosis of Blood culture is the gold standard for the JAN CHARMAINE BACK-UP
Typhoid fever? diagnosis of typhoid fever PALOMAR, MD MIDTERM
A. Blood culture (TOP 9 - FEB 2014 EXAM AUG
B. Stool culture MED BOARDS; 2014
C. Urine culture TOPNOTCH MD)
D. Serology
E. Bone marrow culture

519 What is the level of dopamine in Parkinson's Disease Im platinum 353 JAN CHARMAINE BACK-UP
and in Schizophrenia, respectively? PALOMAR, MD MIDTERM
A. Increased, Decreased (TOP 9 - FEB 2014 EXAM AUG
B. Decreased, Increased MED BOARDS; 2014
C. Increased, Increased TOPNOTCH MD)
D. Decreased,Decreased
E. None of the above

TOPNOTCH MEDICAL BOARD PREP INTERNAL MEDICINE SUPEREXAM Page 67 of 95


For inquiries visit www.topnotchboardprep.com.ph or email us at topnotchmedicalboardprep@gmail.com
TOPNOTCH MEDICAL BOARD PREP INTERNAL MEDICINE SUPEREXAM
For inquiries visit www.topnotchboardprep.com.ph or email us at topnotchmedicalboardprep@gmail.com
Item QUESTION EXPLANATION AUTHOR TOPNOTCH
# EXAM
520 What is the hallmark sign of liver disease and is the Jaundice is hallmark of liver disease and the most JAN CHARMAINE BACK-UP
most reliable marker of severity? reliable marker of severity
PALOMAR, MD MIDTERM
A. Fatigue Fatigue- most common and most characteristic (TOP 9 - FEB 2014 EXAM AUG
B. Abdominal distention symptom of liver disease -IM platinum 267 MED BOARDS; 2014
C. Elevated ALT TOPNOTCH MD)
D. Jaundice
E. Right upper quadrant tenderness

521 A 43 year old hypertensive man comes to the MIGUEL RAFAEL MIDTERM 1
physician because of headache, blurred vision, and RAMOS, MD (TOP 3 EXAM - FEB
confusion for 2 days. He has not had weakness or - FEB 2012 MED 2013
numbness, difficulty ambulating, chest pain or BOARDS;
discomfort, diplopia, or vertigo. Blood pressure is TOPNOTCH MD)
190/120 mmHg on presentation. The neck is supple.
Fundoscopic examination shows enlargement of
optic discs with indistinct border. Blood vessels near
the discs are indistinct. The lungs are clear to
auscultation. There is an S4 with no murmur.
Neurologic examination shows no abnormalities.
Which of the following is the most likely diagnosis?
A) Cerebral infarction
B) Malignant hypertension
C) Subarachnoid hemorrhage
D) Transient ischemic attack

522 A 27 year old woman is brought to the physician MIGUEL RAFAEL MIDTERM 1
because of a 3 days history of visual loss and aching RAMOS, MD (TOP 3 EXAM - FEB
discomfort of her right eye. Examination shows - FEB 2012 MED 2013
markedly reduced visual acuity on the right; the BOARDS;
peripheral visual fields are full to confrontation. TOPNOTCH MD)
Color vision is decreased in the right eye. The right
eye does not react to direct light but has a normal
consensual response. Fundoscopic examination
shows no abnormalities. She has an ataxic gait.
Babinski sign is present bilaterally. Which of the
following is the most appropriate next step in
diagnosis?
A) Carotid ultrasonography
B) Visual evoked potentials
C) CT scan of the head with contrast
D) MRI of the brain with contrast

523 A 52 year old man comes to the physician because MIGUEL RAFAEL MIDTERM 1
he has had a 14kg weight loss during the past 6 RAMOS, MD (TOP 3 EXAM - FEB
months. He has noticed oily, floating stools during - FEB 2012 MED 2013
the past 2 months. He received the diagnosis of acute BOARDS;
pancreatitis 2 years ago and has had 1 to 3 hour TOPNOTCH MD)
episodes of severe abdominal pain since ten. The
patient is a 30 year pack smoker. He is an alcoholic
but has been abstinent for the past 2 years. The
abdomen is scaphoid with mild diffuse tenderness.
The liver edge is firm and is palpated 2 cm below the
right costal margin. Laboratories show Amylase 90
(slightly increased), Lipase 43 (normal), alkaline
phosphatase 120 (normal), AST 23, ALT 29. Which of
the following is the most appropriate step in
management?
A) Dietary supplementation with multivitamins and
iron
B) Insulin therapy
C) Pancreatic enzyme replacement therapy
D) Parenteral nutrition
524 A 42 year old woman comes to the physician MIGUEL RAFAEL MIDTERM 1
because of progressive shortness of breath during RAMOS, MD (TOP 3 EXAM - FEB
the past 6 months. She now has to rest three or four - FEB 2012 MED 2013
times when climbing one flight of stairs. She is a 30 BOARDS;
pack year smoker. She has a chronic nonproductive TOPNOTCH MD)
cough and has wheezing controlled with an albuterol
inhaler two to three times weekly. Arterial blood gas
analysis reveals pH 7.37 pCO2 48 pO2 62 O2 sat
92%. Her FEV1 is 75% of predicted, and total lung
capacity is 50% of predicted. Which of the following
is the most likely diagnosis?
A) Chronic obstructive pulmonary disease
B) Congestive heart failure
C) Interstitial pneumonia
D) Restrictive lung disease

TOPNOTCH MEDICAL BOARD PREP INTERNAL MEDICINE SUPEREXAM Page 68 of 95


For inquiries visit www.topnotchboardprep.com.ph or email us at topnotchmedicalboardprep@gmail.com
TOPNOTCH MEDICAL BOARD PREP INTERNAL MEDICINE SUPEREXAM
For inquiries visit www.topnotchboardprep.com.ph or email us at topnotchmedicalboardprep@gmail.com
Item QUESTION EXPLANATION AUTHOR TOPNOTCH
# EXAM
525 An asymptomatic 47 year old woman comes to the MIGUEL RAFAEL MIDTERM 1
physician for a routine health maintenance RAMOS, MD (TOP 3 EXAM - FEB
examination. She has no history of rheumatic fever. - FEB 2012 MED 2013
She takes no medications. Her pulse is 70 bpm, and BOARDS;
blood pressure is 150/60 mmHg. A grade 2/6 TOPNOTCH MD)
decrescendo murmur that begins after S2 is heard at
the sternal border. Which of the following is the
most likely diagnosis?
A) Aortic valve insufficiency
B) Aortic valve stenosis
C) Mitral valve regurgitation
D) Mitral valve stenosis

526 A 37 year old woman comes to the physician MIGUEL RAFAEL MIDTERM 1
because of a 3 week history of discomfort in both RAMOS, MD (TOP 3 EXAM - FEB
knees. She is an avid runner, 15 pack year smoker, - FEB 2012 MED 2013
with a BMI of 31. Her parents both have mild BOARDS;
osteoarthritis. The patient asks how she can TOPNOTCH MD)
decrease her risk for osteoarthritis in the future.
Which of the following is the most appropriate
recommendation?
A) Avoidance of high-impact physical activities
B) Smoking cessation
C) Weight loss
D) Daily use of calcium supplement

527 A 24 year old man is brought to the emergency MIGUEL RAFAEL MIDTERM 1
department comatose after he sustained severe head RAMOS, MD (TOP 3 EXAM - FEB
trauma in a head on motor vehicle collision. His - FEB 2012 MED 2013
medical history is unknown. Over the next hour, he BOARDS;
receives 80 mL of 0.45% saline and has a urine TOPNOTCH MD)
output of 900 mL. Laboratory studies show Na 147,
glucose 124, osmolality 294, urine specific gravity
1.001. A CT scan of the head shows scattered
contusions and a subarachnoid hemorrhage. Which
of the following is the most likely cause of this
patient’s increased urinary output?
A) Syndrome of inappropriate secretion of ADH
B) Traumatic nephropathy
C) Hypernatremia
D) Diabetes insipidus

528 A 21-year-old African American college student has MIGUEL RAFAEL MIDTERM 1
had increasing fatigue over the past 3 weeks. Since RAMOS, MD (TOP 3 EXAM - FEB
an episode of cystitis treated with trimethoprim- - FEB 2012 MED 2013
sulfamethoxazole 3 weeks ago, he has been unable BOARDS;
to keep up with his physical education classes. For 6 TOPNOTCH MD)
months, he has been following a vegetarian diet that
has been supervised by student health services.
Examination shows no abnormalities. His
hemoglobin level is 10 g/dL, mean corpuscular
volume is 85 μm3, and reticulocyte count is 15%.
Which of the following is the most likely cause of
anemia in this patient?
A) Anemia secondary to infection
B) Antibiotic therapy
C) Gastrointestinal blood loss
D) Sickle cell disease
529 A 28-year-old woman at 28 weeks' gestation reports MIGUEL RAFAEL MIDTERM 1
excessive fatigability and dyspnea. Her blood RAMOS, MD (TOP 3 EXAM - FEB
pressure is 118/74 mm Hg, pulse is 110/min and - FEB 2012 MED 2013
regular, and lungs are clear to auscultation. The BOARDS;
cardiac apex is not palpable. S1 is loud, and there is a TOPNOTCH MD)
sharp sound after S2. A low-frequency diastolic
murmur is heard at the apex that increases in
intensity before S1. Which of the following is the
most likely diagnosis?
A) Aortic regurgitation
B) Ebstein's anomaly
C) Mitral regurgitation
D) Mitral stenosis

TOPNOTCH MEDICAL BOARD PREP INTERNAL MEDICINE SUPEREXAM Page 69 of 95


For inquiries visit www.topnotchboardprep.com.ph or email us at topnotchmedicalboardprep@gmail.com
TOPNOTCH MEDICAL BOARD PREP INTERNAL MEDICINE SUPEREXAM
For inquiries visit www.topnotchboardprep.com.ph or email us at topnotchmedicalboardprep@gmail.com
Item QUESTION EXPLANATION AUTHOR TOPNOTCH
# EXAM
530 A 42-year-old woman comes to the physician for MIGUEL RAFAEL MIDTERM 1
evaluation of persistently increased blood pressures. RAMOS, MD (TOP 3 EXAM - FEB
At her last two office visits during the past 3 months, - FEB 2012 MED 2013
her blood pressure has ranged between 150– BOARDS;
170/105–115 mm Hg. During this period, she has TOPNOTCH MD)
had occasional headaches. In addition, she has had
an increased urine output over the past 6 weeks that
she attributes to a diet high in sodium. She is
otherwise healthy and takes no medications. Her
blood pressure today is 168/115 mm Hg, pulse is
68/min, and respirations are 14/min. Funduscopic
examination shows mild arteriovenous nicking. The
point of maximal impulse is not displaced. There is
no edema, abdominal bruits, or masses. Serum
studies show Na 144, Cl 90, K 2.9, HCO3 32, BUN 20,
Creatinine 1.2. Which of the following is the most
likely underlying cause of this patient's
hypertension?
A) Autonomous production of aldosterone
B) Catecholamine-producing tumor
C) Decreased arterial distensibility caused by
atherosclerosis
D) Excess production of atrial natriuretic peptide
531 A previously healthy 16-year-old high school MIGUEL RAFAEL MIDTERM 1
wrestler comes to the physician because of a rash on RAMOS, MD (TOP 3 EXAM - FEB
his forearms and the back of his legs for 1 week. He - FEB 2012 MED 2013
is allergic to pollen and dust. Examination shows BOARDS;
patches of erythema with mild lichenification over TOPNOTCH MD)
the antecubital and popliteal fossae. There are
clusters of painful umbilicated vesicles at sites of
active skin inflammation. Which of the following is
the most likely diagnosis?
A) Eczema herpeticum
B) Herpes zoster
C) Keratosis pilaris
D) Lichen planus

532 A 20-year-old man is brought to the emergency MIGUEL RAFAEL MIDTERM 1


department on a summer day 20 minutes after RAMOS, MD (TOP 3 EXAM - FEB
developing headache, nausea, and unsteady gait - FEB 2012 MED 2013
while running the last 2 miles of a marathon. On BOARDS;
arrival, he is confused and disoriented. His TOPNOTCH MD)
temperature is 40 C (104 F), blood pressure is
100/60 mm Hg, and pulse is 155/min. His skin is
warm and dry. Neurologic examination shows no
focal findings. Which of the following is the most
likely mechanism of this patient's condition?
A) Depletion of total body potassium
B) Depletion of total body sodium
C) Release of creatine kinase from muscle cells
D) Inadequate dissipation of body heat

533 A 72-year-old man with hypertension has had Osteoblastic lesions >> prostate mets MIGUEL RAFAEL MIDTERM 1
increasingly severe back pain over the past 2 RAMOS, MD (TOP 3 EXAM - FEB
months. He had a myocardial infarction 4 years ago. - FEB 2012 MED 2013
He has marked tenderness over T11, T12, L1, and BOARDS;
L2. An x-ray film of the lumbosacral spine shows TOPNOTCH MD)
osteoblastic lesions in these vertebrae. Which of the
following is the most likely diagnosis?
A) Abdominal aneurysm
B) Fibrosarcoma
C) Metastatic prostate carcinoma
D) Multiple myeloma

TOPNOTCH MEDICAL BOARD PREP INTERNAL MEDICINE SUPEREXAM Page 70 of 95


For inquiries visit www.topnotchboardprep.com.ph or email us at topnotchmedicalboardprep@gmail.com
TOPNOTCH MEDICAL BOARD PREP INTERNAL MEDICINE SUPEREXAM
For inquiries visit www.topnotchboardprep.com.ph or email us at topnotchmedicalboardprep@gmail.com
Item QUESTION EXPLANATION AUTHOR TOPNOTCH
# EXAM
534 A 50-year-old woman has had progressive dyspnea Echo to diagnose cardiac tamponade MIGUEL RAFAEL MIDTERM 1
over the past 2 weeks and constant, sharp chest pain RAMOS, MD (TOP 3 EXAM - FEB
for 4 days. The pain is localized to the center of the - FEB 2012 MED 2013
chest and is worse while supine. She underwent a BOARDS;
right, modified radical mastectomy and adjuvant TOPNOTCH MD)
chemotherapy for breast cancer 3 years ago. She has
a history of hypothyroidism treated with thyroid
replacement therapy. She has smoked one pack of
cigarettes daily for 30 years and drinks two ounces
of alcohol daily. She is dyspneic and diaphoretic. Her
temperature is 37.2 C (99 F), blood pressure is
90/70 mm Hg with a pulsus paradoxus of 20 mm Hg,
pulse is 110/min, and respirations are 28/min.
Examination shows jugular venous distention to the
angle of the mandible. The liver span is 14 cm with 4
cm of shifting abdominal dullness. Arterial blood gas
analysis on room air shows a pH of 7.50, PCO2 of 30
mm Hg, and PO2 of 70 mm Hg. An x-ray film of the
chest shows an enlarged cardiac silhouette with a
globular configuration. An ECG shows sinus
tachycardia with nonspecific ST-segment changes
diffusely. Which of the following is the most
appropriate next step in management?
A ) Echocardiography
B ) CT scan of the abdomen
C ) Ventilation-perfusion lung scans
D ) Bronchoscopy
535 A 52-year-old woman with alcoholism comes to the MIGUEL RAFAEL MIDTERM 1
physician after a serum cholesterol level of 290 RAMOS, MD (TOP 3 EXAM - FEB
mg/dL was found on a routine screening. She drinks - FEB 2012 MED 2013
a pint of vodka daily. She takes captopril for BOARDS;
hypertension and glyburide for type 2 diabetes TOPNOTCH MD)
mellitus. She also has intermittent episodes of gout.
Fasting serum studies show:

Total cholesterol 252 mg/dL
HDL-cholesterol 80 mg/dL
Triglycerides 300 mg/dL
Glucose 118 mg/dL
Thyroid-stimulating hormone 4.5 μU/mL

Which of the following is the most appropriate next
step in management?
A) Alcohol cessation
B) Thyroid replacement therapy
C) Switch from captopril to calcium-channel
blocking agent therapy
D) Gemfibrozil therapy

536 A 67-year-old man is brought to the emergency hints >> calcifications anterior to the vertebral MIGUEL RAFAEL MIDTERM 1
department 4 hours after the onset of severe bodies+ BP 105/65 mm Hg RAMOS, MD (TOP 3 EXAM - FEB
midlumbar back pain. He is anxious, pale, and - FEB 2012 MED 2013
diaphoretic. His temperature is 37.1 C (98.8 F), BOARDS;
blood pressure is 105/65 mm Hg, and pulse is TOPNOTCH MD)
120/min. Examination shows no other
abnormalities. X-ray films of the lumbar spine show
degenerative disc disease with calcifications anterior
to the vertebral bodies. Which of the following is the
most likely diagnosis?
A) Aortoiliac occlusion
B) Herniated nucleus pulposus
C) Lumbar discitis
D) Ruptured aortic aneurysm

537 A 24-year-old woman comes to the physician hint >> multiple tender spots over the neck, MIGUEL RAFAEL MIDTERM 1
because of constant, severe pain in her neck, shoulders, and lumbar spine RAMOS, MD (TOP 3 EXAM - FEB
shoulders, and back for 3 months. She has been - FEB 2012 MED 2013
unable to enjoy her usual activities because of the BOARDS;
pain. Use of over-the-counter ibuprofen and aspirin TOPNOTCH MD)
has not relieved her symptoms. She has a history of
irritable bowel syndrome. Examination shows
multiple tender spots over the neck, shoulders, and
lumbar spine. Range of motion of all joints is full.
There is no evidence of synovitis. Fluorescent serum
antinuclear antibody and rheumatoid factor assays
are negative. Which of the following is the most
likely diagnosis?
A) Ankylosing spondylitis
B) Fibromyalgia
C) Polymyalgia rheumatica
D) Polymyositis

TOPNOTCH MEDICAL BOARD PREP INTERNAL MEDICINE SUPEREXAM Page 71 of 95


For inquiries visit www.topnotchboardprep.com.ph or email us at topnotchmedicalboardprep@gmail.com
TOPNOTCH MEDICAL BOARD PREP INTERNAL MEDICINE SUPEREXAM
For inquiries visit www.topnotchboardprep.com.ph or email us at topnotchmedicalboardprep@gmail.com
Item QUESTION EXPLANATION AUTHOR TOPNOTCH
# EXAM
538 A 72-year-old man comes to the physician because ACEI show mortality benefit, ARBs do no MIGUEL RAFAEL MIDTERM 1
of a 6-month history of mild to moderate shortness RAMOS, MD (TOP 3 EXAM - FEB
of breath when climbing stairs. He had a myocardial - FEB 2012 MED 2013
infarction 2 years ago and has had an ejection BOARDS;
fraction of 35% since then. His only medication is a TOPNOTCH MD)
β-adrenergic blocking agent. The lungs are clear to
auscultation. Cardiac examination shows an S4
gallop. There is no peripheral edema. Laboratory
studies are within normal limits. An ECG shows no
acute changes. Which of the following is the most
appropriate pharmacotherapy?
A) α-Adrenergic blocking agent
B) Angiotensin-converting enzyme (ACE) inhibitor
C) Angiotensin2-receptor blocking agent
D) Nitrates

539 A 57-year-old man is brought to the emergency ECG changes with hyperkalemia >> immediate IV MIGUEL RAFAEL MIDTERM 1
department 30 minutes after he was found on the calcium gluconate RAMOS, MD (TOP 3 EXAM - FEB
floor of his house. He has renal failure but has - FEB 2012 MED 2013
missed his last two dialysis treatments. His renal BOARDS;
failure was caused by inadvertent ingestion of TOPNOTCH MD)
ethylene glycol. Medications include amlodipine and
doxazosin. On arrival, his temperature is 37.5 C
(99.5 F), blood pressure is 150/100 mm Hg, pulse is
95/min and regular, and respirations are 24/min.
His breathing is rapid and deep. Crackles are heard
in the lung bases. Laboratory studies show:

Na+ 135 mEq/L
Cl– 102 mEq/L
K+ 7.1 mEq/L
HCO3– 12 mEq/L
pH 7.22
PCO2 31 mm Hg
PO2 61 mm Hg

An ECG shows peaked T-waves. It will be at least 45
minutes before dialysis can be started. Which of the
following is the most appropriate next step in
management?
A) Intravenous sodium bicarbonate
B) Intravenous calcium gluconate
C) Intravenous glucose and insulin
D) Intravenous 0.9% saline

TOPNOTCH MEDICAL BOARD PREP INTERNAL MEDICINE SUPEREXAM Page 72 of 95


For inquiries visit www.topnotchboardprep.com.ph or email us at topnotchmedicalboardprep@gmail.com
TOPNOTCH MEDICAL BOARD PREP INTERNAL MEDICINE SUPEREXAM
For inquiries visit www.topnotchboardprep.com.ph or email us at topnotchmedicalboardprep@gmail.com
Item QUESTION EXPLANATION AUTHOR TOPNOTCH
# EXAM
540 A 26-year-old woman is brought to the emergency Addison'a Disease MIGUEL RAFAEL MIDTERM 1
department because of marked confusion for 2 RAMOS, MD (TOP 3 EXAM - FEB
hours; she also has had a flu-like illness for 3 days. - FEB 2012 MED 2013
Over the past 6 weeks, she has had increased fatigue, BOARDS;
weakness, and nausea. She recently started thyroid TOPNOTCH MD)
hormone replacement therapy for autoimmune
thyroiditis; 1 week ago, her serum thyroid-
stimulating hormone level was 3 μU/mL. Her
temperature is 38 C (100.4 F), blood pressure is
80/40 mm Hg, and pulse is 140/min. She appears
confused and lethargic. Examination shows cool,
mottled skin. There is generalized
hyperpigmentation, especially involving the palmar
creases. The lungs are clear to auscultation.
Abdominal examination shows diffuse mild
tenderness and no rebound. Laboratory studies
show:

Hemoglobin 10 g/dL
Leukocyte count 9000/mm3
Segmented neutrophils 55%
Eosinophils 20%
Lymphocytes 25%
Serum
Na+ 124 mEq/L
Cl– 92 mEq/L
K+ 6.4 mEq/L
HCO3– 16 mEq/L

An x-ray film of the chest and urinalysis show
normal findings. An ECG shows sinus tachycardia
with peaked T waves. Which of the following is most
likely to confirm the primary cause of this patient's
condition?
A) Measurement of serum antithyroglobulin
antibody level
B) Dexamethasone suppression test
C) Measurement of serum thyroid-stimulating
hormone level
D) ACTH stimulation test
541 A 45/M comes in with severe, prolonged substernal The rest are actions of nitrates. ABDELSIMAR FINAL EXAM -
chest pain associated with diaphoresis and nausea. OMAR II, MD (TOP FEB 2014
ECG reveals ST segment elevation in the anterior 2 - AUG 2013 MED
leads. Among other medications, you start him on BOARDS;
Metoprolol. What is the mechanism of action of this TOPNOTCH MD -
drug in the treatment of ischemia in myocardial 200 QUESTIONS)
infarction? AND MARC
A. Systemic venodilation with reduction in LV end- DENVER
diastolic volume TIONGSON, MD (40
B. Dilation of epicardial coronary vessels QUESTIONS)
C. Increased blood flow in collateral vessels
D. Reduction of myocardial O2 demand by inhibiting
increases in HR, BP and myocardial contractility
E. All of the above
542 Four months after an MI, a 45/M starts experiencing In heart failure, fluid restriction is generally ABDELSIMAR FINAL EXAM -
gradually worsening exertional dyspnea associated unnecessary unless with hyponatremia and OMAR II, MD (TOP FEB 2014
with 3-pillow orthopnea and paroxysmal nocturnal volume overload. Caloric supplementation is 2 - AUG 2013 MED
dyspnea. On PE, you note bibasilar rales on warranted for those with cardiac cachexia. BOARDS;
auscultation and Grade 2 bipedal edema. Which of Diuretics are only used to restore normovolemic TOPNOTCH MD -
the following are cornerstones of treatment? status; while digoxin is used only for symptomatic 200 QUESTIONS)
A. Fluid restriction LV dysfunction with concomitant AF as add on to AND MARC
B. Caloric supplementation standard therapy. DENVER
C. ACE inhibitors and beta blockers TIONGSON, MD (40
D. Diuretics and digoxin QUESTIONS)
E. All of the above

543 According to JNC7, which of the following lifestyle Weight reduction = 5 - 20 mmHg/10 kg ABDELSIMAR FINAL EXAM -
modification recommendations leads to the greatest OMAR II, MD (TOP FEB 2014
reduction in average systolic blood pressure? 2 - AUG 2013 MED
A. DASH eating plan BOARDS;
B. Dietary sodium restriction TOPNOTCH MD -
C. Moderation of alcohol consumption 200 QUESTIONS)
D. Weight reduction (10 kg) AND MARC
E. Aerobic physical activity DENVER
TIONGSON, MD (40
QUESTIONS)

TOPNOTCH MEDICAL BOARD PREP INTERNAL MEDICINE SUPEREXAM Page 73 of 95


For inquiries visit www.topnotchboardprep.com.ph or email us at topnotchmedicalboardprep@gmail.com
TOPNOTCH MEDICAL BOARD PREP INTERNAL MEDICINE SUPEREXAM
For inquiries visit www.topnotchboardprep.com.ph or email us at topnotchmedicalboardprep@gmail.com
Item QUESTION EXPLANATION AUTHOR TOPNOTCH
# EXAM
544 You are performing the chest PE on a patient who Emphysema and pneumothorax: hyperresonant, ABDELSIMAR FINAL EXAM -
came in for dyspnea. The left lung fields were dull on decreased. Pleural effusion: dull, decreased. OMAR II, MD (TOP FEB 2014
percussion while vocal fremitus was increased. This Asthma: hyper-resonant, normal 2 - AUG 2013 MED
is consistent with: BOARDS;
A. Emphysema TOPNOTCH MD -
B. Pneumothorax 200 QUESTIONS)
C. Pleural effusion AND MARC
D. Asthma DENVER
E. Consolidation TIONGSON, MD (40
QUESTIONS)
545 A 55/M heavy smoker comes in for cough In COPD, only 3 interventions influence the natural ABDELSIMAR FINAL EXAM -
productive of sputum and progressive and history: smoking cessation, lung volume reduction OMAR II, MD (TOP FEB 2014
persistent exertional dyspnea. On PE, you note surgery and oxygen therapy in the chronically 2 - AUG 2013 MED
hyperresonant lung fields and poor diagphragmatic hypoxemic patients. BOARDS;
excursion on percussion. Which of the following TOPNOTCH MD -
interventions have been demonstrated to influence 200 QUESTIONS)
the natural history of his condition? AND MARC
A. Smoking cessation DENVER
B. Treatment with inhaled corticosteroids TIONGSON, MD (40
C. Treatment with long acting beta agonists QUESTIONS)
D. Treatment with anticholinergics
E. All of the above

546 High levels of adenosine deaminase (>40 IU/L) in ABDELSIMAR FINAL EXAM -
pleural fluid is virtually diagnostic of: OMAR II, MD (TOP FEB 2014
A. Parapneumonic effusion 2 - AUG 2013 MED
B. Malignant pleural effusion BOARDS;
C. Effusion secondary to congestive heart failure TOPNOTCH MD -
D. Tuberculous pleural effusion 200 QUESTIONS)
E. Cirrhosis AND MARC
DENVER
TIONGSON, MD (40
QUESTIONS)
547 A 35/F, diagnosed case of SLE, comes in for a follow Lab data consistent with NORMAL ANION GAP ABDELSIMAR FINAL EXAM -
up. Routine labs done revealed the following results: METABOLIC ACIDOSIS. Only E causes NAGMA OMAR II, MD (TOP FEB 2014
ABG: pH 7.35, pCO2 34, pO2 of 98; Na 132; K 3.2; Cl among choices. Lupus is a known cause of type 1 2 - AUG 2013 MED
108; and HCO3 12. The patient's metabolic disorder RTA. BOARDS;
is likely due to: TOPNOTCH MD -
A. Uremia 200 QUESTIONS)
B. Diarrhea AND MARC
C. Diabetic ketoacidosis DENVER
D. Lactic acidosis TIONGSON, MD (40
E. Renal tubular acidosis QUESTIONS)
548 The presence of ECG changes in hyperkalemia Classic ECG findings: peaked T waves (5.5 - 6.5 ABDELSIMAR FINAL EXAM -
should be considered as an emergency and thus mM); loss of P waves (6.5 - 7.5); widened QRS (7 - OMAR II, MD (TOP FEB 2014
should be managed immediately. Which of the 8 mM); and sinusoidal pattern (>8 mM). 2 - AUG 2013 MED
following ECG abnormalities is characteristic of BOARDS;
hyperkalemia? TOPNOTCH MD -
A. Prolonged PR interval 200 QUESTIONS)
B. Presence of U waves AND MARC
C. ST segment depression DENVER
D. Loss of P waves TIONGSON, MD (40
E. Shortened QT interval QUESTIONS)
549 A 25/M, recently diagnosed case of type 1 DM, Usually, DM nephropathy develops after 10 years ABDELSIMAR FINAL EXAM -
comes in for routine follow up after he noted passing duration; occurring in the presence of other OMAR II, MD (TOP FEB 2014
foamy urine. You order a urinalysis which reveals 3+ microvascular complications, e.g. retinopathy. 2 - AUG 2013 MED
proteinuria. You refer the patient to an PEARL: Presence of nephropathy in diabetic in the BOARDS;
Ophthalmologist who found no evidence of absence of eye disease should warrant TOPNOTCH MD -
retinopathy. The most likely cause of proteinuria in investigation for other causes of nephropathy. The 200 QUESTIONS)
the patient is: most common cause of nephrotic syndrome in AND MARC
A. Diabetic nephropathy adults is MGN; the most common cause in kids is DENVER
B. Hypertensive kidney disease MCD. TIONGSON, MD (40
C. Minimal change disease QUESTIONS)
D. Membranous glomerulonephritis
E. Focal segmental glomerulosclerosis

550 You are assessing glycemic control in a diabetic Agents which target PPG in particular include ABDELSIMAR FINAL EXAM -
patient who does self monitoring of blood glucose. meglitinides, incretin-related drugs (sitagliptin, OMAR II, MD (TOP FEB 2014
You note that the patient's post-prandial glucose saxagliptin) and a-glucosidase inhibitors 2 - AUG 2013 MED
levels are elevated. Which of the following (acarbose and miglitol.) BOARDS;
hypoglycemic agents target post-prandial glucose in TOPNOTCH MD -
particular? 200 QUESTIONS)
A. Metformin AND MARC
B. Glibenclamide DENVER
C. Pioglitazone TIONGSON, MD (40
D. Miglitol QUESTIONS)
E. All of the above
551 What is the MOST COMMON sign of SIMILAR TO PREVIOUS BOARD EXAM ABDELSIMAR FINAL EXAM -
hyperthyroidism? CONCEPT/PRINCIPLE. The most common OMAR II, MD (TOP FEB 2014
A. Tremor SYMPTOM is hyperactivity, irritability and 2 - AUG 2013 MED
B. Goiter dysphoria. BOARDS;
C. Warm, moist skin TOPNOTCH MD -
D. Tachycardia 200 QUESTIONS)
E. Lid lag AND MARC
DENVER

TOPNOTCH MEDICAL BOARD PREP INTERNAL MEDICINE SUPEREXAM Page 74 of 95


For inquiries visit www.topnotchboardprep.com.ph or email us at topnotchmedicalboardprep@gmail.com
TOPNOTCH MEDICAL BOARD PREP INTERNAL MEDICINE SUPEREXAM
For inquiries visit www.topnotchboardprep.com.ph or email us at topnotchmedicalboardprep@gmail.com
Item QUESTION EXPLANATION AUTHOR TOPNOTCH
# EXAM
TIONGSON, MD (40
QUESTIONS)

552 A 28/F with Cushingoid features complained of A high-dose dexamethasone suppression test can ABDELSIMAR FINAL EXAM -
blurring of vision. Formal visual field testing differentiate pituitary Cushing's syndrome OMAR II, MD (TOP FEB 2014
revealed bitemporal hemianopsia. You suspect a (cortisol is suppressed) from adrenal/ectopic 2 - AUG 2013 MED
functioning pituitary adenoma. Which of the Cushing's syndrome (cortisol is NOT suppressed). BOARDS;
following laboratory findings is NOT consistent with TOPNOTCH MD -
your primary impression? 200 QUESTIONS)
A. Elevated serum cortisol AND MARC
B. Elevated urine free cortisol DENVER
C. Cortisol level not suppressed with low-dose TIONGSON, MD (40
dexamethasone QUESTIONS)
D. Cortisol level not suppressed with high-dose
dexamethasone
E. None of the above
553 After a night of binge-drinking, a 53/M comes in to Classic pancreatitis. Prophylactic antibiotics have ABDELSIMAR FINAL EXAM -
the ER complaining of sudden onset severe mid NO role in either interstitial or necrotizing OMAR II, MD (TOP FEB 2014
epigastric abdominal pain radiating to the back; pancreatitis 2 - AUG 2013 MED
associated with nausea, vomiting and anorexia. Vital BOARDS;
signs are as follows: BP 90/50, HR 114, RR 24, T TOPNOTCH MD -
39C. Cornerstones for the management of this 200 QUESTIONS)
condition include the following except: AND MARC
A. Analgesivs DENVER
B. IV hydration TIONGSON, MD (40
C. No oral alimentation (NPO) QUESTIONS)
D. Prophylactic antibiotics
E. None of the above

554 A 28/M who presents with fever and jaundice has ABDELSIMAR FINAL EXAM -
the following hepatitis profile: IgM Anti-HAV (+); OMAR II, MD (TOP FEB 2014
HBsAg (+); IgM Anti-HBc (-); and Anti-HCV (-). He 2 - AUG 2013 MED
has: BOARDS;
A. Acute hepatitis A TOPNOTCH MD -
B. Acute hepatitis B 200 QUESTIONS)
C. Acute hepatitis A and B AND MARC
D. Acute hepatitis A superimposed on chronic DENVER
hepatitis B TIONGSON, MD (40
E. Acute hepatitis C QUESTIONS)
555 The following are recommendations of the Surviving Start with norepinephrine and dopamine. ABDELSIMAR FINAL EXAM -
Sepsis Guidelines in the management of sepsis, OMAR II, MD (TOP FEB 2014
EXCEPT: 2 - AUG 2013 MED
A. Start IV fluids to maintain CVP > 12 mmHg in BOARDS;
mechanically ventilated patients. TOPNOTCH MD -
B. Maintain MAP above 65 mmHg. 200 QUESTIONS)
C. Epinephrine and dopamine are the initial AND MARC
vasopressors of choice. DENVER
D. Start with broad-spectrum IV antibiotics with TIONGSON, MD (40
adequate penetration. QUESTIONS)
E. Hydrocortisone may be indicated for adult septic
shock when hypotension responds poorly to
sufficient fluid resuscitation and vasopressors.
556 A 12/M was attacked by a stray dog and sustained Patient had a Category II exposure (minor ABDELSIMAR FINAL EXAM -
minor abrasions without bleeding. The patient has abrasions without bleeding). Start vaccine only. OMAR II, MD (TOP FEB 2014
had no rabies vaccination. Which of the following is 2 - AUG 2013 MED
MOST appropriate? BOARDS;
A. Wash exposed skin with immediately with soap TOPNOTCH MD -
and water. No vaccine or RIG is needed. 200 QUESTIONS)
B. Administer rabies immunoglobulin immediately. AND MARC
C. Start rabies vaccination and rabies DENVER
immunoglobulin. TIONGSON, MD (40
D. Start rabies vaccination without rabies QUESTIONS)
immunoglobulin.
E. Start rabies vaccination, rabies immunoglobulin,
and oral antibiotics.
557 A 40/M complains of excruciating pain and swelling Do NOT start hypouricemic therapy during acute ABDELSIMAR FINAL EXAM -
in his left big toe that developed suddenly. He is attacks of gout. OMAR II, MD (TOP FEB 2014
obese and is a heavy beer drinker. On PE, you note 2 - AUG 2013 MED
marked swelling, redness, and warmth of the left big BOARDS;
toe. You may give all of the following for acute TOPNOTCH MD -
attacks except: 200 QUESTIONS)
A. NSAIDs AND MARC
B. Glucocorticoids DENVER
C. Colchicine TIONGSON, MD (40
D. Allopurinol QUESTIONS)
E. Ice compress

TOPNOTCH MEDICAL BOARD PREP INTERNAL MEDICINE SUPEREXAM Page 75 of 95


For inquiries visit www.topnotchboardprep.com.ph or email us at topnotchmedicalboardprep@gmail.com
TOPNOTCH MEDICAL BOARD PREP INTERNAL MEDICINE SUPEREXAM
For inquiries visit www.topnotchboardprep.com.ph or email us at topnotchmedicalboardprep@gmail.com
Item QUESTION EXPLANATION AUTHOR TOPNOTCH
# EXAM
558 Which of the following autoantibodies is SLE-specific C is highly specific but has NO definite clinical ABDELSIMAR FINAL EXAM -
and correlates with disease activity, nephritis and correlations. A is the best screening test. D is OMAR II, MD (TOP FEB 2014
vasculitis? nonspecific for SLE and associated with sicca 2 - AUG 2013 MED
A. ANA syndrome, and neonatal lupus. E is associated BOARDS;
B. Anti-dsDNA more with drug-induced lupus. TOPNOTCH MD -
C. Anti-Sm 200 QUESTIONS)
D. Anti-Ro AND MARC
E. Antihistone DENVER
TIONGSON, MD (40
QUESTIONS)
559 Anemia with a reticulocyte index > 2.5 is consistent RI > 2.5 suggests hemolysis or hemorrhage. ABDELSIMAR FINAL EXAM -
with: OMAR II, MD (TOP FEB 2014
A. Iron deficiency anemia 2 - AUG 2013 MED
B. Thalassemia BOARDS;
C. Hereditary spherocytosis TOPNOTCH MD -
D. Anemia of chronic disease 200 QUESTIONS)
E. Bone marrow infiltration AND MARC
DENVER
TIONGSON, MD (40
QUESTIONS)
560 Platelet count and bleeding time within normal ABDELSIMAR FINAL EXAM -
limits, associated with prolonged PT and PTT, is OMAR II, MD (TOP FEB 2014
classically seen in: 2 - AUG 2013 MED
A. Immune thrombocytopenic pupura BOARDS;
B. Bernard-Soulier syndrome TOPNOTCH MD -
C. von Willebrand disease 200 QUESTIONS)
D. Hemophilia AND MARC
E. Vitamin K deficiency DENVER
TIONGSON, MD (40
QUESTIONS)
561 Obesity is the driving force behind the metabolic Obesity is the driving force behind the metabolic BLAKE WARREN MIDTERM 2
syndrome, thus, the primary approach to this syndrome. Thus, weight reduction is the primary ANG, MD (TOP 1 - EXAM - FEB
disorder is/are approach to the disorder. AUG 2013 MED 2014
A. Treatment of dyslipidemia With weight reduction, the improvement in insulin BOARDS;
B. ACE-inhibitors sensitivity is often accompanied by favorable TOPNOTCH MD)
C. Smoking cessation modifications in many components of the
D. Weight reduction metabolic syndrome.
In general, recommendations for weight loss
include a combination of caloric restriction,
increased physical activity, and behavior
modification
562 The drug of choice for lowering fasting triglycerides A fibrate (gemfibrozil or fenofibrate) is the drug of BLAKE WARREN MIDTERM 2
A. Gemfibrozil choice to lower fasting triglycerides and typically ANG, MD (TOP 1 - EXAM - FEB
B. Atorvastatin achieve a 35–50% reduction AUG 2013 MED 2014
C. Nicotinic Acid BOARDS;
D. Ezetimibe TOPNOTCH MD)

563 Indications for coronary arteriography EXCEPT: Answer: Coronary arteriography is indicated in: BLAKE WARREN MIDTERM 2
(HPIM 18, p2005) (1) patients with chronic stable angina pectoris ANG, MD (TOP 1 - EXAM - FEB
A. Patients with known or possible angina pectoris who are severely symptomatic despite medical AUG 2013 MED 2014
who have survived cardiac arrest therapy and are being considered for BOARDS;
B. Patients with chronic stable angina pectoris who revascularization, i.e., a percutaneous coronary TOPNOTCH MD)
are severely symptomatic despite medical therapy intervention (PCI) or coronary artery bypass
and who are being considered for revascularization grafting (CABG),
C. Patients with troublesome symptoms that present (2) patients with troublesome symptoms that
diagnostic difficulties in whom there is a need to present diagnostic difficulties in whom there is a
confirm or rule out IHD need to confirm or rule out the diagnosis of IHD,
D. Patients with no evidence of ischemia on non- (3) patients with known or possible angina
invasive testing and no clinical or laboratory pectoris who have survived cardiac arrest,
evidence of ventricular dysfunction (4) patients with angina or evidence of ischemia
on noninvasive testing with clinical or laboratory
evidence of ventricular dysfunction, and
(5) patients judged to be at high risk of sustaining
coronary events based on signs of severe ischemia
on noninvasive testing, regardless of the presence
or severity of symptoms

564 At least how many hours each day must a patient be Different preparations and/or administration BLAKE WARREN MIDTERM 2
kept nitrate-free to minimize tolerance and restore during the daytime should be tried only to prevent ANG, MD (TOP 1 - EXAM - FEB
any useful response (HPIM 18, p2010) discomfort while avoiding side effects such as AUG 2013 MED 2014
A. 4 hours headache and dizziness. Individual dose titration BOARDS;
B. 6 hours is important to prevent side effects. To minimize TOPNOTCH MD)
C. 8 hours the effects of tolerance, the minimum effective
D.12 hours dose should be used and a minimum of 8 h each
day kept free of the drug to restore any useful
response(s).
565 Mechanism of action of thienopyridines (HPIM 18, Inhibition of P2Y12 ADP Receptor BLAKE WARREN MIDTERM 2
p2011, 2017) ANG, MD (TOP 1 - EXAM - FEB
A. Inhibition of thromboxane synthesis AUG 2013 MED 2014
B. Inhibition of the IIB/IIIA receptor BOARDS;
C. Inhibition of clotting factor Xa TOPNOTCH MD)
D. Inhibition of P2Y12 ADP receptor

TOPNOTCH MEDICAL BOARD PREP INTERNAL MEDICINE SUPEREXAM Page 76 of 95


For inquiries visit www.topnotchboardprep.com.ph or email us at topnotchmedicalboardprep@gmail.com
TOPNOTCH MEDICAL BOARD PREP INTERNAL MEDICINE SUPEREXAM
For inquiries visit www.topnotchboardprep.com.ph or email us at topnotchmedicalboardprep@gmail.com
Item QUESTION EXPLANATION AUTHOR TOPNOTCH
# EXAM
566 NOT an indication for discontinuation of stress Answer: Exercise duration is usually symptom- BLAKE WARREN MIDTERM 2
testing: limited, and the test is discontinued upon evidence ANG, MD (TOP 1 - EXAM - FEB
A. Chest discomfort of chest discomfort, severe shortness of breath, AUG 2013 MED 2014
B. Ventricular tachyarrhythmia dizziness, severe fatigue, ST-segment depression BOARDS;
C. Fall in systolic blood pressure <10 mmHg >0.2 mV (2 mm), a fall in systolic blood pressure TOPNOTCH MD)
D. Dizziness >10 mmHg, or the development of a ventricular
tachyarrhythmia.

567 NOT a contraindication to exercise stress testing: Contraindications to exercise stress testing BLAKE WARREN MIDTERM 2
(HPIM 18, p2004) include rest angina within 48 h, unstable rhythm, ANG, MD (TOP 1 - EXAM - FEB
A. Severe pulmonary hypertension severe aortic stenosis, acute myocarditis, AUG 2013 MED 2014
B. Severe aortic regurgitation uncontrolled heart failure, severe pulmonary BOARDS;
C. Uncontrolled heart failure hypertension, and active infective endocarditis. TOPNOTCH MD)
D. Active infective endocarditis

568 The most common clinical indication for The most common clinical indication for PCI is BLAKE WARREN MIDTERM 2
Percutaneous Coronary Intervention symptom-limiting angina pectoris, despite medical ANG, MD (TOP 1 - EXAM - FEB
a. Decompensated Heart Failure therapy, accompanied by evidence of ischemia AUG 2013 MED 2014
b. Ventricular Tachyarrythmias during a stress test. PCI is more effective than BOARDS;
c. Symptom – limiting Angina Pectoris medical therapy for the relief of angina. PCI TOPNOTCH MD)
d. Cardiogenic shock improves outcomes in patients with unstable
angina or when used early in the course of
myocardial infarction with and without
cardiogenic shock.
569 After verification of precocious pubertal Answer: To determine if it is gonadotropin BLAKE WARREN MIDTERM 2
development, which of the following lab tests should dependent or independent ANG, MD (TOP 1 - EXAM - FEB
first be measured? • Gonadotropin Dependent AUG 2013 MED 2014
A. Serum LH and FSH Gonadotropins increased in relation to BOARDS;
B. Serum testosterone chronologic age TOPNOTCH MD)
C. Serum DHEA S CNS lesions, idiopathic central precocity
D. Serum B-HCG • Gonadotropin Indenpendent
High testosterone/suppressed LH
CAH, adrenal neoplasms, testicular neoplasms

570 Most important step in the evaluation of male Answer: Most important step in the evaluation of BLAKE WARREN MIDTERM 2
infertility male infertility ANG, MD (TOP 1 - EXAM - FEB
A. Serum testosterone • Normal ejaculate AUG 2013 MED 2014
B. HCG stimulation test Volume 2-6 mL BOARDS;
C. Semen Analysis Sperm count of >20million/mL TOPNOTCH MD)
D. Testicular biopsy Motility >50%
>15% normal morphology

571 What malignancy is associated with a mutated RET BLAKE WARREN MIDTERM 2
proto-oncogene? ANG, MD (TOP 1 - EXAM - FEB
A. Follicular carcinoma AUG 2013 MED 2014
B. Medullary carcinoma BOARDS;
C. Parathyroid carcinoma TOPNOTCH MD)
D. Papillary carcinoma

572 Agents proven to reduce the risk of fractures in Answer: Only bisphosphonates have been BLAKE WARREN MIDTERM 2
patients being treated with glucocorticoids demonstrated in large clinical trials to reduce the ANG, MD (TOP 1 - EXAM - FEB
risk of fractures in patients being treated with AUG 2013 MED 2014
a. Bisphosphonates glucocorticoids. Risedronate prevents bone loss BOARDS;
b. Selective estrogen response modulators (SERMs) and reduces vertebral fracture risk by ~70%. TOPNOTCH MD)
c. Calcium supplements
d. Hormone replacement therapy

573 Substance that is produced by macrophages in Answer: In patients with sarcoidosis and other BLAKE WARREN MIDTERM 2
chronic granulomatous diseases (tuberculosis, granulomatous diseases, such as tuberculosis and ANG, MD (TOP 1 - EXAM - FEB
sarcoidosis) that causes hypercalcemia fungal infections, excess 1,25(OH)2D is AUG 2013 MED 2014
a. PTHrP synthesized in macrophages or other cells in the BOARDS;
b. 1,25(OH)2D granulomas. TOPNOTCH MD)
c. PTH
d. 25(OH)D

574 First line anti-epileptic drug for petit mal and grand ANSWER: first line drugs are as follows BLAKE WARREN MIDTERM 2
mal seizures (HPIM p2507 table 363-8) • Primary Generalized Tonic-Clonic ANG, MD (TOP 1 - EXAM - FEB
A. Phenytoin Valproic Acid AUG 2013 MED 2014
B. Phenobarbital Lamotrigine BOARDS;
C. Levetiracetam Topiramate TOPNOTCH MD)
D. Valproic acid • Absence Seizures
Valproic Acid
Ethosuximide

TOPNOTCH MEDICAL BOARD PREP INTERNAL MEDICINE SUPEREXAM Page 77 of 95


For inquiries visit www.topnotchboardprep.com.ph or email us at topnotchmedicalboardprep@gmail.com
TOPNOTCH MEDICAL BOARD PREP INTERNAL MEDICINE SUPEREXAM
For inquiries visit www.topnotchboardprep.com.ph or email us at topnotchmedicalboardprep@gmail.com
Item QUESTION EXPLANATION AUTHOR TOPNOTCH
# EXAM
575 Acute nephrotic syndrome associated with hepatitis MGN and MPGN are causes of NEPHROTIC BLAKE WARREN MIDTERM 2
B and C infections: SYNDROME. IgA Neph causes SSx of both. ANG, MD (TOP 1 - EXAM - FEB
A. Mesangioproliferative GN Mesangioproliferative GN is type II lupus AUG 2013 MED 2014
B. Membranoproliferative GN nephritis. MGN and MPGN are both associated BOARDS;
C. IgA Nephropathy with hep B and C. TOPNOTCH MD)
D. MCD
576 The most common cause of nephrotic syndrome in BLAKE WARREN MIDTERM 2
the elderly ANG, MD (TOP 1 - EXAM - FEB
A. Focal segmental glomerulosclerosis AUG 2013 MED 2014
B. Membranous glomerulonephritis BOARDS;
C. Diabetic nephropathy TOPNOTCH MD)
D. Minimal change disease
577 Kimmelstiel-Wilson nodules are seen in: Answer: Some patients (with Diabetic BLAKE WARREN MIDTERM 2
A. Fabry’s disease nephropathy) also develop eosinophilic, PAS+ ANG, MD (TOP 1 - EXAM - FEB
B. Focal segmental glmerulosclerosis nodules called nodular glomerulosclerosis or AUG 2013 MED 2014
C. Diabetic Nephropathy Kimmelstiel-Wilson nodules. BOARDS;
D. Membranous Glomerulonephritis TOPNOTCH MD)

578 Hematuria, thinning and splitting of the GBMs, mild BLAKE WARREN MIDTERM 2
proteinuria, chronic glomerulosclerosis leading to ANG, MD (TOP 1 - EXAM - FEB
renal failure and Sensorineural deafness is seen in: AUG 2013 MED 2014
A. Anti-GBM disease BOARDS;
B. Alport’s Syndrome TOPNOTCH MD)
C. Thin Basement Membrane disease
D. Nail-Patella Syndrome

579 Sclerotherapy is a treatment option for which stage III BLAKE WARREN MIDTERM 2
of hemorrhoidal disease? ANG, MD (TOP 1 - EXAM - FEB
a. Stage I – enlargement with bleeding AUG 2013 MED 2014
b. Stage II – protrusion with spontaneous reduction BOARDS;
c. Stage III – protrusion requiring manual reduction TOPNOTCH MD)
d. Stage IV – irreducible protrusion

580 After a person is infected with HBV, the first BLAKE WARREN MIDTERM 2
virologic marker detectable in serum within 1-12 ANG, MD (TOP 1 - EXAM - FEB
weeks is (H-18, C-304, P-2540): AUG 2013 MED 2014
a) HBeAg BOARDS;
b) HBsAg TOPNOTCH MD)
c) HBcAg
d) Anti-HBc IgM

581 Finkelstein's test is used to diagnose what Prayer test and Tinel test is for carpal tunnel TIMOTHY TANG MIDTERM 1
condition? LEE SAY, MD (TOP EXAM - FEB
A. De Quervain tenosynovitis 4 - AUG 2013 MED 2014
B. Ulnar nerve palsy BOARDS;
C. Compartment syndrome TOPNOTCH MD)
D. Carpal tunnel syndrome
E. Osteoarthritis of the hands

582 A patient with meningitis suddenly went to coma. One of the treatment for ICP increase is inducing a TIMOTHY TANG MIDTERM 1
The cause is? state of coma. The most likely cause for a patient LEE SAY, MD (TOP EXAM - FEB
A. Sepsis with meningitis is sepsis or bacterial seeding 4 - AUG 2013 MED 2014
B. Increased ICP resulting in a depressed brain. BOARDS;
C. Pulmonary embolism TOPNOTCH MD)
D. Hyperthermia
E. Subarachnoid hemorrhage

583 Risk factors associated with the development of Obesity prevents osteoporosis. (Pre test questions TIMOTHY TANG MIDTERM 1
osteoporosis include all the following EXCEPT? may appear in IM, no. 270, Pretest 7th ed) LEE SAY, MD (TOP EXAM - FEB
A. Excessive alcohol intake 4 - AUG 2013 MED 2014
B. Obesity BOARDS;
C. Genetics TOPNOTCH MD)
D. Poor calcium intake during adolescence
E. Postmenopausal state

584 Which antibodies are associated with Anti-Sm - specific for SLE TIMOTHY TANG MIDTERM 1
neuropsychiatric lupus? Ro and La - neonatal lupus LEE SAY, MD (TOP EXAM - FEB
A. Anti-Sm nRNP - mixed connective tissue disease 4 - AUG 2013 MED 2014
B. Anti-Ro and Anti-La anti-centromere - scleroderma BOARDS;
C. Anti-nRNP TOPNOTCH MD)
D. Anti-ribosomal
E. Anti-centromere

585 What is the standard immunization schedule for The schedule for initial vaccination in adults is TIMOTHY TANG MIDTERM 1
primary Hepatitis B vaccination among adults? similar to children. 0, 1 (or 2) and 6 months. LEE SAY, MD (TOP EXAM - FEB
A. 0, 1 and 6 months from initial vaccination 4 - AUG 2013 MED 2014
B. 0, 1 and 2 months (4 weeks interval) BOARDS;
C. 0, 6 and 12 months (6 months interval) TOPNOTCH MD)
D. 0, 1 and 2 months plus a booster dose after 1
year of the last dose
E. 0, 2 months (2 doses 6-8 weeks apart)

TOPNOTCH MEDICAL BOARD PREP INTERNAL MEDICINE SUPEREXAM Page 78 of 95


For inquiries visit www.topnotchboardprep.com.ph or email us at topnotchmedicalboardprep@gmail.com
TOPNOTCH MEDICAL BOARD PREP INTERNAL MEDICINE SUPEREXAM
For inquiries visit www.topnotchboardprep.com.ph or email us at topnotchmedicalboardprep@gmail.com
Item QUESTION EXPLANATION AUTHOR TOPNOTCH
# EXAM
586 An 18 year old male complains of polyuria. What is The most common cause of polyuria in both adults TIMOTHY TANG MIDTERM 1
the most likely diagnosis? and children is uncontrolled diabetes mellitus, LEE SAY, MD (TOP EXAM - FEB
A. Diabetes Milletus causing an osmotic diuresis. In the absence of 4 - AUG 2013 MED 2014
B. Diabetes Insipidus diabetes mellitus, the most common causes are BOARDS;
C. Primary polydipsia primary polydipsia (excessive fluid drinking), TOPNOTCH MD)
D. Renal tubular necrosis central diabetes insipidus and nephrogenic
E. Idiopathic polyuria diabetes insipidus.

587 Uncontrolled hypertension not amenable to lifestyle The first line for hypertension withour compelling TIMOTHY TANG MIDTERM 1
modifications is treated with? indications after lifestyle modification is a thiazide LEE SAY, MD (TOP EXAM - FEB
A. Calcium channel blocker diuretic based on JNC 7. 4 - AUG 2013 MED 2014
B. Thiazide diuretic BOARDS;
C. ACE Inhibitor TOPNOTCH MD)
D. β-blocker
E. Central acting adrenergic agents

588 In the setting of myocardial infarction, what is the The first biomarker to be elevated is myoglobin. TIMOTHY TANG MIDTERM 1
first cardiac biomarker to be detected? The most specific for MI is troponin I or T. LEE SAY, MD (TOP EXAM - FEB
A. Troponin I 4 - AUG 2013 MED 2014
B. CK-MB BOARDS;
C. Total CK TOPNOTCH MD)
D. LDH
E. Myoglobin

589 A systolic murmur is detected upon ausculatation of An enlarged and pulsatile liver is virtually TIMOTHY TANG MIDTERM 1
a heart failure patient. Prominent findings include an diagnostic of Tricuspid Insufficiency. LEE SAY, MD (TOP EXAM - FEB
enlarged and pulsatile liver. What is the heart valve 4 - AUG 2013 MED 2014
defect? BOARDS;
A. Aortic stenosis TOPNOTCH MD)
B. Ventral septal defect
C. Tricuspid insufficiecny
D. Mitral regurgitation
E. Pulmonary stenosis

590 The goal of BP reduction in a diabetic patient is? <130/80 is the goal of aggressive BP lowering for TIMOTHY TANG MIDTERM 1
A. <140/90 patients with diabetes or chronic kidney disease LEE SAY, MD (TOP EXAM - FEB
B. <130/90 based on the ACCF. 4 - AUG 2013 MED 2014
C. <130/80 BOARDS;
D. <120/80 TOPNOTCH MD)
E. <110/80

591 Which tissue in the body is LEAST affected by The CNS is made up of permanet neural cells and TIMOTHY TANG MIDTERM 1
radiation? labile glial cells and they need 50 Gy of radiation LEE SAY, MD (TOP EXAM - FEB
A. Intestinal lining before they are damaged. Lymphocytes need 1 Gy. 4 - AUG 2013 MED 2014
B. Lymphocytes Skin, about 10 Gy. Bone is also damaged by BOARDS;
C. Bone absorption of radiation and myelosuppression. TOPNOTCH MD)
D. CNS
E. Skin

592 Which of the following interventions increase Smoking cessation, oxygen therapy and limited TIMOTHY TANG MIDTERM 1
survival in COPD patients? lung resection have been proven to increase LEE SAY, MD (TOP EXAM - FEB
A. Smoking cessation survival in COPD patients. 4 - AUG 2013 MED 2014
B. Corticosteroids BOARDS;
C. β adrenergic agonist TOPNOTCH MD)
D. Pneumococcal vaccination
E. All of the above

593 A 20 year old previously healthy female suddenly Hemiplegic migraine and not atherosclerotic TIMOTHY TANG MIDTERM 1
have blurring of vision and hemiplegia of the right lesions (TIA, stroke) are responsible for LEE SAY, MD (TOP EXAM - FEB
arms and legs that lasted for 3 hours. What is the temporaray hemiparesis in a normal individual 4 - AUG 2013 MED 2014
likely diagnosis? esp females. Seizures do not give rise to weakness BOARDS;
A. TIA that last for hours. TOPNOTCH MD)
B. Mild stroke
C. Migraine
D. Partial seizure
E. Intracranial aneurysm

594 What is the initial imaging modality of choice for Computed tomography angiography (CTA) is the TIMOTHY TANG MIDTERM 1
patients suspected of having pulmonary embolism? initial imaging modality of choice for stable LEE SAY, MD (TOP EXAM - FEB
A. Ventillation-perfusion scan (V/Q scan) patients with suspected pulmonary embolism. The 4 - AUG 2013 MED 2014
B. Computed tomography angiography (CTA) American College of Radiology (ACR) considers BOARDS;
C. Pulmonary angiography chest CTA to be the current standard of care for TOPNOTCH MD)
D. Magnetic resonance angiography (MRA) the detection of pulmonary embolism. Pulmonary
E. Chest radiograph (PA and Lateral) angiography is the gold standard.

TOPNOTCH MEDICAL BOARD PREP INTERNAL MEDICINE SUPEREXAM Page 79 of 95


For inquiries visit www.topnotchboardprep.com.ph or email us at topnotchmedicalboardprep@gmail.com
TOPNOTCH MEDICAL BOARD PREP INTERNAL MEDICINE SUPEREXAM
For inquiries visit www.topnotchboardprep.com.ph or email us at topnotchmedicalboardprep@gmail.com
Item QUESTION EXPLANATION AUTHOR TOPNOTCH
# EXAM
595 A 65 year old male, non-smoker, with a chronic A patient with a chronic history of GERD can be a TIMOTHY TANG MIDTERM 1
history of gastric esophageal reflux disease consults risk factor for development of esophageal LEE SAY, MD (TOP EXAM - FEB
your clinic for dysphagia. The diagnosis is? adenocarcinoma esp if Barrett esophagus is 4 - AUG 2013 MED 2014
A. Squamous cell carcinoma diagnosed. BOARDS;
B. Adenocarcinoma TOPNOTCH MD)
C. MALT Lymphoma
D. Linitis plastica
E. Hypertrophic pyloric stenosis

596 In the chest radiograph of a 22 year old male patient You do not give amoxicillin for Mycoplasma TIMOTHY TANG MIDTERM 1
shows multiple infilttrates inconsisitent with the pneumoniae or atypical pneumonis since they are LEE SAY, MD (TOP EXAM - FEB
normal PE findings and the only complain of not effective because Mycoplasma does not have 4 - AUG 2013 MED 2014
occasional cough. What antibiotic will you AVOID cell walls. BOARDS;
giving this patient? TOPNOTCH MD)
A. Azithromycin
B. Amoxicillin
C. Erythromycin
D. Ciprofloxacin
E. Doxycycline
597 Which of the following is a risk factor for breast Nulliparity, early menarche, late menopause and TIMOTHY TANG MIDTERM 1
cancer? history of a first degree relative with breast cancer LEE SAY, MD (TOP EXAM - FEB
A. Fibrocystic disease are all risk factors for breast cancer. Fibrocystic 4 - AUG 2013 MED 2014
B. First-degree relative with breast cancer disease is not a risk factor. BOARDS;
C. Late menarche TOPNOTCH MD)
D. Multiple pregnancies
E. Early menopause

598 Magnetic resonance imaging is more sensitive than Diagnosing hemorrhage and bony fractures are TIMOTHY TANG MIDTERM 1
computed tomography in the diagnosis of which of better in CT. TIA and Seizure disorders cannot be LEE SAY, MD (TOP EXAM - FEB
the following? diagnose by imaging. Early cerebral infarction is 4 - AUG 2013 MED 2014
A. Intracranial hemorrhage detectable in MRI but not CT. BOARDS;
B. Transient ischemic attack TOPNOTCH MD)
C. Generalized tonic-clonic seizures
D. Early cerebral infarction
E. Skull fracture

599 What is the most common trigger in an asthmatic The presence of URTI is the most common cause of TIMOTHY TANG MIDTERM 1
attack? exacerbations for Asthmatic and COPD patients. LEE SAY, MD (TOP EXAM - FEB
A. Failure of intake of medications All of the choices can possibly trigger an asthma 4 - AUG 2013 MED 2014
B. Excitement attack. BOARDS;
C. Upper respiratory tract infection TOPNOTCH MD)
D. Air pollution
E. Strenuous activities

600 The pathophysiology of which disease CANNOT be AIDS is an infection and cannot be explain by the TIMOTHY TANG MIDTERM 1
explain by the Hygiene Hypothesis? hygiene hypothesis. The hygiene hypothesis states LEE SAY, MD (TOP EXAM - FEB
A. AIDS that a lack of early childhood exposure to 4 - AUG 2013 MED 2014
B. Allergy infectious agents, symbiotic microorganisms, and BOARDS;
C. Asthma parasites increases susceptibility to allergic TOPNOTCH MD)
D. Autoimmunde disease diseases by suppressing natural development of
E. ALL the immune system.

601 A 27-year old male is brought to the ER due to The small, raised yellowish leions around the eyes RACHELLE FINAL EXAM -
severe chest pain radiating to his left arm, with and volar surfaces of his arms are xanthomas. MENDOZA, MD FEB 2013
associated body weakness and diaphoresis. PE Biopsy of these will show lipid-laden macrophages (TOP 9 - AUG 2012
revealed small, raised yellowish lesions around the (foam cells). In this condition, there is a low MED BOARDS;
eye and on volar surfaces of both arms. ECG number of functioning LDL receptors, hence LDL TOPNOTCH MD)
revealed ST elevations. His brother died at the age of accumulates within the blood strean and gets
24 due to a heart problem. What is the underlying deposited on different tissues, such as
genetic condition of this patient? endothelium (may ccause blockage, in this case
A. Li-Fraumeni syndrome MI) and subcutaneous tissue.
B. Autosomal dominant polycystic kidney disease
C. Neurofibromatosis type 1
D. Familial hypercholesterolemia
E. Factor V Leiden Mutation

602 A 45 year old male, a chronic alcoholic, is brought to Treatment for hepatic enceph include: aminoleban RACHELLE FINAL EXAM -
the ER for agitation, confusion and bizarre behavior. for nutrition, neomycin orally or metronidazole to MENDOZA, MD FEB 2013
The patient is cachectic, jaundiced with ascites. ON decrease gut bacteria and thereby decrease (TOP 9 - AUG 2012
PE, you further note spider angiomas, palmar production of ammonia, which is responsible for MED BOARDS;
erythema and flipping hand tremors and ankle encephalopathy. Lactulose is given to neutralize TOPNOTCH MD)
clonus. Which of the following is correct regarding existing ammonia.
treatment of this condition?
A. Lactulose should be given for acute
encephalopathy 30-80 ml every hour until diarrhea
occurs.
B. Neomycin is administered to decrease ammonia
production in the gut
C. Metronidazole and Neomycin have comparable
benefits.
D. All of the above
E. A and B

TOPNOTCH MEDICAL BOARD PREP INTERNAL MEDICINE SUPEREXAM Page 80 of 95


For inquiries visit www.topnotchboardprep.com.ph or email us at topnotchmedicalboardprep@gmail.com
TOPNOTCH MEDICAL BOARD PREP INTERNAL MEDICINE SUPEREXAM
For inquiries visit www.topnotchboardprep.com.ph or email us at topnotchmedicalboardprep@gmail.com
Item QUESTION EXPLANATION AUTHOR TOPNOTCH
# EXAM
603 According to recent studies, microorganisms Epidemiologic studies have demonstrated an RACHELLE FINAL EXAM -
associated with coronary heart disease are: association between serologic evidence of C. MENDOZA, MD FEB 2013
A. Chlamydia pneumoniae and Streptococcus pneumoniae infection and atherosclerotic disease (TOP 9 - AUG 2012
pneumoniae of the coronary and other arteries. In addition, C. MED BOARDS;
B. Chlamydia pneumoniae, Streptococcus pneumoniae has been identified in atherosclerotic TOPNOTCH MD)
pneumoniae and Helicobacter pylori plaques by electron microscopy, DNA
C. Chlamydia pneumoniae and Helicobacter pylori hybridization, and immunocytochemistry. A
D. Streptococcus pneumoniae and Helicobacter potentially important but even more controversial
pylori association is the association of H. pylori with
ischemic heart disease and cerebrovascular
disease.
604 A 24-year old female started to note fatigue, The given patient has SLE. Antinuclear antibodies RACHELLE FINAL EXAM -
occasional fever, muscle and joint pains over the (ANA) are positive in >98% of patients during the MENDOZA, MD FEB 2013
past 3 months. At the clinic, she was observed to course of disease; repeated negative tests suggest (TOP 9 - AUG 2012
have fixed raised erythema over the malar that the diagnosis is not SLE, unless other MED BOARDS;
eminences, along with several oral ulcers. autoantibodies are present. High-titer IgG TOPNOTCH MD)
Auscultation revealed pericardial friction rub. What antibodies to double-stranded DNA and antibodies
is the most specific test for this condition? to the Sm antigen are both specific for SLE and,
A. ANA therefore, favor the diagnosis in the presence of
B. anti-dsDNA compatible clinical manifestations.
C. anti-Smith
D. A and B
E. B and C

605 A 45-year old patient suddenly developed diffuse The term SJS is now used to describe cases with RACHELLE FINAL EXAM -
blisters and eventual skin detachment after injection blisters developing on dusky or purpuric macules MENDOZA, MD FEB 2013
of a certain drug. Upon evaluation, the physician in which total body surface area blistering and (TOP 9 - AUG 2012
documented that the 25% of the total body surface eventual detachment is <10%. The term SJS/TEN MED BOARDS;
area of the patient is affected. The most likely is used to describe cases with 10–30% TOPNOTCH MD)
diagnosis is: detachment, and TEN is used to describe cases
A. Stevens-Johnson Syndrome (SJS) with >30% detachment. (Harrison's, 17th ed)
B. Toxic Epidermal Necrolysis (TEN)
C. SJS/TEN
D. Serum sickness
E. Acute generalized exanthematous pustulosis
606 Which of the following is TRUE in anaphylaxis? A characteristic feature is the eruption of well- RACHELLE FINAL EXAM -
A. Anaphylactic urticarial eruptions are intensely circumscribed, discrete cutaneous wheals with MENDOZA, MD FEB 2013
pruritic and frequently persist beyond 48 h erythematous, raised, serpiginous borders and (TOP 9 - AUG 2012
B. Atopy predisposes individuals to anaphylaxis blanched centers. These urticarial eruptions are MED BOARDS;
from penicillin therapy intensely pruritic and may be localized or TOPNOTCH MD)
C. The diagnosis of an anaphylactic reaction depends disseminated. They may coalesce to form giant
largely on an accurate history hives, and they seldom persist beyond 48 h. The
D. NSAIDS may produce IgE related acute airway diagnosis of an anaphylactic reaction depends on a
obstruction history revealing the onset of the symptoms and
signs within minutes after the responsible
material is encountered. Aspirin and other NSAIDs
such as indomethacin, aminopyrine, and
mefenamic acid may precipitate a life-threatening
episode of obstruction of upper or lower airways,
especially in patients with asthma, that is clinically
indistinguishable from anaphylaxis but is not
associated with the presence of specific IgE or
elevation of blood tryptase. According to most
studies, atopy does not predispose individuals to
anaphylaxis from penicillin therapy or venom of a
stinging insect but is a risk factor for allergens in
food or latex. (Harrison's, 17th ed)
607 What part of the immune system does the nitroblue Deficiencies of oxidative metabolism are detected RACHELLE FINAL EXAM -
tetrazolium test (NBT) evaluate? with either the nitroblue tetrazolium (NBT) dye MENDOZA, MD FEB 2013
A. B cell test or the dihydrorhodamine (DHR) oxidation (TOP 9 - AUG 2012
B. T cell test. These tests are based on the ability of MED BOARDS;
C. Phagocytic products of oxidative metabolism to alter the TOPNOTCH MD)
D. Complement system oxidation states of reporter molecules so that they
E. All of the above can be detected microscopically (NBT) or by flow
cytometry (DHR). Qualitative studies of
superoxide and hydrogen peroxide production
may further define neutrophil oxidative function.
(Harrison's 17th ed)
608 Dementia with Lewy bodies is associated with the in DLB, the first symptoms include: Visual RACHELLE FINAL EXAM -
following syndromes/symptoms, EXCEPT? hallucinations, REM sleep disorder, delirium, MENDOZA, MD FEB 2013
A. Capgras' syndrome Capgras' syndrome, parkinsonism. The mental (TOP 9 - AUG 2012
B. Parkinsonism status affected is the drawing and MED BOARDS;
C. Visual hallucinations frontal/executive part. It spares memory, but is TOPNOTCH MD)
D. Memory loss delirium prone. Patients with DLB also complain
E. Major depression of visual hallucinations, depression, sleep
disorder, delusions.

TOPNOTCH MEDICAL BOARD PREP INTERNAL MEDICINE SUPEREXAM Page 81 of 95


For inquiries visit www.topnotchboardprep.com.ph or email us at topnotchmedicalboardprep@gmail.com
TOPNOTCH MEDICAL BOARD PREP INTERNAL MEDICINE SUPEREXAM
For inquiries visit www.topnotchboardprep.com.ph or email us at topnotchmedicalboardprep@gmail.com
Item QUESTION EXPLANATION AUTHOR TOPNOTCH
# EXAM
609 A 44 year old renal transplant patient consulted at The patient has herpes zoster along a dermatomal RACHELLE FINAL EXAM -
the OPD for painful blisters located along his right distribution. Tzanck smear is done to confirm MENDOZA, MD FEB 2013
breast. Tzanck smear was done, revealing which of diagnosis. Pathognomonic of herpes zoster is the (TOP 9 - AUG 2012
the follwoing results? presence of multinucleated giant cells. MED BOARDS;
A. Hyphae and budding yeast TOPNOTCH MD)
B. Multinucleated giant cells
C. Granules
D. Sea-blue histiocytes
E. Hirano bodies

610 Urobilinogen is produced in the: Following secretion into bile, conjugated bilirubin RACHELLE FINAL EXAM -
A. Renal tubules reaches the duodenum and passes down the MENDOZA, MD FEB 2013
B. Red blood cell gastrointestinal tract without reabsorption by the (TOP 9 - AUG 2012
C. Liver intestinal mucosa. An appreciable fraction is MED BOARDS;
D. Intestinal mucosa converted by bacterial metabolism in the gut to TOPNOTCH MD)
E. Spleen the water-soluble colorless compound,
urobilinogen. Urobilinogen undergoes
enterohepatic cycling. Urobilinogen not taken up
by the liver reaches the systemic circulation, from
which some is cleared by the kidneys.
611 A 62-year old male was observed to be moving more Either a dopamine agonist or levodopa/carbidopa RACHELLE FINAL EXAM -
slowly than usual. He has not been sleeping well and can be used as initial treatment for parkinson's MENDOZA, MD FEB 2013
was always anxious. He was brought to your clinic (TOP 9 - AUG 2012
and you observed wide-based, shuffling gait, with MED BOARDS;
cogwheel rigidity and resting, pill-rolling tremors. TOPNOTCH MD)
What is the mechanism of action of the most
appropriate treatment for this condition?
A. Dopamine precursor with peripheral carboxylase
inhbitor
B. Catechol-O-methyltransferase (COMT) inhibitor
C. Dopamine agonist that acts directly on
postsynaptic dopamine receptors (D2)
D. A or B
E. A or C
612 Which of the following statements is/are correct All the others describe Type 1 DM RACHELLE FINAL EXAM -
regarding the pathogenesis of Type 2 DM? MENDOZA, MD FEB 2013
A. There is a strong genetic component for the (TOP 9 - AUG 2012
development of the disease. MED BOARDS;
B. The presence of islet cell autoantibodies TOPNOTCH MD)
characterizes the initial stage of the disease
C. Exposure to nitrosureas, coxsackie and rubella
viruses has been identified as putative triggers of the
disease.
D. All of the above
613 A patient with complete biliary obstruction Patients with complete biliary obstruction will RACHELLE FINAL EXAM -
developed bleeding tendencies. Which of the have impaired absorption of fat-soluble vitamins, MENDOZA, MD FEB 2013
following parameters may be prolonged? one of which is vitamin K. It is expected in this (TOP 9 - AUG 2012
A. Partial thromboplastin time patient that his PTT and PT may be prolonged, due MED BOARDS;
B. Prothrombin time to inadequate amounts of factors 10, 9, 7 and 2. TOPNOTCH MD)
C. Bleeding time Factor 10 and 2 is part of both intrnsic and
D. All of the above extrinsic coagulation pathway (common pathway),
E. A and B such that both PTT and PT may be affected.

614 A 21-year old male developed fever and sore throat. The patient is diagnosed with infectious RACHELLE FINAL EXAM -
On PE, there was a note of lymphadenopathy. Blood mononucleosis. Most cases are self-limited. MENDOZA, MD FEB 2013
exam revealed leukocytosis, with predominance of Deaths are very rare and most often are due to (TOP 9 - AUG 2012
atypical lymphocytes. Which of the following may central nervous system (CNS) complications, MED BOARDS;
be a complication of this condition? splenic rupture, upper airway obstruction, or TOPNOTCH MD)
A. Splenic rupture bacterial superinfection. Acute EBV infection has
B. Guillain-Barré syndrome also been associated with cranial nerve palsies
C. Autoimmune hemolytic anemia (especially those involving cranial nerve VII),
D. All of the above Guillain-Barré syndrome, acute transverse
E. A and B myelitis, and peripheral neuritis. Autoimmune
hemolytic anemia occurs in ~2% of cases during
the first 2 weeks. Hypertrophy of lymphoid tissue
in the tonsils or adenoids can result in upper
airway obstruction, as can inflammation and
edema of the epiglottis, pharynx, or uvula. About
10% of patients with IM develop streptococcal
pharyngitis after their initial sore throat resolves.
615 For disorders along the wall of the gastrointestinal Radiograph studies, such as barium swallow and RACHELLE FINAL EXAM -
tract, such as ulcers, diverticula, strictures, new barium enema, can provide an actual picture of MENDOZA, MD FEB 2013
growths and motility disorders, the most useful motility, filling defects and mucosal defects when (TOP 9 - AUG 2012
diagnostic test to order is: taken in series. Intetsinal obstruction is also MED BOARDS;
A. Radiograph preferred to be viewed in upright and supine TOPNOTCH MD)
B. MRI radiographic studies.
C. CT scan
D. Ultrasound
E. Scintigraphy

TOPNOTCH MEDICAL BOARD PREP INTERNAL MEDICINE SUPEREXAM Page 82 of 95


For inquiries visit www.topnotchboardprep.com.ph or email us at topnotchmedicalboardprep@gmail.com
TOPNOTCH MEDICAL BOARD PREP INTERNAL MEDICINE SUPEREXAM
For inquiries visit www.topnotchboardprep.com.ph or email us at topnotchmedicalboardprep@gmail.com
Item QUESTION EXPLANATION AUTHOR TOPNOTCH
# EXAM
616 This syndrome results from defective hypothalamic McCune-Albright syndrome consists of polyostotic RACHELLE FINAL EXAM -
gonadotropin-releasing hormone (GnRH) synthesis fibrous dysplasia, pigmented skin patches, and a MENDOZA, MD FEB 2013
and is associated with anosmia or hyposmia due to variety of endocrine disorders, including GH- (TOP 9 - AUG 2012
olfactory bulb agenesis or hypoplasia: secreting pituitary tumors, adrenal adenomas, and MED BOARDS;
A. Bardet-Biedl Syndrome autonomous ovarian function. Carney syndrome is TOPNOTCH MD)
B. Prader-Willi Syndrome characterized by spotty skin pigmentation,
C. Carney syndrome myxomas, and endocrine tumors including
D. McCune-Albright syndrome testicular, adrenal, and pituitary adenomas.
E. Kallmann syndrome Acromegaly occurs in about 20% of patients.
Prader-Willi Syndrome is associated with
hypogonadotropic hypogonadism, hyperphagia-
obesity, chronic muscle hypotonia, mental
retardation, and adult-onset diabetes mellitus.
Bardet-Biedl Syndrome is a rare genetically
heterogeneous disorder characterized by mental
retardation, renal abnormalities, obesity, and
hexadactyly, brachydactyly, or syndactyly. Central
diabetes insipidus may or may not be associated.
617 A 29-year old male complains of progressive attacks Alkaptonuria is a rare disorder of tyrosine RACHELLE FINAL EXAM -
of joint pains and stiffness (hip and knee joints), catabolism in which deficiency of homogentisate MENDOZA, MD FEB 2013
back pain and darkening of urine. Physical exam 1,2-dioxygenase (also known as homogentisic acid (TOP 9 - AUG 2012
revealed generalized darkening of the ear, along oxidase) leads to excretion of large amounts of MED BOARDS;
with limited range of motion for both lower homogentisic acid in urine and accumulation of TOPNOTCH MD)
extremities due to pain. The underlying defect is: oxidized homogentisic acid pigment in connective
A. Lack of homogentisic acid oxidase tissues (ochronosis). Alkaptonuria may go
B. Abnormal overproduction of melanin granules unrecognized until middle life, when degenerative
C. Reduced activity of cystathionine synthase joint disease develops.
D. Overactivity of tyrosinase
E. None of the above
618 Which of the following is NOT a major criterion Framingham Major Criteria include: paroxysmal RACHELLE FINAL EXAM -
included in the Framingham criteria of congestive nocturnal dyspnea, rales, incresed CVP, neck vein MENDOZA, MD FEB 2013
heart failure? distention, cardiomegaly, acute pulmonary edema, (TOP 9 - AUG 2012
A. Rales s3 gallop, hepatojugular reflux, weight loss. MED BOARDS;
B. Cardiomegaly Dyspnea on exertion is included in the minor TOPNOTCH MD)
C. Dypnea on exertion criteria.
D. S3 gallop
E. Neck vein distention

619 A 63-year old congestive heart failure patient Orthopnea is due to redistribution of fluid from RACHELLE FINAL EXAM -
frequently complains of worsening orthopnea, the splanchnic circulation and lower extremities MENDOZA, MD FEB 2013
requiring him to use 4 pillows when sleeping. The into the central circulation during recumbency. (TOP 9 - AUG 2012
most probable mechanism for this is: PNH is due to increased pressure on bronchial MED BOARDS;
A. Worsening interstitial pulmonary edema arteries leading to airway compression, along with TOPNOTCH MD)
B. Redistribution of fluid from the splanchnic interstitial edema. Cheyne-Stokes respiration is
circulation and lower extremities into central due to diminished sensitivty of respiratory center
circulation to arterial PCO2. Crackles/rales is due to
C. Increased pressure in the bronchial arteries transudation of fluid from the intravscular space
leading to airway compression into the alveoli.
D. Transudation of fluid from the intravscular space
into the alveoli
E. Diminished sensitivity of the respiratory canter to
arterial PCO2
620 A 30-year old male sought consult due to presistent Complete tumor removal is the ultimate RACHELLE FINAL EXAM -
headache and palpitations. He was pale, restless and therapeutic goal. Preoperative patient preparation MENDOZA, MD FEB 2013
sweating profusely during the interview. BP was is essential for safe surgery. -Adrenergic blockers (TOP 9 - AUG 2012
220/120 mmHg, HR was 124 bpm, PE was otherwise (phenoxybenzamine) should be initiated at MED BOARDS;
unremarkable. CT scan was requested revealing an relatively low doses (e.g., 5–10 mg orally three TOPNOTCH MD)
adrenal mass. Surgery was planned. What is the times per day) and increased as tolerated every
most appropriate pre-oprative treatment for this few days. Because patients are volume constricted,
patient? liberal salt intake and hydration are necessary to
A. Nitroprusside drip 12 hours prior to surgery avoid orthostasis. Adequate alpha blockade
B. Nicardipine IV infusion 12 hours prior to surgery generally requires 10–14 days, with a typical final
C. Propranolol 10mg TID-QID 10-14 days prior to dose of 20–30 mg phenoxybenzamine three times
surgery per day. Oral prazosin or intravenous
D. Phenoxybenzamine 20-30mg TID 10-14 days phentolamine can be used to manage paroxysms
prior to surgery while awaiting adequate alpha blockade. Before
E. Captopril 25mg TID-QID 10-14 days prior to surgery, the blood pressure should be consistently
surgery below 160/90 mmHg, with moderate orthostasis.
Beta blockers (e.g., 10 mg propranolol three to
four times per day) can be added after starting
alpha blockers, and increased as needed, if
tachycardia persists. Because beta blockers can
induce a paradoxical increase in blood pressure in
the absence of alpha blockade, they should be
administered only after effective alpha blockade.
Other antihypertensives, such as calcium-channel
blockers or angiotensin-converting enzyme
inhibitors, have also been used when blood
pressure is difficult to control with
phenoxybenzamine alone.

TOPNOTCH MEDICAL BOARD PREP INTERNAL MEDICINE SUPEREXAM Page 83 of 95


For inquiries visit www.topnotchboardprep.com.ph or email us at topnotchmedicalboardprep@gmail.com
TOPNOTCH MEDICAL BOARD PREP INTERNAL MEDICINE SUPEREXAM
For inquiries visit www.topnotchboardprep.com.ph or email us at topnotchmedicalboardprep@gmail.com
Item QUESTION EXPLANATION AUTHOR TOPNOTCH
# EXAM
621 A 36 year old man presents to the clinic with This case is a PRIMARY SYPHILIS. It may lead to VON ANDRE DIAGNOSTIC
complaints of a genital sore. The patient is sexually tertiary syphilis which can cause ascending aortic MEDINA, MD (TOP EXAM - AUG
active, heterosexual involved with 3 partners and aneurysm 4 - FEB 2012 MED 2012
practices unprotected intercourse. Four days ago he BOARDS;
noted, a painless sore on his penis. He is afebrile TOPNOTCH MD)
with a HR of 80bpm, BP of 120/80 mmHg. PE reveals
a solitary ulcerated lesion located on the lateral
aspect of his penis. The lesion is non tender and is
associated with with bilateral inguinal
lymphadenopathy. PE is otherwise normal. If left
untreated, this man is at increased risk for which of
the following?
A. Ascending aortic aneurysm
B. Coronary artery aneurysm
C. Endocarditis
D. Mitral valve stenosis
E. Rupture of ventricular free wall
622 A 65 year old woman presents to the ER department patient has symptomatic bradycardia. Atropine is VON ANDRE DIAGNOSTIC
complaining of dizziness. She is disoriented to the the dug of choice. MEDINA, MD (TOP EXAM - AUG
date and her location and it is difficult to gather an 4 - FEB 2012 MED 2012
accurate history. Her pulse is 48 bpm, BP of 80/60 BOARDS;
mmHg, and RR of 12 cpm. On examination, her TOPNOTCH MD)
extremities are cool and clammy. her capplary refill
time is 5 seconds. What is the most appropriate
therapy?
A. Adenosine
B. Amiodarone
C. Atropine
D. Isoproterenol
E. Metoprolol
623 A 25 year old man is recovering in the hospital from- This is a case of Pulmonary Embolism in which the VON ANDRE DIAGNOSTIC
an open rapair of his broken femur, which he compliance of the lungs (volume change per unit MEDINA, MD (TOP EXAM - AUG
suffered during an automobile accident. On pressure change) decreases. 4 - FEB 2012 MED 2012
postoperative day 3, he develops sudden onset of BOARDS;
shortness of breath and vague chest pain. His TOPNOTCH MD)
temperature is 37.6 C and HR of 108 bpm, BP of
90/60 mmHg, RR of 42 cpm and O2 sat of is 89 % on
room air. PE is significant for jugular venous
distention to 9 cm and an accentuated pulmonic
component of S2. Which of the following is most
likely DECREASED?
A. Airway resistance
B. Alveolar dead space
C. Alveolar ventilation
D. Pulmonary compliance
E. Pulmonary vascular resistance
624 A 48 year old man is brought to the emergency As uremic toxins increases, patients will complain VON ANDRE DIAGNOSTIC
department confused and disoriented. He is a of pruritus, nausea and vomiting, hiccups, and a MEDINA, MD (TOP EXAM - AUG
diagnosed case of DM type 1. He reports recent metallic taste in her mouth. Pericardial friction rub 4 - FEB 2012 MED 2012
onset of nausea and has had several episodes of in cases of uremic pericarditis BOARDS;
emesis in the past 4 days. On further questioning, he TOPNOTCH MD)
also notes a metallic taste in his mouth, frequent
hiccups, and pruritus. On PE there is rough, velcro-
like sound heard accross his precordium. Which of
the following is the most likely diagnosis?
A. Addisonian crisis
B. Fulminant hepatic failure
C. Heroin withdrawal
D. Renal insufficiency
E. Vit B 12 insufficiency
625 A 43 year old man develops fever, headache, and encapsulated organism such as Neisseria, Strep VON ANDRE DIAGNOSTIC
altered mental status. His past medical history is pneumoniae and H influenza may cause infection MEDINA, MD (TOP EXAM - AUG
notable only for a motor vehicle accident 2 years in asplenic patients, as they are unable to 4 - FEB 2012 MED 2012
ago, during which he sustained a splenic laceration manufacture a new antibody immune response BOARDS;
requiring splenectomy. Which of the following is the TOPNOTCH MD)
most likely causing this patient's syndrome?
A. Cryptococcus neoformans and Listeria
monocytogenes
B. Cryptococcus neoformans and Streptococcus
pneumonia
C. Haemophilus influenza and Staphylococcus
aureus
D. N. meningitides and S. Aureus
E. N. meningitides and Streptococcus pneumoniae

TOPNOTCH MEDICAL BOARD PREP INTERNAL MEDICINE SUPEREXAM Page 84 of 95


For inquiries visit www.topnotchboardprep.com.ph or email us at topnotchmedicalboardprep@gmail.com
TOPNOTCH MEDICAL BOARD PREP INTERNAL MEDICINE SUPEREXAM
For inquiries visit www.topnotchboardprep.com.ph or email us at topnotchmedicalboardprep@gmail.com
Item QUESTION EXPLANATION AUTHOR TOPNOTCH
# EXAM
626 A 34 year old man presents with fever and night This is a case of Miliary TB. It refers to VON ANDRE DIAGNOSTIC
sweats for 3 weeks and productive cough. A recent hematogenous spread of the bacteria following MEDINA, MD (TOP EXAM - AUG
HIV test was negative. A PPD test performed on reactivation of a latent infection. The important 4 - FEB 2012 MED 2012
admission shows 15 mm induration. His ESR is 97 information here is is the classic MILIARY BOARDS;
mm/hr. Past medical history is significant for PATTERN ON XRAY of the chest indicative of TOPNOTCH MD)
relapse of alcoholism. rebiew of systems reveal hematogenous spread.
generalized fatigue over the past month and a 3.2 kg
weight loss. His temperature is 39.6 C, RR is 25 cpm,
and O2 sat is 86 % on room air. Bilateral pulmonary
rales are noted on P.E and moderate sternal
retractions are present. X ray of the chest reveals
reticulonodular infiltrates spread evenly throughout
both lung fields. Which of the following is the most
likely diagnosis?
A. Latent tuberculosis
B. Miliary tuberculosis
C. Pneumocystus jiroveci pneumonia
D. Primary tuberculosis
E. Reactivation pulmonary tuberculosis
627 A 54 year old man presents to his physician This is a cse of BPH. Urinary obstructive VON ANDRE DIAGNOSTIC
complaining of difficulty urinating. He urinates more symptoms (hesitancy, weak stream, intermittent MEDINA, MD (TOP EXAM - AUG
often during the day and often wakes at night to stream and etc). Serum creatinine is used to assess 4 - FEB 2012 MED 2012
urinate. He also has difficulty starting and for renal insufficiency caused by obstructive BOARDS;
maintaining a stream of urine. DRE reveals a fusely, uropathy 2ndary to BPH. TOPNOTCH MD)
large, rubbery prostate. Urinalysis and urine culture
reveal no hematuria or signs of infection. What is the
BEST next step in the diagnosis?
A. Cystoscopy
B. Measure creatinine levels
C. Measure PSA level
D. UTZ guided transrectal prostate biopsy
E. X ray of the chest and bone scan

628 A 29 year old type 1 diabetic presented to the ER VON ANDRE DIAGNOSTIC
with clouded sensorium. He is febrile and MEDINA, MD (TOP EXAM - AUG
tachypneic, sweating profusely and pale. His BP is 4 - FEB 2012 MED 2012
90/60 mmHg, CR of 116 bpm and CBG of 300 mg/dL. BOARDS;
Relatives reported that he has cough and yellow TOPNOTCH MD)
sputum for about a week now and has not sought
consult nor took antibiotics. he has decreased
appetite for the last 3 days. His breath has a
characteristic fruity odor. These statements are
TRUE regarding the pathophysiology of the case
EXCEPT:
A. Increase hepatic gluconeogenesis
B. Insulin deficiency
C. Low peripheral glucose utilization
D. Blunted counter-regulatory response
E. Insulin resistance
629 The MOST useful physiologic marker of thyroid TSH is the most useful physiologic marker of VON ANDRE DIAGNOSTIC
hormone synthesis is: thyroid hormone synthesis MEDINA, MD (TOP EXAM - AUG
A. Triiodithyronine 4 - FEB 2012 MED 2012
B. TSH BOARDS;
C. TRH TOPNOTCH MD)
D. Thyroxine
E. Thyroglobulin

630 The gold standard in the diagnosis of 24 hr pH monitoring VON ANDRE DIAGNOSTIC
gastroesophageal reflux disease: MEDINA, MD (TOP EXAM - AUG
A. Esophagoscopy 4 - FEB 2012 MED 2012
B. 24 hr pH monitoring BOARDS;
C. Barium swallow TOPNOTCH MD)
D. Berstein test
E. Breath test

631 A 37 year old seaman came to your clinic for a Active Hepa B with high infectivity and previous VON ANDRE DIAGNOSTIC
second opinion. In his pre-employment medical Hepa A MEDINA, MD (TOP EXAM - AUG
examination, he turned (+) for HBs Ag. His industrial 4 - FEB 2012 MED 2012
physician requested for a complete hepatitis BOARDS;
serology. The following are the results: anti-HBs (-), TOPNOTCH MD)
anti -HBc (-), HBeAg (+), anti-HBe (-), anti-HAV IgG
(+), anti-HCV (-). The CORRECT interpretation of the
results is:
A. Active Hepa B with high infectivity and active
Hepa A
B. Active Hepa B with high infectivity and previous
Hepa A
C. Active Hepa B in the window period and
previous Hepa A
D. Active Hepa B with low infectivity and previous
Hepa A
E. None of the above

TOPNOTCH MEDICAL BOARD PREP INTERNAL MEDICINE SUPEREXAM Page 85 of 95


For inquiries visit www.topnotchboardprep.com.ph or email us at topnotchmedicalboardprep@gmail.com
TOPNOTCH MEDICAL BOARD PREP INTERNAL MEDICINE SUPEREXAM
For inquiries visit www.topnotchboardprep.com.ph or email us at topnotchmedicalboardprep@gmail.com
Item QUESTION EXPLANATION AUTHOR TOPNOTCH
# EXAM
632 A 42 year old female came in due to diarrhea for 6 Zollinger-Ellison Syndrome VON ANDRE DIAGNOSTIC
months. She has stopped all dairy and milk products MEDINA, MD (TOP EXAM - AUG
for the past few months as advised by a physician 4 - FEB 2012 MED 2012
who told her that her symptoms were caused by BOARDS;
lactase deficiency. The diarrhea occurs throughout TOPNOTCH MD)
the day and she has not noticed blood or pus in the
stools. Past medical history revealed PUD for many
years that has been relatively resistant to medical
treatment. She takes maximum doses of omeprazole
and famotidine and still has symptoms. Her PE is
unrevealing. her lab results are normal except for
mild hypercalcemia. What is the most likely
diagnosis?
A. H. pylori associated PUD
B. Zollinger-Ellison Syndrome
C. Irritable Bowel syndrome
D. Autoimmune atrophic gastritis
E. None of the above
633 What is the cornerstone of the treatment for NON- weight loss VON ANDRE DIAGNOSTIC
ALCOHOLIC fatty liver diasease? MEDINA, MD (TOP EXAM - AUG
A. Thiazolidinediones 4 - FEB 2012 MED 2012
B. Statins BOARDS;
C. Fibrates TOPNOTCH MD)
D. Metformin
E. Weight Loss

634 A 24 year old man is brought to the ER department This is a cse of Cardiac tamponande. Beck's triad: VON ANDRE DIAGNOSTIC
after suffering blunt trauma to his chest in a motor hypotension, muffled heart sounds and JUGULAR MEDINA, MD (TOP EXAM - AUG
vehicle accident. His RR is 32 cpm, CR of 125 bpm, VENOUS DISTENTION 4 - FEB 2012 MED 2012
and BP is 80/40 mmHg, with a decrease to 60/40 BOARDS;
mmHg on inspiration. PE reveals decreased heart TOPNOTCH MD)
sounds and a pericardial friction rub. Which other
symptom would the physician expect to see as part
of the patient's presentation?
A. Bounding pulse
B. Bradycardia
C. Holosystolic murmur
D. Hypertension
E. Jugular venous distention

635 A 47 year old man with hypertension is prescribed VON ANDRE DIAGNOSTIC
with hydochlorothiazide by his primary care MEDINA, MD (TOP EXAM - AUG
physician. Which of the following is a potential effect 4 - FEB 2012 MED 2012
of this medication? BOARDS;
A. Hypokalemia TOPNOTCH MD)
B. Dilutional Hyponatremia
C. Hyperuricemia
D. Hyperlipidemia
E. All of the above

636 An 85 year old man with hypertension and Anterior inferior cerebellar artery , distribution VON ANDRE DIAGNOSTIC
cerebrovascular disease presents to his physician's typically present with gaze palsy, DEAFNESS, MEDINA, MD (TOP EXAM - AUG
office for a routine check up. Review of systems is TINNITUS and ipsilateral facial weakness. 4 - FEB 2012 MED 2012
positive only for "periodic ringing in the ears". PE BOARDS;
reveals left facial paralysis and poof hearing on the TOPNOTCH MD)
left side. A stroke in which vascular territory is
MOST likely?
A. Anterior cerebral artery
B. Anterior inferior cerebellar artery
C. Middle cerebral artery
D. Posterior cerebral artery
E. Posterior inferior cerebellar artery

637 Which of the following is associated with Virtually all cases of primary CNS lymphoma in VON ANDRE DIAGNOSTIC
development of primary central nervous system patients with HIV are associated with EBV MEDINA, MD (TOP EXAM - AUG
lymphoma in ppatients with HIV? 4 - FEB 2012 MED 2012
A. EBV BOARDS;
B. JC virus TOPNOTCH MD)
C. Mycobacterium avium complex
D. Previous radiation exposure
E. Toxoplasma gondii

638 What is the best treatment for anemia in patients VON ANDRE DIAGNOSTIC
with chronic kidney disease? MEDINA, MD (TOP EXAM - AUG
A. Thrice weekly erythropoietin 4 - FEB 2012 MED 2012
B. Periodic blood transfusion BOARDS;
C. Daily intake of ferrous sulfate TOPNOTCH MD)
D. Bone marrow transplantation
E. Any of the above choices

TOPNOTCH MEDICAL BOARD PREP INTERNAL MEDICINE SUPEREXAM Page 86 of 95


For inquiries visit www.topnotchboardprep.com.ph or email us at topnotchmedicalboardprep@gmail.com
TOPNOTCH MEDICAL BOARD PREP INTERNAL MEDICINE SUPEREXAM
For inquiries visit www.topnotchboardprep.com.ph or email us at topnotchmedicalboardprep@gmail.com
Item QUESTION EXPLANATION AUTHOR TOPNOTCH
# EXAM
639 What is the gold standard in the diagnosis of gold standard: pulmonary angiography; principal VON ANDRE DIAGNOSTIC
pulmonary embolism? imaging technique: CT scan with IV contrast MEDINA, MD (TOP EXAM - AUG
A. V/Q scan 4 - FEB 2012 MED 2012
B. CT Scan with IV contrast BOARDS;
C. Pulmonary angiography TOPNOTCH MD)
D. MRI contrast enhanced
E. Chest Xray

640 A 69 year old male was diagnosed to have COPD. smoking cessation and oxygen therapy VON ANDRE DIAGNOSTIC
What is/are the only intervention/s proven to MEDINA, MD (TOP EXAM - AUG
improve survival in severe COPD patients? 4 - FEB 2012 MED 2012
A. Smoking cessation BOARDS;
B. Oxygen therapy TOPNOTCH MD)
C. Cyclic antibiotics
D. Inhaled corticosteroids
E. A and B

641 Primary Syphilis is best treated with? The DOC for syphilis is still Pen G LITO JAY DIAGNOSTIC
A. Ampicillin MACARAIG, MD EXAM - AUG
B. Amikacin (TOP 8 - FEB 2013 2013
C. Penicillin G MED BOARDS;
D. Spectinomycin TOPNOTCH MD)
E. Ceftriaxone

642 The most common complication of Peptic Ulcer LITO JAY DIAGNOSTIC
disease is? MACARAIG, MD EXAM - AUG
A. Perforation (TOP 8 - FEB 2013 2013
B. Bleeding MED BOARDS;
C. Fistula formation TOPNOTCH MD)
D. Hypertrophy of the pylorus
E. Peritonitis

643 A 24 year old male consulted you due to purulent Ceftriaxone should definitely be given. However, LITO JAY DIAGNOSTIC
penile discharge. Gram stain showed garm(-) gonorrhea is almost always accompanied by MACARAIG, MD EXAM - AUG
intracellular diplococci. What is/are the best Chlamydial infection, hence Azithromycin or (TOP 8 - FEB 2013 2013
treatment? Doxycycline should be given too. MED BOARDS;
A. Ceftriaxone 250mg IM TOPNOTCH MD)
B. Metronodazole 500mg tablet, 4 tablets single
dose
C. Azithromycin 1 gram single dose
D. A and B
E. A and C
644 A 32 year old cave explorer came to your clinic due LITO JAY DIAGNOSTIC
to a 3 week history of cough that became associated MACARAIG, MD EXAM - AUG
with blood-streaked sputum. What is the best (TOP 8 - FEB 2013 2013
differential for this case? MED BOARDS;
A. Community Acquired Pneumonia TOPNOTCH MD)
B. Pulmonary tuberculosis
C. Paragonimus westermani infection
D. Histoplasma capsulatum infection

645 The lowest grade of murmur that you can observe LITO JAY DIAGNOSTIC
thrill is MACARAIG, MD EXAM - AUG
A. Grade I (TOP 8 - FEB 2013 2013
B. Grade II MED BOARDS;
C. Grade III TOPNOTCH MD)
D. Grade IV
E. Grade V

646 A 42 year old teacher came to your clinic to consult LITO JAY DIAGNOSTIC
for lesions in her skin. Upon PE, you saw vesicular, MACARAIG, MD EXAM - AUG
erythematous and crusted lesions on her (TOP 8 - FEB 2013 2013
extremities, chest and back. Some lesions have the MED BOARDS;
characteristic dew-drop on a rose petal morphology. TOPNOTCH MD)
You are sure this is
A. Scabies
B. Psoriasis
C. Impetigo
D. Varicella zoster
E. Herpes simplex
647 A 56 year year old alcoholic male came to ER due to LITO JAY DIAGNOSTIC
severe hematemesis. Bleeding is uncontrollable, MACARAIG, MD EXAM - AUG
hence you considered rupture of distal esophagus (TOP 8 - FEB 2013 2013
due to retching which is called MED BOARDS;
A. Mallory-Weiss Syndrome TOPNOTCH MD)
B. Boerhaave Syndrome
C. Plummer-Vinson Syndrome
D. Zenker's Diverticulum
E. Traction Diverticulum

TOPNOTCH MEDICAL BOARD PREP INTERNAL MEDICINE SUPEREXAM Page 87 of 95


For inquiries visit www.topnotchboardprep.com.ph or email us at topnotchmedicalboardprep@gmail.com
TOPNOTCH MEDICAL BOARD PREP INTERNAL MEDICINE SUPEREXAM
For inquiries visit www.topnotchboardprep.com.ph or email us at topnotchmedicalboardprep@gmail.com
Item QUESTION EXPLANATION AUTHOR TOPNOTCH
# EXAM
648 The most common presenting symptom of LITO JAY DIAGNOSTIC
Pulmonary Thromboembolism is MACARAIG, MD EXAM - AUG
A. Syncope (TOP 8 - FEB 2013 2013
B. Tachycardia MED BOARDS;
C. Dyspnea TOPNOTCH MD)
D. Hemoptysis
E. "sense of impending doom"

649 A tumor in the descending colon commonly presents LITO JAY DIAGNOSTIC
as MACARAIG, MD EXAM - AUG
A. Fatigue and palpitations (TOP 8 - FEB 2013 2013
B. tenesmus MED BOARDS;
C. hematochezia TOPNOTCH MD)
D. Hypochromic, microcytic anemia
E. Applecore lesion on radiograph

650 During your internship, you saw a patient with LITO JAY DIAGNOSTIC
psoriasis scraping off her lesions and punctate MACARAIG, MD EXAM - AUG
hemorrhages appeared. This is called (TOP 8 - FEB 2013 2013
A. Koebner phenomenon MED BOARDS;
B. Wickham striae TOPNOTCH MD)
C. Auspitz sign
D. Forscheimer phenomenon
E. Nagayama spots

651 A 27 year old male was noted to have BP of 150/90 Beta blockers can cause impotence LITO JAY DIAGNOSTIC
mmHg. Which drug should you prescribe the least? MACARAIG, MD EXAM - AUG
A. Amlodipine (TOP 8 - FEB 2013 2013
B. Losartan MED BOARDS;
C. Captopril TOPNOTCH MD)
D. Propranolol
E. Hydrochlorothiazide

652 You are given a patient with thalassemia major who LITO JAY DIAGNOSTIC
had hemochromatosis. This is also known as MACARAIG, MD EXAM - AUG
A. Bronze diabetes (TOP 8 - FEB 2013 2013
B. Wilson's disease MED BOARDS;
C. Ondine's curse TOPNOTCH MD)
D. Gustilo's syndrome
E. Beckmann's Disease

653 The Philippine CPG for Community Acquired LITO JAY DIAGNOSTIC
Pneumonia requires ______ as basis for its diagnosis. MACARAIG, MD EXAM - AUG
A. History of cough of >2 weeks (TOP 8 - FEB 2013 2013
B. Clinical findings alone MED BOARDS;
C. Routine sputum gram stain TOPNOTCH MD)
D. Radiologic confirmation
E. Blood culture

654 the Global Initiative Against Asthma (GINA) LITO JAY DIAGNOSTIC
recommends addition of long acting beta-agonist as MACARAIG, MD EXAM - AUG
controller medication for (TOP 8 - FEB 2013 2013
A. Mild intermittent asthma MED BOARDS;
B. Mild persistent asthma TOPNOTCH MD)
C. Moderate persistent asthma
D. Severe acute attacks
E. Status asthmaticus

655 In a patient with goiter on hyperthyroid state, this LITO JAY DIAGNOSTIC
medication is usually given to reduce vascularity and MACARAIG, MD EXAM - AUG
size of the tumor (TOP 8 - FEB 2013 2013
A. Lugol's iodine MED BOARDS;
B. Propanolol TOPNOTCH MD)
C. PTU
D. Methimazole
E. Bromocriptine

656 A 45 year old patient presented with bone pain. LITO JAY DIAGNOSTIC
Radiographic findings showed lytic bone lesions and MACARAIG, MD EXAM - AUG
serology showed monoclonal antibodies. This (TOP 8 - FEB 2013 2013
patient would probably be suffering from? MED BOARDS;
A. Chronic Myelogenous Leukemia TOPNOTCH MD)
B. Primary Myeloproliferative disease
C. Multiple Myeloma
D. Multiple Sclerosis
E. Reynaud's Disease

TOPNOTCH MEDICAL BOARD PREP INTERNAL MEDICINE SUPEREXAM Page 88 of 95


For inquiries visit www.topnotchboardprep.com.ph or email us at topnotchmedicalboardprep@gmail.com
TOPNOTCH MEDICAL BOARD PREP INTERNAL MEDICINE SUPEREXAM
For inquiries visit www.topnotchboardprep.com.ph or email us at topnotchmedicalboardprep@gmail.com
Item QUESTION EXPLANATION AUTHOR TOPNOTCH
# EXAM
657 A 29 year old female came to ER due to severe flank LITO JAY DIAGNOSTIC
pain and decreased amount of urine. Urinalysis MACARAIG, MD EXAM - AUG
showed gross hematuria with leukocytes, few (TOP 8 - FEB 2013 2013
squamous cells and coffin-lid crystals. You know this MED BOARDS;
is caused be renal stones composed by TOPNOTCH MD)
A. Magnesium Ammonium Phosphate
B. Calcium oxalate
C. Uric acid
D. Calcium carbonate
E. None of the above

658 This is the pathognomonic lesion for Rheumatic LITO JAY DIAGNOSTIC
Fever. MACARAIG, MD EXAM - AUG
A. Anitschkow cells (TOP 8 - FEB 2013 2013
B. Caterpillar cells MED BOARDS;
C. Aschoff bodies TOPNOTCH MD)
D. McCullough Plaques
E. Vegetations

659 The following are Type III hypersensitivity LITO JAY DIAGNOSTIC
reactions, EXCEPT? MACARAIG, MD EXAM - AUG
A. Systemic LupusErythematosus (TOP 8 - FEB 2013 2013
B. Arthus Reaction MED BOARDS;
C. PSGN TOPNOTCH MD)
D. Serum sickness
E. Goodpasteur's Syndrome

660 You are presented with a 34 year old call center LITO JAY DIAGNOSTIC
agent who developed severe coughing episodes MACARAIG, MD EXAM - AUG
associated with high grade fever. Further history (TOP 8 - FEB 2013 2013
revealed previous diagnosis of HIV. You requested MED BOARDS;
for CD4 count and revealed <200/µL. What is the TOPNOTCH MD)
most likely etiologic agent?
A. Nocardia
B. Cytomegalovirus
C. Mycobacterium Avium-Intracellulare
D. Pneumocystis jirovecii
E. None of the above
661 Patient presented with dizziness and dyspnea at the VSD is an acyanotic congenital heart defect, aka a HAZEL KAREN MIDTERM 2 -
ER. Physical examination showed a holosystolic Left-to-right shunt, so there are no signs of RAZ, MD (TOP 6 - AUG 2013
harsh sounding murmur loudest at the left lateral cyanosis. FEB 2013 MED
sternal border. This describes? BOARDS;
A. Mitral stenosis TOPNOTCH MD)
B. Mitral regurgitation
C. Ventricular septal defect
D. Mitral valve prolapse
E. Patent dusctus arteriosus

662 On ECG, PR interval shows increased duration of Atrial flutter is descbed to have a "sawtooth HAZEL KAREN MIDTERM 2 -
more than 5 small squares. What would be the pattern" on ECG. 2nd degree AV block is divided RAZ, MD (TOP 6 - AUG 2013
diagnosis? into to subcategories, namely Mobitz I ( FEB 2013 MED
A. Atrial flutter Wenckebach) with progressive lengthening od PR BOARDS;
B. Mobitz Type II interval and dropped beats, and Mobitz II with TOPNOTCH MD)
C. Wenckebach type AV block dropped beats not preceeded by change in length
D. 1st degree AV block of PR interval.
E. None

663 Class of anti-arrythmic that blocks Na channels? I - Na channel blockers, II - B - blockers, III - K HAZEL KAREN MIDTERM 2 -
A. I channel blockers, IV - Ca channel blockers RAZ, MD (TOP 6 - AUG 2013
B. II FEB 2013 MED
C. III BOARDS;
D. IV TOPNOTCH MD)
E. None

664 This disease is characterized by acute adrenocortical The bacterial infection leads to HAZEL KAREN MIDTERM 2 -
insufficiency and adrenal hemorrhage secondary to massive hemorrhage into one or (usually) both RAZ, MD (TOP 6 - AUG 2013
infection with N. menigitidis? adrenal glands. It is characterized by FEB 2013 MED
A. Waterhouse - Friedrichsen Syndrome overwhelming bacterial BOARDS;
B. Pheochromocytoma infection meningococcemia leading to massive TOPNOTCH MD)
C. Conn's disease blood invasion, organ failure, coma, low blood
D. Addison's disease pressure and shock,disseminated intravascular
E. None coagulation (DIC) with widespread purpura,
rapidly developing adrenocortical
insufficiency and death.
665 Manifestation of MEN I except? Multiple endocrine neoplasia type 1 (MEN-1 HAZEL KAREN MIDTERM 2 -
A. Pheochromocytoma syndrome) or Wermer's syndrome is part of a RAZ, MD (TOP 6 - AUG 2013
B. Kidney stones group of disorders that affect theendocrine FEB 2013 MED
C. Stomach ulcers system through development neoplastic lesions BOARDS;
D. Pituitary tumors in pituitary, parathyroid gland and pancreas. TOPNOTCH MD)
E. Zollinger - Ellison Syndrome Kidney stones are due to excess calcium
deposition due to PTH tumor, stomach ulcers are
secondary to ZES, a form of pancreatic tumor.

TOPNOTCH MEDICAL BOARD PREP INTERNAL MEDICINE SUPEREXAM Page 89 of 95


For inquiries visit www.topnotchboardprep.com.ph or email us at topnotchmedicalboardprep@gmail.com
TOPNOTCH MEDICAL BOARD PREP INTERNAL MEDICINE SUPEREXAM
For inquiries visit www.topnotchboardprep.com.ph or email us at topnotchmedicalboardprep@gmail.com
Item QUESTION EXPLANATION AUTHOR TOPNOTCH
# EXAM
666 The following are causes of high anion gap metabolic All of the examples cause HAGMA, except HAZEL KAREN MIDTERM 2 -
acidosis, except? acetazolamide which causes NAGMA. RAZ, MD (TOP 6 - AUG 2013
A. Methanol FEB 2013 MED
B. Ethylene glycol BOARDS;
C. Paraldehyde TOPNOTCH MD)
D. Acetazolamide
E. Aspirin

667 GFR of stage III CKD? Chronic kidney disease (CKD), also known HAZEL KAREN MIDTERM 2 -
A. >90 ml/min/1.73m2 as chronic renal disease, is a progressive loss RAZ, MD (TOP 6 - AUG 2013
B. 60 - 89 in renal function over a period of months or FEB 2013 MED
C. 30 - 59 years.All individuals with a glomerular filtration BOARDS;
D. 15 - 29 rate (GFR) <60 mL/min/1.73 m2 for 3 months are TOPNOTCH MD)
E. < 15% classified as having chronic kidney disease,
irrespective of the presence or absence of kidney
damage.
668 This is the most common cause of glomerular origin IgA nephropathy (also known as IgA HAZEL KAREN MIDTERM 2 -
hematuria. nephritis, IgAN, Berger's disease, Berger's RAZ, MD (TOP 6 - AUG 2013
A. PSGN syndrome and synpharyngitic glomerulonephritis) FEB 2013 MED
B. Goodpasture syndrome is a form of glomerulonephritis (inflammation of BOARDS;
C. SLE Nephritis the glomeruli of the kidney). IgA nephropathy is TOPNOTCH MD)
D. Minimal Change DIsease the most common glomerulonephritis throughout
E. Berger's Syndrome the world

669 Most common cause of bacterial meningitis in adults The most common causes per age group are as HAZEL KAREN MIDTERM 2 -
>20 y/o? follows: a - infants, b - adolescents, c - RAZ, MD (TOP 6 - AUG 2013
A. H. influenzae adults >20, d - immunosuppressed and elderly FEB 2013 MED
B. N. meningitidis BOARDS;
C. S. pneumoniae TOPNOTCH MD)
D. L. monocytogenes
E. None

670 Used in diagnosis of diabetes insipidus? Metyrapone test is used to diagnose excess HAZEL KAREN MIDTERM 2 -
A. Metyrapone test cortisol production in ACTH, demeclocycline is RAZ, MD (TOP 6 - AUG 2013
B. Water deprivation test used as treatment for SIADH/ FEB 2013 MED
C. Demeclocycline administration BOARDS;
D. 24 hr urine osmolality TOPNOTCH MD)
E. None

671 A 65 year old female came in due to joint pain The case is a classic presentation of osteoarthritis HAZEL KAREN MIDTERM 2 -
invloving her knees and hip joints. Her symptoms which is due to decreased synovial fluid and RAZ, MD (TOP 6 - AUG 2013
appeared gradually with morning stiffness lasting leading to"wear and tear" of the joint. Treatment FEB 2013 MED
less than 30 minutes. Pain worsens with activity. On includes lifestyle modification, exercise and BOARDS;
PE, there are nodes at the DIP and PIP joints of the NSAIDS, and surgery id with severe debilitation. TOPNOTCH MD)
hands. what is the diagnosis?
A. Rheumatoid arthritis
B. Osteoarthritis
C. Gouty arthritis
D. Septic arthritis
E. none
672 True of hereditary spherocytosis? Hereditary spherocytosis is an auto-hemolytic HAZEL KAREN MIDTERM 2 -
A. A form of intravascular hemolysis anemia characterized by the production of red RAZ, MD (TOP 6 - AUG 2013
B. Presents during early adulthood blood cells that are sphere-shaped rather than bi- FEB 2013 MED
C. Cells have decreased surface - volume ratio concave disk shaped (donut-shaped), and BOARDS;
D. A and b therefore more prone to hemolysis. It is an TOPNOTCH MD)
E. None example of extravascular hemolysis which can
lead to heppsplenomegaly and usually present
during childhood.
673 Mechanism of lead poisoning, except? One of the main causes for the pathology of lead is HAZEL KAREN MIDTERM 2 -
A. Inhibition of ferrochelatase that it interferes with the activity of an essential RAZ, MD (TOP 6 - AUG 2013
B. Decreased heme synthesis enzyme called ALA dehydratase, which is FEB 2013 MED
C. Inhibition of ALA dehydrogenase important in the biosynthesis of heme, the BOARDS;
D. A and b only cofactor found in hemoglobin.Lead also inhibits TOPNOTCH MD)
E. None the enzyme ferrochelatase, another enzyme
involved in the formation of heme

674 Type of skin cancer associated with exposure to Basal cell carcinoma is the most common form of HAZEL KAREN MIDTERM 2 -
sunlight. Lesions are described to have rolled edges skin cancer, it is locally invasive and rarely RAZ, MD (TOP 6 - AUG 2013
with central ulceration and "palisading nuclei" on metastasize. SCC is mainly caused by cumulative FEB 2013 MED
microscopy? UV exposure over the course of a lifetime BOARDS;
A. Squamous cell carcinoma TOPNOTCH MD)
B. Melanoma
C. Basal Cell Carcinoma
D. Actinic Keratosis
E. Dysplastic Nevus

675 Disease characterized by decreased production of Depression is due to a decreased production of HAZEL KAREN MIDTERM 2 -
neurotransmitters from the locus ceruleus, ventral neurotransmitters, namely NE, dopamine and 5- RAZ, MD (TOP 6 - AUG 2013
tegmentum and raphe nucleus? HT from locus ceruleus, tegmentum of septal FEB 2013 MED
A. Anxiety nucleus and raphe nucleus respectively. BOARDS;
B. Depression TOPNOTCH MD)
C. Huntington's
D. Schizophrenia
E. None

TOPNOTCH MEDICAL BOARD PREP INTERNAL MEDICINE SUPEREXAM Page 90 of 95


For inquiries visit www.topnotchboardprep.com.ph or email us at topnotchmedicalboardprep@gmail.com
TOPNOTCH MEDICAL BOARD PREP INTERNAL MEDICINE SUPEREXAM
For inquiries visit www.topnotchboardprep.com.ph or email us at topnotchmedicalboardprep@gmail.com
Item QUESTION EXPLANATION AUTHOR TOPNOTCH
# EXAM
676 Lesion in this areal eads to development of fluent Fluent aphasia, also known as sensory aphasia/ HAZEL KAREN MIDTERM 2 -
aphasia? Wernicke's aphasia, wherein articulation is RAZ, MD (TOP 6 - AUG 2013
A. BA 22 normal, however, perception is impaired. This FEB 2013 MED
B. BA 44, 45 involves BA 22 also known as the superior BOARDS;
C. BA 3, 1, 2 temporal gyrus, or Wernicke's area. TOPNOTCH MD)
D. BA 19
E. Arcuate fasciculus

677 Most common location of berry aneurysms in the he most common sites include the anterior HAZEL KAREN MIDTERM 2 -
Circle of WIllis? cerebral artery and anterior communicating RAZ, MD (TOP 6 - AUG 2013
A. ICA artery (30–35%), the bifurcation, division of two FEB 2013 MED
B. Junction between the posterior cerebral and branches, of the internal carotid and posterior BOARDS;
posterior communicating artery communicating artery (30–35%), the bifurcation TOPNOTCH MD)
C. Junction between anterior cerebral artery and of the middle cerebral artery (20%), the
anterior communicating artery bifurcation of the basilar artery, and the remaining
D. basilar artery posterior circulation arteries (5%).
E. none

678 Side effects of anti - TB medications, except? Rifampicin causes orange discoloration of body HAZEL KAREN MIDTERM 2 -
A. INH : hepatotoxicity fluids, hepatotoxicity, pruritus. RAZ, MD (TOP 6 - AUG 2013
B. Rifampicin : peripheral neuropathy FEB 2013 MED
C. PZA : hyperuricemia BOARDS;
D. Ethambutol : optic neuropathy TOPNOTCH MD)
E. Streptomycin : ototoxic

679 Drug of choice for the treatment of anaerobic The first three antibiotics are treatment for HAZEL KAREN MIDTERM 2 -
infections above the diaphragm? penicillin - susceptible microorganisms. RAZ, MD (TOP 6 - AUG 2013
A. Amoxicillin Metronidazole is used for treatment of anaerobic FEB 2013 MED
B. Cloxacillin infections below the diaphragm. BOARDS;
C. Co - amoxiclav TOPNOTCH MD)
D. Metronidazole
E. Clindamycin

680 Lumbar tap showed increased lymphocytes, protein Bacterial - inc pressure, predominance of PMN, HAZEL KAREN MIDTERM 2 -
and decreased sugar in a patient with nuchal rigidity increased protein, decreased sugar
RAZ, MD (TOP 6 - AUG 2013
and irritability. Cause? Viral - n/inc pressure, lymphocytes, n/inc protein, FEB 2013 MED
A. Fungal normal sugar BOARDS;
B. Bacterial TOPNOTCH MD)
C. Viral
D. Protozoan
E. None of the above

681 Component therapy is encouraged in transfusion Answer: B. 3 units of PRBC MICHELLE JAY MIDTERM 1 -
medicine to avoid volume overload most especially Notes: 1 unit of PRBC increases haemoglobin by 1 FRANCISCO, MD AUG 2013
in elderly patients. In a male patient with a current g/dL in adults (3 g/dL in children) and hematocrit (TOP 9 - FEB 2013
hemoglobin level of 96 g/L, how many units of by 3% (0.03). MED BOARDS;
packed red blood cell (PRBC) is needed to be TOPNOTCH MD)
transfused to raise the hematocrit level to 0.38?
A. 2 units
B. 3 units
C. 4 units
D. 5 units
682 Which seronegative spondyloarthropathies Answer: D. Ankylosing Spondylitis MICHELLE JAY MIDTERM 1 -
associated with Human Leukocyte Antigen B27 most FRANCISCO, MD AUG 2013
likely presents in a young man complaining of (TOP 9 - FEB 2013
chronic lower back pain with morning stiffness for MED BOARDS;
more than an hour which improves with exercise? TOPNOTCH MD)
A. Rieter’s Syndrome
B. MCTD
C. Psoriatic Arthritis
D. Ankylosing Spondylitis

683 The following miscellaneous cardiac Answer: D. Bifid pulse: atrial-septal defect MICHELLE JAY MIDTERM 1 -
“pathognomonisms” are correctly matched; except, Notes: FRANCISCO, MD AUG 2013
A. Parvus et Tardus pulses: aortic stenosis • Bifid pulse: hypertrophic cardiomyopathy (from (TOP 9 - FEB 2013
B. Continuous machinery murmur: patent ductus midsystolic obstruction) MED BOARDS;
arteriosus • Fixed, split S2 heart sound: atrial-septal defect TOPNOTCH MD)
C. Pulsus paradoxus: cardiac tamponade • Opening snap: mitral stenosis
D. Bifid pulse: atrial-septal defect • Parvus only: low output cardiomyopathy

684 A 20 yo male complained of chest pains and Answer: D. Patent Ductus Arteriosus MICHELLE JAY MIDTERM 1 -
difficulty in breathing. He was seen in the ED, and his Notes: This patient has a continuous murmur that FRANCISCO, MD AUG 2013
vital signs were as follows: T = 37OC; PR = 88 bpm, straddles the entire spectrum of the cardiac cycle. (TOP 9 - FEB 2013
regular; BP = 110/70 mmHg, RR = 20 cpm; and O2 Such murmur is called a “machinery murmur,” and MED BOARDS;
sat on room air = 94%. PE revealed a rather anxious is observed in patients having PDA or an TOPNOTCH MD)
patient with no pallor, icterus, or cyanosis. JVP was arteriovenous fistula.
normal. The apical pulse was in the 5th ICS MCL, and
a murmur was heard that extended throughout the
cardiac cycle. The most likely cause for this murmur
is which of the following?
A. Aortic Stenosis
B. Mitral regurgitation
C. Pericarditis
D. Patent Ductus Arteriosus

TOPNOTCH MEDICAL BOARD PREP INTERNAL MEDICINE SUPEREXAM Page 91 of 95


For inquiries visit www.topnotchboardprep.com.ph or email us at topnotchmedicalboardprep@gmail.com
TOPNOTCH MEDICAL BOARD PREP INTERNAL MEDICINE SUPEREXAM
For inquiries visit www.topnotchboardprep.com.ph or email us at topnotchmedicalboardprep@gmail.com
Item QUESTION EXPLANATION AUTHOR TOPNOTCH
# EXAM
685 A 69 yo man with a history of coronary artery Answer: C. Bell’s Palsy MICHELLE JAY MIDTERM 1 -
disease and hypertension presents with acute onset Notes: Bell’s palsy is idiopathic and nearly always FRANCISCO, MD AUG 2013
of right facial weakness and numbness. On acute. All of the patient’s deficits are referable to (TOP 9 - FEB 2013
examination, his speech and extremity strength are the peripheral nervous system, including the loss MED BOARDS;
normal, but he has significant weakness of the right of taste (chorda tympani branch of the facial TOPNOTCH MD)
side of the face, including the orbicularis oculi. In nerve) and hyperacusis (branch of the stapedius
addition, he complains of roaring in the right ear, muscle of the ear).
and his taste sensation is absent on the right side of
the anterior tongue. Sensation is normal to
prinprick. Which of the following would best explain
these findings?
A. Lacunar stroke of the left internal capsule
B. Brainstem glioma
C. Bell’s palsy
D. A stroke due to occlusion of the left middle
cerebral artery
686 A 34 yo man comes to your clinic complaining of Answer: C. Do a barium swallow MICHELLE JAY MIDTERM 1 -
“crushing” chest discomfort for 1 hour. He has no Notes: The most likely diagnosis for the patient in FRANCISCO, MD AUG 2013
siginifcant medical history. The ECG is normal but this case is DES (Diffuse Esophageal Spasm). (TOP 9 - FEB 2013
you still give sublingual nitroglycerin which MED BOARDS;
immediately improves the pain. What is the next TOPNOTCH MD)
step in the management?
A. Request for Troponin T
B. Repeat the ECG
C. Do a barium swallow
D. Do an endoscopy
687 A 60 yo known diabetic patient was rushed to the ER Answer: B. Insert a line and administer plain NSS MICHELLE JAY MIDTERM 1 -
for unresponsiveness. On physical examination, she FRANCISCO, MD AUG 2013
was noted to have dry skin and oral mucosa with BP (TOP 9 - FEB 2013
100/70mmHg, HR 100 bpm, RR 28 cpm. RBS = MED BOARDS;
600mg/dL, urine – negative for ketones. What is the TOPNOTCH MD)
initial step in the management of this patient?
A. Give IV insulin as soon as possible
B. Insert a line and administer plain NSS
C. Give IV bolus of sodium bicarbonate
D. Intubate the patient

688 A 30 yo female is brought to the ER for lethargy. You Answer: D. Secondary Hypothyroidism MICHELLE JAY MIDTERM 1 -
noted dry skin and hair, periorbital edema and FRANCISCO, MD AUG 2013
sparse hair on the armpit. Serum TSH and FT4 levels (TOP 9 - FEB 2013
are low. What is your most probable diagnosis? MED BOARDS;
A. Primary hyperthyroidism TOPNOTCH MD)
B. Primary hypothyroidism
C. Secondary hyperthyroidism
D. Secondary hypothyroidism

689 A 35 yo patient is brought for evaluation of Answer: B. Systemic lupus erythematosus MICHELLE JAY MIDTERM 1 -
confusion for a day. You find that she has an elevated FRANCISCO, MD AUG 2013
BP, decreased air entry on right lung base with (TOP 9 - FEB 2013
dullness to percussion, and symmetrical joint MED BOARDS;
swelling of the wrists. Chemistry profile shows TOPNOTCH MD)
elevated creatinine with proteinuria on urinalysis.
What is your diagnosis?
A. Rheumatoid arthritis
B. Systemic lupus erythematosus
C. Mixed connective tissue disease
D. Nephritic syndrome
690 Characteristic of Obstructive Lung Disease: Answer: C. Decreased FEV/FVC, increased RV, MICHELLE JAY MIDTERM 1 -
A. Increased FEV/FVC, normal RV, increased VC decreased VC FRANCISCO, MD AUG 2013
B. Decreased FEV/FVC, normal RV, increased VC (TOP 9 - FEB 2013
C. Decreased FEV/FVC, increased RV, decreased VC MED BOARDS;
D. Increased FEV/FVC, increased RV, decreased VC TOPNOTCH MD)

691 Which of the following is characteristic of a transient Answer: B MICHELLE JAY MIDTERM 1 -
ischemic attack? TIA completely resolves within 24hrs, involves FRANCISCO, MD AUG 2013
A. Weakness, paralysis or dysarthria in one side of unilateral sensory or motor deficit. (TOP 9 - FEB 2013
the face or extremity MED BOARDS;
B. Incontinence of bowel & bladder TOPNOTCH MD)
C. Bilateral paresthesias, numbness or aphasia
D. Symptoms lasting longer than 24 hrs
692 An 80 yr-old male previously diagnosed to have Answer: C MICHELLE JAY MIDTERM 1 -
duodenal bulb ulcer was admitted because of Achalasia is a motor disorder of the esophagus FRANCISCO, MD AUG 2013
repeated bouts of vomiting previously ingested food characterized by dysphagia, nocturnal vomiting (TOP 9 - FEB 2013
of 2 months duration associated with weight loss and regurgitation but w/out succusion splash. MED BOARDS;
and weakness. On PE, he was noted to have poor Peptic stricture occurs at CEJ brought about by TOPNOTCH MD)
skin turgor and succusion splash. What is the chronic reflux disease. It manifests w/ progressive
probable diagnosis? dysphagia but w/out succusion splash.Perforated
A. Achalasia duodenal ulcer will present w/ sudden epigastric
B. Peptic stricture pain, pallor, hypotension, and tachycardia but still
C. Gastric outlet obstruction no succusion splash.
D. Perforated duodenal ulcer

TOPNOTCH MEDICAL BOARD PREP INTERNAL MEDICINE SUPEREXAM Page 92 of 95


For inquiries visit www.topnotchboardprep.com.ph or email us at topnotchmedicalboardprep@gmail.com
TOPNOTCH MEDICAL BOARD PREP INTERNAL MEDICINE SUPEREXAM
For inquiries visit www.topnotchboardprep.com.ph or email us at topnotchmedicalboardprep@gmail.com
Item QUESTION EXPLANATION AUTHOR TOPNOTCH
# EXAM
693 A significant amount of pleural effusion Answer: B MICHELLE JAY MIDTERM 1 -
accompanying pneumonia is an indication for: Thoracentesis allows for analysis of pleural fluid FRANCISCO, MD AUG 2013
A. CT Scan and microbiologic testing for etiologic diagnosis. (TOP 9 - FEB 2013
B. Thoracentesis CTT is not indicated unless there is empyema of MED BOARDS;
C. Closed tube thoracostomy complicated effusion. VATS and CT Scan are TOPNOTCH MD)
D. Video-assisted thoracoscopy usually not needed.

694 A 60 yr-old female with known coronary heart Answer: B MICHELLE JAY MIDTERM 1 -
disease was recently diagnosed to have In patients with coexisting medical conditions FRANCISCO, MD AUG 2013
hyperthyroidism probably due to Grave’s. The best particularly heart disease, best treatment is RAI. (TOP 9 - FEB 2013
treatment option for this patient is: MED BOARDS;
A. Anti-thyroid drugs for 3 years TOPNOTCH MD)
B. RAI therapy
C. Subtotal thyroidectomy
D. Total thyroidectomy

695 An acute myocardial infarction that is transmural Answer: A MICHELLE JAY MIDTERM 1 -
produces dramatic and characteristic Usually T wave peaking in the leads reflecting FRANCISCO, MD AUG 2013
electrocardiographic changes. The earliest change electrical activity from the necrosing area of the (TOP 9 - FEB 2013
on ECG with an acute transmural infarction is: myocardial infarction is the first sign. These are MED BOARDS;
A. Peaking of T waves referred to as hyperacute T wave changes. TOPNOTCH MD)
B. ST segment elevation
C. Development of new Q waves
D. T wave inversion

696 The patient is a 43 yr-old male. He is anemic with a Answer: C MICHELLE JAY MIDTERM 1 -
hemoglobin of 12.2 g/dl an MCV of 120 fL. Which of Choice A,B & D will reveal macrocytic RBCS but FRANCISCO, MD AUG 2013
the following is the least likely diagnosis? IDA will have microcytic RBC (Normal MVC = 80- (TOP 9 - FEB 2013
A. Acute bleeding 100femtoliters). Vit. K deficiency will have MED BOARDS;
B. Folate deficiency problems w/ coagulation but not with RBC TOPNOTCH MD)
C. Iron deficiency morphology.
D. Vit. B2 deficiency
E. Vit. K deficiency

697 The ECG of a 66 yr-old male with a history of Answer: A MICHELLE JAY MIDTERM 1 -
atherosclerotic heart disease reveals an irregular A-fib is a continuous chaotic re-entry of electrical FRANCISCO, MD AUG 2013
but rapid heart rate. The QRS complexes are normal impulses within the atrial myocardium that arises (TOP 9 - FEB 2013
but no P waves can be seen. The most likely reason in a diseased or stretched L atrium. The chaotic MED BOARDS;
for this finding is: patterns of atrial excitation prevents P-waves TOPNOTCH MD)
A. Atrial fibrillation from being seen in the ECG. He ventricular
B. Paroxysmal ventricular tachycardia response is rapid and irregular. In all other
C. Right bundle branch block choices, p wave should be distinguishable.
D. Sinus tachycardia
E. Wolf-Parkinson-White syndrome
698 One of you diabetic patient has a blood glucose level Answer: D MICHELLE JAY MIDTERM 1 -
of 200 mg/dl. Surprisingly, a dipstick test is negative Glucose excretion by the kidneys depends FRANCISCO, MD AUG 2013
for urinary glucose. How could this finding be glomerular filtration and tubular reabsorption. \At (TOP 9 - FEB 2013
explained? a renal threshold of 180 mg/dl, excess glucose is MED BOARDS;
A. Dipstick tests are more sensitive for reducing spilled into the urine and is detected by dipstick in TOPNOTCH MD)
sugars other than glucose as little as 100 mg/dl.
B. Patient has defective tubular glucose transporters
C. Patient has diabetes insipidus
D. Patient has significant renal damage
E. Patient is in a state of antidiuresis
699 A 67 yr-old woman complains of gradually Answer: B MICHELLE JAY MIDTERM 1 -
increasing fatigue. On physical examination, she is The inability to absorb Vit.B12 leads to a FRANCISCO, MD AUG 2013
found to be anemic and has peripheral neuropathy macrocytic pancytopenia and peripheral (TOP 9 - FEB 2013
characterized by loss of position and vibratory neuropathy MED BOARDS;
sense. Labs show macrocytic anemia and low WBC TOPNOTCH MD)
and platelet counts. Your impression is:
A. Folate defiency anemia
B. Pernicious anemia
C. Chronic blood loss
D. Diabetes mellitus
E. Myelodysplastic sideroblastic anemia
700 Calcium and aluminium-containing antacids should Answer: D MICHELLE JAY MIDTERM 1 -
not be given to patients taking which of the Tetracyclines such as minocycline chelates cations FRANCISCO, MD AUG 2013
following? (Ca & Al). (TOP 9 - FEB 2013
A. amoxicillin MED BOARDS;
B. erythromycin TOPNOTCH MD)
C. isoniazid
D. minocycline
E. rifampin

TOPNOTCH MEDICAL BOARD PREP INTERNAL MEDICINE SUPEREXAM Page 93 of 95


For inquiries visit www.topnotchboardprep.com.ph or email us at topnotchmedicalboardprep@gmail.com
TOPNOTCH MEDICAL BOARD PREP INTERNAL MEDICINE SUPEREXAM
For inquiries visit www.topnotchboardprep.com.ph or email us at topnotchmedicalboardprep@gmail.com

Item # ANSWER 88 C 176 A 264 B 352 C


1 D 89 B 177 E 265 B 353 D
2 B 90 E 178 E, C 266 C 354 B
3 A 91 B 179 A 267 E 355 A
4 E 92 C 180 E 268 E 356 B
5 A 93 D 181 B 269 A 357 D
6 A 94 A 182 C 270 C 358 A
7 C 95 B 183 D 271 A 359 D
8 D 96 D 184 B 272 D 360 D
9 B 97 A 185 A 273 C 361 B
10 C 98 E 186 E 274 E 362 B
11 B 99 B 187 A 275 E 363 E
12 A 100 B 188 D 276 A 364 B
13 E 101 B 189 C 277 C 365 C
14 C 102 C 190 A 278 B 366 E
15 A 103 C 191 B 279 B 367 D
16 C 104 D 192 B 280 C 368 A
17 D 105 A 193 C 281 E 369 C
18 A 106 E 194 E 282 B 370 C
19 B 107 A 195 A 283 B 371 C
20 A 108 D 196 C 284 A 372 C
21 A 109 C 197 C 285 D 373 A
22 C 110 B 198 E 286 E 374 B
23 D 111 B 199 D 287 C 375 E
24 E 112 A 200 E 288 A 376 A
25 D 113 B 201 E 289 D 377 E
26 B 114 B 202 B 290 D 378 E
27 A 115 C 203 D 291 C 379 A
28 C 116 D 204 C 292 C 380 D
29 B 117 E 205 A 293 E 381 A
30 A 118 A 206 C 294 A 382 B
31 B 119 E 207 C 295 D 383 E
32 D 120 D 208 B 296 B 384 C
33 E 121 C 209 D 297 C 385 C
34 C 122 A 210 B 298 B 386 A
35 B 123 A 211 C 299 D 387 D
36 D 124 B 212 E 300 E 388 C
37 B 125 D 213 C 301 D 389 B
38 D 126 A 214 E 302 B 390 A
39 A 127 C 215 D 303 B 391 A
40 A 128 A 216 B 304 D 392 C
41 C 129 D 217 A 305 D 393 D
42 B 130 D 218 C 306 B 394 A
43 A 131 A 219 D 307 A 395 E
44 C 132 E 220 C 308 B 396 C
45 B 133 E 221 A 309 C 397 C
46 D 134 C 222 A 310 C 398 E
47 B 135 D 223 E 311 D 399 B
48 A 136 C 224 A 312 C 400 D
49 E 137 B 225 D 313 E 401 B
50 D 138 A 226 C 314 D 402 E
51 A 139 A 227 D 315 C 403 E
52 A 140 D 228 C 316 D 404 D
53 A 141 A 229 B 317 E 405 B
54 D 142 A 230 B 318 C 406 A
55 B 143 B 231 D 319 B 407 C
56 A 144 A 232 C 320 A 408 B
57 C 145 B 233 C 321 A 409 D
58 C 146 C 234 E 322 D 410 D
59 A 147 D 235 A 323 C 411 A
60 E 148 C 236 C 324 B 412 C
61 B 149 A 237 B 325 C 413 D
62 E 150 B 238 A 326 C 414 B
63 A 151 D 239 C 327 D 415 A
64 B 152 B 240 B 328 A 416 A
65 C 153 D 241 B 329 E 417 D
66 C 154 C or E 242 C 330 D 418 A
67 D 155 C 243 B 331 C 419 B
68 B 156 A 244 C 332 D 420 D
69 A 157 B 245 A 333 D 421 A
70 E 158 C 246 A 334 B 422 C
71 B 159 B 247 D 335 E 423 D
72 B 160 BONUS 248 E 336 D 424 C
73 C 161 D 249 C 337 A 425 C
74 A 162 A 250 A 338 C 426 A
75 D 163 C 251 B 339 D 427 B
76 D 164 B 252 A 340 A 428 C
77 B 165 C 253 C 341 B 429 B
78 C 166 D 254 E 342 A 430 C
79 E 167 B 255 A 343 D 431 D
80 A 168 A 256 B 344 B 432 B
81 D 169 C 257 B 345 A 433 A
82 E 170 C 258 C 346 C 434 D
83 A 171 B 259 B 347 A 435 C
84 D 172 C 260 A 348 B 436 A
85 A 173 A 261 D 349 A 437 A
86 E 174 E 262 C 350 A 438 D
87 C 175 C 263 A 351 C 439 D
TOPNOTCH MEDICAL BOARD PREP INTERNAL MEDICINE SUPEREXAM Page 94 of 95
For inquiries visit www.topnotchboardprep.com.ph or email us at topnotchmedicalboardprep@gmail.com
TOPNOTCH MEDICAL BOARD PREP INTERNAL MEDICINE SUPEREXAM
For inquiries visit www.topnotchboardprep.com.ph or email us at topnotchmedicalboardprep@gmail.com
440 B 529 D 618 C
441 A 530 A 619 B
442 B 531 A 620 D
443 C 532 D 621 A
444 B 533 C 622 C
445 B 534 A 623 D
446 C 535 A 624 D
447 D 536 D 625 E
448 B 537 B 626 B
449 C 538 B 627 B
450 D 539 B 628 E
451 C 540 D 629 B
452 B 541 D 630 B
453 D 542 C 631 B
454 D 543 D 632 B
455 C 544 E 633 E
456 E 545 A 634 E
457 C 546 D 635 E
458 A 547 E 636 B
459 B 548 D 637 A
460 C 549 D 638 A
461 E 550 D 639 C
462 A 551 D 640 E
463 D 552 D 641 C
464 E 553 D 642 B
465 A 554 D 643 E
466 D 555 C 644 D
467 C 556 D 645 D
468 A 557 D 646 D
469 C 558 B 647 B
470 B 559 C 648 C
471 B 560 D 649 E
472 C 561 D 650 C
473 D 562 A 651 D
474 C 563 D 652 A
475 B 564 C 653 D
476 D 565 D 654 C
477 B 566 C 655 A
478 C 567 B 656 C
479 E 568 C 657 A
480 A 569 A 658 C
481 B 570 C 659 E
482 D 571 B 660 D
483 C 572 A 661 C
484 C 573 B 662 D
485 A 574 D 663 A
486 C 575 B 664 A
487 C 576 A 665 A
488 D 577 C 666 D
489 A 578 B 667 C
490 B 579 A 668 E
491 C 580 B 669 C
492 A 581 A 670 B
493 D 582 A 671 B
494 A 583 B 672 C
495 C 584 D 673 C
496 A 585 A 674 C
497 A 586 A 675 B
498 B 587 B 676 A
499 D 588 E 677 C
500 A 589 C 678 B
501 A 590 C 679 E
502 D 591 D 680 A
503 B 592 A 681 B
504 C 593 C 682 D
505 E 594 B 683 D
506 D 595 B 684 D
507 A 596 B 685 C
508 B 597 B 686 C
509 C 598 D 687 B
510 B 599 C 688 D
511 A 600 A 689 B
512 B 601 D 690 C
513 A 602 D 691 B
514 C 603 C 692 C
515 E 604 E 693 B
516 E 605 C 694 B
517 A 606 C 695 A
518 A 607 C 696 C
519 B 608 D 697 A
520 D 609 B 698 D
521 B 610 D 699 B
522 D 611 E 700 D
523 C 612 A
524 D 613 E
525 A 614 D
526 C 615 A
527 D 616 E
528 B 617 A
TOPNOTCH MEDICAL BOARD PREP INTERNAL MEDICINE SUPEREXAM Page 95 of 95
For inquiries visit www.topnotchboardprep.com.ph or email us at topnotchmedicalboardprep@gmail.com

You might also like